[go: up one dir, main page]

Academia.eduAcademia.edu
This page intentionally left blank bee80288_ifc.indd Page 1 10/26/10 4:39:07 PM user-f499 /Volumes/201/MHDQ251/bee80288_disk1of1/0073380288/bee80288_pagefiles SI Prefixes Multiplication Factor 12 1 000 000 000 1 000 000 1 000 1 000 5 10 000 5 109 000 5 106 000 5 103 100 5 102 10 5 101 0.1 5 1021 0.01 5 1022 0.001 5 1023 0.000 001 5 1026 0.000 000 001 5 1029 0.000 000 000 001 5 10212 0.000 000 000 000 001 5 10215 0.000 000 000 000 000 001 5 10218 Prefix† Symbol tera giga mega kilo hecto‡ deka‡ deci‡ centi‡ milli micro nano pico femto atto T G M k h da d c m m n p f a † The first syllable of every prefix is accented so that the prefix will retain its identity. Thus, the preferred pronunciation of kilometer places the accent on the first syllable, not the second. ‡ The use of these prefixes should be avoided, except for the measurement of areas and volumes and for the nontechnical use of centimeter, as for body and clothing measurements. U.S. Customary Units and Their SI Equivalents Quantity U.S. Customary Units SI Equivalent Acceleration ft/s2 in./s2 ft2 in2 ft ? lb kip lb oz lb ? s ft in. mi oz mass lb mass slug ton lb ? ft lb ? in. 0.3048 m/s2 0.0254 m/s2 0.0929 m2 645.2 mm2 1.356 J 4.448 kN 4.448 N 0.2780 N 4.448 N ? s 0.3048 m 25.40 mm 1.609 km 28.35 g 0.4536 kg 14.59 kg 907.2 kg 1.356 N ? m 0.1130 N ? m in4 lb ? ft ? s2 ft ? lb/s hp lb/ft2 lb/in2 (psi) ft/s in./s mi/h (mph) mi/h (mph) ft3 in3 gal qt ft ? lb 0.4162 3 106 mm4 1.356 kg ? m2 1.356 W 745.7 W 47.88 Pa 6.895 kPa 0.3048 m/s 0.0254 m/s 0.4470 m/s 1.609 km/h 0.02832 m3 16.39 cm3 3.785 L 0.9464 L 1.356 J Area Energy Force Impulse Length Mass Moment of a force Principal SI Units Used in Mechanics Quantity Unit Symbol Formula Acceleration Angle Angular acceleration Angular velocity Area Density Energy Force Frequency Impulse Length Mass Moment of a force Power Pressure Stress Time Velocity Volume, solids Liquids Work Meter per second squared Radian Radian per second squared Radian per second Square meter Kilogram per cubic meter Joule Newton Hertz Newton-second Meter Kilogram Newton-meter Watt Pascal Pascal Second Meter per second Cubic meter Liter Joule p rad p p p p J N Hz p m kg p W Pa Pa s p p L J m/s2 † rad/s2 rad/s m2 kg/m3 N?m kg ? m/s2 s21 kg ? m/s ‡ ‡ N?m J/s N/m2 N/m2 ‡ m/s m3 1023 m3 N?m † Supplementary unit (1 revolution 5 2p rad 5 3608). ‡ Base unit. ISBN: 0073380288 Author: Beer, Johnston, Dewolf, and Mazurek Title: MECHANICS OF MATERIALS Front endsheets Color: 4 Pages: 2, 3 Moment of inertia Of an area Of a mass Power Pressure or stress Velocity Volume, solids Liquids Work bee80288_ifc.indd Page 1 10/26/10 4:39:07 PM user-f499 /Volumes/201/MHDQ251/bee80288_disk1of1/0073380288/bee80288_pagefiles SI Prefixes Multiplication Factor 12 1 000 000 000 1 000 000 1 000 1 000 5 10 000 5 109 000 5 106 000 5 103 100 5 102 10 5 101 0.1 5 1021 0.01 5 1022 0.001 5 1023 0.000 001 5 1026 0.000 000 001 5 1029 0.000 000 000 001 5 10212 0.000 000 000 000 001 5 10215 0.000 000 000 000 000 001 5 10218 Prefix† Symbol tera giga mega kilo hecto‡ deka‡ deci‡ centi‡ milli micro nano pico femto atto T G M k h da d c m m n p f a † The first syllable of every prefix is accented so that the prefix will retain its identity. Thus, the preferred pronunciation of kilometer places the accent on the first syllable, not the second. ‡ The use of these prefixes should be avoided, except for the measurement of areas and volumes and for the nontechnical use of centimeter, as for body and clothing measurements. U.S. Customary Units and Their SI Equivalents Quantity U.S. Customary Units SI Equivalent Acceleration ft/s2 in./s2 ft2 in2 ft ? lb kip lb oz lb ? s ft in. mi oz mass lb mass slug ton lb ? ft lb ? in. 0.3048 m/s2 0.0254 m/s2 0.0929 m2 645.2 mm2 1.356 J 4.448 kN 4.448 N 0.2780 N 4.448 N ? s 0.3048 m 25.40 mm 1.609 km 28.35 g 0.4536 kg 14.59 kg 907.2 kg 1.356 N ? m 0.1130 N ? m in4 lb ? ft ? s2 ft ? lb/s hp lb/ft2 lb/in2 (psi) ft/s in./s mi/h (mph) mi/h (mph) ft3 in3 gal qt ft ? lb 0.4162 3 106 mm4 1.356 kg ? m2 1.356 W 745.7 W 47.88 Pa 6.895 kPa 0.3048 m/s 0.0254 m/s 0.4470 m/s 1.609 km/h 0.02832 m3 16.39 cm3 3.785 L 0.9464 L 1.356 J Area Energy Force Impulse Length Mass Moment of a force Principal SI Units Used in Mechanics Quantity Unit Symbol Formula Acceleration Angle Angular acceleration Angular velocity Area Density Energy Force Frequency Impulse Length Mass Moment of a force Power Pressure Stress Time Velocity Volume, solids Liquids Work Meter per second squared Radian Radian per second squared Radian per second Square meter Kilogram per cubic meter Joule Newton Hertz Newton-second Meter Kilogram Newton-meter Watt Pascal Pascal Second Meter per second Cubic meter Liter Joule p rad p p p p J N Hz p m kg p W Pa Pa s p p L J m/s2 † rad/s2 rad/s m2 kg/m3 N?m kg ? m/s2 s21 kg ? m/s ‡ ‡ N?m J/s N/m2 N/m2 ‡ m/s m3 1023 m3 N?m † Supplementary unit (1 revolution 5 2p rad 5 3608). ‡ Base unit. ISBN: 0073380288 Author: Beer, Johnston, Dewolf, and Mazurek Title: MECHANICS OF MATERIALS Front endsheets Color: 4 Pages: 2, 3 Moment of inertia Of an area Of a mass Power Pressure or stress Velocity Volume, solids Liquids Work bee80288_fm_i-xx_1.indd Page i 11/19/10 7:20:16 PM user-f499 /Users/user-f499/Desktop/Temp Work/Don't Delete Job/MHDQ251:Beer:201/fm MECHANICS OF MATERIALS This page intentionally left blank bee80288_fm_i-xx_1.indd Page iii 11/19/10 7:20:16 PM user-f499 /Users/user-f499/Desktop/Temp Work/Don't Delete Job/MHDQ251:Beer:201/fm SIXTH EDITION MECHANICS OF MATERIALS Ferdinand P. Beer Late of Lehigh University E. Russell Johnston, Jr. Late of University of Connecticut John T. Dewolf University of Connecticut David F. Mazurek United States Coast Guard Academy TM bee80288_fm_i-xx_1.indd Page iv 11/29/10 6:37:55 PM user-f499 /Users/user-f499/Desktop/Temp Work/Don't Delete Job/MHDQ251:Beer:201 TM MECHANICS OF MATERIALS, SIXTH EDITION Published by McGraw-Hill, a business unit of The McGraw-Hill Companies, Inc., 1221 Avenue of the Americas, New York, NY 10020. Copyright © 2012 by The McGraw-Hill Companies, Inc. All rights reserved. Previous editions © 2009, 2006, and 2002. No part of this publication may be reproduced or distributed in any form or by any means, or stored in a database or retrieval system, without the prior written consent of The McGraw-Hill Companies, Inc., including, but not limited to, in any network or other electronic storage or transmission, or broadcast for distance learning. Some ancillaries, including electronic and print components, may not be available to customers outside the United States. This book is printed on acid-free paper. 1 2 3 4 5 6 7 8 9 0 QVR/QVR 1 0 9 8 7 6 5 4 3 2 1 ISBN 978-0-07-338028-5 MHID 0-07-338028-8 Vice President, Editor-in-Chief: Marty Lange Vice President, EDP: Kimberly Meriwether David Senior Director of Development: Kristine Tibbetts Global Publisher: Raghothaman Srinivasan Executive Editor: Bill Stenquist Developmental Editor: Lora Neyens Senior Marketing Manager: Curt Reynolds Lead Project Manager: Sheila M. Frank Buyer II: Sherry L. Kane Senior Designer: Laurie B. Janssen Cover Designer: Ron Bissell Cover Image: (front) © Ervin Photography, Inc. Lead Photo Research Coordinator: Carrie K. Burger Photo Research: Sabina Dowell Compositor: Aptara®, Inc. Typeface: 10.5/12 New Caledonia Printer: Quad/Graphics All credits appearing on page or at the end of the book are considered to be an extension of the copyright page. The photos on the front and back cover show the Bob Kerrey Pedestrian Bridge, which spans the Missouri River between Omaha, Nebraska, and Council Bluffs, lowa. This S-curved structure utilizes a cable-stayed design, and is the longest pedestrian bridge to connect two states. Library of Congress Cataloging-in-Publication Data Mechanics of materials / Ferdinand Beer ... [et al.]. — 6th ed. p. cm. Includes index. ISBN 978-0-07-338028-5 ISBN 0-07-338028-8 (alk. paper) 1. Strength of materials—Textbooks. I. Beer, Ferdinand Pierre, 1915– TA405.B39 2012 620.1’12—dc22 2010037852 www.mhhe.com bee80288_fm_i-xx_1.indd Page v 11/19/10 7:20:17 PM user-f499 /Users/user-f499/Desktop/Temp Work/Don't Delete Job/MHDQ251:Beer:201/fm About the Authors As publishers of the books written by Ferd Beer and Russ Johnston, we are often asked how did they happen to write the books together, with one of them at Lehigh and the other at the University of Connecticut. The answer to this question is simple. Russ Johnston’s first teaching appointment was in the Department of Civil Engineering and Mechanics at Lehigh University. There he met Ferd Beer, who had joined that department two years earlier and was in charge of the courses in mechanics. Born in France and educated in France and Switzerland (he held an M.S. degree from the Sorbonne and an Sc.D. degree in the field of theoretical mechanics from the University of Geneva), Ferd had come to the United States after serving in the French army during the early part of World War II and had taught for four years at Williams College in the Williams-MIT joint arts and engineering program. Born in Philadelphia, Russ had obtained a B.S. degree in civil engineering from the University of Delaware and an Sc.D. degree in the field of structural engineering from MIT. Ferd was delighted to discover that the young man who had been hired chiefly to teach graduate structural engineering courses was not only willing but eager to help him reorganize the mechanics courses. Both believed that these courses should be taught from a few basic principles and that the various concepts involved would be best understood and remembered by the students if they were presented to them in a graphic way. Together they wrote lecture notes in statics and dynamics, to which they later added problems they felt would appeal to future engineers, and soon they produced the manuscript of the first edition of Mechanics for Engineers. The second edition of Mechanics for Engineers and the first edition of Vector Mechanics for Engineers found Russ Johnston at Worcester Polytechnic Institute and the next editions at the University of Connecticut. In the meantime, both Ferd and Russ had assumed administrative responsibilities in their departments, and both were involved in research, consulting, and supervising graduate students—Ferd in the area of stochastic processes and random vibrations, and Russ in the area of elastic stability and structural analysis and design. However, their interest in improving the teaching of the basic mechanics courses had not subsided, and they both taught sections of these courses as they kept revising their texts and began writing together the manuscript of the first edition of Mechanics of Materials. Ferd and Russ’s contributions to engineering education earned them a number of honors and awards. They were presented with the Western Electric Fund Award for excellence in the instruction of engineering students by their respective regional sections of the American Society for Engineering Education, and they both received the Distinguished Educator Award from the Mechanics Division of the v bee80288_fm_i-xx_1.indd Page vi 11/20/10 3:27:43 PM user-f499 vi About the Authors /Users/user-f499/Desktop/Temp Work/Don't Delete Job/MHDQ251:Beer:201/fm same society. In 1991 Russ received the Outstanding Civil Engineer Award from the Connecticut Section of the American Society of Civil Engineers, and in 1995 Ferd was awarded an honorary Doctor of Engineering degree by Lehigh University. John T. DeWolf, Professor of Civil Engineering at the University of Connecticut, joined the Beer and Johnston team as an author on the second edition of Mechanics of Materials. John holds a B.S. degree in civil engineering from the University of Hawaii and M.E. and Ph.D. degrees in structural engineering from Cornell University. His research interests are in the area of elastic stability, bridge monitoring, and structural analysis and design. He is a registered Professional Engineering and a member of the Connecticut Board of Professional Engineers. He was selected as the University of Connecticut Teaching Fellow in 2006. David F. Mazurek, Professor of Civil Engineering at the United States Coast Guard Academy, joined the team in the fourth edition. David holds a B.S. degree in ocean engineering and an M.S. degree in civil engineering from the Florida Institute of Technology, and a Ph.D. degree in civil engineering from the University of Connecticut. He is a registered Professional Engineer. He has served on the American Railway Engineering & Maintenance of Way Association’s Committee 15—Steel Structures for the past seventeen years. Professional interests include bridge engineering, structural forensics, and blastresistant design. bee80288_fm_i-xx_1.indd Page vii 11/20/10 3:27:43 PM user-f499 /Users/user-f499/Desktop/Temp Work/Don't Delete Job/MHDQ251:Beer:201/fm Contents Preface xii List of Symbols 1 1.1 1.2 1.3 1.4 1.5 1.6 1.7 1.8 1.9 1.10 1.11 1.12 1.13 xviii Introduction—Concept of Stress Introduction 4 A Short Review of the Methods of Statics 4 Stresses in the Members of a Structure 7 Analysis and Design 8 Axial Loading; Normal Stress 9 Shearing Stress 11 Bearing Stress in Connections 13 Application to the Analysis and Design of Simple Structures 13 Method of Problem Solution 16 Numerical Accuracy 17 Stress on an Oblique Plane under Axial Loading 26 Stress under General Loading Conditions; Components of Stress 27 Design Considerations 30 Review and Summary for Chapter 1 2 2.1 2.2 2.3 *2.4 2.5 2.6 2.7 2.8 2.9 2.10 2.11 2.12 *2.13 2 42 Stress and Strain—Axial Loading 52 Introduction 54 Normal Strain under Axial Loading 55 Stress-Strain Diagram 57 True Stress and True Strain 61 Hooke’s Law; Modulus of Elasticity 62 Elastic versus Plastic Behavior of a Material 64 Repeated Loadings; Fatigue 66 Deformations of Members under Axial Loading 67 Statically Indeterminate Problems 78 Problems Involving Temperature Changes 82 Poisson’s Ratio 93 Multiaxial Loading; Generalized Hooke’s Law 94 Dilatation; Bulk Modulus 96 vii bee80288_fm_i-xx_1.indd Page viii 11/19/10 7:20:18 PM user-f499 viii Contents /Users/user-f499/Desktop/Temp Work/Don't Delete Job/MHDQ251:Beer:201/fm 2.14 Shearing Strain 98 2.15 Further Discussion of Deformations under Axial Loading; Relation among E, n, and G 101 *2.16 Stress-Strain Relationships for Fiber-Reinforced Composite Materials 103 2.17 Stress and Strain Distribution under Axial Loading; Saint-Venant’s Principle 113 2.18 Stress Concentrations 115 2.19 Plastic Deformations 117 *2.20 Residual Stresses 121 Review and Summary for Chapter 2 3 3.1 3.2 3.3 3.4 3.5 3.6 3.7 3.8 *3.9 *3.10 *3.11 *3.12 *3.13 Torsion 140 Introduction 142 Preliminary Discussion of the Stresses in a Shaft 144 Deformations in a Circular Shaft 145 Stresses in the Elastic Range 148 Angle of Twist in the Elastic Range 159 Statically Indeterminate Shafts 163 Design of Transmission Shafts 176 Stress Concentrations in Circular Shafts 179 Plastic Deformations in Circular Shafts 184 Circular Shafts Made of an Elastoplastic Material 186 Residual Stresses in Circular Shafts 189 Torsion of Noncircular Members 197 Thin-Walled Hollow Shafts 200 Review and Summary for Chapter 3 4 4.1 4.2 4.3 4.4 4.5 4.6 4.7 *4.8 *4.9 *4.10 129 Pure Bending 210 220 Introduction 222 Symmetric Member in Pure Bending 224 Deformations in a Symmetric Member in Pure Bending 226 Stresses and Deformations in the Elastic Range 229 Deformations in a Transverse Cross Section 233 Bending of Members Made of Several Materials 242 Stress Concentrations 246 Plastic Deformations 255 Members Made of an Elastoplastic Material 256 Plastic Deformations of Members with a Single Plane of Symmetry 260 *4.11 Residual Stresses 261 4.12 Eccentric Axial Loading in a Plane of Symmetry 270 bee80288_fm_i-xx_1.indd Page ix 11/19/10 7:20:18 PM user-f499 4.13 Unsymmetric Bending 279 4.14 General Case of Eccentric Axial Loading *4.15 Bending of Curved Members 294 Review and Summary for Chapter 4 5 5.1 5.2 5.3 5.4 *5.5 *5.6 6.1 6.2 6.3 6.4 *6.5 6.6 6.7 *6.8 *6.9 7.1 7.2 7.3 7.4 7.5 284 305 Introduction 316 Shear and Bending-Moment Diagrams 319 Relations among Load, Shear, and Bending Moment 329 Design of Prismatic Beams for Bending 339 Using Singularity Functions to Determine Shear and Bending Moment in a Beam 350 Nonprismatic Beams 361 370 Shearing Stresses in Beams and Thin-Walled Members 380 Introduction 382 Shear on the Horizontal Face of a Beam Element 384 Determination of the Shearing Stresses in a Beam 386 Shearing Stresses txy in Common Types of Beams 387 Further Discussion of the Distribution of Stresses in a Narrow Rectangular Beam 390 Longitudinal Shear on a Beam Element of Arbitrary Shape 399 Shearing Stresses in Thin-Walled Members 401 Plastic Deformations 404 Unsymmetric Loading of Thin-Walled Members; Shear Center 414 Review and Summary for Chapter 6 7 Contents Analysis and Design of Beams for Bending 314 Review and Summary for Chapter 5 6 /Users/user-f499/Desktop/Temp Work/Don't Delete Job/MHDQ251:Beer:201/fm 427 Transformations of Stress and Strain 436 Introduction 438 Transformation of Plane Stress 440 Principal Stresses: Maximum Shearing Stress 443 Mohr’s Circle for Plane Stress 452 General State of Stress 462 ix bee80288_fm_i-xx_1.indd Page x 11/19/10 7:20:18 PM user-f499 x Contents /Users/user-f499/Desktop/Temp Work/Don't Delete Job/MHDQ251:Beer:201/fm 7.6 *7.7 *7.8 7.9 *7.10 *7.11 *7.12 *7.13 Application of Mohr’s Circle to the Three-Dimensional Analysis of Stress 464 Yield Criteria for Ductile Materials under Plane Stress 467 Fracture Criteria for Brittle Materials under Plane Stress 469 Stresses in Thin-Walled Pressure Vessels 478 Transformation of Plane Strain 486 Mohr’s Circle for Plane Strain 489 Three-Dimensional Analysis of Strain 491 Measurements of Strain; Strain Rosette 494 Review and Summary for Chapter 7 8 *8.1 *8.2 *8.3 *8.4 Principal Stresses under a Given Loading 512 Introduction 514 Principal Stresses in a Beam 515 Design of Transmission Shafts 518 Stresses under Combined Loadings 527 Review and Summary for Chapter 8 9 9.1 9.2 9.3 *9.4 9.5 *9.6 9.7 9.8 *9.9 *9.10 *9.11 *9.12 *9.13 *9.14 502 540 Deflection of Beams 548 Introduction 550 Deformation of a Beam under Transverse Loading 552 Equation of the Elastic Curve 553 Direct Determination of the Elastic Curve from the Load Distribution 559 Statically Indeterminate Beams 561 Using Singularity Functions to Determine the Slope and Deflection of a Beam 571 Method of Superposition 580 Application of Superposition to Statically Indeterminate Beams 582 Moment-Area Theorems 592 Application to Cantilever Beams and Beams with Symmetric Loadings 595 Bending-Moment Diagrams by Parts 597 Application of Moment-Area Theorems to Beams with Unsymmetric Loadings 605 Maximum Deflection 607 Use of Moment-Area Theorems with Statically Indeterminate Beams 609 Review and Summary for Chapter 9 618 bee80288_fm_i-xx_1.indd Page xi 11/19/10 7:20:18 PM user-f499 10 10.1 10.2 10.3 10.4 *10.5 10.6 10.7 Columns 11.1 11.2 11.3 11.4 11.5 11.6 11.7 11.8 11.9 11.10 *11.11 *11.12 *11.13 *11.14 Introduction 632 Stability of Structures 632 Euler’s Formula for Pin-Ended Columns 635 Extension of Euler’s Formula to Columns with Other End Conditions 638 Eccentric Loading; the Secant Formula 649 Design of Columns under a Centric Load 660 Design of Columns under an Eccentric Load 675 692 Introduction 694 Strain Energy 694 Strain-Energy Density 696 Elastic Strain Energy for Normal Stresses 698 Elastic Strain Energy for Shearing Stresses 701 Strain Energy for a General State of Stress 704 Impact Loading 716 Design for Impact Loads 718 Work and Energy under a Single Load 719 Deflection under a Single Load by the Work-Energy Method 722 Work and Energy under Several Loads 732 Castigliano’s Theorem 734 Deflections by Castigliano’s Theorem 736 Statically Indeterminate Structures 740 Appendices C D E 684 Energy Methods Review and Summary for Chapter 11 A B Contents 630 Review and Summary for Chapter 10 11 /Users/user-f499/Desktop/Temp Work/Don't Delete Job/MHDQ251:Beer:201/fm 750 A1 Moments of Areas A2 Typical Properties of Selected Materials Used in Engineering A12 Properties of Rolled-Steel Shapes A16 Beam Deflections and Slopes A28 Fundamentals of Engineering Examination A29 Photo Credits C1 Index I1 Answers to Problems An1 xi bee80288_fm_i-xx_1.indd Page xii 11/19/10 7:20:18 PM user-f499 /Users/user-f499/Desktop/Temp Work/Don't Delete Job/MHDQ251:Beer:201/fm Preface OBJECTIVES The main objective of a basic mechanics course should be to develop in the engineering student the ability to analyze a given problem in a simple and logical manner and to apply to its solution a few fundamental and well-understood principles. This text is designed for the first course in mechanics of materials—or strength of materials— offered to engineering students in the sophomore or junior year. The authors hope that it will help instructors achieve this goal in that particular course in the same way that their other texts may have helped them in statics and dynamics. GENERAL APPROACH In this text the study of the mechanics of materials is based on the understanding of a few basic concepts and on the use of simplified models. This approach makes it possible to develop all the necessary formulas in a rational and logical manner, and to clearly indicate the conditions under which they can be safely applied to the analysis and design of actual engineering structures and machine components. Free-body Diagrams Are Used Extensively. Throughout the text free-body diagrams are used to determine external or internal forces. The use of “picture equations” will also help the students understand the superposition of loadings and the resulting stresses and deformations. Design Concepts Are Discussed Throughout the Text Whenever Appropriate. A discussion of the application of the factor of safety to design can be found in Chap. 1, where the concepts of both allowable stress design and load and resistance factor design are presented. A Careful Balance Between SI and U.S. Customary Units Is Consistently Maintained. Because it is essential that students be able to handle effectively both SI metric units and U.S. customary units, half the examples, sample problems, and problems to be assigned have been stated in SI units and half in U.S. customary units. Since a large number of problems are available, instructors can assign problems using each system of units in whatever proportion they find most desirable for their class. Optional Sections Offer Advanced or Specialty Topics. Topics such as residual stresses, torsion of noncircular and thin-walled members, bending of curved beams, shearing stresses in non-symmetrical xii bee80288_fm_i-xx_1.indd Page xiii 11/19/10 7:20:18 PM user-f499 /Users/user-f499/Desktop/Temp Work/Don't Delete Job/MHDQ251:Beer:201/fm members, and failure criteria, have been included in optional sections for use in courses of varying emphases. To preserve the integrity of the subject, these topics are presented in the proper sequence, wherever they logically belong. Thus, even when not covered in the course, they are highly visible and can be easily referred to by the students if needed in a later course or in engineering practice. For convenience all optional sections have been indicated by asterisks. CHAPTER ORGANIZATION It is expected that students using this text will have completed a course in statics. However, Chap. 1 is designed to provide them with an opportunity to review the concepts learned in that course, while shear and bending-moment diagrams are covered in detail in Secs. 5.2 and 5.3. The properties of moments and centroids of areas are described in Appendix A; this material can be used to reinforce the discussion of the determination of normal and shearing stresses in beams (Chaps. 4, 5, and 6). The first four chapters of the text are devoted to the analysis of the stresses and of the corresponding deformations in various structural members, considering successively axial loading, torsion, and pure bending. Each analysis is based on a few basic concepts, namely, the conditions of equilibrium of the forces exerted on the member, the relations existing between stress and strain in the material, and the conditions imposed by the supports and loading of the member. The study of each type of loading is complemented by a large number of examples, sample problems, and problems to be assigned, all designed to strengthen the students’ understanding of the subject. The concept of stress at a point is introduced in Chap. 1, where it is shown that an axial load can produce shearing stresses as well as normal stresses, depending upon the section considered. The fact that stresses depend upon the orientation of the surface on which they are computed is emphasized again in Chaps. 3 and 4 in the cases of torsion and pure bending. However, the discussion of computational techniques—such as Mohr’s circle—used for the transformation of stress at a point is delayed until Chap. 7, after students have had the opportunity to solve problems involving a combination of the basic loadings and have discovered for themselves the need for such techniques. The discussion in Chap. 2 of the relation between stress and strain in various materials includes fiber-reinforced composite materials. Also, the study of beams under transverse loads is covered in two separate chapters. Chapter 5 is devoted to the determination of the normal stresses in a beam and to the design of beams based on the allowable normal stress in the material used (Sec. 5.4). The chapter begins with a discussion of the shear and bending-moment diagrams (Secs. 5.2 and 5.3) and includes an optional section on the use of singularity functions for the determination of the shear and bending moment in a beam (Sec. 5.5). The chapter ends with an optional section on nonprismatic beams (Sec. 5.6). Preface xiii bee80288_fm_i-xx_1.indd Page xiv 11/19/10 7:20:19 PM user-f499 xiv Preface /Users/user-f499/Desktop/Temp Work/Don't Delete Job/MHDQ251:Beer:201/fm Chapter 6 is devoted to the determination of shearing stresses in beams and thin-walled members under transverse loadings. The formula for the shear flow, q 5 VQyI, is derived in the traditional way. More advanced aspects of the design of beams, such as the determination of the principal stresses at the junction of the flange and web of a W-beam, are in Chap. 8, an optional chapter that may be covered after the transformations of stresses have been discussed in Chap. 7. The design of transmission shafts is in that chapter for the same reason, as well as the determination of stresses under combined loadings that can now include the determination of the principal stresses, principal planes, and maximum shearing stress at a given point. Statically indeterminate problems are first discussed in Chap. 2 and considered throughout the text for the various loading conditions encountered. Thus, students are presented at an early stage with a method of solution that combines the analysis of deformations with the conventional analysis of forces used in statics. In this way, they will have become thoroughly familiar with this fundamental method by the end of the course. In addition, this approach helps the students realize that stresses themselves are statically indeterminate and can be computed only by considering the corresponding distribution of strains. The concept of plastic deformation is introduced in Chap. 2, where it is applied to the analysis of members under axial loading. Problems involving the plastic deformation of circular shafts and of prismatic beams are also considered in optional sections of Chaps. 3, 4, and 6. While some of this material can be omitted at the choice of the instructor, its inclusion in the body of the text will help students realize the limitations of the assumption of a linear stress-strain relation and serve to caution them against the inappropriate use of the elastic torsion and flexure formulas. The determination of the deflection of beams is discussed in Chap. 9. The first part of the chapter is devoted to the integration method and to the method of superposition, with an optional section (Sec. 9.6) based on the use of singularity functions. (This section should be used only if Sec. 5.5 was covered earlier.) The second part of Chap. 9 is optional. It presents the moment-area method in two lessons. Chapter 10 is devoted to columns and contains material on the design of steel, aluminum, and wood columns. Chapter 11 covers energy methods, including Castigliano’s theorem. PEDAGOGICAL FEATURES Each chapter begins with an introductory section setting the purpose and goals of the chapter and describing in simple terms the material to be covered and its application to the solution of engineering problems. Chapter Lessons. The body of the text has been divided into units, each consisting of one or several theory sections followed by sample problems and a large number of problems to be assigned. bee80288_fm_i-xx_1.indd Page xv 11/20/10 3:27:48 PM user-f499 /Users/user-f499/Desktop/Temp Work/Don't Delete Job/MHDQ251:Beer:201/fm Each unit corresponds to a well-defined topic and generally can be covered in one lesson. Examples and Sample Problems. The theory sections include many examples designed to illustrate the material being presented and facilitate its understanding. The sample problems are intended to show some of the applications of the theory to the solution of engineering problems. Since they have been set up in much the same form that students will use in solving the assigned problems, the sample problems serve the double purpose of amplifying the text and demonstrating the type of neat and orderly work that students should cultivate in their own solutions. Homework Problem Sets. Most of the problems are of a practical nature and should appeal to engineering students. They are primarily designed, however, to illustrate the material presented in the text and help the students understand the basic principles used in mechanics of materials. The problems have been grouped according to the portions of material they illustrate and have been arranged in order of increasing difficulty. Problems requiring special attention have been indicated by asterisks. Answers to problems are given at the end of the book, except for those with a number set in italics. Chapter Review and Summary. Each chapter ends with a review and summary of the material covered in the chapter. Notes in the margin have been included to help the students organize their review work, and cross references provided to help them find the portions of material requiring their special attention. Review Problems. A set of review problems is included at the end of each chapter. These problems provide students further opportunity to apply the most important concepts introduced in the chapter. Computer Problems. Computers make it possible for engineering students to solve a great number of challenging problems. A group of six or more problems designed to be solved with a computer can be found at the end of each chapter. These problems can be solved using any computer language that provides a basis for analytical calculations. Developing the algorithm required to solve a given problem will benefit the students in two different ways: (1) it will help them gain a better understanding of the mechanics principles involved; (2) it will provide them with an opportunity to apply the skills acquired in their computer programming course to the solution of a meaningful engineering problem. These problems can be solved using any computer language that provide a basis for analytical calculations. Fundamentals of Engineering Examination. Engineers who seek to be licensed as Professional Engineers must take two exams. The first exam, the Fundamentals of Engineering Examination, includes subject material from Mechanics of Materials. Appendix E lists the topics in Mechanics of Materials that are covered in this exam along with problems that can be solved to review this material. Preface xv bee80288_fm_i-xx_1.indd Page xvi 11/29/10 6:42:12 PM user-f499 xvi Preface /Users/user-f499/Desktop/Temp Work/Don't Delete Job/MHDQ251:Beer:201 SUPPLEMENTAL RESOURCES Instructor’s Solutions Manual. The Instructor’s and Solutions Manual that accompanies the sixth edition continues the tradition of exceptional accuracy and keeping solutions contained to a single page for easier reference. The manual also features tables designed to assist instructors in creating a schedule of assignments for their courses. The various topics covered in the text are listed in Table I, and a suggested number of periods to be spent on each topic is indicated. Table II provides a brief description of all groups of problems and a classification of the problems in each group according to the units used. Sample lesson schedules are also found within the manual. MCGRAW-HILL CONNECT ENGINEERING McGraw-Hill Connect EngineeringTM is a web-based assignment and assessment platform that gives students the means to better connect with their coursework, with their instructors, and with the important concepts that they will need to know for success now and in the future. With Connect Engineering, instructors can deliver assignments, quizzes, and tests easily online. Students can practice important skills at their own pace and on their own schedule. With Connect Engineering Plus, students also get 24/7 online access to an eBook— an online edition of the text—to aid them in successfully completing their work, wherever and whenever they choose. Connect Engineering for Mechanics of Materials is available at www.mcgrawhillconnect.com McGRAW-HILL CREATE™ Craft your teaching resources to match the way you teach! With McGraw-Hill CreateTM, www.mcgrawhillcreate.com, you can easily rearrange chapters, combine material from other content sources, and quickly upload content you have written like your course syllabus or teaching notes. Arrange your book to fit your teaching style. Create even allows you to personalize your book’s appearance by selecting the cover and adding your name, school, and course information. Order a Create book and you’ll receive a complimentary print review copy in 3–5 business days or a complimentary electronic review copy (eComp) via email in minutes. Go to www.mcgrawhillcreate.com today and register to experience how McGraw-Hill Create empowers you to teach your students your way. McGraw-Hill Higher Education and Blackboard® have teamed up. Blackboard, the Web-based course-management system, has partnered with McGraw-Hill to better allow students and faculty to use online materials and activities to complement face-to-face teaching. Blackboard features exciting social learning and teaching tools that foster more logical, visually impactful and active learning opportunities for students. You’ll transform your closed-door classrooms into communities where students remain connected to their educational experience 24 hours a day. This partnership allows you and your students access to McGraw-Hill’s Connect and Create right from within your Blackboard course—all with one single sign-on. bee80288_fm_i-xx_1.indd Page xvii 11/29/10 6:57:17 PM user-f499 /Users/user-f499/Desktop/Temp Work/Don't Delete Job/MHDQ251:Beer:201 Not only do you get single sign-on with Connect and Create, you also get deep integration of McGraw-Hill content and content engines right in Blackboard. Whether you’re choosing a book for your course or building Connect assignments, all the tools you need are right where you want them—inside of Blackboard. Gradebooks are now seamless. When a student completes an integrated Connect assignment, the grade for that assignment automatically (and instantly) feeds your Blackboard grade center. McGraw-Hill and Blackboard can now offer you easy access to industry leading technology and content, whether your campus hosts it, or we do. Be sure to ask your local McGraw-Hill representative for details. ADDITIONAL ONLINE RESOURCES Mechanics of Materials 6e also features a companion website (www. mhhe.com/beerjohnston) for instructors. Included on the website are lecture PowerPoints, an image library, and animations. Via the website, instructors can also request access to C.O.S.M.O.S., a complete online solutions manual organization system that allows instructors to create custom homework, quizzes, and tests using end-of-chapter problems from the text. For access to this material, contact your sales representative for a user name and password. Hands-On Mechanics. Hands-On Mechanics is a website designed for instructors who are interested in incorporating threedimensional, hands-on teaching aids into their lectures. Developed through a partnership between McGraw-Hill and the Department of Civil and Mechanical Engineering at the United States Military Academy at West Point, this website not only provides detailed instructions for how to build 3-D teaching tools using materials found in any lab or local hardware store but also provides a community where educators can share ideas, trade best practices, and submit their own demonstrations for posting on the site. Visit www. handsonmechanics.com to see how you can put this to use in your classroom. ACKNOWLEDGMENTS The authors thank the many companies that provided photographs for this edition. We also wish to recognize the determined efforts and patience of our photo researcher Sabina Dowell. Our special thanks go to Professor Dean Updike, of the Department of Mechanical Engineering and Mechanics, Lehigh University for his patience and cooperation as he checked the solutions and answers of all the problems in this edition. We also gratefully acknowledge the help, comments and suggestions offered by the many reviewers and users of previous editions of Mechanics of Materials. John T. DeWolf David F. Mazurek Preface xvii bee80288_fm_i-xx_1.indd Page xviii 11/20/10 3:27:49 PM user-f499 /Users/user-f499/Desktop/Temp Work/Don't Delete Job/MHDQ251:Beer:201/fm List of Symbols a A, B, C, . . . A, B, C, . . . A, A b c C C1, C2, . . . CP d D e E f F F.S. G h H H, J, K I, Ix, . . . Ixy, . . . J k K l L Le m M M, Mx, . . . MD ML MU n p P PD PL PU q Q Q xviii Constant; distance Forces; reactions Points Area Distance; width Constant; distance; radius Centroid Constants of integration Column stability factor Distance; diameter; depth Diameter Distance; eccentricity; dilatation Modulus of elasticity Frequency; function Force Factor of safety Modulus of rigidity; shear modulus Distance; height Force Points Moment of inertia Product of inertia Polar moment of inertia Spring constant; shape factor; bulk modulus; constant Stress concentration factor; torsional spring constant Length; span Length; span Effective length Mass Couple Bending moment Bending moment, dead load (LRFD) Bending moment, live load (LRFD) Bending moment, ultimate load (LRFD) Number; ratio of moduli of elasticity; normal direction Pressure Force; concentrated load Dead load (LRFD) Live load (LRFD) Ultimate load (LRFD) Shearing force per unit length; shear flow Force First moment of area bee80288_fm_i-xx_1.indd Page xix 11/20/10 3:27:50 PM user-f499 r R R s S t T T u, v u U v V V w W, W x, y, z x, y, z Z a, b, g a g gD gL d e u l n r s t f v /Users/user-f499/Desktop/Temp Work/Don't Delete Job/MHDQ251:Beer:201/fm Radius; radius of gyration Force; reaction Radius; modulus of rupture Length Elastic section modulus Thickness; distance; tangential deviation Torque Temperature Rectangular coordinates Strain-energy density Strain energy; work Velocity Shearing force Volume; shear Width; distance; load per unit length Weight, load Rectangular coordinates; distance; displacements; deflections Coordinates of centroid Plastic section modulus Angles Coefficient of thermal expansion; influence coefficient Shearing strain; specific weight Load factor, dead load (LRFD) Load factor, live load (LRFD) Deformation; displacement Normal strain Angle; slope Direction cosine Poisson’s ratio Radius of curvature; distance; density Normal stress Shearing stress Angle; angle of twist; resistance factor Angular velocity List of Symbols xix This page intentionally left blank bee80288_fm_i-xx_1.indd Page Sec1:1 11/19/10 7:20:21 PM user-f499 /Users/user-f499/Desktop/Temp Work/Don't Delete Job/MHDQ251:Beer:201/fm MECHANICS OF MATERIALS bee80288_ch01_002-051.indd Page 2 11/1/10 4:54:15 PM user-f499 /Users/user-f499/Desktop/Temp Work/Don't Delete Job/MHDQ251:Beer:201/ch This chapter is devoted to the study of the stresses occurring in many of the elements contained in these excavators, such as two-force members, axles, bolts, and pins. 2 bee80288_ch01_002-051.indd Page 3 11/1/10 4:54:22 PM user-f499 /Users/user-f499/Desktop/Temp Work/Don't Delete Job/MHDQ251:Beer:201/ch C H A P T E R Introduction—Concept of Stress 3 bee80288_ch01_002-051.indd Page 4 11/2/10 2:54:53 PM user-f499 Chapter 1 Introduction—Concept of Stress 1.1 1.2 1.3 1.4 1.5 1.6 1.7 1.8 1.9 1.10 1.11 1.12 1.13 Introduction A Short Review of the Methods of Statics Stresses in the Members of a Structure Analysis and Design Axial Loading; Normal Stress Shearing Stress Bearing Stress in Connections Application to the Analysis and Design of Simple Structures Method of Problem Solution Numerical Accuracy Stress on an Oblique Plane Under Axial Loading Stress Under General Loading Conditions; Components of Stress Design Considerations /Users/user-f499/Desktop/Temp Work/Don't Delete Job/MHDQ251:Beer:201/ch01 1.1 INTRODUCTION The main objective of the study of the mechanics of materials is to provide the future engineer with the means of analyzing and designing various machines and load-bearing structures. Both the analysis and the design of a given structure involve the determination of stresses and deformations. This first chapter is devoted to the concept of stress. Section 1.2 is devoted to a short review of the basic methods of statics and to their application to the determination of the forces in the members of a simple structure consisting of pin-connected members. Section 1.3 will introduce you to the concept of stress in a member of a structure, and you will be shown how that stress can be determined from the force in the member. After a short discussion of engineering analysis and design (Sec. 1.4), you will consider successively the normal stresses in a member under axial loading (Sec. 1.5), the shearing stresses caused by the application of equal and opposite transverse forces (Sec. 1.6), and the bearing stresses created by bolts and pins in the members they connect (Sec. 1.7). These various concepts will be applied in Sec. 1.8 to the determination of the stresses in the members of the simple structure considered earlier in Sec. 1.2. The first part of the chapter ends with a description of the method you should use in the solution of an assigned problem (Sec. 1.9) and with a discussion of the numerical accuracy appropriate in engineering calculations (Sec. 1.10). In Sec. 1.11, where a two-force member under axial loading is considered again, it will be observed that the stresses on an oblique plane include both normal and shearing stresses, while in Sec. 1.12 you will note that six components are required to describe the state of stress at a point in a body under the most general loading conditions. Finally, Sec. 1.13 will be devoted to the determination from test specimens of the ultimate strength of a given material and to the use of a factor of safety in the computation of the allowable load for a structural component made of that material. 1.2 A SHORT REVIEW OF THE METHODS OF STATICS In this section you will review the basic methods of statics while determining the forces in the members of a simple structure. Consider the structure shown in Fig. 1.1, which was designed to support a 30-kN load. It consists of a boom AB with a 30 3 50-mm rectangular cross section and of a rod BC with a 20-mm-diameter circular cross section. The boom and the rod are connected by a pin at B and are supported by pins and brackets at A and C, respectively. Our first step should be to draw a free-body diagram of the structure by detaching it from its supports at A and C, and showing the reactions that these supports exert on the structure (Fig. 1.2). Note that the sketch of the structure has been simplified by omitting all unnecessary details. Many of you may have recognized at this point that AB and BC are two-force members. For those of you who have not, we will pursue our analysis, ignoring that fact and assuming that the directions of the reactions at A and C are unknown. Each of these 4 bee80288_ch01_002-051.indd Page 5 9/4/10 5:33:01 PM user-f499 /Users/user-f499/Desktop/Temp Work/Don't Delete Job/MHDQ251:Beer:201/ch01 1.2 A Short Review of the Methods of Statics C d ⫽ 20 mm 600 mm A 50 mm B 800 mm 30 kN Fig. 1.1 Boom used to support a 30-kN load. Cy reactions, therefore, will be represented by two components, Ax and Ay at A, and Cx and Cy at C. We write the following three equilibrium equations: 1l o MC 5 0: 1 y o Fx 5 0: 1x o Fy 5 0: Ax 10.6 m2 2 130 kN2 10.8 m2 5 0 Ax 5 140 kN Ax 1 Cx 5 0 Cx 5 2Ax Cx 5 240 kN Ay 1 Cy 2 30 kN 5 0 Ay 1 Cy 5 130 kN (1.1) 2Ay 10.8 m2 5 0 Ay 5 0 Ay 0.6 m (1.2) (1.3) We have found two of the four unknowns, but cannot determine the other two from these equations, and no additional independent equation can be obtained from the free-body diagram of the structure. We must now dismember the structure. Considering the freebody diagram of the boom AB (Fig. 1.3), we write the following equilibrium equation: 1l o MB 5 0: C Cx Ax 0.8 m 30 kN Fig. 1.2 (1.4) Cx 5 40 kN z , Cy 5 30 kNx We note that the reaction at A is directed along the axis of the boom AB and causes compression in that member. Observing that the components Cx and Cy of the reaction at C are, respectively, proportional to the horizontal and vertical components of the distance from B to C, we conclude that the reaction at C is equal to 50 kN, is directed along the axis of the rod BC, and causes tension in that member. By Ay Substituting for Ay from (1.4) into (1.3), we obtain Cy 5 130 kN. Expressing the results obtained for the reactions at A and C in vector form, we have A 5 40 kN y B A Ax A B 0.8 m 30 kN Fig. 1.3 Bz 5 bee80288_ch01_002-051.indd Page 6 9/4/10 5:33:09 PM user-f499 6 These results could have been anticipated by recognizing that AB and BC are two-force members, i.e., members that are subjected to forces at only two points, these points being A and B for member AB, and B and C for member BC. Indeed, for a two-force member the lines of action of the resultants of the forces acting at each of the two points are equal and opposite and pass through both points. Using this property, we could have obtained a simpler solution by considering the free-body diagram of pin B. The forces on pin B are the forces FAB and FBC exerted, respectively, by members AB and BC, and the 30-kN load (Fig. 1.4a). We can express that pin B is in equilibrium by drawing the corresponding force triangle (Fig. 1.4b). Since the force FBC is directed along member BC, its slope is the same as that of BC, namely, 3/4. We can, therefore, write the proportion Introduction—Concept of Stress FBC FBC 30 kN 5 3 4 B FAB FAB 30 kN (a) /Users/user-f499/Desktop/Temp Work/Don't Delete Job/MHDQ251:Beer:201/ch01 (b) Fig. 1.4 FBC FAB 30 kN 5 5 3 4 5 from which we obtain FAB 5 40 kN FBC 5 50 kN The forces F9AB and F9BC exerted by pin B, respectively, on boom AB and rod BC are equal and opposite to FAB and FBC (Fig. 1.5). FBC FBC C C D FBC F'BC D B FAB Fig. 1.5 A B F'BC B F'AB F'BC Fig. 1.6 Knowing the forces at the ends of each of the members, we can now determine the internal forces in these members. Passing a section at some arbitrary point D of rod BC, we obtain two portions BD and CD (Fig. 1.6). Since 50-kN forces must be applied at D to both portions of the rod to keep them in equilibrium, we conclude that an internal force of 50 kN is produced in rod BC when a 30-kN load is applied at B. We further check from the directions of the forces FBC and F9BC in Fig. 1.6 that the rod is in tension. A similar procedure would enable us to determine that the internal force in boom AB is 40 kN and that the boom is in compression. bee80288_ch01_002-051.indd Page 7 9/4/10 5:33:15 PM user-f499 1.3 /Users/user-f499/Desktop/Temp Work/Don't Delete Job/MHDQ251:Beer:201/ch01 1.3 Stresses in the Members of a Structure STRESSES IN THE MEMBERS OF A STRUCTURE While the results obtained in the preceding section represent a first and necessary step in the analysis of the given structure, they do not tell us whether the given load can be safely supported. Whether rod BC, for example, will break or not under this loading depends not only upon the value found for the internal force FBC, but also upon the cross-sectional area of the rod and the material of which the rod is made. Indeed, the internal force FBC actually represents the resultant of elementary forces distributed over the entire area A of the cross section (Fig. 1.7) and the average intensity of these distributed forces is equal to the force per unit area, FBCyA, in the section. Whether or not the rod will break under the given loading clearly depends upon the ability of the material to withstand the corresponding value FBCyA of the intensity of the distributed internal forces. It thus depends upon the force FBC, the cross-sectional area A, and the material of the rod. The force per unit area, or intensity of the forces distributed over a given section, is called the stress on that section and is denoted by the Greek letter s (sigma). The stress in a member of cross-sectional area A subjected to an axial load P (Fig. 1.8) is therefore obtained by dividing the magnitude P of the load by the area A: P s5 A FBC ⫽ FBC A A Fig. 1.7 P ⫽ (1.5) A positive sign will be used to indicate a tensile stress (member in tension) and a negative sign to indicate a compressive stress (member in compression). Since SI metric units are used in this discussion, with P expressed in newtons (N) and A in square meters (m2), the stress s will be expressed in N/m2. This unit is called a pascal (Pa). However, one finds that the pascal is an exceedingly small quantity and that, in practice, multiples of this unit must be used, namely, the kilopascal (kPa), the megapascal (MPa), and the gigapascal (GPa). We have 1 kPa 5 103 Pa 5 103 N/m2 1 MPa 5 106 Pa 5 106 N/m2 1 GPa 5 109 Pa 5 109 N/m2 When U.S. customary units are used, the force P is usually expressed in pounds (lb) or kilopounds (kip), and the cross-sectional area A in square inches (in2). The stress s will then be expressed in pounds per square inch (psi) or kilopounds per square inch (ksi).† †The principal SI and U.S. customary units used in mechanics are listed in tables inside the front cover of this book. From the table on the right-hand side, we note that 1 psi is approximately equal to 7 kPa, and 1 ksi approximately equal to 7 MPa. P A A P' (a) Fig. 1.8 P' (b) Member with an axial load. 7 bee80288_ch01_002-051.indd Page 8 9/4/10 5:33:20 PM user-f499 8 Introduction—Concept of Stress /Users/user-f499/Desktop/Temp Work/Don't Delete Job/MHDQ251:Beer:201/ch01 1.4 ANALYSIS AND DESIGN Considering again the structure of Fig. 1.1, let us assume that rod BC is made of a steel with a maximum allowable stress sall 5 165 MPa. Can rod BC safely support the load to which it will be subjected? The magnitude of the force FBC in the rod was found earlier to be 50 kN. Recalling that the diameter of the rod is 20 mm, we use Eq. (1.5) to determine the stress created in the rod by the given loading. We have P 5 FBC 5 150 kN 5 150 3 103 N 20 mm 2 A 5 pr2 5 pa b 5 p110 3 1023 m2 2 5 314 3 1026 m2 2 P 150 3 103 N 6 s5 5 26 2 5 1159 3 10 Pa 5 1159 MPa A 314 3 10 m Since the value obtained for s is smaller than the value sall of the allowable stress in the steel used, we conclude that rod BC can safely support the load to which it will be subjected. To be complete, our analysis of the given structure should also include the determination of the compressive stress in boom AB, as well as an investigation of the stresses produced in the pins and their bearings. This will be discussed later in this chapter. We should also determine whether the deformations produced by the given loading are acceptable. The study of deformations under axial loads will be the subject of Chap. 2. An additional consideration required for members in compression involves the stability of the member, i.e., its ability to support a given load without experiencing a sudden change in configuration. This will be discussed in Chap. 10. The engineer’s role is not limited to the analysis of existing structures and machines subjected to given loading conditions. Of even greater importance to the engineer is the design of new structures and machines, that is, the selection of appropriate components to perform a given task. As an example of design, let us return to the structure of Fig. 1.1, and assume that aluminum with an allowable stress sall 5 100 MPa is to be used. Since the force in rod BC will still be P 5 FBC 5 50 kN under the given loading, we must have, from Eq. (1.5), sall 5 P A A5 P 50 3 103 N 5 5 500 3 1026 m2 sall 100 3 106 Pa and, since A 5 pr2, r5 A 500 3 1026 m2 5 5 12.62 3 1023 m 5 12.62 mm p Bp B d 5 2r 5 25.2 mm We conclude that an aluminum rod 26 mm or more in diameter will be adequate. bee80288_ch01_002-051.indd Page 9 9/4/10 5:33:21 PM user-f499 1.5 /Users/user-f499/Desktop/Temp Work/Don't Delete Job/MHDQ251:Beer:201/ch01 1.5 Axial Loading; Normal Stress AXIAL LOADING; NORMAL STRESS As we have already indicated, rod BC of the example considered in the preceding section is a two-force member and, therefore, the forces FBC and F9BC acting on its ends B and C (Fig. 1.5) are directed along the axis of the rod. We say that the rod is under axial loading. An actual example of structural members under axial loading is provided by the members of the bridge truss shown in Photo 1.1. Photo 1.1 This bridge truss consists of two-force members that may be in tension or in compression. Returning to rod BC of Fig. 1.5, we recall that the section we passed through the rod to determine the internal force in the rod and the corresponding stress was perpendicular to the axis of the rod; the internal force was therefore normal to the plane of the section (Fig. 1.7) and the corresponding stress is described as a normal stress. Thus, formula (1.5) gives us the normal stress in a member under axial loading: s5 P A ¢Ay0 ⌬A Q (1.5) We should also note that, in formula (1.5), s is obtained by dividing the magnitude P of the resultant of the internal forces distributed over the cross section by the area A of the cross section; it represents, therefore, the average value of the stress over the cross section, rather than the stress at a specific point of the cross section. To define the stress at a given point Q of the cross section, we should consider a small area DA (Fig. 1.9). Dividing the magnitude of DF by DA, we obtain the average value of the stress over DA. Letting DA approach zero, we obtain the stress at point Q: s 5 lim ⌬F ¢F ¢A (1.6) P' Fig. 1.9 9 bee80288_ch01_002-051.indd Page 10 9/4/10 5:33:26 PM user-f499 10 Introduction—Concept of Stress P   /Users/user-f499/Desktop/Temp Work/Don't Delete Job/MHDQ251:Beer:201/ch01 In general, the value obtained for the stress s at a given point Q of the section is different from the value of the average stress given by formula (1.5), and s is found to vary across the section. In a slender rod subjected to equal and opposite concentrated loads P and P9 (Fig. 1.10a), this variation is small in a section away from the points of application of the concentrated loads (Fig. 1.10c), but it is quite noticeable in the neighborhood of these points (Fig. 1.10b and d). It follows from Eq. (1.6) that the magnitude of the resultant of the distributed internal forces is # dF 5 # s dA  A But the conditions of equilibrium of each of the portions of rod shown in Fig. 1.10 require that this magnitude be equal to the magnitude P of the concentrated loads. We have, therefore, P' (a) P' (b) P' (c) P' (d) Fig. 1.10 Stress distributions at different sections along axially loaded member.  P C Fig. 1.11 P5 # dF 5 # s dA (1.7) A which means that the volume under each of the stress surfaces in Fig. 1.10 must be equal to the magnitude P of the loads. This, however, is the only information that we can derive from our knowledge of statics, regarding the distribution of normal stresses in the various sections of the rod. The actual distribution of stresses in any given section is statically indeterminate. To learn more about this distribution, it is necessary to consider the deformations resulting from the particular mode of application of the loads at the ends of the rod. This will be discussed further in Chap. 2. In practice, it will be assumed that the distribution of normal stresses in an axially loaded member is uniform, except in the immediate vicinity of the points of application of the loads. The value s of the stress is then equal to save and can be obtained from formula (1.5). However, we should realize that, when we assume a uniform distribution of stresses in the section, i.e., when we assume that the internal forces are uniformly distributed across the section, it follows from elementary statics† that the resultant P of the internal forces must be applied at the centroid C of the section (Fig. 1.11). This means that a uniform distribution of stress is possible only if the line of action of the concentrated loads P and P9 passes through the centroid of the section considered (Fig. 1.12). This type of loading is called centric loading and will be assumed to take place in all straight two-force members found in trusses and pin-connected structures, such as the one considered in Fig. 1.1. However, if a two-force member is loaded axially, but eccentrically as shown in Fig. 1.13a, we find from the conditions of equilibrium of the portion of member shown in Fig. 1.13b that the internal forces in a given section must be †See Ferdinand P. Beer and E. Russell Johnston, Jr., Mechanics for Engineers, 5th ed., McGraw-Hill, New York, 2008, or Vector Mechanics for Engineers, 9th ed., McGraw-Hill, New York, 2010, Secs. 5.2 and 5.3. bee80288_ch01_002-051.indd Page 11 9/4/10 5:33:30 PM user-f499 /Users/user-f499/Desktop/Temp Work/Don't Delete Job/MHDQ251:Beer:201/ch01 P 1.6 Shearing Stress P C P C d d M P' Fig. 1.12 P' equivalent to a force P applied at the centroid of the section and a couple M of moment M 5 Pd. The distribution of forces—and, thus, the corresponding distribution of stresses—cannot be uniform. Nor can the distribution of stresses be symmetric as shown in Fig. 1.10. This point will be discussed in detail in Chap. 4. 1.6 P' (a) (b) Fig. 1.13 Eccentric axial loading. SHEARING STRESS The internal forces and the corresponding stresses discussed in Secs. 1.2 and 1.3 were normal to the section considered. A very different type of stress is obtained when transverse forces P and P9 are applied to a member AB (Fig. 1.14). Passing a section at C between the points of application of the two forces (Fig. 1.15a), we obtain the diagram of portion AC shown in Fig. 1.15b. We conclude that internal forces must exist in the plane of the section, and that their resultant is equal to P. These elementary internal forces are called shearing forces, and the magnitude P of their resultant is the shear in the section. Dividing the shear P by the area A of the cross section, we P A C B P P⬘ A (a) B A P' Fig. 1.14 Member with transverse loads. C P' (b) Fig. 1.15 P 11 bee80288_ch01_002-051.indd Page 12 9/4/10 5:33:39 PM user-f499 12 /Users/user-f499/Desktop/Temp Work/Don't Delete Job/MHDQ251:Beer:201/ch01 obtain the average shearing stress in the section. Denoting the shearing stress by the Greek letter t (tau), we write Introduction—Concept of Stress tave 5 P A (1.8) It should be emphasized that the value obtained is an average value of the shearing stress over the entire section. Contrary to what we said earlier for normal stresses, the distribution of shearing stresses across the section cannot be assumed uniform. As you will see in Chap. 6, the actual value t of the shearing stress varies from zero at the surface of the member to a maximum value tmax that may be much larger than the average value tave. Photo 1.2 Cutaway view of a connection with a bolt in shear. Shearing stresses are commonly found in bolts, pins, and rivets used to connect various structural members and machine components (Photo 1.2). Consider the two plates A and B, which are connected by a bolt CD (Fig. 1.16). If the plates are subjected to tension forces of magnitude F, stresses will develop in the section of bolt corresponding to the plane EE9. Drawing the diagrams of the bolt and of the portion located above the plane EE9 (Fig. 1.17), we conclude that the shear P in the section is equal to F. The average shearing stress in the section is obtained, according to formula (1.8), by dividing the shear P 5 F by the area A of the cross section: tave 5 P F 5 A A (1.9) C C C A E F' E E⬘ B P F' E' D (a) D Fig. 1.16 F F F Bolt subject to single shear. Fig. 1.17 (b) bee80288_ch01_002-051.indd Page 13 9/4/10 5:33:44 PM user-f499 E F' /Users/user-f499/Desktop/Temp Work/Don't Delete Job/MHDQ251:Beer:201/ch01 C K K' B A L Fig. 1.18 F L' D G J Bolts subject to double shear. The bolt we have just considered is said to be in single shear. Different loading situations may arise, however. For example, if splice plates C and D are used to connect plates A and B (Fig. 1.18), shear will take place in bolt HJ in each of the two planes KK9 and LL9 (and similarly in bolt EG). The bolts are said to be in double shear. To determine the average shearing stress in each plane, we draw free-body diagrams of bolt HJ and of the portion of bolt located between the two planes (Fig. 1.19). Observing that the shear P in each of the sections is P 5 Fy2, we conclude that the average shearing stress is tave 5 1.7 Fy2 P F 5 5 A A 2A H FC F K P K' L F L' P FD J (a) (b) Fig. 1.19 (1.10) BEARING STRESS IN CONNECTIONS Bolts, pins, and rivets create stresses in the members they connect, along the bearing surface, or surface of contact. For example, consider again the two plates A and B connected by a bolt CD that we have discussed in the preceding section (Fig. 1.16). The bolt exerts on plate A a force P equal and opposite to the force F exerted by the plate on the bolt (Fig. 1.20). The force P represents the resultant of elementary forces distributed on the inside surface of a halfcylinder of diameter d and of length t equal to the thickness of the plate. Since the distribution of these forces—and of the corresponding stresses—is quite complicated, one uses in practice an average nominal value sb of the stress, called the bearing stress, obtained by dividing the load P by the area of the rectangle representing the projection of the bolt on the plate section (Fig. 1.21). Since this area is equal to td, where t is the plate thickness and d the diameter of the bolt, we have sb 5 P P 5 A td t A F Fig. 1.20 t A We are now in a position to determine the stresses in the members and connections of various simple two-dimensional structures and, thus, to design such structures. d F' D (1.11) APPLICATION TO THE ANALYSIS AND DESIGN OF SIMPLE STRUCTURES C P Fig. 1.21 1.8 13 1.8 Application to the Analysis and Design of Simple Structures H d bee80288_ch01_002-051.indd Page 14 9/4/10 5:33:51 PM user-f499 14 Introduction—Concept of Stress /Users/user-f499/Desktop/Temp Work/Don't Delete Job/MHDQ251:Beer:201/ch01 As an example, let us return to the structure of Fig. 1.1 that we have already considered in Sec. 1.2 and let us specify the supports and connections at A, B, and C. As shown in Fig. 1.22, the 20-mmdiameter rod BC has flat ends of 20 3 40-mm rectangular cross section, while boom AB has a 30 3 50-mm rectangular cross section and is fitted with a clevis at end B. Both members are connected at B by a pin from which the 30-kN load is suspended by means of a U-shaped bracket. Boom AB is supported at A by a pin fitted into a double bracket, while rod BC is connected at C to a single bracket. All pins are 25 mm in diameter. d ⫽ 25 mm C 20 mm Flat end TOP VIEW OF ROD BC 40 mm d ⫽ 20 mm C d ⫽ 20 mm 600 mm d ⫽ 25 mm FRONT VIEW B Flat end 50 mm A B B 800 mm Q ⫽ 30 kN Q ⫽ 30 kN END VIEW 25 mm 20 mm 30 mm 25 mm A TOP VIEW OF BOOM AB 20 mm B d ⫽ 25 mm Fig. 1.22 a. Determination of the Normal Stress in Boom AB and Rod BC. As we found in Secs. 1.2 and 1.4, the force in rod BC is FBC 5 50 kN (tension) and the area of its circular cross section is A 5 314 3 1026 m2; the corresponding average normal stress is sBC 5 1159 MPa. However, the flat parts of the rod are also under tension and at the narrowest section, where a hole is located, we have A 5 120 mm2 140 mm 2 25 mm2 5 300 3 10 26 m2 bee80288_ch01_002-051.indd Page 15 11/1/10 4:54:27 PM user-f499/Users/user-f499/Desktop/Temp user-f499/Users/user-f499/Desktop/Temp Work/Don't Delete Job/MHDQ251:Beer:201/ch 1.8 Application to the Analysis and Design of Simple Structures The corresponding average value of the stress, therefore, is 1sBC 2 end 5 P 50 3 103 N 5 5 167 MPa A 300 3 1026 m2 Note that this is an average value; close to the hole, the stress will actually reach a much larger value, as you will see in Sec. 2.18. It is clear that, under an increasing load, the rod will fail near one of the holes rather than in its cylindrical portion; its design, therefore, could be improved by increasing the width or the thickness of the flat ends of the rod. Turning now our attention to boom AB, we recall from Sec. 1.2 that the force in the boom is FAB 5 40 kN (compression). Since the area of the boom’s rectangular cross section is A 5 30 mm 3 50 mm 5 1.5 3 1023 m2, the average value of the normal stress in the main part of the rod, between pins A and B, is C 50 kN (a) d ⫽ 25 mm 3 sAB 5 2 15 40 3 10 N 5 226.7 3 106 Pa 5 226.7 MPa 1.5 3 1023 m2 Note that the sections of minimum area at A and B are not under stress, since the boom is in compression, and, therefore, pushes on the pins (instead of pulling on the pins as rod BC does). 50 kN D P 50 kN D' Fb (c) (b) Fig. 1.23 b. Determination of the Shearing Stress in Various Connections. To determine the shearing stress in a connection such as a bolt, pin, or rivet, we first clearly show the forces exerted by the various members it connects. Thus, in the case of pin C of our example (Fig. 1.23a), we draw Fig. 1.23b, showing the 50-kN force exerted by member BC on the pin, and the equal and opposite force exerted by the bracket. Drawing now the diagram of the portion of the pin located below the plane DD9 where shearing stresses occur (Fig. 1.23c), we conclude that the shear in that plane is P 5 50 kN. Since the cross-sectional area of the pin is A 40 kN 25 mm 2 b 5 p112.5 3 1023 m2 2 5 491 3 1026 m2 A 5 pr2 5 pa 2 we find that the average value of the shearing stress in the pin at C is P 50 3 103 N 5 102 MPa tave 5 5 A 491 3 1026 m2 Considering now the pin at A (Fig. 1.24), we note that it is in double shear. Drawing the free-body diagrams of the pin and of the portion of pin located between the planes DD9 and EE9 where shearing stresses occur, we conclude that P 5 20 kN and that tave 5 P 20 kN 5 5 40.7 MPa A 491 3 1026 m2 (a) d ⫽ 25 mm Fb Fb D D' E E' (b) Fig. 1.24 P 40 kN 40 kN P (c) bee80288_ch01_002-051.indd Page 16 9/4/10 5:33:58 PM user-f499 16 Introduction—Concept of Stress 1 2 FAB ⫽ 1 2 FAB ⫽ 20 kN J 20 kN Pin B 1 2Q E D ⫽ 15 kN H G 1 2Q ⫽ 15 kN /Users/user-f499/Desktop/Temp Work/Don't Delete Job/MHDQ251:Beer:201/ch01 Considering the pin at B (Fig. 1.25a), we note that the pin may be divided into five portions which are acted upon by forces exerted by the boom, rod, and bracket. Considering successively the portions DE (Fig. 1.25b) and DG (Fig. 1.25c), we conclude that the shear in section E is PE 5 15 kN, while the shear in section G is PG 5 25 kN. Since the loading of the pin is symmetric, we conclude that the maximum value of the shear in pin B is PG 5 25 kN, and that the largest shearing stresses occur in sections G and H, where tave 5 FBC ⫽ 50 kN PG 25 kN 5 5 50.9 MPa A 491 3 1026 m2 (a) c. Determination of the Bearing Stresses. To determine the nominal bearing stress at A in member AB, we use formula (1.11) of Sec. 1.7. From Fig. 1.22, we have t 5 30 mm and d 5 25 mm. Recalling that P 5 FAB 5 40 kN, we have PE E D sb 5 1 2Q ⫽ 15 kN To obtain the bearing stress in the bracket at A, we use t 5 2(25 mm) 5 50 mm and d 5 25 mm: (b) 1 2 FAB ⫽ 20 kN sb 5 G D 1 2Q P 40 kN 5 53.3 MPa 5 td 130 mm2 125 mm2 PG P 40 kN 5 5 32.0 MPa td 150 mm2 125 mm2 The bearing stresses at B in member AB, at B and C in member BC, and in the bracket at C are found in a similar way. ⫽ 15 kN (c) Fig. 1.25 1.9 METHOD OF PROBLEM SOLUTION You should approach a problem in mechanics of materials as you would approach an actual engineering situation. By drawing on your own experience and intuition, you will find it easier to understand and formulate the problem. Once the problem has been clearly stated, however, there is no place in its solution for your particular fancy. Your solution must be based on the fundamental principles of statics and on the principles you will learn in this course. Every step you take must be justified on that basis, leaving no room for your “intuition.” After an answer has been obtained, it should be checked. Here again, you may call upon your common sense and personal experience. If not completely satisfied with the result obtained, you should carefully check your formulation of the problem, the validity of the methods used in its solution, and the accuracy of your computations. The statement of the problem should be clear and precise. It should contain the given data and indicate what information is required. A simplified drawing showing all essential quantities involved should be included. The solution of most of the problems you will encounter will necessitate that you first determine the reactions at supports and internal forces and couples. This will require bee80288_ch01_002-051.indd Page 17 9/4/10 5:34:00 PM user-f499 /Users/user-f499/Desktop/Temp Work/Don't Delete Job/MHDQ251:Beer:201/ch01 the drawing of one or several free-body diagrams, as was done in Sec. 1.2, from which you will write equilibrium equations. These equations can be solved for the unknown forces, from which the required stresses and deformations will be computed. After the answer has been obtained, it should be carefully checked. Mistakes in reasoning can often be detected by carrying the units through your computations and checking the units obtained for the answer. For example, in the design of the rod discussed in Sec. 1.4, we found, after carrying the units through our computations, that the required diameter of the rod was expressed in millimeters, which is the correct unit for a dimension; if another unit had been found, we would have known that some mistake had been made. Errors in computation will usually be found by substituting the numerical values obtained into an equation which has not yet been used and verifying that the equation is satisfied. The importance of correct computations in engineering cannot be overemphasized. 1.10 NUMERICAL ACCURACY The accuracy of the solution of a problem depends upon two items: (1) the accuracy of the given data and (2) the accuracy of the computations performed. The solution cannot be more accurate than the less accurate of these two items. For example, if the loading of a beam is known to be 75,000 lb with a possible error of 100 lb either way, the relative error which measures the degree of accuracy of the data is 100 lb 5 0.0013 5 0.13% 75,000 lb In computing the reaction at one of the beam supports, it would then be meaningless to record it as 14,322 lb. The accuracy of the solution cannot be greater than 0.13%, no matter how accurate the computations are, and the possible error in the answer may be as large as (0.13y100)(14,322 lb) < 20 lb. The answer should be properly recorded as 14,320 6 20 lb. In engineering problems, the data are seldom known with an accuracy greater than 0.2%. It is therefore seldom justified to write the answers to such problems with an accuracy greater than 0.2%. A practical rule is to use 4 figures to record numbers beginning with a “1” and 3 figures in all other cases. Unless otherwise indicated, the data given in a problem should be assumed known with a comparable degree of accuracy. A force of 40 lb, for example, should be read 40.0 lb, and a force of 15 lb should be read 15.00 lb. Pocket calculators and computers are widely used by practicing engineers and engineering students. The speed and accuracy of these devices facilitate the numerical computations in the solution of many problems. However, students should not record more significant figures than can be justified merely because they are easily obtained. As noted above, an accuracy greater than 0.2% is seldom necessary or meaningful in the solution of practical engineering problems. 1.10 Numerical Accuracy 17 bee80288_ch01_002-051.indd Page 18 11/1/10 4:54:32 PM user-f499/Users/user-f499/Desktop/Temp user-f499/Users/user-f499/Desktop/Temp Work/Don't Delete Job/MHDQ251:Beer:201/ch SAMPLE PROBLEM 1.1 D A 1.25 in. B 6 in. In the hanger shown, the upper portion of link ABC is 38 in. thick and the lower portions are each 14 in. thick. Epoxy resin is used to bond the upper and lower portions together at B. The pin at A is of 38-in. diameter while a 1 4 -in.-diameter pin is used at C. Determine (a) the shearing stress in pin A, (b) the shearing stress in pin C, (c) the largest normal stress in link ABC, (d) the average shearing stress on the bonded surfaces at B, (e) the bearing stress in the link at C. 1.75 in. 7 in. C E SOLUTION 10 in. 500 lb Free Body: Entire Hanger. Since the link ABC is a two-force member, the reaction at A is vertical; the reaction at D is represented by its components Dx and Dy. We write 5 in. Dy FAC A D 1l oMD 5 0: 1500 lb2 115 in.2 2 FAC 110 in.2 5 0 FAC 5 1750 lb FAC 5 750 lb tension Dx a. Shearing Stress in Pin A. Since this 38-in.-diameter pin is in single shear, we write 5 in. 10 in. tA 5 E FAC 750 lb 51 A p10.375 in.2 2 4 tA 5 6790 psi ◀ C b. Shearing Stress in Pin C. shear, we write 500 lb 750 lb FAC ⫽ 750 lb FAC ⫽ 750 lb C A 3 8 tC 5 1 2 -in. diameter 1 4 FAC ⫽ 375 lb 3 8 in. A 5 375 lb in.2 2 1 4 p 10.25 c. Largest Normal Stress in Link ABC. 1 2 -in. diameter 1 2 FAC Since this 14-in.-diameter pin is in double tC 5 7640 psi ◀ The largest stress is found FAC ⫽ 375 lb where the area is smallest; this occurs at the cross section at A where the 38-in. FAC ⫽ 750 lb 1.25 in. hole is located. We have sA 5 FAC 750 lb 750 lb 5 3 5 Anet 1 8 in.2 11.25 in. 2 0.375 in.2 0.328 in2 sA 5 2290 psi ◀ 1.25 in. 3 8 A -in. diameter B FAC F1 ⫽ F2 ⫽ 12 FAC ⫽ 375 lb 375 lb F2 1.75 in. F1 d. Average Shearing Stress at B. We note that bonding exists on both sides of the upper portion of the link and that the shear force on each side is F1 5 (750 lb)y2 5 375 lb. The average shearing stress on each surface is thus tB 5 F1 375 lb 5 A 11.25 in.2 11.75 in.2 F1 ⫽ 375 lb 1 4 in. e. Bearing Stress in Link at C. For each portion of the link, F1 5 375 lb and the nominal bearing area is (0.25 in.)(0.25 in.) 5 0.0625 in2. sb 5 1 4 18 tB 5 171.4 psi ◀ -in. diameter F1 375 lb 5 A 0.0625 in2 sb 5 6000 psi ◀ bee80288_ch01_002-051.indd Page 19 11/1/10 A 4:54:44 PM user-f499/Users/user-f499/Desktop/Temp user-f499/Users/user-f499/Desktop/Temp Work/Don't Delete Job/MHDQ251:Beer:201/ch SAMPLE PROBLEM 1.2 B The steel tie bar shown is to be designed to carry a tension force of magnitude P 5 120 kN when bolted between double brackets at A and B. The bar will be fabricated from 20-mm-thick plate stock. For the grade of steel to be used, the maximum allowable stresses are: s 5 175 MPa, t 5 100 MPa, sb 5 350 MPa. Design the tie bar by determining the required values of (a) the diameter d of the bolt, (b) the dimension b at each end of the bar, (c) the dimension h of the bar. F1 F1 SOLUTION d F1  a. Diameter of the Bolt. Since the bolt is in double shear, F1 5 12 P 5 P 60 kN. 1 P 2 t  20 mm h t5 F1 60 kN 5 1 2 A 4p d 100 MPa 5 60 kN 1 4p d 5 27.6 mm d2 We will use d 5 28 mm ◀ At this point we check the bearing stress between the 20-mm-thick plate and the 28-mm-diameter bolt. d b tb 5 t a b d a 1 2 P P'  120 kN 1 2 P P 120 kN 5 5 214 MPa , 350 MPa 10.020 m2 10.028 m2 td b. Dimension b at Each End of the Bar. We consider one of the end portions of the bar. Recalling that the thickness of the steel plate is t 5 20 mm and that the average tensile stress must not exceed 175 MPa, we write s5 1 2P ta 175 MPa 5 60 kN 10.02 m2a a 5 17.14 mm b 5 d 1 2a 5 28 mm 1 2(17.14 mm) t  20 mm b 5 62.3 mm ◀ c. Dimension h of the Bar. Recalling that the thickness of the steel plate is t 5 20 mm, we have s5 P  120 kN h OK P th 175 MPa 5 120 kN 10.020 m2h h 5 34.3 mm We will use h 5 35 mm ◀ 19 bee80288_ch01_002-051.indd Page 20 11/1/10 4:54:55 PM user-f499/Users/user-f499/Desktop/Temp user-f499/Users/user-f499/Desktop/Temp Work/Don't Delete Job/MHDQ251:Beer:201/ch PROBLEMS 1.1 Two solid cylindrical rods AB and BC are welded together at B and loaded as shown. Knowing that the average normal stress must not exceed 175 MPa in rod AB and 150 MPa in rod BC, determine the smallest allowable values of d1 and d2. A 300 mm d1 B 40 kN 250 mm d2 1.2 Two solid cylindrical rods AB and BC are welded together at B and loaded as shown. Knowing that d1 5 50 mm and d2 5 30 mm, find the average normal stress at the midsection of (a) rod AB, (b) rod BC. 1.3 Two solid cylindrical rods AB and BC are welded together at B and loaded as shown. Determine the magnitude of the force P for which the tensile stress in rod AB has the same magnitude as the compressive stress in rod BC. C 2 in. 30 kN 30 kips B 3 in. C A Fig. P1.1 and P1.2 P 30 kips 30 in. 40 in. Fig. P1.3 1.4 In Prob. 1.3, knowing that P 5 40 kips, determine the average normal stress at the midsection of (a) rod AB, (b) rod BC. 1.5 Two steel plates are to be held together by means of 16-mm-diameter high-strength steel bolts fitting snugly inside cylindrical brass spacers. Knowing that the average normal stress must not exceed 200 MPa in the bolts and 130 MPa in the spacers, determine the outer diameter of the spacers that yields the most economical and safe design. A a 15 mm B Fig. P1.5 100 m b 10 mm C Fig. P1.6 20 1.6 Two brass rods AB and BC, each of uniform diameter, will be brazed together at B to form a nonuniform rod of total length 100 m which will be suspended from a support at A as shown. Knowing that the density of brass is 8470 kg/m3, determine (a) the length of rod AB for which the maximum normal stress in ABC is minimum, (b) the corresponding value of the maximum normal stress. bee80288_ch01_002-051.indd Page 21 9/4/10 5:34:29 PM user-f499 /Users/user-f499/Desktop/Temp Work/Don't Delete Job/MHDQ251:Beer:201/ch01 21 Problems 1.7 Each of the four vertical links has an 8 3 36-mm uniform rectangular cross section and each of the four pins has a 16-mm diameter. Determine the maximum value of the average normal stress in the links connecting (a) points B and D, (b) points C and E. 0.4 m C 0.25 m 0.2 m B E 20 kN D A 4 in. 4 in. 12 in. E 2 in. 1.8 Knowing that link DE is 18 in. thick and 1 in. wide, determine the normal stress in the central portion of that link when (a) u 5 0, (b) u 5 908. 1.9 Link AC has a uniform rectangular cross section 161 in. thick and 1 4 in. wide. Determine the normal stress in the central portion of the link. 240 lb B 6 in. 7 in. A 30 240 lb C Fig. P1.9 1.10 Three forces, each of magnitude P 5 4 kN, are applied to the mechanism shown. Determine the cross-sectional area of the uniform portion of rod BE for which the normal stress in that portion is 1100 MPa. 0.100 m E P A 0.150 m Fig. P1.10 D C Fig. P1.7 3 in. B B P P C 0.300 m 0.250 m D 8 in. J 6 in. D A F 60 lb Fig. P1.8  bee80288_ch01_002-051.indd Page 22 9/4/10 5:34:39 PM user-f499 22 Introduction—Concept of Stress /Users/user-f499/Desktop/Temp Work/Don't Delete Job/MHDQ251:Beer:201/ch01 1.11 The frame shown consists of four wooden members, ABC, DEF, BE, and CF. Knowing that each member has a 2 3 4-in. rectangular cross section and that each pin has a 12-in. diameter, determine the maximum value of the average normal stress (a) in member BE, (b) in member CF. 45 in. A 30 in. B C 480 lb 4 in. 40 in. D 15 in. E 4 in. F 30 in. Fig. P1.11 1.12 For the Pratt bridge truss and loading shown, determine the average normal stress in member BE, knowing that the cross-sectional area of that member is 5.87 in2. D B F 12 ft H A C 9 ft E G 9 ft 80 kips 9 ft 80 kips 9 ft 80 kips Fig. P1.12 1.13 An aircraft tow bar is positioned by means of a single hydraulic cylinder connected by a 25-mm-diameter steel rod to two identical arm-and-wheel units DEF. The mass of the entire tow bar is 200 kg, and its center of gravity is located at G. For the position shown, determine the normal stress in the rod. Dimensions in mm 1150 A 100 C G F 850 Fig. P1.13 D B 250 E 500 450 675 825 bee80288_ch01_002-051.indd Page 23 11/1/10 4:55:04 PM user-f499/Users/user-f499/Desktop/Temp user-f499/Users/user-f499/Desktop/Temp Work/Don't Delete Job/MHDQ251:Beer:201/ch Problems 1.14 A couple M of magnitude 1500 N ? m is applied to the crank of an engine. For the position shown, determine (a) the force P required to hold the engine system in equilibrium, (b) the average normal stress in the connecting rod BC, which has a 450-mm2 uniform cross section. P 1.15 When the force P reached 8 kN, the wooden specimen shown failed in shear along the surface indicated by the dashed line. Determine the average shearing stress along that surface at the time of failure. C 200 mm 15 mm B P P' Steel 90 mm 80 mm M A Wood Fig. P1.15 1.16 The wooden members A and B are to be joined by plywood splice plates that will be fully glued on the surfaces in contact. As part of the design of the joint, and knowing that the clearance between the ends of the members is to be 14 in., determine the smallest allowable length L if the average shearing stress in the glue is not to exceed 120 psi. 1.17 A load P is applied to a steel rod supported as shown by an aluminum plate into which a 0.6-in.-diameter hole has been drilled. Knowing that the shearing stress must not exceed 18 ksi in the steel rod and 10 ksi in the aluminum plate, determine the largest load P that can be applied to the rod. 60 mm Fig. P1.14 5.8 kips A 1 4 L in. 1.6 in. 4 in. B 0.4 in. 0.25 in. 5.8 kips 0.6 in. Fig. P1.16 P Fig. P1.17 1.18 Two wooden planks, each 22 mm thick and 160 mm wide, are joined by the glued mortise joint shown. Knowing that the joint will fail when the average shearing stress in the glue reaches 820 kPa, determine the smallest allowable length d of the cuts if the joint is to withstand an axial load of magnitude P 5 7.6 kN. d P' 20 0 mm Glue ue 160 mm 160 m 20 mm 20 Fig. P1.18 P 23 bee80288_ch01_002-051.indd Page 24 9/4/10 5:34:58 PM user-f499 24 /Users/user-f499/Desktop/Temp Work/Don't Delete Job/MHDQ251:Beer:201/ch01 1.19 The load P applied to a steel rod is distributed to a timber support by an annular washer. The diameter of the rod is 22 mm and the inner diameter of the washer is 25 mm, which is slightly larger than the diameter of the hole. Determine the smallest allowable outer diameter d of the washer, knowing that the axial normal stress in the steel rod is 35 MPa and that the average bearing stress between the washer and the timber must not exceed 5 MPa. Introduction—Concept of Stress d 22 mm P 1.20 The axial force in the column supporting the timber beam shown is P 5 20 kips. Determine the smallest allowable length L of the bearing plate if the bearing stress in the timber is not to exceed 400 psi. Fig. P1.19 L 6 in. a P a P Fig. P1.20 1.21 An axial load P is supported by a short W8 3 40 column of crosssectional area A 5 11.7 in2 and is distributed to a concrete foundation by a square plate as shown. Knowing that the average normal stress in the column must not exceed 30 ksi and that the bearing stress on the concrete foundation must not exceed 3.0 ksi, determine the side a of the plate that will provide the most economical and safe design. Fig. P1.21 P  40 kN 120 mm b Fig. P1.22 100 mm b 1.22 A 40-kN axial load is applied to a short wooden post that is supported by a concrete footing resting on undisturbed soil. Determine (a) the maximum bearing stress on the concrete footing, (b) the size of the footing for which the average bearing stress in the soil is 145 kPa. 1.23 A 58-in.-diameter steel rod AB is fitted to a round hole near end C of the wooden member CD. For the loading shown, determine (a) the maximum average normal stress in the wood, (b) the distance b for which the average shearing stress is 100 psi on the surfaces indicated by the dashed lines, (c) the average bearing stress on the wood. 1500 lb 1 in. 750 lb A 4 in. D 750 lb B C b Fig. P1.23 bee80288_ch01_002-051.indd Page 25 9/6/10 7:27:03 PM user-f499 /Users/user-f499/Desktop/Temp Work/Don't Delete Job/MHDQ251:Beer:201/ch01 Problems 1.24 Knowing that u 5 408 and P 5 9 kN, determine (a) the smallest allowable diameter of the pin at B if the average shearing stress in the pin is not to exceed 120 MPa, (b) the corresponding average bearing stress in member AB at B, (c) the corresponding average bearing stress in each of the support brackets at B. P A 16 mm 750 mm 750 mm ␪ 50 mm B C 12 mm A Fig. P1.24 and P1.25 d 1.25 Determine the largest load P that can be applied at A when u 5 608, knowing that the average shearing stress in the 10-mm-diameter pin at B must not exceed 120 MPa and that the average bearing stress in member AB and in the bracket at B must not exceed 90 MPa. 1.26 Link AB, of width b 5 50 mm and thickness t 5 6 mm, is used to support the end of a horizontal beam. Knowing that the average normal stress in the link is 2140 MPa, and that the average shearing stress in each of the two pins is 80 MPa, determine (a) the diameter d of the pins, (b) the average bearing stress in the link. 1.27 For the assembly and loading of Prob. 1.7, determine (a) the average shearing stress in the pin at B, (b) the average bearing stress at B in member BD, (c) the average bearing stress at B in member ABC, knowing that this member has a 10 3 50-mm uniform rectangular cross section. 1.28 The hydraulic cylinder CF, which partially controls the position of rod DE, has been locked in the position shown. Member BD is 5 3 8 in. thick and is connected to the vertical rod by a 8 -in.-diameter bolt. Determine (a) the average shearing stress in the bolt, (b) the bearing stress at C in member BD. 4 in. 7 in. D B 20⬚ C 8 in. 75⬚ E 400 lb A F 1.8 in. Fig. P1.28 b t B d Fig. P1.26 25 bee80288_ch01_002-051.indd Page 26 9/4/10 5:35:14 PM user-f499 26 1.11 Introduction—Concept of Stress P' P (a) P' /Users/user-f499/Desktop/Temp Work/Don't Delete Job/MHDQ251:Beer:201/ch01 P  P' STRESS ON AN OBLIQUE PLANE UNDER AXIAL LOADING In the preceding sections, axial forces exerted on a two-force member (Fig. 1.26a) were found to cause normal stresses in that member (Fig. 1.26b), while transverse forces exerted on bolts and pins (Fig. 1.27a) were found to cause shearing stresses in those connections (Fig. 1.27b). The reason such a relation was observed between axial forces and normal stresses on one hand, and transverse forces and shearing stresses on the other, was because stresses were being determined only on planes perpendicular to the axis of the member or connection. As you will see in this section, axial forces cause both normal and shearing stresses on planes which are not perpendicular to the axis of the member. Similarly, transverse forces exerted on a bolt or a pin cause both normal and shearing stresses on planes which are not perpendicular to the axis of the bolt or pin. (b) Fig. 1.26 Axial forces. P P  P' P' (a) Fig. 1.27 P' P  (a) P' P A A0 F  P' (c) V  P'  (d) Fig. 1.28 (b) Transverse forces. Consider the two-force member of Fig. 1.26, which is subjected to axial forces P and P9. If we pass a section forming an angle u with a normal plane (Fig. 1.28a) and draw the free-body diagram of the portion of member located to the left of that section (Fig. 1.28b), we find from the equilibrium conditions of the free body that the distributed forces acting on the section must be equivalent to the force P. Resolving P into components F and V, respectively normal and tangential to the section (Fig. 1.28c), we have F 5 P cos u (b) P P' V 5 P sin u (1.12) The force F represents the resultant of normal forces distributed over the section, and the force V the resultant of shearing forces (Fig. 1.28d). The average values of the corresponding normal and shearing stresses are obtained by dividing, respectively, F and V by the area Au of the section: s5 F Au t5 V Au (1.13) Substituting for F and V from (1.12) into (1.13), and observing from Fig. 1.28c that A0 5 Au cos u, or Au 5 A0ycos u, where A0 denotes bee80288_ch01_002-051.indd Page 27 9/4/10 5:35:20 PM user-f499 /Users/user-f499/Desktop/Temp Work/Don't Delete Job/MHDQ251:Beer:201/ch01 the area of a section perpendicular to the axis of the member, we obtain s5 P cos u A0ycos u t5 1.12 Stress Under General Loading Conditions; Components of Stress P sin u A0ycos u or s5 P cos2 u A0 t5 P sin u cos u A0 (1.14) We note from the first of Eqs. (1.14) that the normal stress s is maximum when u 5 0, i.e., when the plane of the section is perpendicular to the axis of the member, and that it approaches zero as u approaches 908. We check that the value of s when u 5 0 is sm 5 P A0 P' (a) Axial loading (1.15) m ⫽ P/A0 as we found earlier in Sec. 1.3. The second of Eqs. (1.14) shows that the shearing stress t is zero for u 5 0 and u 5 908, and that for u 5 458 it reaches its maximum value P P tm 5 sin 45° cos 45° 5 A0 2A0 (b) Stresses for  ⫽ 0  ' ⫽ P/2A0 (1.16)  m ⫽ P/2A0 The first of Eqs. (1.14) indicates that, when u 5 458, the normal stress s9 is also equal to Py2A0: s¿ 5 P P cos2 45° 5 A0 2A0 P (c) Stresses for  ⫽ 45° m ⫽ P/2A0 (1.17) The results obtained in Eqs. (1.15), (1.16), and (1.17) are shown graphically in Fig. 1.29. We note that the same loading may produce either a normal stress sm 5 PyA0 and no shearing stress (Fig. 1.29b), or a normal and a shearing stress of the same magnitude s9 5 tm 5 Py2A0 (Fig. 1.29 c and d), depending upon the orientation of the section.  '⫽ P/2A0 (d) Stresses for  ⫽ –45° Fig. 1.29 y 1.12 STRESS UNDER GENERAL LOADING CONDITIONS; COMPONENTS OF STRESS The examples of the previous sections were limited to members under axial loading and connections under transverse loading. Most structural members and machine components are under more involved loading conditions. Consider a body subjected to several loads P1, P2, etc. (Fig. 1.30). To understand the stress condition created by these loads at some point Q within the body, we shall first pass a section through Q, using a plane parallel to the yz plane. The portion of the body to the left of the section is subjected to some of the original loads, and to normal and shearing forces distributed over the section. We shall denote by DFx and DVx, respectively, the normal and the shearing P2 P3 P1 P4 x z Fig. 1.30 27 bee80288_ch01_002-051.indd Page 28 9/4/10 5:35:25 PM user-f499 28 /Users/user-f499/Desktop/Temp Work/Don't Delete Job/MHDQ251:Beer:201/ch01 P2 y Introduction—Concept of Stress y P2 Vxy A Vxz x V  Fx Q Q P1  Fx P1 x x z z (a) (b) Fig. 1.31 forces acting on a small area DA surrounding point Q (Fig. 1.31a). Note that the superscript x is used to indicate that the forces DFx and DVx act on a surface perpendicular to the x axis. While the normal force DF x has a well-defined direction, the shearing force DV x may have any direction in the plane of the section. We therefore resolve DVx into two component forces, DV yx and DV zx, in directions parallel to the y and z axes, respectively (Fig. 1.31b). Dividing now the magnitude of each force by the area DA, and letting DA approach zero, we define the three stress components shown in Fig. 1.32: y xy xz x Q sx 5 lim ¢AS0 x z txy 5 lim ¢AS0 Fig. 1.32 y xz Q x xy x z Fig. 1.33 ¢Vyx ¢A ¢F x ¢A (1.18) ¢Vzx txz 5 lim ¢AS0 ¢A We note that the first subscript in sx, txy, and txz is used to indicate that the stresses under consideration are exerted on a surface perpendicular to the x axis. The second subscript in txy and txz identifies the direction of the component. The normal stress sx is positive if the corresponding arrow points in the positive x direction, i.e., if the body is in tension, and negative otherwise. Similarly, the shearing stress components txy and txz are positive if the corresponding arrows point, respectively, in the positive y and z directions. The above analysis may also be carried out by considering the portion of body located to the right of the vertical plane through Q (Fig. 1.33). The same magnitudes, but opposite directions, are obtained for the normal and shearing forces DF x, DV yx, and DV xz. Therefore, the same values are also obtained for the corresponding stress components, but since the section in Fig. 1.33 now faces the negative x axis, a positive sign for sx will indicate that the corresponding arrow points in the negative x direction. Similarly, positive signs for txy and txz will indicate that the corresponding arrows point, respectively, in the negative y and z directions, as shown in Fig. 1.33. bee80288_ch01_002-051.indd Page 29 9/4/10 5:35:32 PM user-f499 /Users/user-f499/Desktop/Temp Work/Don't Delete Job/MHDQ251:Beer:201/ch01 Passing a section through Q parallel to the zx plane, we define in the same manner the stress components, sy, tyz, and tyx. Finally, a section through Q parallel to the xy plane yields the components sz, tzx, and tzy. To facilitate the visualization of the stress condition at point Q, we shall consider a small cube of side a centered at Q and the stresses exerted on each of the six faces of the cube (Fig. 1.34). The stress components shown in the figure are sx, sy, and sz, which represent the normal stress on faces respectively perpendicular to the x, y, and z axes, and the six shearing stress components txy, txz, etc. We recall that, according to the definition of the shearing stress components, txy represents the y component of the shearing stress exerted on the face perpendicular to the x axis, while tyx represents the x component of the shearing stress exerted on the face perpendicular to the y axis. Note that only three faces of the cube are actually visible in Fig. 1.34, and that equal and opposite stress components act on the hidden faces. While the stresses acting on the faces of the cube differ slightly from the stresses at Q, the error involved is small and vanishes as side a of the cube approaches zero. Important relations among the shearing stress components will now be derived. Let us consider the free-body diagram of the small cube centered at point Q (Fig. 1.35). The normal and shearing forces acting on the various faces of the cube are obtained by multiplying the corresponding stress components by the area DA of each face. We first write the following three equilibrium equations: oFx 5 0 oFy 5 0 oFz 5 0 (1.19) 1.12 Stress Under General Loading Conditions; Components of Stress y y a a yx yz xy zy Q x  z zx xz a z x Fig. 1.34 y y  A yz  A zy  A yx A xy A Q z  A zx  A xA xz A z x Fig. 1.35 Since forces equal and opposite to the forces actually shown in Fig. 1.35 are acting on the hidden faces of the cube, it is clear that Eqs. (1.19) are satisfied. Considering now the moments of the forces about axes x9, y9, and z9 drawn from Q in directions respectively parallel to the x, y, and z axes, we write the three additional equations oM x¿ 5 0 oM y¿ 5 0 oM z¿ 5 0 (1.20) Using a projection on the x9y9 plane (Fig. 1.36), we note that the only forces with moments about the z axis different from zero are the shearing forces. These forces form two couples, one of counterclockwise (positive) moment (txy DA)a, the other of clockwise (negative) moment 2(tyx DA)a. The last of the three Eqs. (1.20) yields, therefore, 1l oMz 5 0: (txy DA)a 2 (tyx DA)a 5 0 y' y A x A xy A yx A from which we conclude that Fig. 1.36 txy 5 tyx (1.21) The relation obtained shows that the y component of the shearing stress exerted on a face perpendicular to the x axis is equal to the x yx A xy A z' a x A y A x' 29 bee80288_ch01_002-051.indd Page 30 9/4/10 5:35:38 PM user-f499 30 component of the shearing stress exerted on a face perpendicular to the y axis. From the remaining two equations (1.20), we derive in a similar manner the relations Introduction—Concept of Stress  P Q  P' (b) Fig. 1.37 y P' P x x ⫽ P A z (a) P' ' ' 45 m ⫽ P 2A ' m ' ⫽ P 2A (b) Fig. 1.38 tyz 5 tzy   (a) P /Users/user-f499/Desktop/Temp Work/Don't Delete Job/MHDQ251:Beer:201/ch01 x tzx 5 txz (1.22) We conclude from Eqs. (1.21) and (1.22) that only six stress components are required to define the condition of stress at a given point Q, instead of nine as originally assumed. These six components are sx, sy, sz, txy, tyz, and tzx. We also note that, at a given point, shear cannot take place in one plane only; an equal shearing stress must be exerted on another plane perpendicular to the first one. For example, considering again the bolt of Fig. 1.27 and a small cube at the center Q of the bolt (Fig. 1.37a), we find that shearing stresses of equal magnitude must be exerted on the two horizontal faces of the cube and on the two faces that are perpendicular to the forces P and P9 (Fig. 1.37b). Before concluding our discussion of stress components, let us consider again the case of a member under axial loading. If we consider a small cube with faces respectively parallel to the faces of the member and recall the results obtained in Sec. 1.11, we find that the conditions of stress in the member may be described as shown in Fig. 1.38a; the only stresses are normal stresses sx exerted on the faces of the cube which are perpendicular to the x axis. However, if the small cube is rotated by 458 about the z axis so that its new orientation matches the orientation of the sections considered in Fig. 1.29c and d, we conclude that normal and shearing stresses of equal magnitude are exerted on four faces of the cube (Fig. 1.38b). We thus observe that the same loading condition may lead to different interpretations of the stress situation at a given point, depending upon the orientation of the element considered. More will be said about this in Chap 7. 1.13 DESIGN CONSIDERATIONS In the preceding sections you learned to determine the stresses in rods, bolts, and pins under simple loading conditions. In later chapters you will learn to determine stresses in more complex situations. In engineering applications, however, the determination of stresses is seldom an end in itself. Rather, the knowledge of stresses is used by engineers to assist in their most important task, namely, the design of structures and machines that will safely and economically perform a specified function. a. Determination of the Ultimate Strength of a Material. An important element to be considered by a designer is how the material that has been selected will behave under a load. For a given material, this is determined by performing specific tests on prepared samples of the material. For example, a test specimen of steel may be prepared and placed in a laboratory testing machine to be subjected to a known centric axial tensile force, as described in Sec. 2.3. As the magnitude of the force is increased, various changes in the specimen are measured, for example, changes in its length and its diameter. bee80288_ch01_002-051.indd Page 31 9/4/10 5:35:42 PM user-f499 /Users/user-f499/Desktop/Temp Work/Don't Delete Job/MHDQ251:Beer:201/ch01 Eventually the largest force which may be applied to the specimen is reached, and the specimen either breaks or begins to carry less load. This largest force is called the ultimate load for the test specimen and is denoted by PU. Since the applied load is centric, we may divide the ultimate load by the original cross-sectional area of the rod to obtain the ultimate normal stress of the material used. This stress, also known as the ultimate strength in tension of the material, is sU 5 PU A b. Allowable Load and Allowable Stress; Factor of Safety. The maximum load that a structural member or a machine component will be allowed to carry under normal conditions of utilization is considerably smaller than the ultimate load. This smaller load is referred to as the allowable load and, sometimes, as the working load or design load. Thus, only a fraction of the ultimate-load capacity of the member is utilized when the allowable load is applied. The remaining portion of the load-carrying capacity of the member is kept in reserve to assure its safe performance. The ratio of the ultimate load to the allowable load is used to define the factor of safety.† We have ultimate load allowable load (1.24) An alternative definition of the factor of safety is based on the use of stresses: Factor of safety 5 F.S. 5 P (1.23) Several test procedures are available to determine the ultimate shearing stress, or ultimate strength in shear, of a material. The one most commonly used involves the twisting of a circular tube (Sec. 3.5). A more direct, if less accurate, procedure consists in clamping a rectangular or round bar in a shear tool (Fig. 1.39) and applying an increasing load P until the ultimate load PU for single shear is obtained. If the free end of the specimen rests on both of the hardened dies (Fig. 1.40), the ultimate load for double shear is obtained. In either case, the ultimate shearing stress tU is obtained by dividing the ultimate load by the total area over which shear has taken place. We recall that, in the case of single shear, this area is the crosssectional area A of the specimen, while in double shear it is equal to twice the cross-sectional area. Factor of safety 5 F.S. 5 1.13 Design Considerations ultimate stress allowable stress (1.25) The two expressions given for the factor of safety in Eqs. (1.24) and (1.25) are identical when a linear relationship exists between the load and the stress. In most engineering applications, however, this relationship ceases to be linear as the load approaches its ultimate value, and the factor of safety obtained from Eq. (1.25) does not provide a †In some fields of engineering, notably aeronautical engineering, the margin of safety is used in place of the factor of safety. The margin of safety is defined as the factor of safety minus one; that is, margin of safety 5 F.S. 2 1.00. Fig. 1.39 Single shear test. P Fig. 1.40 Double shear test. 31 bee80288_ch01_002-051.indd Page 32 9/4/10 5:35:46 PM user-f499 32 Introduction—Concept of Stress /Users/user-f499/Desktop/Temp Work/Don't Delete Job/MHDQ251:Beer:201/ch01 true assessment of the safety of a given design. Nevertheless, the allowable-stress method of design, based on the use of Eq. (1.25), is widely used. c. Selection of an Appropriate Factor of Safety. The selection of the factor of safety to be used for various applications is one of the most important engineering tasks. On the one hand, if a factor of safety is chosen too small, the possibility of failure becomes unacceptably large; on the other hand, if a factor of safety is chosen unnecessarily large, the result is an uneconomical or nonfunctional design. The choice of the factor of safety that is appropriate for a given design application requires engineering judgment based on many considerations, such as the following: 1. Variations that may occur in the properties of the member 2. 3. 4. 5. 6. 7. under consideration. The composition, strength, and dimensions of the member are all subject to small variations during manufacture. In addition, material properties may be altered and residual stresses introduced through heating or deformation that may occur during manufacture, storage, transportation, or construction. The number of loadings that may be expected during the life of the structure or machine. For most materials the ultimate stress decreases as the number of load applications is increased. This phenomenon is known as fatigue and, if ignored, may result in sudden failure (see Sec. 2.7). The type of loadings that are planned for in the design, or that may occur in the future. Very few loadings are known with complete accuracy—most design loadings are engineering estimates. In addition, future alterations or changes in usage may introduce changes in the actual loading. Larger factors of safety are also required for dynamic, cyclic, or impulsive loadings. The type of failure that may occur. Brittle materials fail suddenly, usually with no prior indication that collapse is imminent. On the other hand, ductile materials, such as structural steel, normally undergo a substantial deformation called yielding before failing, thus providing a warning that overloading exists. However, most buckling or stability failures are sudden, whether the material is brittle or not. When the possibility of sudden failure exists, a larger factor of safety should be used than when failure is preceded by obvious warning signs. Uncertainty due to methods of analysis. All design methods are based on certain simplifying assumptions which result in calculated stresses being approximations of actual stresses. Deterioration that may occur in the future because of poor maintenance or because of unpreventable natural causes. A larger factor of safety is necessary in locations where conditions such as corrosion and decay are difficult to control or even to discover. The importance of a given member to the integrity of the whole structure. Bracing and secondary members may in many cases be designed with a factor of safety lower than that used for primary members. bee80288_ch01_002-051.indd Page 33 9/4/10 5:35:46 PM user-f499 /Users/user-f499/Desktop/Temp Work/Don't Delete Job/MHDQ251:Beer:201/ch01 In addition to the these considerations, there is the additional consideration concerning the risk to life and property that a failure would produce. Where a failure would produce no risk to life and only minimal risk to property, the use of a smaller factor of safety can be considered. Finally, there is the practical consideration that, unless a careful design with a nonexcessive factor of safety is used, a structure or machine might not perform its design function. For example, high factors of safety may have an unacceptable effect on the weight of an aircraft. For the majority of structural and machine applications, factors of safety are specified by design specifications or building codes written by committees of experienced engineers working with professional societies, with industries, or with federal, state, or city agencies. Examples of such design specifications and building codes are 1. Steel: American Institute of Steel Construction, Specification for Structural Steel Buildings 2. Concrete: American Concrete Institute, Building Code Require- ment for Structural Concrete 3. Timber: American Forest and Paper Association, National Design Specification for Wood Construction 4. Highway bridges: American Association of State Highway Offi- cials, Standard Specifications for Highway Bridges *d. Load and Resistance Factor Design. As we saw previously, the allowable-stress method requires that all the uncertainties associated with the design of a structure or machine element be grouped into a single factor of safety. An alternative method of design, which is gaining acceptance chiefly among structural engineers, makes it possible through the use of three different factors to distinguish between the uncertainties associated with the structure itself and those associated with the load it is designed to support. This method, referred to as Load and Resistance Factor Design (LRFD), further allows the designer to distinguish between uncertainties associated with the live load, PL, that is, with the load to be supported by the structure, and the dead load, PD, that is, with the weight of the portion of structure contributing to the total load. When this method of design is used, the ultimate load, PU, of the structure, that is, the load at which the structure ceases to be useful, should first be determined. The proposed design is then acceptable if the following inequality is satisfied: gD PD 1 gL PL # fPU (1.26) The coefficient f is referred to as the resistance factor; it accounts for the uncertainties associated with the structure itself and will normally be less than 1. The coefficients gD and gL are referred to as the load factors; they account for the uncertainties associated, respectively, with the dead and live load and will normally be greater than 1, with gL generally larger than gD. While a few examples or assigned problems using LRFD are included in this chapter and in Chaps. 5 and 10, the allowable-stress method of design will be used in this text. 1.13 Design Considerations 33 bee80288_ch01_002-051.indd Page 34 11/1/10 dAB P SAMPLE PROBLEM 1.3 B A 50 kN 0.6 m t 4:55:13 PM user-f499/Users/user-f499/Desktop/Temp user-f499/Users/user-f499/Desktop/Temp Work/Don't Delete Job/MHDQ251:Beer:201/ch 15 kN t C D 0.3 m 0.3 m P SOLUTION B 50 kN 0.6 m Two forces are applied to the bracket BCD as shown. (a) Knowing that the control rod AB is to be made of a steel having an ultimate normal stress of 600 MPa, determine the diameter of the rod for which the factor of safety with respect to failure will be 3.3. (b) The pin at C is to be made of a steel having an ultimate shearing stress of 350 MPa. Determine the diameter of the pin C for which the factor of safety with respect to shear will also be 3.3. (c) Determine the required thickness of the bracket supports at C knowing that the allowable bearing stress of the steel used is 300 MPa. 15 kN Free Body: Entire Bracket. The reaction at C is represented by its components Cx and Cy. 1 l oMC 5 0: P(0.6 m) 2 (50 kN)(0.3 m) 2 (15 kN)(0.6 m) 5 0 P 5 40 kN oFx 5 0: Cx 5 40 k C 5 2C 2x 1 C 2y 5 76.3 kN oFy 5 0: Cy 5 65 kN C Cx D Cy 0.3 m a. Control Rod AB. able stress is Since the factor of safety is to be 3.3, the allow- 0.3 m sall 5 sU 600 MPa 5 181.8 MPa 5 F.S. 3.3 For P 5 40 kN the cross-sectional area required is P 40 kN 5 220 3 1026 m 2 5 s all 181.8 MPa p 2 dAB 5 16.74 mm 5 dAB 5 220 3 1026 m 2 4 A req 5 C A req dC b. Shear in Pin C. F2 For a factor of safety of 3.3, we have tU 350 MPa 5 5 106.1 MPa F.S. 3.3 tall 5 F1  F2  12 C F1 Since the pin is in double shear, we write Areq 5 A req 5 1 2C t d  22 mm 1 2C 176.3 kN2y2 Cy2 5 5 360 mm2 tall 106.1 MPa p 2 dC 5 360 mm 2 4 dC 5 21.4 mm Use: dC 5 22 mm ◀ The next larger size pin available is of 22-mm diameter and should be used. c. Bearing at C. Using d 5 22 mm, the nominal bearing area of each bracket is 22t. Since the force carried by each bracket is Cy2 and the allowable bearing stress is 300 MPa, we write Areq 5 Thus 22t 5 127.2 34 ◀ 176.3 kN2y2 Cy2 5 5 127.2 mm2 sall 300 MPa t 5 5.78 mm Use: t 5 6 mm ◀ bee80288_ch01_002-051.indd Page 35 11/1/10 4:55:21 PM user-f499/Users/user-f499/Desktop/Temp user-f499/Users/user-f499/Desktop/Temp Work/Don't Delete Job/MHDQ251:Beer:201/ch SAMPLE PROBLEM 1.4 C The rigid beam BCD is attached by bolts to a control rod at B, to a hydraulic cylinder at C, and to a fixed support at D. The diameters of the bolts used are: dB 5 dD 5 38 in., dC 5 12 in. Each bolt acts in double shear and is made from a steel for which the ultimate shearing stress is tU 5 40 ksi. The control rod AB has a diameter dA 5 167 in. and is made of a steel for which the ultimate tensile stress is sU 5 60 ksi. If the minimum factor of safety is to be 3.0 for the entire unit, determine the largest upward force which may be applied by the hydraulic cylinder at C. D 8 in. B 6 in. A SOLUTION C D C B B D 6 in. 8 in. The factor of safety with respect to failure must be 3.0 or more in each of the three bolts and in the control rod. These four independent criteria will be considered separately. Free Body: Beam BCD. We first determine the force at C in terms of the force at B and in terms of the force at D. B114 in.2 2 C18 in.2 5 0 2D114 in.2 1 C16 in.2 5 0 1l oMD 5 0: 1l oMB 5 0: Control Rod. C 5 1.750B C 5 2.33D (1) (2) For a factor of safety of 3.0 we have sall 5 sU 60 ksi 5 5 20 ksi F.S. 3.0 The allowable force in the control rod is B 5 sall 1A2 5 120 ksi2 14p 1 167 in.2 2 5 3.01 kips F1 3 8 Using Eq. (1) we find the largest permitted value of C: in. C 5 1.750B 5 1.75013.01 kips2 F1 B  2F1 C 5 5.27 kips ◀ Bolt at B. tall 5 tUyF.S. 5 (40 ksi)y3 5 13.33 ksi. Since the bolt is in double shear, the allowable magnitude of the force B exerted on the bolt is B 5 2F 1 5 21tall A2 5 2113.33 ksi2 1 14 p2 1 38 in.2 2 5 2.94 kips B C From Eq. (1): 1 2 in. C 5 1.750B 5 1.75012.94 kips2 C 5 5.15 kips ◀ Bolt at D. Since this bolt is the same as bolt B, the allowable force is D 5 B 5 2.94 kips. From Eq. (2): C 5 2.33D 5 2.3312.94 kips2 Bolt at C. F2 C ⫽ 2F2 ◀ We again have tall 5 13.33 ksi and write C 5 2F 2 5 21tall A2 5 2113.33 ksi2 1 14 p2 1 12 in.2 2 F2 C 5 6.85 kips C 5 5.23 kips ◀ Summary. We have found separately four maximum allowable values of the force C. In order to satisfy all these criteria we must choose the C 5 5.15 kips ◀ smallest value, namely: 35 bee80288_ch01_002-051.indd Page 36 9/6/10 7:27:12 PM user-f499 /Users/user-f499/Desktop/Temp Work/Don't Delete Job/MHDQ251:Beer:201/ch01 PROBLEMS P 5.0 in. 3.0 in. 1.29 The 1.4-kip load P is supported by two wooden members of uniform cross section that are joined by the simple glued scarf splice shown. Determine the normal and shearing stresses in the glued splice. 1.30 Two wooden members of uniform cross section are joined by the simple scarf splice shown. Knowing that the maximum allowable tensile stress in the glued splice is 75 psi, determine (a) the largest load P that can be safely supported, (b) the corresponding shearing stress in the splice. 60⬚ 1.31 Two wooden members of uniform rectangular cross section are joined by the simple glued scarf splice shown. Knowing that P 5 11 kN, determine the normal and shearing stresses in the glued splice. P' P' Fig. P1.29 and P1.30 150 mm P 45⬚⬚ 45 75 mm Fig. P1.31 and P1.32 1.32 Two wooden members of uniform rectangular cross section are joined by the simple glued scarf splice shown. Knowing that the maximum allowable shearing stress in the glued splice is 620 kPa, determine (a) the largest load P that can be safely applied, (b) the corresponding tensile stress in the splice. P 1 4 Weld 25⬚ Fig. P1.33 and P1.34 36 in. 1.33 A steel pipe of 12-in. outer diameter is fabricated from 14-in.-thick plate by welding along a helix that forms an angle of 258 with a plane perpendicular to the axis of the pipe. Knowing that the maximum allowable normal and shearing stresses in the directions respectively normal and tangential to the weld are s 5 12 ksi and t 5 7.2 ksi, determine the magnitude P of the largest axial force that can be applied to the pipe. 1.34 A steel pipe of 12-in. outer diameter is fabricated from 14-in.-thick plate by welding along a helix that forms an angle of 258 with a plane perpendicular to the axis of the pipe. Knowing that a 66 kip axial force P is applied to the pipe, determine the normal and shearing stresses in directions respectively normal and tangential to the weld. bee80288_ch01_002-051.indd Page 37 9/6/10 7:27:33 PM user-f499 /Users/user-f499/Desktop/Temp Work/Don't Delete Job/MHDQ251:Beer:201/ch01 Problems 1.35 A 1060-kN load P is applied to the granite block shown. Determine the resulting maximum value of (a) the normal stress, (b) the shearing stress. Specify the orientation of that plane on which each of these maximum values occurs. P 1.36 A centric load P is applied to the granite block shown. Knowing that the resulting maximum value of the shearing stress in the block is 18 MPa, determine (a) the magnitude of P, (b) the orientation of the surface on which the maximum shearing stress occurs, (c) the normal stress exerted on that surface, (d) the maximum value of the normal stress in the block. 1.37 Link BC is 6 mm thick, has a width w 5 25 mm, and is made of a steel with a 480-MPa ultimate strength in tension. What is the safety factor used if the structure shown was designed to support a 16-kN load P? 1.38 Link BC is 6 mm thick and is made of a steel with a 450-MPa ultimate strength in tension. What should be its width w if the structure shown is being designed to support a 20-kN load P with a factor of safety of 3? 1.39 A 34-in.-diameter rod made of the same material as rods AC and AD in the truss shown was tested to failure and an ultimate load of 29 kips was recorded. Using a factor of safety of 3.0, determine the required diameter (a) of rod AC, (b) of rod AD. 140 mm 140 mm Fig. P1.35 and P1.36 600 mm A B w 90⬚ 480 mm C D A P 5 ft D B 10 ft C Fig. P1.37 and P1.38 10 ft 10 kips 10 kips Fig. P1.39 and P1.40 1.40 In the truss shown, members AC and AD consist of rods made of the same metal alloy. Knowing that AC is of 1-in. diameter and that the ultimate load for that rod is 75 kips, determine (a) the factor of safety for AC, (b) the required diameter of AD if it is desired that both rods have the same factor of safety. 1.41 Link AB is to be made of a steel for which the ultimate normal stress is 450 MPa. Determine the cross-sectional area of AB for which the factor of safety will be 3.50. Assume that the link will be adequately reinforced around the pins at A and B. A 8 kN/m 35⬚ C B D 20 kN 0.4 m Fig. P1.41 0.4 m 0.4 m E 37 bee80288_ch01_002-051.indd Page 38 9/4/10 5:36:17 PM user-f499 38 1.42 A steel loop ABCD of length 1.2 m and of 10-mm diameter is placed as shown around a 24-mm-diameter aluminum rod AC. Cables BE and DF, each of 12-mm diameter, are used to apply the load Q. Knowing that the ultimate strength of the steel used for the loop and the cables is 480 MPa and that the ultimate strength of the aluminum used for the rod is 260 MPa, determine the largest load Q that can be applied if an overall factor of safety of 3 is desired. Introduction—Concept of Stress Q 240 mm 240 mm E B 180 mm 24 mm C A 180 mm 10 mm D 1.43 Two wooden members shown, which support a 3.6-kip load, are joined by plywood splices fully glued on the surfaces in contact. The ultimate shearing stress in the glue is 360 psi and the clearance between the members is 14 in. Determine the required length L of each splice if a factor of safety of 2.75 is to be achieved. 5.0 in. 3.6 kips 12 mm F /Users/user-f499/Desktop/Temp Work/Don't Delete Job/MHDQ251:Beer:201/ch01 L Q' Fig. P1.42 1 4 in. 3.6 kips Fig. P1.43 5 8 in. 3 4 1.44 Two plates, each 18-in. thick, are used to splice a plastic strip as shown. Knowing that the ultimate shearing stress of the bonding between the surfaces is 130 psi, determine the factor of safety with respect to shear when P 5 325 lb. in. P' 2 14 in. P 1 4 Fig. P1.44 in. 1.45 A load P is supported as shown by a steel pin that has been inserted in a short wooden member hanging from the ceiling. The ultimate strength of the wood used is 60 MPa in tension and 7.5 MPa in shear, while the ultimate strength of the steel is 145 MPa in shear. Knowing that b 5 40 mm, c 5 55 mm, and d 5 12 mm, determine the load P if an overall factor of safety of 3.2 is desired. 1 2 d P 1 2 c 40 mm Fig. P1.45 b P bee80288_ch01_002-051.indd Page 39 11/1/10 4:55:31 PM user-f499/Users/user-f499/Desktop/Temp user-f499/Users/user-f499/Desktop/Temp Work/Don't Delete Job/MHDQ251:Beer:201/ch Problems 1.46 For the support of Prob. 1.45, knowing that the diameter of the pin is d 5 16 mm and that the magnitude of the load is P 5 20 kN, determine (a) the factor of safety for the pin, (b) the required values of b and c if the factor of safety for the wooden member is the same as that found in part a for the pin. 1.47 Three steel bolts are to be used to attach the steel plate shown to a wooden beam. Knowing that the plate will support a 110-kN load, that the ultimate shearing stress for the steel used is 360 MPa, and that a factor of safety of 3.35 is desired, determine the required diameter of the bolts. 1.48 Three 18-mm-diameter steel bolts are to be used to attach the steel plate shown to a wooden beam. Knowing that the plate will support a 110-kN load and that the ultimate shearing stress for the steel used is 360 MPa, determine the factor of safety for this design. 1.49 A steel plate 165 in. thick is embedded in a horizontal concrete slab and is used to anchor a high-strength vertical cable as shown. The diameter of the hole in the plate is 34 in., the ultimate strength of the steel used is 36 ksi, and the ultimate bonding stress between plate and concrete is 300 psi. Knowing that a factor of safety of 3.60 is desired when P 5 2.5 kips, determine (a) the required width a of the plate, (b) the minimum depth b to which a plate of that width should be embedded in the concrete slab. (Neglect the normal stresses between the concrete and the bottom edge of the plate.) P 5 in. 16 3 4 b in. a Fig. P1.49 1.50 Determine the factor of safety for the cable anchor in Prob. 1.49 when P 5 3 kips, knowing that a 5 2 in. and b 5 7.5 in. 110 kN Fig. P1.47 and P1.48 39 bee80288_ch01_002-051.indd Page 40 9/4/10 5:36:29 PM user-f499 40 Introduction—Concept of Stress /Users/user-f499/Desktop/Temp Work/Don't Delete Job/MHDQ251:Beer:201/ch01 1.51 In the steel structure shown, a 6-mm-diameter pin is used at C and 10-mm-diameter pins are used at B and D. The ultimate shearing stress is 150 MPa at all connections, and the ultimate normal stress is 400 MPa in link BD. Knowing that a factor of safety of 3.0 is desired, determine the largest load P that can be applied at A. Note that link BD is not reinforced around the pin holes. D Front view D 6 mm 18 mm B A P B 160 mm A 120 mm B Top view C Side view C Fig. P1.51 1.52 Solve Prob. 1.51, assuming that the structure has been redesigned to use 12-mm-diameter pins at B and D and no other change has been made. 1.53 Each of the two vertical links CF connecting the two horizontal members AD and EG has a uniform rectangular cross section 14 in. thick and 1 in. wide, and is made of a steel with an ultimate strength in tension of 60 ksi. The pins at C and F each have a 12-in. diameter and are made of a steel with an ultimate strength in shear of 25 ksi. Determine the overall factor of safety for the links CF and the pins connecting them to the horizontal members. 10 in. 16 in. A 10 in. B C D E F G 2 kips Fig. P1.53 bee80288_ch01_002-051.indd Page 41 11/1/10 4:55:35 PM user-f499/Users/user-f499/Desktop/Temp user-f499/Users/user-f499/Desktop/Temp Work/Don't Delete Job/MHDQ251:Beer:201/ch Problems 1.54 Solve Prob. 1.53, assuming that the pins at C and F have been replaced by pins with a 34-in. diameter. 1.55 In the structure shown, an 8-mm-diameter pin is used at A, and 12-mm-diameter pins are used at B and D. Knowing that the ultimate shearing stress is 100 MPa at all connections and that the ultimate normal stress is 250 MPa in each of the two links joining B and D, determine the allowable load P if an overall factor of safety of 3.0 is desired. Top view 200 mm 180 mm 12 mm 8 mm A B C B A C B 20 mm P D Front view 8 mm 8 mm D 12 mm Side view P P Fig. P1.55 1.56 In an alternative design for the structure of Prob. 1.55, a pin of 10-mm diameter is to be used at A. Assuming that all other specifications remain unchanged, determine the allowable load P if an overall factor of safety of 3.0 is desired. *1.57 The Load and Resistance Factor Design method is to be used to select the two cables that will raise and lower a platform supporting two window washers. The platform weighs 160 lb and each of the window washers is assumed to weigh 195 lb with equipment. Since these workers are free to move on the platform, 75% of their total weight and the weight of their equipment will be used as the design live load of each cable. (a) Assuming a resistance factor f 5 0.85 and load factors gD 5 1.2 and gL 5 1.5, determine the required minimum ultimate load of one cable. (b) What is the conventional factor of safety for the selected cables? *1.58 A 40-kg platform is attached to the end B of a 50-kg wooden beam AB, which is supported as shown by a pin at A and by a slender steel rod BC with a 12-kN ultimate load. (a) Using the Load and Resistance Factor Design method with a resistance factor f 5 0.90 and load factors gD 5 1.25 and gL 5 1.6, determine the largest load that can be safely placed on the platform. (b) What is the corresponding conventional factor of safety for rod BC? Fig. P1.57 C 1.8 m A B 2.4 m Fig. P1.58 41 bee80288_ch01_002-051.indd Page 42 9/4/10 5:36:38 PM user-f499 /Users/user-f499/Desktop/Temp Work/Don't Delete Job/MHDQ251:Beer:201/ch01 REVIEW AND SUMMARY This chapter was devoted to the concept of stress and to an introduction to the methods used for the analysis and design of machines and load-bearing structures. Section 1.2 presented a short review of the methods of statics and of their application to the determination of the reactions exerted by its supports on a simple structure consisting of pin-connected members. Emphasis was placed on the use of a free-body diagram to obtain equilibrium equations which were solved for the unknown reactions. Free-body diagrams were also used to find the internal forces in the various members of the structure. Axial loading. Normal stress P The concept of stress was first introduced in Sec. 1.3 by considering a two-force member under an axial loading. The normal stress in that member was obtained by dividing the magnitude P of the load by the cross-sectional area A of the member (Fig. 1.41). We wrote s5 P' 42 (1.5) Section 1.4 was devoted to a short discussion of the two principal tasks of an engineer, namely, the analysis and the design of structures and machines. As noted in Sec. 1.5, the value of s obtained from Eq. (1.5) represents the average stress over the section rather than the stress at a specific point Q of the section. Considering a small area DA surrounding Q and the magnitude DF of the force exerted on DA, we defined the stress at point Q as A Fig. 1.41 P A s 5 lim ¢ Ay 0 ¢F ¢A (1.6) In general, the value obtained for the stress s at point Q is different from the value of the average stress given by formula (1.5) and is found to vary across the section. However, this variation is small in any section away from the points of application of the loads. In practice, therefore, the distribution of the normal stresses in an axially loaded member is assumed to be uniform, except in the immediate vicinity of the points of application of the loads. However, for the distribution of stresses to be uniform in a given section, it is necessary that the line of action of the loads P and P9 pass through the centroid C of the section. Such a loading is called a centric axial loading. In the case of an eccentric axial loading, the distribution of stresses is not uniform. Stresses in members subjected to an eccentric axial loading will be discussed in Chap 4. bee80288_ch01_002-051.indd Page 43 9/4/10 5:36:41 PM user-f499 /Users/user-f499/Desktop/Temp Work/Don't Delete Job/MHDQ251:Beer:201/ch01 When equal and opposite transverse forces P and P9 of magnitude P are applied to a member AB (Fig. 1.42), shearing stresses t are created over any section located between the points of application of the two forces [Sec 1.6]. These stresses vary greatly across the section and their distribution cannot be assumed uniform. However, dividing the magnitude P—referred to as the shear in the section— by the cross-sectional area A, we defined the average shearing stress over the section: tave 5 P A 43 Review and Summary Transverse forces. Shearing stress P A C (1.8) B P⬘ Fig. 1.42 Shearing stresses are found in bolts, pins, or rivets connecting two structural members or machine components. For example, in the case of bolt CD (Fig. 1.43), which is in single shear, we wrote tave 5 P F 5 A A (1.9) while, in the case of bolts EG and HJ (Fig. 1.44), which are both in double shear, we had tave 5 Fy2 P F 5 5 A A 2A P P 5 A td C F A E E' B F' D (1.10) Fig. 1.43 Bolts, pins, and rivets also create stresses in the members they connect, along the bearing surface, or surface of contact [Sec. 1.7]. The bolt CD of Fig. 1.43, for example, creates stresses on the semicylindrical surface of plate A with which it is in contact (Fig. 1.45). Since the distribution of these stresses is quite complicated, one uses in practice an average nominal value sb of the stress, called bearing stress, obtained by dividing the load P by the area of the rectangle representing the projection of the bolt on the plate section. Denoting by t the thickness of the plate and by d the diameter of the bolt, we wrote sb 5 Single and double shear Bearing stress E H C K F' L L' D G J Fig. 1.44 t C P A d F F' D Fig. 1.45 The method you should use in solving a problem in mechanics of materials was described in Sec. 1.9. Your solution should begin with a clear and precise statement of the problem. You will then draw one or several free-body diagrams that you will use to write equilibrium equations. These equations will be solved for unknown forces, from which the required stresses and deformations can be computed. Once the answer has been obtained, it should be carefully checked. F A (1.11) In Sec. 1.8, we applied the concept introduced in the previous sections to the analysis of a simple structure consisting of two pinconnected members supporting a given load. We determined successively the normal stresses in the two members, paying special attention to their narrowest sections, the shearing stresses in the various pins, and the bearing stress at each connection. K' B Method of Solution bee80288_ch01_002-051.indd Page 44 9/4/10 5:36:50 PM user-f499 44 The first part of the chapter ended with a discussion of numerical accuracy in engineering, which stressed the fact that the accuracy of an answer can never be greater than the accuracy of the given data [Sec. 1.10]. Introduction—Concept of Stress Stresses on an oblique section P' /Users/user-f499/Desktop/Temp Work/Don't Delete Job/MHDQ251:Beer:201/ch01 P  Fig. 1.46 In Sec. 1.11, we considered the stresses created on an oblique section in a two-force member under axial loading. We found that both normal and shearing stresses occurred in such a situation. Denoting by u the angle formed by the section with a normal plane (Fig. 1.46) and by A0 the area of a section perpendicular to the axis of the member, we derived the following expressions for the normal stress s and the shearing stress t on the oblique section: s5 P cos2 u A0 t5 P sin u cos u A0 (1.14) We observed from these formulas that the normal stress is maximum and equal to sm 5 PyA0 for u 5 0, while the shearing stress is maximum and equal to tm 5 Py2A0 for u 5 458. We also noted that t 5 0 when u 5 0, while s 5 Py2A0 when u 5 458. Stress under general loading y y a a yz yx zy Q  z zx xz xy x a z x Fig. 1.47 Factor of safety Next, we discussed the state of stress at a point Q in a body under the most general loading condition [Sec. 1.12]. Considering a small cube centered at Q (Fig. 1.47), we denoted by sx the normal stress exerted on a face of the cube perpendicular to the x axis, and by txy and txz, respectively, the y and z components of the shearing stress exerted on the same face of the cube. Repeating this procedure for the other two faces of the cube and observing that txy 5 tyx, tyz 5 tzy, and tzx 5 txz, we concluded that six stress components are required to define the state of stress at a given point Q, namely, sx, sy, sz, txy, tyz, and tzx. Section 1.13 was devoted to a discussion of the various concepts used in the design of engineering structures. The ultimate load of a given structural member or machine component is the load at which the member or component is expected to fail; it is computed from the ultimate stress or ultimate strength of the material used, as determined by a laboratory test on a specimen of that material. The ultimate load should be considerably larger than the allowable load, i.e., the load that the member or component will be allowed to carry under normal conditions. The ratio of the ultimate load to the allowable load is defined as the factor of safety: Factor of safety 5 F.S. 5 ultimate load allowable load (1.24) The determination of the factor of safety that should be used in the design of a given structure depends upon a number of considerations, some of which were listed in this section. Load and Resistance Factor Design Section 1.13 ended with the discussion of an alternative approach to design, known as Load and Resistance Factor Design, which allows the engineer to distinguish between the uncertainties associated with the structure and those associated with the load. bee80288_ch01_002-051.indd Page 45 9/4/10 5:36:54 PM user-f499 /Users/user-f499/Desktop/Temp Work/Don't Delete Job/MHDQ251:Beer:201/ch01 REVIEW PROBLEMS 1.59 A strain gage located at C on the surface of bone AB indicates that the average normal stress in the bone is 3.80 MPa when the bone is subjected to two 1200-N forces as shown. Assuming the cross section of the bone at C to be annular and knowing that its outer diameter is 25 mm, determine the inner diameter of the bone’s cross section at C. 1200 N A C B 1200 N Fig. P1.59 1.60 Two horizontal 5-kip forces are applied to pin B of the assembly shown. Knowing that a pin of 0.8-in. diameter is used at each connection, determine the maximum value of the average normal stress (a) in link AB, (b) in link BC. 0.5 in. B 1.8 in. A 5 kips 5 kips 60 45 0.5 in. 1.8 in. C Fig. P1.60 1.61 For the assembly and loading of Prob. 1.60, determine (a) the average shearing stress in the pin at C, (b) the average bearing stress at C in member BC, (c) the average bearing stress at B in member BC. 45 bee80288_ch01_002-051.indd Page 46 9/6/10 7:27:48 PM user-f499 46 /Users/user-f499/Desktop/Temp Work/Don't Delete Job/MHDQ251:Beer:201/ch01 1.62 In the marine crane shown, link CD is known to have a uniform Introduction—Concept of Stress cross section of 50 3 150 mm. For the loading shown, determine the normal stress in the central portion of that link. 15 m 25 m 3m B 35 m 80 Mg C 15 m D A Fig. P1.62 1 2 in. thick and 9 in. wide, are joined by the dry mortise joint shown. Knowing that the wood used shears off along its grain when the average shearing stress reaches 1.20 ksi, determine the magnitude P of the axial load that will cause the joint to fail. 1.63 Two wooden planks, each 5 8 in. 5 8 P' 1 in. 2 in. in. 2 in. 1 in. 9 in. P Fig. P1.63 P' 1.64 Two wooden members of uniform rectangular cross section of a b ␣ P Fig. P1.64 sides a 5 100 mm and b 5 60 mm are joined by a simple glued joint as shown. Knowing that the ultimate stresses for the joint are sU 5 1.26 MPa in tension and tU 5 1.50 MPa in shear and that P 5 6 kN, determine the factor of safety for the joint when (a) a 5 208, (b) a 5 358, (c) a 5 458. For each of these values of a, also determine whether the joint will fail in tension or in shear if P is increased until rupture occurs. bee80288_ch01_002-051.indd Page 47 9/4/10 5:37:07 PM user-f499 /Users/user-f499/Desktop/Temp Work/Don't Delete Job/MHDQ251:Beer:201/ch01 1.65 Member ABC, which is supported by a pin and bracket at C and a cable BD, was designed to support the 16-kN load P as shown. Knowing that the ultimate load for cable BD is 100 kN, determine the factor of safety with respect to cable failure. 40⬚ D P A 30⬚ B 0.6 m C 0.8 m 0.4 m Fig. P1.65 1.66 The 2000-lb load may be moved along the beam BD to any posi- tion between stops at E and F. Knowing that sall 5 6 ksi for the steel used in rods AB and CD, determine where the stops should be placed if the permitted motion of the load is to be as large as possible. 60 in. A -in. diameter C 1 2 5 8 -in. diameter xF xE F E D B x 2000 lb Fig. P1.66 1.67 Knowing that a force P of magnitude 750 N is applied to the pedal shown, determine (a) the diameter of the pin at C for which the average shearing stress in the pin is 40 MPa, (b) the corresponding bearing stress in the pedal at C, (c) the corresponding bearing stress in each support bracket at C. 75 mm 9 mm A B 300 mm P 125 mm C 5 mm Fig. P1.67 C D Review Problems 47 bee80288_ch01_002-051.indd Page 48 9/4/10 5:37:14 PM user-f499 48 Introduction—Concept of Stress /Users/user-f499/Desktop/Temp Work/Don't Delete Job/MHDQ251:Beer:201/ch01 1.68 A force P is applied as shown to a steel reinforcing bar that has been embedded in a block of concrete. Determine the smallest length L for which the full allowable normal stress in the bar can be developed. Express the result in terms of the diameter d of the bar, the allowable normal stress sall in the steel, and the average allowable bond stress tall between the concrete and the cylindrical surface of the bar. (Neglect the normal stresses between the concrete and the end of the bar.) d L P Fig. P1.68 1.69 The two portions of member AB are glued together along a plane forming an angle u with the horizontal. Knowing that the ultimate stress for the glued joint is 2.5 ksi in tension and 1.3 ksi in shear, determine the range of values of u for which the factor of safety of the members is at least 3.0. 2.4 kips A ␪ B 2.0 in. 1.25 in. Fig. P1.69 and P1.70 1.70 The two portions of member AB are glued together along a plane forming an angle u with the horizontal. Knowing that the ultimate stress for the glued joint is 2.5 ksi in tension and 1.3 ksi in shear, determine (a) the value of u for which the factor of safety of the member is maximum, (b) the corresponding value of the factor of safety. (Hint: Equate the expressions obtained for the factors of safety with respect to normal stress and shear.) bee80288_ch01_002-051.indd Page 49 11/2/10 2:55:25 PM user-f499 /Users/user-f499/Desktop/Temp Work/Don't Delete Job/MHDQ251:Beer:201/ch01 COMPUTER PROBLEMS The following problems are designed to be solved with a computer. 1.C1 A solid steel rod consisting of n cylindrical elements welded together is subjected to the loading shown. The diameter of element i is denoted by di and the load applied to its lower end by Pi, with the magnitude Pi of this load being assumed positive if Pi is directed downward as shown and negative otherwise. (a) Write a computer program that can used with either SI or U.S. customary units to determine the average stress in each element of the rod. (b) Use this program to solve Probs. 1.2 and 1.4. 1.C2 A 20-kN load is applied as shown to the horizontal member ABC. Member ABC has a 10 3 50-mm uniform rectangular cross section and is supported by four vertical links, each of 8 3 36-mm uniform rectangular cross section. Each of the four pins at A, B, C, and D has the same diameter d and is in double shear. (a) Write a computer program to calculate for values of d from 10 to 30 mm, using 1-mm increments, (1) the maximum value of the average normal stress in the links connecting pins B and D, (2) the average normal stress in the links connecting pins C and E, (3) the average shearing stress in pin B, (4) the average shearing stress in pin C, (5) the average bearing stress at B in member ABC, (6) the average bearing stress at C in member ABC. (b) Check your program by comparing the values obtained for d 5 16 mm with the answers given for Probs. 1.7 and 1.27. (c) Use this program to find the permissible values of the diameter d of the pins, knowing that the allowable values of the normal, shearing, and bearing stresses for the steel used are, respectively, 150 MPa, 90 MPa, and 230 MPa. (d) Solve part c, assuming that the thickness of member ABC has been reduced from 10 to 8 mm. Element n Pn Element 1 P1 Fig. P1.C1 0.4 m C 0.25 m 0.5 in. 0.2 m B E 20 kN D B 1.8 in. 5 kips 5 kips A A 60⬚ 45⬚ 0.5 in. 1.8 in. Fig. P1.C2 C 1.C3 Two horizontal 5-kip forces are applied to pin B of the assembly shown. Each of the three pins at A, B, and C has the same diameter d and is in double shear. (a) Write a computer program to calculate for values of d from 0.50 to 1.50 in., using 0.05-in. increments, (1) the maximum value of the average normal stress in member AB, (2) the average normal stress Fig. P1.C3 49 bee80288_ch01_002-051.indd Page 50 11/2/10 2:55:36 PM user-f499 50 /Users/user-f499/Desktop/Temp Work/Don't Delete Job/MHDQ251:Beer:201/ch01 in member BC, (3) the average shearing stress in pin A, (4) the average shearing stress in pin C, (5) the average bearing stress at A in member AB, (6) the average bearing stress at C in member BC, (7) the average bearing stress at B in member BC. (b) Check your program by comparing the values obtained for d 5 0.8 in. with the answers given for Probs. 1.60 and 1.61. (c) Use this program to find the permissible values of the diameter d of the pins, knowing that the allowable values of the normal, shearing, and bearing stresses for the steel used are, respectively, 22 ksi, 13 ksi, and 36 ksi. (d) Solve part c, assuming that a new design is being investigated in which the thickness and width of the two members are changed, respectively, from 0.5 to 0.3 in. and from 1.8 to 2.4 in. Introduction—Concept of Stress 1.C4 A 4-kip force P forming an angle a with the vertical is applied as shown to member ABC, which is supported by a pin and bracket at C and by a cable BD forming an angle b with the horizontal. (a) Knowing that the ultimate load of the cable is 25 kips, write a computer program to construct a table of the values of the factor of safety of the cable for values of a and b from 0 to 458, using increments in a and b corresponding to 0.1 increments in tan a and tan b. (b) Check that for any given value of a, the maximum value of the factor of safety is obtained for b 5 38.668 and explain why. (c) Determine the smallest possible value of the factor of safety for b 5 38.668, as well as the corresponding value of a, and explain the result obtained. ␣ D P ␤ A B 15 in. C P b ␣ P' Fig. P1.C5 18 in. a 12 in. Fig. P1.C4 1.C5 A load P is supported as shown by two wooden members of uniform rectangular cross section that are joined by a simple glued scarf splice. (a) Denoting by sU and tU, respectively, the ultimate strength of the joint in tension and in shear, write a computer program which, for given values of a, b, P, sU and tU, expressed in either SI or U.S. customary units, and for values of a from 5 to 858 at 58 intervals, can calculate (1) the normal stress in the joint, (2) the shearing stress in the joint, (3) the factor of safety relative to failure in tension, (4) the factor of safety relative to failure in shear, (5) the overall factor of safety for the glued joint. (b) Apply this program, using the dimensions and loading of the members of Probs. 1.29 and 1.31, knowing that sU 5 150 psi and tU 5 214 psi for the glue used in Prob. 1.29, and that sU 5 1.26 MPa and tU 5 1.50 MPa for the glue used in Prob. 1.31. (c) Verify in each of these two cases that the shearing stress is maximum for a 5 458. bee80288_ch01_002-051.indd Page 51 11/1/10 4:55:42 PM user-f499/Users/user-f499/Desktop/Temp user-f499/Users/user-f499/Desktop/Temp Work/Don't Delete Job/MHDQ251:Beer:201/ch 1.C6 Member ABC is supported by a pin and bracket at A, and by two links that are pin-connected to the member at B and to a fixed support at D. (a) Write a computer program to calculate the allowable load Pall for any given values of (1) the diameter d1 of the pin at A, (2) the common diameter d2 of the pins at B and D, (3) the ultimate normal stress sU in each of the two links, (4) the ultimate shearing stress tU in each of the three pins, (5) the desired overall factor of safety F.S. Your program should also indicate which of the following three stresses is critical: the normal stress in the links, the shearing stress in the pin at A, or the shearing stress in the pins at B and D (b and c). Check your program by using the data of Probs. 1.55 and 1.56, respectively, and comparing the answers obtained for Pall with those given in the text. (d) Use your program to determine the allowable load Pall, as well as which of the stresses is critical, when d1 5 d2 5 15 mm, sU 5 110 MPa for aluminum links, tU 5 100 MPa for steel pins, and F.S. 5 3.2. Top view 200 mm 180 mm 12 mm 8 mm A B C B A C B 20 mm P D Front view Fig. P1.C6 8 mm 8 mm D 12 mm Side view Computer Problems 51 bee80288_ch02_052-139.indd Page 52 11/1/10 11:26:12 PM user-f499 This chapter is devoted to the study of deformations occurring in structural components subjected to axial loading. The change in length of the diagonal stays was carefully accounted for in the design of this cable-stayed bridge. 52 /Users/user-f499/Desktop/Temp Work/Don't Delete Job/MHDQ251:Beer:201/ch02 bee80288_ch02_052-139.indd Page 53 11/1/10 11:28:53 PM user-f499 /Users/user-f499/Desktop/Temp Work/Don't Delete Job/MHDQ251:Beer:201/ch02 C H A P T E R Stress and Strain—Axial Loading 53 bee80288_ch02_052-139.indd Page 54 11/2/10 2:59:48 PM user-f499 Chapter 2 Stress and Strain— Axial Loading 2.1 2.2 2.3 *2.4 2.5 2.6 2.7 2.8 2.9 2.10 2.11 2.12 *2.13 2.14 2.15 *2.16 2.17 2.18 2.19 *2.20 54 Introduction Normal Strain Under Axial Loading Stress-Strain Diagram True Stress and True Strain Hooke’s Law; Modulus of Elasticity Elastic versus Plastic Behavior of a Material Repeated Loadings; Fatigue Deformations of Members Under Axial Loading Statically Indeterminate Problems Problems Involving Temperature Changes Poisson’s Ratio Multiaxial Loading; Generalized Hooke’s Law Dilatation; Bulk Modulus Shearing Strain Further Discussions of Deformations Under Axial Loading; Relation Among E, n, and G Stress-Strain Relationships for Fiber-Reinforced Composite Materials Stress and Strain Distribution Under Axial Loading; SaintVenant’s Principle Stress Concentrations Plastic Deformations Residual Stresses /Users/user-f499/Desktop/Temp Work/Don't Delete Job/MHDQ251:Beer:201/ch02 2.1 INTRODUCTION In Chap. 1 we analyzed the stresses created in various members and connections by the loads applied to a structure or machine. We also learned to design simple members and connections so that they would not fail under specified loading conditions. Another important aspect of the analysis and design of structures relates to the deformations caused by the loads applied to a structure. Clearly, it is important to avoid deformations so large that they may prevent the structure from fulfilling the purpose for which it was intended. But the analysis of deformations may also help us in the determination of stresses. Indeed, it is not always possible to determine the forces in the members of a structure by applying only the principles of statics. This is because statics is based on the assumption of undeformable, rigid structures. By considering engineering structures as deformable and analyzing the deformations in their various members, it will be possible for us to compute forces that are statically indeterminate, i.e., indeterminate within the framework of statics. Also, as we indicated in Sec. 1.5, the distribution of stresses in a given member is statically indeterminate, even when the force in that member is known. To determine the actual distribution of stresses within a member, it is thus necessary to analyze the deformations that take place in that member. In this chapter, you will consider the deformations of a structural member such as a rod, bar, or plate under axial loading. First, the normal strain P in a member will be defined as the deformation of the member per unit length. Plotting the stress s versus the strain P as the load applied to the member is increased will yield a stress-strain diagram for the material used. From such a diagram we can determine some important properties of the material, such as its modulus of elasticity, and whether the material is ductile or brittle (Secs. 2.2 to 2.5). You will also see in Sec. 2.5 that, while the behavior of most materials is independent of the direction in which the load is applied, the response of fiber-reinforced composite materials depends upon the direction of the load. From the stress-strain diagram, we can also determine whether the strains in the specimen will disappear after the load has been removed—in which case the material is said to behave elastically—or whether a permanent set or plastic deformation will result (Sec. 2.6). Section 2.7 is devoted to the phenomenon of fatigue, which causes structural or machine components to fail after a very large number of repeated loadings, even though the stresses remain in the elastic range. The first part of the chapter ends with Sec. 2.8, which is devoted to the determination of the deformation of various types of members under various conditions of axial loading. In Secs. 2.9 and 2.10, statically indeterminate problems will be considered, i.e., problems in which the reactions and the internal forces cannot be determined from statics alone. The equilibrium equations derived from the free-body diagram of the member under consideration must be complemented by relations involving deformations; these relations will be obtained from the geometry of the problem. bee80288_ch02_052-139.indd Page 55 11/1/10 11:29:12 PM user-f499 /Users/user-f499/Desktop/Temp Work/Don't Delete Job/MHDQ251:Beer:201/ch02 In Secs. 2.11 to 2.15, additional constants associated with isotropic materials—i.e., materials with mechanical characteristics independent of direction—will be introduced. They include Poisson’s ratio, which relates lateral and axial strain, the bulk modulus, which characterizes the change in volume of a material under hydrostatic pressure, and the modulus of rigidity, which relates the components of the shearing stress and shearing strain. Stress-strain relationships for an isotropic material under a multiaxial loading will also be derived. In Sec. 2.16, stress-strain relationships involving several distinct values of the modulus of elasticity, Poisson’s ratio, and the modulus of rigidity, will be developed for fiber-reinforced composite materials under a multiaxial loading. While these materials are not isotropic, they usually display special properties, known as orthotropic properties, which facilitate their study. In the text material described so far, stresses are assumed uniformly distributed in any given cross section; they are also assumed to remain within the elastic range. The validity of the first assumption is discussed in Sec. 2.17, while stress concentrations near circular holes and fillets in flat bars are considered in Sec. 2.18. Sections 2.19 and 2.20 are devoted to the discussion of stresses and deformations in members made of a ductile material when the yield point of the material is exceeded. As you will see, permanent plastic deformations and residual stresses result from such loading conditions. 2.2 2.2 Normal Strain under Axial Loading B B NORMAL STRAIN UNDER AXIAL LOADING Let us consider a rod BC, of length L and uniform cross-sectional area A, which is suspended from B (Fig. 2.1a). If we apply a load P to end C, the rod elongates (Fig. 2.1b). Plotting the magnitude P of the load against the deformation d (Greek letter delta), we obtain a certain load-deformation diagram (Fig. 2.2). While this diagram contains information useful to the analysis of the rod under consideration, it cannot be used directly to predict the deformation of a rod of the same material but of different dimensions. Indeed, we observe that, if a deformation d is produced in rod BC by a load P, a load 2P is required to cause the same deformation in a rod B9C9 of the same length L, but of cross-sectional area 2A (Fig. 2.3). We note that, in both cases, the value of the stress is the same: s 5 PyA. On the other hand, a load P applied to a rod B0C0, of the same L C ␦ C A P (a) (b) Fig. 2.1 Deformation of axially-loaded rod. B' P B' L C' ␦ C' 2A  Fig. 2.2 Load-deformation diagram. 2P Fig. 2.3 55 bee80288_ch02_052-139.indd Page 56 11/2/10 1:16:38 AM user-f499 56 Stress and Strain—Axial Loading B⬙ B⬙ /Users/user-f499/Desktop/Temp Work/Don't Delete Job/MHDQ251:Beer:201/ch02 cross-sectional area A, but of length 2L, causes a deformation 2d in that rod (Fig. 2.4), i.e., a deformation twice as large as the deformation d it produces in rod BC. But in both cases the ratio of the deformation over the length of the rod is the same; it is equal to dyL. This observation brings us to introduce the concept of strain: We define the normal strain in a rod under axial loading as the deformation per unit length of that rod. Denoting the normal strain by P (Greek letter epsilon), we write 2L C⬙ P5 2␦ A C⬙ P Fig. 2.4 d L (2.1) Plotting the stress s 5 PyA against the strain P 5 dyL, we obtain a curve that is characteristic of the properties of the material and does not depend upon the dimensions of the particular specimen used. This curve is called a stress-strain diagram and will be discussed in detail in Sec. 2.3. Since the rod BC considered in the preceding discussion had a uniform cross section of area A, the normal stress s could be assumed to have a constant value PyA throughout the rod. Thus, it was appropriate to define the strain P as the ratio of the total deformation d over the total length L of the rod. In the case of a member of variable cross-sectional area A, however, the normal stress s 5 PyA varies along the member, and it is necessary to define the strain at a given point Q by considering a small element of undeformed length Dx (Fig. 2.5). Denoting by Dd the deformation of the element under the given loading, we define the normal strain at point Q as Q P 5 lim ⌬x x P Q x+ ␦ ⌬ x + ⌬␦ Fig. 2.5 Deformation of axiallyloaded member of variable crosssectional area. ¢d ¢xy0 ¢x 5 dd dx (2.2) Since deformation and length are expressed in the same units, the normal strain P obtained by dividing d by L (or dd by dx) is a dimensionless quantity. Thus, the same numerical value is obtained for the normal strain in a given member, whether SI metric units or U.S. customary units are used. Consider, for instance, a bar of length L 5 0.600 m and uniform cross section, which undergoes a deformation d 5 150 3 1026 m. The corresponding strain is P5 d 150 3 1026 m 5 5 250 3 1026 m/m 5 250 3 1026 L 0.600 m Note that the deformation could have been expressed in micrometers: d 5 150 mm. We would then have written P5 150 mm d 5 5 250 mm/m 5 250 m L 0.600 m and read the answer as “250 micros.” If U.S. customary units are used, the length and deformation of the same bar are, respectively, bee80288_ch02_052-139.indd Page 57 11/1/10 11:29:24 PM user-f499 /Users/user-f499/Desktop/Temp Work/Don't Delete Job/MHDQ251:Beer:201/ch02 L 5 23.6 in. and d 5 5.91 3 1023 in. The corresponding strain is P5 2.3 Stress-Strain Diagram d 5.91 3 1023 in. 5 250 3 1026 in./in. 5 L 23.6 in. which is the same value that we found using SI units. It is customary, however, when lengths and deformations are expressed in inches or microinches (min.), to keep the original units in the expression obtained for the strain. Thus, in our example, the strain would be recorded as P 5 250 3 1026 in./in. or, alternatively, as P 5 250 min./in. 2.3 STRESS-STRAIN DIAGRAM We saw in Sec. 2.2 that the diagram representing the relation between stress and strain in a given material is an important characteristic of the material. To obtain the stress-strain diagram of a material, one usually conducts a tensile test on a specimen of the material. One type of specimen commonly used is shown in Photo 2.1. The crosssectional area of the cylindrical central portion of the specimen has been accurately determined and two gage marks have been inscribed on that portion at a distance L0 from each other. The distance L0 is known as the gage length of the specimen. The test specimen is then placed in a testing machine (Photo 2.2), which is used to apply a centric load P. As the load P increases, the Photo 2.2 This machine is used to test tensile test specimens, such as those shown in this chapter. Photo 2.1 Typical tensile-test specimen. 57 bee80288_ch02_052-139.indd Page 58 11/1/10 11:29:30 PM user-f499 distance L between the two gage marks also increases (Photo 2.3). The distance L is measured with a dial gage, and the elongation d 5 L 2 L0 is recorded for each value of P. A second dial gage is often used simultaneously to measure and record the change in diameter of the specimen. From each pair of readings P and d, the stress s is computed by dividing P by the original cross-sectional area A0 of the specimen, and the strain P by dividing the elongation d by the original distance L0 between the two gage marks. The stress-strain diagram may then be obtained by plotting P as an abscissa and s as an ordinate. Stress-strain diagrams of various materials vary widely, and different tensile tests conducted on the same material may yield different results, depending upon the temperature of the specimen and the speed of loading. It is possible, however, to distinguish some common characteristics among the stress-strain diagrams of various groups of materials and to divide materials into two broad categories on the basis of these characteristics, namely, the ductile materials and the brittle materials. Ductile materials, which comprise structural steel, as well as many alloys of other metals, are characterized by their ability to yield at normal temperatures. As the specimen is subjected to an increasing load, its length first increases linearly with the load and at a very slow rate. Thus, the initial portion of the stress-strain diagram is a straight line with a steep slope (Fig. 2.6). However, after a critical value sY of the stress has been reached, the specimen undergoes a large deformation with a relatively small increase in the applied load. This deformation is caused by slippage of the material along oblique surfaces and is due, therefore, primarily to shearing stresses. As we can note from the stress-strain diagrams of two typical ductile materials (Fig. 2.6), the elongation of the specimen after it has started to yield can be 200 times as large as its deformation before yield. After a certain maximum value of the load has been reached, the diameter of a portion of the specimen begins to decrease, because of local instability (Photo 2.4a). This phenomenon is known as necking. After necking has begun, somewhat lower loads are sufficient to keep the specimen elongating further, until it finally ruptures (Photo 2.4b). We note that rupture occurs along a cone-shaped surface that forms an angle of approximately 458 with the original surface of the specimen. This indicates that shear is primarily responsible for the failure of ductile materials, and confirms the fact that, under an axial load, P⬘ P Photo 2.3 Test specimen with tensile load. 60 U 60 U Rupture 40 Y  (ksi) Stress and Strain—Axial Loading  (ksi) 58 /Users/user-f499/Desktop/Temp Work/Don't Delete Job/MHDQ251:Beer:201/ch02 B Rupture 40 Y B 20 20 Yield Strain-hardening Necking 0.02 0.2 0.0012 (a) Low-carbon steel Fig. 2.6 0.25  0.2 0.004 (b) Aluminum alloy Stress-strain diagrams of two typical ductile materials.  bee80288_ch02_052-139.indd Page 59 11/1/10 11:29:35 PM user-f499 /Users/user-f499/Desktop/Temp Work/Don't Delete Job/MHDQ251:Beer:201/ch02 2.3 Stress-Strain Diagram  U  B Rupture Photo 2.4 Tested specimen of a ductile material. shearing stresses are largest on surfaces forming an angle of 458 with the load (cf. Sec. 1.11). The stress sY at which yield is initiated is called the yield strength of the material, the stress sU corresponding to the maximum load applied to the specimen is known as the ultimate strength, and the stress sB corresponding to rupture is called the breaking strength. Brittle materials, which comprise cast iron, glass, and stone, are characterized by the fact that rupture occurs without any noticeable prior change in the rate of elongation (Fig. 2.7). Thus, for brittle materials, there is no difference between the ultimate strength and the breaking strength. Also, the strain at the time of rupture is much smaller for brittle than for ductile materials. From Photo 2.5, we note the absence of any necking of the specimen in the case of a brittle material, and observe that rupture occurs along a surface perpendicular to the load. We conclude from this observation that normal stresses are primarily responsible for the failure of brittle materials.† The stress-strain diagrams of Fig. 2.6 show that structural steel and aluminum, while both ductile, have different yield characteristics. In the case of structural steel (Fig. 2.6a), the stress remains constant over a large range of values of the strain after the onset of yield. Later the stress must be increased to keep elongating the specimen, until the maximum value sU has been reached. This is due to a property of the material known as strain-hardening. The †The tensile tests described in this section were assumed to be conducted at normal temperatures. However, a material that is ductile at normal temperatures may display the characteristics of a brittle material at very low temperatures, while a normally brittle material may behave in a ductile fashion at very high temperatures. At temperatures other than normal, therefore, one should refer to a material in a ductile state or to a material in a brittle state, rather than to a ductile or brittle material.  Fig. 2.7 Stress-strain diagram for a typical brittle material. Photo 2.5 Tested specimen of a brittle material. 59 bee80288_ch02_052-139.indd Page 60 9/7/10 4:34:25 PM user-f499 60 Stress and Strain—Axial Loading  Y Y Rupture  0.2% offset Fig. 2.8 Determination of yield strength by offset method. /Users/user-f499/Desktop/Temp Work/Don't Delete Job/MHDQ251:Beer:201/ch07 yield strength of structural steel can be determined during the tensile test by watching the load shown on the display of the testing machine. After increasing steadily, the load is observed to suddenly drop to a slightly lower value, which is maintained for a certain period while the specimen keeps elongating. In a very carefully conducted test, one may be able to distinguish between the upper yield point, which corresponds to the load reached just before yield starts, and the lower yield point, which corresponds to the load required to maintain yield. Since the upper yield point is transient, the lower yield point should be used to determine the yield strength of the material. In the case of aluminum (Fig. 2.6b) and of many other ductile materials, the onset of yield is not characterized by a horizontal portion of the stress-strain curve. Instead, the stress keeps increasing— although not linearly—until the ultimate strength is reached. Necking then begins, leading eventually to rupture. For such materials, the yield strength sY can be defined by the offset method. The yield strength at 0.2% offset, for example, is obtained by drawing through the point of the horizontal axis of abscissa P 5 0.2% (or P 5 0.002), a line parallel to the initial straight-line portion of the stress-strain diagram (Fig. 2.8). The stress s Y corresponding to the point Y obtained in this fashion is defined as the yield strength at 0.2% offset. A standard measure of the ductility of a material is its percent elongation, which is defined as Percent elongation 5 100 LB 2 L0 L0 where L0 and LB denote, respectively, the initial length of the tensile test specimen and its final length at rupture. The specified minimum elongation for a 2-in. gage length for commonly used steels with yield strengths up to 50 ksi is 21 percent. We note that this means that the average strain at rupture should be at least 0.21 in./in. Another measure of ductility which is sometimes used is the percent reduction in area, defined as Percent reduction in area 5 100 A0 2 AB A0 where A0 and AB denote, respectively, the initial cross-sectional area of the specimen and its minimum cross-sectional area at rupture. For structural steel, percent reductions in area of 60 to 70 percent are common. Thus far, we have discussed only tensile tests. If a specimen made of a ductile material were loaded in compression instead of tension, the stress-strain curve obtained would be essentially the same through its initial straight-line portion and through the beginning of the portion corresponding to yield and strain-hardening. Particularly noteworthy is the fact that for a given steel, the yield strength is the same in both tension and compression. For larger values of the strain, the tension and compression stress-strain curves diverge, and it should be noted that necking cannot occur in compression. bee80288_ch02_052-139.indd Page 61 11/1/10 11:29:40 PM user-f499 /Users/user-f499/Desktop/Temp Work/Don't Delete Job/MHDQ251:Beer:201/ch02  *2.4 True Stress and True Strain  U, tension Rupture, tension  Linear elastic range Rupture, compression U, compression Fig. 2.9 Stress-strain diagram for concrete. For most brittle materials, one finds that the ultimate strength in compression is much larger than the ultimate strength in tension. This is due to the presence of flaws, such as microscopic cracks or cavities, which tend to weaken the material in tension, while not appreciably affecting its resistance to compressive failure. An example of brittle material with different properties in tension and compression is provided by concrete, whose stress-strain diagram is shown in Fig. 2.9. On the tension side of the diagram, we first observe a linear elastic range in which the strain is proportional to the stress. After the yield point has been reached, the strain increases faster than the stress until rupture occurs. The behavior of the material in compression is different. First, the linear elastic range is significantly larger. Second, rupture does not occur as the stress reaches its maximum value. Instead, the stress decreases in magnitude while the strain keeps increasing until rupture occurs. Note that the modulus of elasticity, which is represented by the slope of the stress-strain curve in its linear portion, is the same in tension and compression. This is true of most brittle materials. *2.4 TRUE STRESS AND TRUE STRAIN We recall that the stress plotted in the diagrams of Figs. 2.6 and 2.7 was obtained by dividing the load P by the cross-sectional area A0 of the specimen measured before any deformation had taken place. Since the cross-sectional area of the specimen decreases as P increases, the stress plotted in our diagrams does not represent the actual stress in the specimen. The difference between the engineering stress s 5 PyA0 that we have computed and the true stress st 5 PyA obtained by dividing P by the cross-sectional area A of the deformed specimen becomes apparent in ductile materials after yield has started. While the engineering stress s, which is directly proportional to the load P, decreases with P during the necking phase, the true stress st, which is proportional to P but also inversely proportional to A, is observed to keep increasing until rupture of the specimen occurs. 61 bee80288_ch02_052-139.indd Page 62 11/29/10 7:06:16 PM user-f499 62 Many scientists also use a definition of strain different from that of the engineering strain P 5 dyL0. Instead of using the total elongation d and the original value L0 of the gage length, they use all the successive values of L that they have recorded. Dividing each increment DL of the distance between the gage marks, by the corresponding value of L, they obtain the elementary strain DP 5 DLyL. Adding the successive values of DP, they define the true strain Pt: Stress and Strain—Axial Loading Rupture ␴t /Users/user-f499/Desktop/Temp Work/Don't Delete Job/MHDQ251:Beer:201 Yield Pt 5 o¢P 5 o1 ¢LyL2 With the summation replaced by an integral, they can also express the true strain as follows: Pt 5 ⑀t Fig. 2.10 True stress versus true strain for a typical ductile material. # L L0 L dL 5 ln L L0 (2.3) The diagram obtained by plotting true stress versus true strain (Fig. 2.10) reflects more accurately the behavior of the material. As we have already noted, there is no decrease in true stress during the necking phase. Also, the results obtained from tensile and from compressive tests will yield essentially the same plot when true stress and true strain are used. This is not the case for large values of the strain when the engineering stress is plotted versus the engineering strain. However, engineers, whose responsibility is to determine whether a load P will produce an acceptable stress and an acceptable deformation in a given member, will want to use a diagram based on the engineering stress s 5 PyA0 and the engineering strain P 5 dyL0, since these expressions involve data that are available to them, namely the cross-sectional area A0 and the length L0 of the member in its undeformed state. 2.5 HOOKE’S LAW; MODULUS OF ELASTICITY Most engineering structures are designed to undergo relatively small deformations, involving only the straight-line portion of the corresponding stress-strain diagram. For that initial portion of the diagram (Fig. 2.6), the stress s is directly proportional to the strain P, and we can write s 5 EP (2.4) This relation is known as Hooke’s law, after Robert Hooke (1635–1703), an English scientist and one of the early founders of applied mechanics. The coefficient E is called the modulus of elasticity of the material involved, or also Young’s modulus, after the English scientist Thomas Young (1773–1829). Since the strain P is a dimensionless quantity, the modulus E is expressed in the same units as the stress s, namely in pascals or one of its multiples if SI units are used, and in psi or ksi if U.S. customary units are used. The largest value of the stress for which Hooke’s law can be used for a given material is known as the proportional limit of that material. In the case of ductile materials possessing a well-defined yield point, as in Fig. 2.6a, the proportional limit almost coincides with the yield point. For other materials, the proportional limit cannot be defined as bee80288_ch02_052-139.indd Page 63 9/4/10 5:15:11 PM user-f499 /Users/user-f499/Desktop/Temp Work/Don't Delete Job/MHDQ251:Beer:201/ch02 easily, since it is difficult to determine with accuracy the value of the stress s for which the relation between s and P ceases to be linear. But from this very difficulty we can conclude for such materials that using Hooke’s law for values of the stress slightly larger than the actual proportional limit will not result in any significant error. Some of the physical properties of structural metals, such as strength, ductility, and corrosion resistance, can be greatly affected by alloying, heat treatment, and the manufacturing process used. For example, we note from the stress-strain diagrams of pure iron and of three different grades of steel (Fig. 2.11) that large variations in the yield strength, ultimate strength, and final strain (ductility) exist among these four metals. All of them, however, possess the same modulus of elasticity; in other words, their “stiffness,” or ability to resist a deformation within the linear range, is the same. Therefore, if a high-strength steel is substituted for a lower-strength steel in a given structure, and if all dimensions are kept the same, the structure will have an increased load-carrying capacity, but its stiffness will remain unchanged. For each of the materials considered so far, the relation between normal stress and normal strain, s 5 EP, is independent of the direction of loading. This is because the mechanical properties of each material, including its modulus of elasticity E, are independent of the direction considered. Such materials are said to be isotropic. Materials whose properties depend upon the direction considered are said to be anisotropic. An important class of anisotropic materials consists of fiberreinforced composite materials. These composite materials are obtained by embedding fibers of a strong, stiff material into a weaker, softer material, referred to as a matrix. Typical materials used as fibers are graphite, glass, and polymers, while various types of resins are used as a matrix. Figure 2.12 shows a layer, or lamina, of a composite material consisting of a large number of parallel fibers embedded in a matrix. An axial load applied to the lamina along the x axis, that is, in a direction parallel to the fibers, will create a normal stress sx in the lamina and a corresponding normal strain Px which will satisfy Hooke’s law as the load is increased and as long as the elastic limit of the lamina is not exceeded. Similarly, an axial load applied along the y axis, that is, in a direction perpendicular to the lamina, will create a normal stress sy and a normal strain Py satisfying Hooke’s law, and an axial load applied along the z axis will create a normal stress sz and a normal strain Pz which again satisfy Hooke’s law. However, the moduli of elasticity Ex, Ey, and Ez corresponding, respectively, to each of the above loadings will be different. Because the fibers are parallel to the x axis, the lamina will offer a much stronger resistance to a loading directed along the x axis than to a loading directed along the y or z axis, and Ex will be much larger than either Ey or Ez. A flat laminate is obtained by superposing a number of layers or laminas. If the laminate is to be subjected only to an axial load causing tension, the fibers in all layers should have the same orientation as the load in order to obtain the greatest possible strength. But if the laminate may be in compression, the matrix material may not be sufficiently strong to prevent the fibers from kinking or buckling. The lateral stability of the laminate may then be increased by positioning 2.5 Hooke’s Law; Modulus of Elasticity ␴ Quenched, tempered alloy steel (A709) High-strength, low-alloy steel (A992) Carbon steel (A36) Pure iron ⑀ Fig. 2.11 Stress-strain diagrams for iron and different grades of steel. y Layer of material z Fibers Fig. 2.12 Layer of fiber-reinforced composite material. x 63 bee80288_ch02_052-139.indd Page 64 9/4/10 5:15:15 PM user-f499 64 some of the layers so that their fibers will be perpendicular to the load. Positioning some layers so that their fibers are oriented at 308, 458, or 608 to the load may also be used to increase the resistance of the laminate to in-plane shear. Fiber-reinforced composite materials will be further discussed in Sec. 2.16, where their behavior under multiaxial loadings will be considered. Stress and Strain—Axial Loading 2.6 ␴ C Rupture B A ⑀ D Fig. 2.13 Stress-strain characteristics of ductile material loaded beyond yield and unloaded. ␴ C Rupture B A D /Users/user-f499/Desktop/Temp Work/Don't Delete Job/MHDQ251:Beer:201/ch02 ⑀ Fig. 2.14 Stress-strain characteristics of ductile material reloaded after prior yielding. ELASTIC VERSUS PLASTIC BEHAVIOR OF A MATERIAL If the strains caused in a test specimen by the application of a given load disappear when the load is removed, the material is said to behave elastically. The largest value of the stress for which the material behaves elastically is called the elastic limit of the material. If the material has a well-defined yield point as in Fig. 2.6a, the elastic limit, the proportional limit (Sec. 2.5), and the yield point are essentially equal. In other words, the material behaves elastically and linearly as long as the stress is kept below the yield point. If the yield point is reached, however, yield takes place as described in Sec. 2.3 and, when the load is removed, the stress and strain decrease in a linear fashion, along a line CD parallel to the straight-line portion AB of the loading curve (Fig. 2.13). The fact that P does not return to zero after the load has been removed indicates that a permanent set or plastic deformation of the material has taken place. For most materials, the plastic deformation depends not only upon the maximum value reached by the stress, but also upon the time elapsed before the load is removed. The stress-dependent part of the plastic deformation is referred to as slip, and the time-dependent part— which is also influenced by the temperature—as creep. When a material does not possess a well-defined yield point, the elastic limit cannot be determined with precision. However, assuming the elastic limit equal to the yield strength as defined by the offset method (Sec. 2.3) results in only a small error. Indeed, referring to Fig. 2.8, we note that the straight line used to determine point Y also represents the unloading curve after a maximum stress sY has been reached. While the material does not behave truly elastically, the resulting plastic strain is as small as the selected offset. If, after being loaded and unloaded (Fig. 2.14), the test specimen is loaded again, the new loading curve will closely follow the earlier unloading curve until it almost reaches point C; it will then bend to the right and connect with the curved portion of the original stress-strain diagram. We note that the straight-line portion of the new loading curve is longer than the corresponding portion of the initial one. Thus, the proportional limit and the elastic limit have increased as a result of the strain-hardening that occurred during the earlier loading of the specimen. However, since the point of rupture R remains unchanged, the ductility of the specimen, which should now be measured from point D, has decreased. We have assumed in our discussion that the specimen was loaded twice in the same direction, i.e., that both loads were tensile loads. Let us now consider the case when the second load is applied in a direction opposite to that of the first one. We assume that the bee80288_ch02_052-139.indd Page 65 11/1/10 11:29:42 PM user-f499 /Users/user-f499/Desktop/Temp Work/Don't Delete Job/MHDQ251:Beer:201/ch02  Y 2.6 Elastic versus Plastic Behavior of a Material C' B C 2 Y K A D K' J' J H D'  H' – Y Fig. 2.15 Stress-strain characteristics for mild steel subjected to reverse loading. material is mild steel, for which the yield strength is the same in tension and in compression. The initial load is tensile and is applied until point C has been reached on the stress-strain diagram (Fig. 2.15). After unloading (point D), a compressive load is applied, causing the material to reach point H, where the stress is equal to 2sY. We note that portion DH of the stress-strain diagram is curved and does not show any clearly defined yield point. This is referred to as the Bauschinger effect. As the compressive load is maintained, the material yields along line HJ. If the load is removed after point J has been reached, the stress returns to zero along line JK, and we note that the slope of JK is equal to the modulus of elasticity E. The resulting permanent set AK may be positive, negative, or zero, depending upon the lengths of the segments BC and HJ. If a tensile load is applied again to the test specimen, the portion of the stress-strain diagram beginning at K (dashed line) will curve up and to the right until the yield stress sY has been reached. If the initial loading is large enough to cause strain-hardening of the material (point C9), unloading takes place along line C9D9. As the reverse load is applied, the stress becomes compressive, reaching its maximum value at H9 and maintaining it as the material yields along line H9J9. We note that while the maximum value of the compressive stress is less than sY, the total change in stress between C9 and H9 is still equal to 2sY. If point K or K9 coincides with the origin A of the diagram, the permanent set is equal to zero, and the specimen may appear to have returned to its original condition. However, internal changes will have taken place and, while the same loading sequence may be repeated, the specimen will rupture without any warning after relatively few repetitions. This indicates that the excessive plastic deformations to which the specimen was subjected have caused a radical change in the characteristics of the material. Reverse loadings into the plastic range, therefore, are seldom allowed, and only under carefully controlled conditions. Such situations occur in the straightening of damaged material and in the final alignment of a structure or machine. 65 bee80288_ch02_052-139.indd Page 66 9/4/10 5:15:22 PM user-f499 66 Stress and Strain—Axial Loading 2.7 /Users/user-f499/Desktop/Temp Work/Don't Delete Job/MHDQ251:Beer:201/ch02 REPEATED LOADINGS; FATIGUE In the preceding sections we have considered the behavior of a test specimen subjected to an axial loading. We recall that, if the maximum stress in the specimen does not exceed the elastic limit of the material, the specimen returns to its initial condition when the load is removed. You might conclude that a given loading may be repeated many times, provided that the stresses remain in the elastic range. Such a conclusion is correct for loadings repeated a few dozen or even a few hundred times. However, as you will see, it is not correct when loadings are repeated thousands or millions of times. In such cases, rupture will occur at a stress much lower than the static breaking strength; this phenomenon is known as fatigue. A fatigue failure is of a brittle nature, even for materials that are normally ductile. Fatigue must be considered in the design of all structural and machine components that are subjected to repeated or to fluctuating loads. The number of loading cycles that may be expected during the useful life of a component varies greatly. For example, a beam supporting an industrial crane may be loaded as many as two million times in 25 years (about 300 loadings per working day), an automobile crankshaft will be loaded about half a billion times if the automobile is driven 200,000 miles, and an individual turbine blade may be loaded several hundred billion times during its lifetime. Some loadings are of a fluctuating nature. For example, the passage of traffic over a bridge will cause stress levels that will fluctuate about the stress level due to the weight of the bridge. A more severe condition occurs when a complete reversal of the load occurs during the loading cycle. The stresses in the axle of a railroad car, for example, are completely reversed after each half-revolution of the wheel. The number of loading cycles required to cause the failure of a specimen through repeated successive loadings and reverse loadings may be determined experimentally for any given maximum stress level. If a series of tests is conducted, using different maximum stress levels, the resulting data may be plotted as a s-n curve. For each test, the maximum stress s is plotted as an ordinate and the number of cycles n as an abscissa; because of the large number of cycles required for rupture, the cycles n are plotted on a logarithmic scale. A typical s-n curve for steel is shown in Fig. 2.16. We note that, if the applied maximum stress is high, relatively few cycles are required to cause rupture. As the magnitude of the maximum stress is reduced, the number of cycles required to cause rupture increases, until a stress, known as the endurance limit, is reached. The endurance limit is the stress for which failure does not occur, even for an indefinitely large number of loading cycles. For a low-carbon steel, such as structural steel, the endurance limit is about one-half of the ultimate strength of the steel. For nonferrous metals, such as aluminum and copper, a typical s-n curve (Fig. 2.16) shows that the stress at failure continues to bee80288_ch02_052-139.indd Page 67 9/4/10 5:15:22 PM user-f499 /Users/user-f499/Desktop/Temp Work/Don't Delete Job/MHDQ251:Beer:201/ch02 2.8 Deformations of Members under Axial Loading 50 Stress (ksi) 40 Steel (1020HR) 30 20 Aluminum (2024) 10 103 104 105 106 107 108 109 Number of completely reversed cycles Fig. 2.16 Typical s-n curves. decrease as the number of loading cycles is increased. For such metals, one defines the fatigue limit as the stress corresponding to failure after a specified number of loading cycles, such as 500 million. Examination of test specimens, of shafts, of springs, and of other components that have failed in fatigue shows that the failure was initiated at a microscopic crack or at some similar imperfection. At each loading, the crack was very slightly enlarged. During successive loading cycles, the crack propagated through the material until the amount of undamaged material was insufficient to carry the maximum load, and an abrupt, brittle failure occurred. Because fatigue failure may be initiated at any crack or imperfection, the surface condition of a specimen has an important effect on the value of the endurance limit obtained in testing. The endurance limit for machined and polished specimens is higher than for rolled or forged components, or for components that are corroded. In applications in or near seawater, or in other applications where corrosion is expected, a reduction of up to 50% in the endurance limit can be expected. 2.8 DEFORMATIONS OF MEMBERS UNDER AXIAL LOADING Consider a homogeneous rod BC of length L and uniform cross section of area A subjected to a centric axial load P (Fig. 2.17). If the resulting axial stress s 5 PyA does not exceed the proportional limit of the material, we may apply Hooke’s law and write s 5 EP B B L (2.4) from which it follows that P5 s P 5 E AE (2.5) Recalling that the strain P was defined in Sec. 2.2 as P 5 dyL, we have d 5 PL (2.6) C ␦ C A P Fig. 2.17 Deformation of axially loaded rod. 67 bee80288_ch02_052-139.indd Page 68 9/4/10 5:15:26 PM user-f499 68 Stress and Strain—Axial Loading /Users/user-f499/Desktop/Temp Work/Don't Delete Job/MHDQ251:Beer:201/ch02 and, substituting for P from (2.5) into (2.6): PL AE d5 (2.7) Equation (2.7) may be used only if the rod is homogeneous (constant E), has a uniform cross section of area A, and is loaded at its ends. If the rod is loaded at other points, or if it consists of several portions of various cross sections and possibly of different materials, we must divide it into component parts that satisfy individually the required conditions for the application of formula (2.7). Denoting, respectively, by Pi, Li, Ai, and Ei the internal force, length, crosssectional area, and modulus of elasticity corresponding to part i, we express the deformation of the entire rod as P iL i d5 a i A iE i (2.8) We recall from Sec. 2.2 that, in the case of a member of variable cross section (Fig. 2.18), the strain P depends upon the position of the point Q where it is computed and is defined as P 5 ddydx. Solving for dd and substituting for P from Eq. (2.5), we express the deformation of an element of length dx as dd 5 P dx 5 P dx AE Q ⌬x x P Q x+ ␦ ⌬ x + ⌬␦ Fig. 2.18 Deformation of axially loaded member of variable crosssectional area. The total deformation d of the member is obtained by integrating this expression over the length L of the member: d5 # L 0 P dx AE (2.9) Formula (2.9) should be used in place of (2.7), not only when the cross-sectional area A is a function of x, but also when the internal force P depends upon x, as is the case for a rod hanging under its own weight. bee80288_ch02_052-139.indd Page 69 9/4/10 5:15:29 PM user-f499 /Users/user-f499/Desktop/Temp Work/Don't Delete Job/MHDQ251:Beer:201/ch02 Determine the deformation of the steel rod shown in Fig. 2.19a under the given loads (E 5 29 3 106 psi). We divide the rod into three component parts shown in Fig. 2.19b and write L 1 5 L2 5 12 in. A 1 5 A 2 5 0.9 in2 EXAMPLE 2.01 A ⫽ 0.3 in2 A ⫽ 0.9 in2 B A L 3 5 16 in. A 3 5 0.3 in2 C 30 kips To find the internal forces P1, P2, and P3, we must pass sections through each of the component parts, drawing each time the free-body diagram of the portion of rod located to the right of the section (Fig. 2.19c). Expressing that each of the free bodies is in equilibrium, we obtain successively 75 kips 12 in. 45 kips B A C 1 P1 5 60 kips 5 60 3 10 lb P2 5 215 kips 5 215 3 103 lb P3 5 30 kips 5 30 3 103 lb (b) D 3 2 75 kips 30 kips 45 kips P3 Carrying the values obtained into Eq. (2.8), we have 30 kips C PiLi P3L3 P2L2 1 P1L1 5 a 1 1 b d5 a E A1 A2 A3 i AiEi 160 3 103 2 1122 1 5 c 0.9 29 3 106 130 3 103 2 1162 1215 3 103 2 1122 d 1 1 0.9 0.3 2.20 3 106 d5 5 75.9 3 1023 in. 29 3 106 D P2 30 kips 45 kips B C D P1 (c) 30 kips 75 kips 45 kips Fig. 2.19 The rod BC of Fig. 2.17, which was used to derive formula (2.7), and the rod AD of Fig. 2.19, which has just been discussed in Example 2.01, both had one end attached to a fixed support. In each case, therefore, the deformation d of the rod was equal to the displacement of its free end. When both ends of a rod move, however, the deformation of the rod is measured by the relative displacement of one end of the rod with respect to the other. Consider, for instance, the assembly shown in Fig. 2.20a, which consists of three elastic bars of length L connected by a rigid pin at A. If a load P is applied at B (Fig. 2.20b), each of the three bars will deform. Since the bars AC and AC9 are attached to fixed supports at C and C9, their common deformation is measured by the displacement dA of point A. On the other hand, since both ends of bar AB move, the deformation of AB is measured by the difference between the displacements dA and dB of points A and B, i.e., by the relative displacement of B with respect to A. Denoting this relative displacement by dByA, we write PL AE 16 in. 12 in. (a) 3 dByA 5 dB 2 dA 5 D A A ␦A L C C' B C C' ␦B B P (2.10) where A is the cross-sectional area of AB and E is its modulus of elasticity. (a) (b) Fig. 2.20 Example of relative end displacement, as exhibited by the middle bar. 69 bee80288_ch02_052-139.indd Page 70 11/1/10 11:29:45 PM user-f499 /Users/user-f499/Desktop/Temp Work/Don't Delete Job/MHDQ251:Beer:201/ch02 SAMPLE PROBLEM 2.1 C A 30 kN 0.4 m 0.3 m D B E 0.4 m 0.2 m 30 kN FCD FAB B SOLUTION E D Free Body: Bar BDE 0.4 m 0.2 m The rigid bar BDE is supported by two links AB and CD. Link AB is made of aluminum (E 5 70 GPa) and has a cross-sectional area of 500 mm2; link CD is made of steel (E 5 200 GPa) and has a cross-sectional area of 600 mm2. For the 30-kN force shown, determine the deflection (a) of B, (b) of D, (c) of E. 2130 kN2 10.6 m2 1 FCD 10.2 m2 5 0 FCD 5 190 kN FCD 5 90 kN tension 2130 kN2 10.4 m2 2 FAB 10.2 m2 5 0 FAB 5 260 kN FAB 5 60 kN compression 1lo MB 5 0: F'AB  60 kN 1lo MD 5 0: A A  500 mm2 E  70 GPa 0.3 m a. Deflection of B. we have P 5 260 kN B dB 5 FAB  60 kN Since the internal force in link AB is compressive, 1260 3 103 N2 10.3 m2 PL 5 5 2514 3 1026 m AE 1500 3 1026 m2 2 170 3 109 Pa2 The negative sign indicates a contraction of member AB, and, thus, an upward deflection of end B: FCD  90 kN C dB 5 0.514 mmx ◀ b. Deflection of D. A  600 mm2 E  200 GPa 0.4 m Since in rod CD, P 5 90 kN, we write 190 3 103 N2 10.4 m2 PL 5 AE 1600 3 1026 m2 2 1200 3 109 Pa2 5 300 3 1026 m dD 5 0.300 mmw ◀ dD 5 D FCD  90 kN  B  0.514 mm  D  0.300 mm B' H D B E D' E x (200 mm – x) 200 mm 70 400 mm E' c. Deflection of E. We denote by B9 and D9 the displaced positions of points B and D. Since the bar BDE is rigid, points B9, D9, and E9 lie in a straight line and we write BH BB¿ 5 DD¿ HD EE¿ HE 5 DD¿ HD 1200 mm2 2 x 0.514 mm 5 x 5 73.7 mm x 0.300 mm 1400 mm2 1 173.7 mm2 dE 5 0.300 mm 73.7 mm dE 5 1.928 mmw ◀ bee80288_ch02_052-139.indd Page 71 11/1/10 11:29:50 PM user-f499 SAMPLE PROBLEM 2.2 18 in. C D A E /Users/user-f499/Desktop/Temp Work/Don't Delete Job/MHDQ251:Beer:201/ch02 F B G H The rigid castings A and B are connected by two 34-in.-diameter steel bolts CD and GH and are in contact with the ends of a 1.5-in.-diameter aluminum rod EF. Each bolt is single-threaded with a pitch of 0.1 in., and after being snugly fitted, the nuts at D and H are both tightened one-quarter of a turn. Knowing that E is 29 3 106 psi for steel and 10.6 3 106 psi for aluminum, determine the normal stress in the rod. 12 in. SOLUTION Deformations Bolts CD and GH. Tightening the nuts causes tension in the bolts. Because of symmetry, both are subjected to the same internal force Pb and undergo the same deformation db. We have C D Pb E P'b F Pr P'r G H P'b Pb db 5 1 Pb 118 in.2 PbLb 5 11.405 3 1026 Pb 5 11 2 6 AbEb p10.75 in.2 129 3 10 psi2 4 (1) Rod EF. The rod is in compression. Denoting by Pr the magnitude of the force in the rod and by dr the deformation of the rod, we write dr 5 2 Pr 112 in.2 PrLr 5 20.6406 3 1026 Pr 5 21 2 6 ArEr 4 p11.5 in.2 110.6 3 10 psi2 (2) Displacement of D Relative to B. Tightening the nuts one-quarter of a turn causes ends D and H of the bolts to undergo a displacement of 1 4 (0.1 in.) relative to casting B. Considering end D, we write dDyB 5 14 10.1 in.2 5 0.025 in. (3) But dDyB 5 dD 2 dB, where dD and dB represent the displacements of D and B. If we assume that casting A is held in a fixed position while the nuts at D and H are being tightened, these displacements are equal to the deformations of the bolts and of the rod, respectively. We have, therefore, dDyB 5 db 2 dr (4) Substituting from (1), (2), and (3) into (4), we obtain 0.025 in. 5 1.405 3 1026 Pb 1 0.6406 3 1026 Pr Pb Pr (5) Free Body: Casting B Pr 2 2Pb 5 0 1 y oF 5 0: (6) Pr 5 2Pb Forces in Bolts and Rod Substituting for Pr from (6) into (5), we have B Pb 0.025 in. 5 1.405 3 1026 Pb 1 0.6406 3 1026 12Pb 2 P b 5 9.307 3 103 lb 5 9.307 kips Pr 5 2P b 5 219.307 kips2 5 18.61 kips Stress in Rod sr 5 18.61 kips Pr 51 2 Ar 4 p11.5 in.2 sr 5 10.53 ksi ◀ 71 bee80288_ch02_052-139.indd Page 72 9/4/10 5:15:48 PM user-f499 /Users/user-f499/Desktop/Temp Work/Don't Delete Job/MHDQ251:Beer:201/ch02 PROBLEMS 2.1 An 80-m-long wire of 5-mm diameter is made of a steel with E 5 200 GPa and an ultimate tensile strength of 400 MPa. If a factor of safety of 3.2 is desired, determine (a) the largest allowable tension in the wire, (b) the corresponding elongation of the wire. 2.2 A steel control rod is 5.5 ft long and must not stretch more than 0.04 in. when a 2-kip tensile load is applied to it. Knowing that E 5 29 3 106 psi, determine (a) the smallest diameter rod that should be used, (b) the corresponding normal stress caused by the load. 2.3 Two gage marks are placed exactly 10 in. apart on a 12-in.-diameter aluminum rod with E 5 10.1 3 106 psi and an ultimate strength of 16 ksi. Knowing that the distance between the gage marks is 10.009 in. after a load is applied, determine (a) the stress in the rod, (b) the factor of safety. 2.4 An 18-m-long steel wire of 5-mm diameter is to be used in the manufacture of a prestressed concrete beam. It is observed that the wire stretches 45 mm when a tensile force P is applied. Knowing that E 5 200 GPa, determine (a) the magnitude of the force P, (b) the corresponding normal stress in the wire. 2.5 A polystyrene rod of length 12 in. and diameter 0.5 in. is subjected to an 800-lb tensile load. Knowing that E 5 0.45 3 106 psi, determine (a) the elongation of the rod, (b) the normal stress in the rod. 2.6 A nylon thread is subjected to a 8.5-N tension force. Knowing that E 5 3.3 GPa and that the length of the thread increases by 1.1%, determine (a) the diameter of the thread, (b) the stress in the thread. 2.7 Two gage marks are placed exactly 250 mm apart on a 12-mmdiameter aluminum rod. Knowing that, with an axial load of 6000 N acting on the rod, the distance between the gage marks is 250.18 mm, determine the modulus of elasticity of the aluminum used in the rod. 2.8 An aluminum pipe must not stretch more than 0.05 in. when it is subjected to a tensile load. Knowing that E 5 10.1 3 106 psi and that the maximum allowable normal stress is 14 ksi, determine (a) the maximum allowable length of the pipe, (b) the required area of the pipe if the tensile load is 127.5 kips. 2.9 An aluminum control rod must stretch 0.08 in. when a 500-lb tensile load is applied to it. Knowing that sall 5 22 ksi and E 5 10.1 3 106 psi, determine the smallest diameter and shortest length that can be selected for the rod. 72 bee80288_ch02_052-139.indd Page 73 9/7/10 5:26:40 PM user-f499 /Users/user-f499/Desktop/Temp Work/Don't Delete Job/MHDQ251:Beer:201/ch07 Problems 2.10 A square yellow-brass bar must not stretch more than 2.5 mm when it is subjected to a tensile load. Knowing that E 5 105 GPa and that the allowable tensile strength is 180 MPa, determine (a) the maximum allowable length of the bar, (b) the required dimensions of the cross section if the tensile load is 40 kN. 2.11 A 4-m-long steel rod must not stretch more than 3 mm and the normal stress must not exceed 150 MPa when the rod is subjected to a 10-kN axial load. Knowing that E 5 200 GPa, determine the required diameter of the rod. 2.12 A nylon thread is to be subjected to a 10-N tension. Knowing that E 5 3.2 GPa, that the maximum allowable normal stress is 40 MPa, and that the length of the thread must not increase by more than 1%, determine the required diameter of the thread. 2.13 The 4-mm-diameter cable BC is made of a steel with E 5 200 GPa. Knowing that the maximum stress in the cable must not exceed 190 MPa and that the elongation of the cable must not exceed 6 mm, find the maximum load P that can be applied as shown. B 2.5 m P 3.5 m A C 4.0 m Fig. P2.13 2.14 The aluminum rod ABC (E 5 10.1 3 106 psi), which consists of two cylindrical portions AB and BC, is to be replaced with a cylindrical steel rod DE (E 5 29 3 106 psi) of the same overall length. Determine the minimum required diameter d of the steel rod if its vertical deformation is not to exceed the deformation of the aluminum rod under the same load and if the allowable stress in the steel rod is not to exceed 24 ksi. 28 kips 28 kips D A 1.5 in. 12 in. B 2.25 in. d 18 in. B C E 4 ft Fig. P2.14 A 2.15 A 4-ft section of aluminum pipe of cross-sectional area 1.75 in2 rests on a fixed support at A. The 58-in.-diameter steel rod BC hangs from a rigid bar that rests on the top of the pipe at B. Knowing that the modulus of elasticity is 29 3 106 psi for steel and 10.4 3 106 psi for aluminum, determine the deflection of point C when a 15-kip force is applied at C. 3 ft C P Fig. P2.15 73 bee80288_ch02_052-139.indd Page 74 9/4/10 5:16:08 PM user-f499 74 Stress and Strain—Axial Loading P D 1 mm A 375 mm /Users/user-f499/Desktop/Temp Work/Don't Delete Job/MHDQ251:Beer:201/ch02 2.16 The brass tube AB (E 5 105 GPa) has a cross-sectional area of 140 mm2 and is fitted with a plug at A. The tube is attached at B to a rigid plate that is itself attached at C to the bottom of an aluminum cylinder (E 5 72 GPa) with a cross-sectional area of 250 mm2. The cylinder is then hung from a support at D. In order to close the cylinder, the plug must move down through 1 mm. Determine the force P that must be applied to the cylinder. 2.17 A 250-mm-long aluminum tube (E 5 70 GPa) of 36-mm outer diameter and 28-mm inner diameter can be closed at both ends by means of single-threaded screw-on covers of 1.5-mm pitch. With one cover screwed on tight, a solid brass rod (E 5 105 GPa) of 25-mm diameter is placed inside the tube and the second cover is screwed on. Since the rod is slightly longer than the tube, it is observed that the cover must be forced against the rod by rotating it one-quarter of a turn before it can be tightly closed. Determine (a) the average normal stress in the tube and in the rod, (b) the deformations of the tube and of the rod. 36 mm C 28 mm B 25 mm Fig. P2.16 250 mm Fig. P2.17 2.18 The specimen shown is made from a 1-in.-diameter cylindrical steel rod with two 1.5-in.-outer-diameter sleeves bonded to the rod as shown. Knowing that E 5 29 3 106 psi, determine (a) the load P so that the total deformation is 0.002 in., (b) the corresponding deformation of the central portion BC. P P' A 20-mm diameter 0.4 m 112 -in. diameter A 1-in. diameter B 112 -in. diameter C 2 in. D 3 in. P 2 in. B Fig. P2.18 Q 0.5 m 60-mm diameter C Fig. P2.19 and P2.20 2.19 Both portions of the rod ABC are made of an aluminum for which E 5 70 GPa. Knowing that the magnitude of P is 4 kN, determine (a) the value of Q so that the deflection at A is zero, (b) the corresponding deflection of B. 2.20 The rod ABC is made of an aluminum for which E 5 70 GPa. Knowing that P 5 6 kN and Q 5 42 kN, determine the deflection of (a) point A, (b) point B. bee80288_ch02_052-139.indd Page 75 9/4/10 5:16:16 PM user-f499 /Users/user-f499/Desktop/Temp Work/Don't Delete Job/MHDQ251:Beer:201/ch02 2.21 Members AB and BC are made of steel (E 5 29 3 106 psi) with cross-sectional areas of 0.80 in2 and 0.64 in2, respectively. For the loading shown, determine the elongation of (a) member AB, (b) member BC. 6 ft 6 ft B Problems P C B C A D 5 ft 6m A D 28 kips E 54 kips Fig. P2.21 2.22 The steel frame (E 5 200 GPa) shown has a diagonal brace BD with an area of 1920 mm2. Determine the largest allowable load P if the change in length of member BD is not to exceed 1.6 mm. 2.23 For the steel truss (E 5 200 GPa) and loading shown, determine the deformations of members AB and AD, knowing that their cross-sectional areas are 2400 mm2 and 1800 mm2, respectively. 228 kN B 2.5 m C D A 4.0 m 4.0 m Fig. P2.23 2.24 For the steel truss (E 5 29 3 106 psi) and loading shown, determine the deformations of members BD and DE, knowing that their cross-sectional areas are 2 in2 and 3 in2, respectively. 8 ft 8 ft 30 kips A 30 kips B 30 kips C D E 8 ft F G 15 ft Fig. P2.24 5m Fig. P2.22 75 bee80288_ch02_052-139.indd Page 76 9/4/10 5:16:25 PM user-f499 76 Stress and Strain—Axial Loading /Users/user-f499/Desktop/Temp Work/Don't Delete Job/MHDQ251:Beer:201/ch02 2.25 Each of the links AB and CD is made of aluminum (E 5 10.9 3 106 psi) and has a cross-sectional area of 0.2 in2. Knowing that they support the rigid member BC, determine the deflection of point E. A D P = 1 kip 18 in. E B C 22 in. 10 in. Fig. P2.25 2.26 The length of the 323 -in.-diameter steel wire CD has been adjusted so that with no load applied, a gap of 161 in. exists between the end B of the rigid beam ACB and a contact point E. Knowing that E 5 29 3 106 psi, determine where a 50-lb block should be placed on the beam in order to cause contact between B and E. D 12.5 in. x C 50 lb B A E 16 in. 4 in. 1 16 in. Fig. P2.26 2.27 Link BD is made of brass (E 5 105 GPa) and has a cross-sectional area of 240 mm2. Link CE is made of aluminum (E 5 72 GPa) and has a cross-sectional area of 300 mm2. Knowing that they support rigid member ABC, determine the maximum force P that can be applied vertically at point A if the deflection of A is not to exceed 0.35 mm. D 225 mm C A B 150 mm P E 125 mm Fig. P2.27 225 mm bee80288_ch02_052-139.indd Page 77 9/4/10 5:16:31 PM user-f499 /Users/user-f499/Desktop/Temp Work/Don't Delete Job/MHDQ251:Beer:201/ch02 2.28 Each of the four vertical links connecting the two rigid horizontal members is made of aluminum (E 5 70 GPa) and has a uniform rectangular cross section of 10 3 40 mm. For the loading shown, determine the deflection of (a) point E, (b) point F, (c) point G. 250 mm 400 mm A 250 mm B 40 mm C D E 300 mm F G 24 kN Fig. P2.28 2.29 The vertical load P is applied at the center A of the upper section of a homogeneous frustum of a circular cone of height h, minimum radius a, and maximum radius b. Denoting by E the modulus of elasticity of the material and neglecting the effect of its weight, determine the deflection of point A. P A a h b Fig. P2.29 2.30 A homogenous cable of length L and uniform cross section is suspended from one end. (a) Denoting by r the density (mass per unit volume) of the cable and by E its modulus of elasticity, determine the elongation of the cable due to its own weight. (b) Show that the same elongation would be obtained if the cable were horizontal and if a force equal to half of its weight were applied at each end. 2.31 The volume of a tensile specimen is essentially constant while plastic deformation occurs. If the initial diameter of the specimen is d1, show that when the diameter is d, the true strain is Pt 5 2 ln(d1yd). 2.32 Denoting by P the “engineering strain” in a tensile specimen, show that the true strain is Pt 5 ln(1 1 P). Problems 77 bee80288_ch02_052-139.indd Page 78 9/4/10 5:16:36 PM user-f499 78 2.9 Stress and Strain—Axial Loading /Users/user-f499/Desktop/Temp Work/Don't Delete Job/MHDQ251:Beer:201/ch02 STATICALLY INDETERMINATE PROBLEMS In the problems considered in the preceding section, we could always use free-body diagrams and equilibrium equations to determine the internal forces produced in the various portions of a member under given loading conditions. The values obtained for the internal forces were then entered into Eq. (2.8) or (2.9) to obtain the deformation of the member. There are many problems, however, in which the internal forces cannot be determined from statics alone. In fact, in most of these problems the reactions themselves—which are external forces— cannot be determined by simply drawing a free-body diagram of the member and writing the corresponding equilibrium equations. The equilibrium equations must be complemented by relations involving deformations obtained by considering the geometry of the problem. Because statics is not sufficient to determine either the reactions or the internal forces, problems of this type are said to be statically indeterminate. The following examples will show how to handle this type of problem. EXAMPLE 2.02 Tube (A2, E2) P Rod (A1, E1) End plate L A rod of length L, cross-sectional area A1, and modulus of elasticity E1, has been placed inside a tube of the same length L, but of cross-sectional area A2 and modulus of elasticity E2 (Fig. 2.21a). What is the deformation of the rod and tube when a force P is exerted on a rigid end plate as shown? Denoting by P1 and P2, respectively, the axial forces in the rod and in the tube, we draw free-body diagrams of all three elements (Fig. 2.21b, c, d). Only the last of the diagrams yields any significant information, namely: (a) P1 P'1 (b) Clearly, one equation is not sufficient to determine the two unknown internal forces P1 and P2. The problem is statically indeterminate. However, the geometry of the problem shows that the deformations d1 and d2 of the rod and tube must be equal. Recalling Eq. (2.7), we write P'2 P2 (c) d1 5 P1L A1E1 d2 5 P2L A2E2 (2.12) Equating the deformations d1 and d2, we obtain: P1 (d) (2.11) P1 1 P2 5 P P2 P2 P1 5 A1E1 A2E2 P (2.13) Equations (2.11) and (2.13) can be solved simultaneously for P1 and P2: Fig. 2.21 P1 5 A1E1P A1E1 1 A2E2 P2 5 A2E2P A1E1 1 A2E2 Either of Eqs. (2.12) can then be used to determine the common deformation of the rod and tube. bee80288_ch02_052-139.indd Page 79 11/8/10 8:06:10 PM user-f499 /Users/user-f499/Desktop/Temp Work/Don't Delete Job/MHDQ251:Beer:201/ch02 A bar AB of length L and uniform cross section is attached to rigid supports at A and B before being loaded. What are the stresses in portions AC and BC due to the application of a load P at point C (Fig. 2.22a)? Drawing the free-body diagram of the bar (Fig. 2.22b), we obtain the equilibrium equation EXAMPLE 2.03 RA A A (2.14) RA 1 RB 5 P Since this equation is not sufficient to determine the two unknown reactions RA and RB, the problem is statically indeterminate. However, the reactions may be determined if we observe from the geometry that the total elongation d of the bar must be zero. Denoting by d1 and d2, respectively, the elongations of the portions AC and BC, we write L1 C C L L2 P P B B RB d 5 d 1 1 d2 5 0 or, expressing d1 and d2 in terms of the corresponding internal forces P1 and P2: d5 P1L1 P2L2 1 50 AE AE (a) (2.15) RA But we note from the free-body diagrams shown respectively in parts b and c of Fig. 2.23 that P1 5 RA and P2 5 2RB. Carrying these values into (2.15), we write RAL1 2 RBL2 5 0 (2.16) Equations (2.14) and (2.16) can be solved simultaneously for RA and RB; we obtain RA 5 PL2yL and RB 5 PL1yL. The desired stresses s1 in AC and s2 in BC are obtained by dividing, respectively, P1 5 RA and P2 5 2RB by the cross-sectional area of the bar: s1 5 PL2 AL s2 5 2 (b) Fig. 2.22 PL1 AL RA A C (b) P1 (a) P2 P (c) B RB RB Fig. 2.23 Superposition Method. We observe that a structure is statically indeterminate whenever it is held by more supports than are required to maintain its equilibrium. This results in more unknown reactions than available equilibrium equations. It is often found convenient to designate one of the reactions as redundant and to eliminate the corresponding support. Since the stated conditions of the problem cannot be arbitrarily changed, the redundant reaction must be maintained in the solution. But it will be treated as an unknown load that, together with the other loads, must produce deformations that are compatible with the original constraints. The actual solution of the problem is carried out by considering separately the deformations caused by the given loads and by the redundant reaction, and by adding—or superposing—the results obtained.† †The general conditions under which the combined effect of several loads can be obtained in this way are discussed in Sec. 2.12. 79 bee80288_ch02_052-139.indd Page 80 9/4/10 5:16:44 PM user-f499 EXAMPLE 2.04 A A ⫽ 250 mm2 150 mm D 300 kN A ⫽ 400 mm2 150 mm C 150 mm K 600 kN B /Users/user-f499/Desktop/Temp Work/Don't Delete Job/MHDQ251:Beer:201/ch02 Determine the reactions at A and B for the steel bar and loading shown in Fig. 2.24, assuming a close fit at both supports before the loads are applied. We consider the reaction at B as redundant and release the bar from that support. The reaction RB is now considered as an unknown load (Fig. 2.25a) and will be determined from the condition that the deformation d of the rod must be equal to zero. The solution is carried out by considering separately the deformation dL caused by the given loads (Fig. 2.25b) and the deformation dR due to the redundant reaction RB (Fig. 2.25c). 150 mm Fig. 2.24 A A 300 kN 300 kN 600 kN A 600 kN ␦⫽ 0 ␦L ␦R RB RB (a) (b) (c) Fig. 2.25 A 150 mm 4 D 300 kN 3 150 mm C 150 mm 2 K 600 kN B 1 150 mm P2 5 P3 5 600 3 103 N P4 5 900 3 103 N P1 5 0 26 2 A3 5 A4 5 250 3 1026 m2 A1 5 A2 5 400 3 10 m L1 5 L2 5 L3 5 L4 5 0.150 m Substituting these values into Eq. (2.8), we obtain Fig. 2.26 A 2 300 mm 1 300 mm C B RB Fig. 2.27 80 The deformation dL is obtained from Eq. (2.8) after the bar has been divided into four portions, as shown in Fig. 2.26. Following the same procedure as in Example 2.01, we write 4 PiLi 600 3 103 N dL 5 a 5 a0 1 400 3 1026 m2 i51 AiE 600 3 103 N 900 3 103 N 0.150 m 1 1 b 26 2 E 250 3 10 m 250 3 1026 m2 1.125 3 109 (2.17) dL 5 E Considering now the deformation dR due to the redundant reaction RB, we divide the bar into two portions, as shown in Fig. 2.27, and write P1 5 P2 5 2RB A1 5 400 3 1026 m2 A2 5 250 3 1026 m2 L1 5 L2 5 0.300 m bee80288_ch02_052-139.indd Page 81 11/8/10 8:06:22 PM user-f499 /Users/user-f499/Desktop/Temp Work/Don't Delete Job/MHDQ251:Beer:201/ch02 Substituting these values into Eq. (2.8), we obtain RA 3 dR 5 11.95 3 10 2RB P1L1 P2L2 1 52 A1E A2E E (2.18) A Expressing that the total deformation d of the bar must be zero, we 300 kN write d 5 dL 1 dR 5 0 (2.19) C and, substituting for dL and dR from (2.17) and (2.18) into (2.19), d5 600 kN 11.95 3 103 2RB 1.125 3 109 2 50 E E B Solving for RB, we have RB 3 RB 5 577 3 10 N 5 577 kN Fig. 2.28 The reaction RA at the upper support is obtained from the freebody diagram of the bar (Fig. 2.28). We write RA 2 300 kN 2 600 kN 1 RB 5 0 1c o Fy 5 0: RA 5 900 kN 2 RB 5 900 kN 2 577 kN 5 323 kN Once the reactions have been determined, the stresses and strains in the bar can easily be obtained. It should be noted that, while the total deformation of the bar is zero, each of its component parts does deform under the given loading and restraining conditions. Determine the reactions at A and B for the steel bar and loading of Example 2.04, assuming now that a 4.50-mm clearance exists between the bar and the ground before the loads are applied (Fig. 2.29). Assume E 5 200 GPa. We follow the same procedure as in Example 2.04. Considering the reaction at B as redundant, we compute the deformations dL and dR caused, respectively, by the given loads and by the redundant reaction RB. However, in this case the total deformation is not zero, but d 5 4.5 mm. We write therefore d 5 dL 1 dR 5 4.5 3 1023 m (2.20) EXAMPLE 2.05 A A A ⫽ 250 mm2 300 mm 300 kN C A ⫽ 400 C mm2 300 mm Substituting for dL and dR from (2.17) and (2.18) into (2.20), and recalling that E 5 200 GPa 5 200 3 109 Pa, we have d5 1.125 3 109 200 3 109 2 11.95 3 103 2R B 200 3 109 5 4.5 3 1023 m Solving for RB, we obtain 600 kN ␦ 4.5 mm B B Fig. 2.29 RB 5 115.4 3 103 N 5 115.4 kN The reaction at A is obtained from the free-body diagram of the bar (Fig. 2.28): RA 2 300 kN 2 600 kN 1 RB 5 0 1c o Fy 5 0: RA 5 900 kN 2 RB 5 900 kN 2 115.4 kN 5 785 kN 81 bee80288_ch02_052-139.indd Page 82 9/4/10 5:17:00 PM user-f499 82 Stress and Strain—Axial Loading 2.10 /Users/user-f499/Desktop/Temp Work/Don't Delete Job/MHDQ251:Beer:201/ch02 PROBLEMS INVOLVING TEMPERATURE CHANGES All of the members and structures that we have considered so far were assumed to remain at the same temperature while they were being loaded. We are now going to consider various situations involving changes in temperature. Let us first consider a homogeneous rod AB of uniform cross section, which rests freely on a smooth horizontal surface (Fig. 2.30a). If the temperature of the rod is raised by DT, we observe that the rod elongates by an amount dT which is proportional to both the temperature change DT and the length L of the rod (Fig. 2.30b). We have (2.21) dT 5 a(DT)L where a is a constant characteristic of the material, called the coefficient of thermal expansion. Since dT and L are both expressed in units of length, a represents a quantity per degree C, or per degree F, depending whether the temperature change is expressed in degrees Celsius or in degrees Fahrenheit. L A B (a) L A ␦T B (b) Fig. 2.30 Elongation of rod due to temperature increase. With the deformation dT must be associated a strain PT 5 dTyL. Recalling Eq. (2.21), we conclude that PT 5 aDT (2.22) The strain PT is referred to as a thermal strain, since it is caused by the change in temperature of the rod. In the case we are considering here, there is no stress associated with the strain PT. Let us now assume that the same rod AB of length L is placed between two fixed supports at a distance L from each other (Fig. 2.31a). Again, there is neither stress nor strain in this initial condition. If we raise the temperature by DT, the rod cannot elongate because of the restraints imposed on its ends; the elongation dT of the rod is thus zero. Since the rod is homogeneous and of uniform cross section, the strain PT at any point is PT 5 dTyL and, thus, also zero. However, the supports will exert equal and opposite forces P and P9 on the rod after the temperature has been raised, to keep it bee80288_ch02_052-139.indd Page 83 9/4/10 5:17:02 PM user-f499 /Users/user-f499/Desktop/Temp Work/Don't Delete Job/MHDQ251:Beer:201/ch02 2.10 Problems Involving Temperature Changes L A B (a) P P' A B (b) Fig. 2.31 Rod with ends restrained against thermal expansion. from elongating (Fig. 2.31b). It thus follows that a state of stress (with no corresponding strain) is created in the rod. As we prepare to determine the stress s created by the temperature change DT, we observe that the problem we have to solve is statically indeterminate. Therefore, we should first compute the magnitude P of the reactions at the supports from the condition that the elongation of the rod is zero. Using the superposition method described in Sec. 2.9, we detach the rod from its support B (Fig. 2.32a) and let it elongate freely as it undergoes the temperature change DT (Fig. 2.32b). According to formula (2.21), the corresponding elongation is dT 5 a(DT)L L A (a) ␦P A B P L (c) Expressing that the total deformation d must be zero, we have PL 50 AE from which we conclude that P 5 2AEa(DT) and that the stress in the rod due to the temperature change DT is s5 B (b) PL AE d 5 dT 1 dP 5 a1 ¢T2L 1 ␦T A Applying now to end B the force P representing the redundant reaction, and recalling formula (2.7), we obtain a second deformation (Fig. 2.32c) dP 5 B P 5 2Ea1 ¢T2 A (2.23) It should be kept in mind that the result we have obtained here and our earlier remark regarding the absence of any strain in the rod apply only in the case of a homogeneous rod of uniform cross section. Any other problem involving a restrained structure undergoing a change in temperature must be analyzed on its own merits. However, the same general approach can be used, i.e., we can consider separately the deformation due to the temperature change and the deformation due to the redundant reaction and superpose the solutions obtained. Fig. 2.32 Superposition method applied to rod restrained against thermal expansion. 83 bee80288_ch02_052-139.indd Page 84 9/4/10 5:17:07 PM user-f499 EXAMPLE 2.06 A ⫽ 0.6 in2 A 12 in. Fig. 2.33 A ⫽ 1.2 in2 B C 12 in. /Users/user-f499/Desktop/Temp Work/Don't Delete Job/MHDQ251:Beer:201/ch02 Determine the values of the stress in portions AC and CB of the steel bar shown (Fig. 2.33) when the temperature of the bar is 2508F, knowing that a close fit exists at both of the rigid supports when the temperature is 1758F. Use the values E 5 29 3 106 psi and a 5 6.5 3 106/8F for steel. We first determine the reactions at the supports. Since the problem is statically indeterminate, we detach the bar from its support at B and let it undergo the temperature change ¢T 5 1250°F2 2 175°F2 5 2125°F The corresponding deformation (Fig. 2.34b) is dT 5 a1 ¢T2L 5 16.5 3 1026/°F2 12125°F2 124 in.2 5 219.50 3 1023 in. C A B (a) ␦T B C A 1 2 L1 L2 ␦R (b) C A 1 B 2 RB (c) Fig. 2.34 Applying now the unknown force RB at end B (Fig. 2.34c), we use Eq. (2.8) to express the corresponding deformation dR. Substituting L 1 5 L2 5 12 in. A 2 5 1.2 in2 A 1 5 0.6 in2 P1 5 P2 5 RB E 5 29 3 106 psi into Eq. (2.8), we write P 1L 1 P 2L 2 1 A 1E A 2E RB 12 in. 12 in. 1 b 5 a 6 2 1.2 in2 29 3 10 psi 0.6 in 5 11.0345 3 1026 in./lb2RB dR 5 Expressing that the total deformation of the bar must be zero as a result of the imposed constraints, we write d 5 dT 1 dR 5 0 5 219.50 3 1023 in. 1 11.0345 3 1026 in./lb2RB 5 0 84 bee80288_ch02_052-139.indd Page 85 11/1/10 11:30:03 PM user-f499 /Users/user-f499/Desktop/Temp Work/Don't Delete Job/MHDQ251:Beer:201/ch02 from which we obtain R B 5 18.85 3 103 lb 5 18.85 kips The reaction at A is equal and opposite. Noting that the forces in the two portions of the bar are P1 5 P2 5 18.85 kips, we obtain the following values of the stress in portions AC and CB of the bar: 18.85 kips P1 5 5 131.42 ksi A1 0.6 in2 18.85 kips P2 5 5 115.71 ksi s2 5 A2 1.2 in2 s1 5 We cannot emphasize too strongly the fact that, while the total deformation of the bar must be zero, the deformations of the portions AC and CB are not zero. A solution of the problem based on the assumption that these deformations are zero would therefore be wrong. Neither can the values of the strain in AC or CB be assumed equal to zero. To amplify this point, let us determine the strain PAC in portion AC of the bar. The strain PAC can be divided into two component parts; one is the thermal strain PT produced in the unrestrained bar by the temperature change DT (Fig. 2.34b). From Eq. (2.22) we write PT 5 a ¢T 5 16.5 3 1026/°F2 12125°F2 5 2812.5 3 1026 in./in. The other component of PAC is associated with the stress s1 due to the force RB applied to the bar (Fig. 2.34c). From Hooke’s law, we express this component of the strain as 131.42 3 103 psi s1 5 5 11083.4 3 1026 in./in. E 29 3 106 psi Adding the two components of the strain in AC, we obtain s1 5 2812.5 3 1026 1 1083.4 3 1026 E 5 1271 3 1026 in./in. PAC 5 PT 1 A similar computation yields the strain in portion CB of the bar: s2 5 2812.5 3 1026 1 541.7 3 1026 E 5 2271 3 1026 in./in. PCB 5 PT 1 The deformations dAC and dCB of the two portions of the bar are expressed respectively as dAC 5 PAC 1AC2 5 11271 3 1026 2 112 in.2 5 13.25 3 1023 in. dCB 5 PCB 1CB2 5 12271 3 1026 2 112 in.2 5 23.25 3 1023 in. We thus check that, while the sum d 5 dAC 1 dCB of the two deformations is zero, neither of the deformations is zero. 85 bee80288_ch02_052-139.indd Page 86 11/1/10 11:30:05 PM user-f499 12 in. 8 in. 18 in. B A SAMPLE PROBLEM 2.3 D C 24 in. 10 kips /Users/user-f499/Desktop/Temp Work/Don't Delete Job/MHDQ251:Beer:201/ch02 30 in. The 12 -in.-diameter rod CE and the 34 -in.-diameter rod DF are attached to the rigid bar ABCD as shown. Knowing that the rods are made of aluminum and using E 5 10.6 3 106 psi, determine (a) the force in each rod caused by the loading shown, (b) the corresponding deflection of point A. E F SOLUTION 12 in. 8 in. 18 in. A B Bx By 10 kips FDF FCE 12 in. 18 in. B A D C C A' A 8 in. D' C' C Statics. Considering the free body of bar ABCD, we note that the reaction at B and the forces exerted by the rods are indeterminate. However, using statics, we may write 110 kips2 118 in.2 2 F CE 112 in.2 2 F DF 120 in.2 5 0 12FCE 1 20FDF 5 180 (1) 1 l o M B 5 0: Geometry. After application of the 10-kip load, the position of the bar is A9BC9D9. From the similar triangles BAA9, BCC9, and BDD9 we have D D FCE FDF C C 24 in. E D 1 2 in. 3 4 in. D Deformations. dC dD 5 12 in. 20 in. dC 5 0.6dD (2) dD dA 5 18 in. 20 in. dA 5 0.9dD (3) Using Eq. (2.7), we have dC 5 30 in. FCELCE ACEE dD 5 FDFLDF ADFE Substituting for dC and dD into (2), we write dC 5 0.6dD F FCE 5 0.6 FCELCE FDFLDF 5 0.6 ACEE ADFE 1 1 2 LDF ACE 30 in. 4p1 2 in.2 FDF 5 0.6 a bc 1 3 d FDF FCE 5 0.333FDF LCE ADF 24 in. 4p1 4 in.2 2 Force in Each Rod. Substituting for FCE into (1) and recalling that all forces have been expressed in kips, we have 1210.333FDF 2 1 20FDF 5 180 F CE 5 0.333F DF 5 0.33317.50 kips2 Deflections. dD 5 FDF 5 7.50 kips ◀ F CE 5 2.50 kips ◀ The deflection of point D is 17.50 3 103 lb2 130 in.2 FDFLDF 51 3 2 6 ADFE 4 p1 4 in.2 110.6 3 10 psi2 dD 5 48.0 3 1023 in. Using (3), we write dA 5 0.9dD 5 0.9148.0 3 1023 in.2 86 dA 5 43.2 3 1023 in. ◀ bee80288_ch02_052-139.indd Page 87 11/1/10 11:30:09 PM user-f499 0.45 m /Users/user-f499/Desktop/Temp Work/Don't Delete Job/MHDQ251:Beer:201/ch02 0.3 m C SAMPLE PROBLEM 2.4 E The rigid bar CDE is attached to a pin support at E and rests on the 30-mmdiameter brass cylinder BD. A 22-mm-diameter steel rod AC passes through a hole in the bar and is secured by a nut which is snugly fitted when the temperature of the entire assembly is 208C. The temperature of the brass cylinder is then raised to 508C while the steel rod remains at 208C. Assuming that no stresses were present before the temperature change, determine the stress in the cylinder. D 0.3 m B 0.9 m Rod AC: Steel E 5 200 GPa a 5 11.7 3 1026/°C A C E D SOLUTION Ex A Statics. Considering the free body of the entire assembly, we write Ey B 1l o M E 5 0: R A 10.75 m2 2 R B 10.3 m2 5 0 R A 5 0.4R B (1) RB Deformations. We use the method of superposition, considering RB as redundant. With the support at B removed, the temperature rise of the cylinder causes point B to move down through dT. The reaction RB must cause a deflection d1 equal to dT so that the final deflection of B will be zero (Fig. 3). 0.3 m Deflection dT. Because of a temperature rise of 508 2 208 5 308C, the length of the brass cylinder increases by dT. dT 5 L1 ¢T2a 5 10.3 m2 130°C2 120.9 3 1026/°C2 5 188.1 3 1026 m w RA 0.45 m Cylinder BD: Brass E 5 105 GPa a 5 20.9 3 1026/°C C C E D D  C 0.3   0.4 C 0.75 C D E B B T A C D C B RB  1 A 1 A 2 3 RA Deflection d1. We note that dD 5 0.4dC and d1 5 dD 1 dByD. RA 10.9 m2 RAL 51 5 11.84 3 1029RA x dC 5 2 AE 4 p10.022 m2 1200 GPa2 dD 5 0.40dC 5 0.4111.84 3 1029RA 2 5 4.74 3 1029RAx RB 10.3 m2 RBL 51 dByD 5 5 4.04 3 1029RB x 2 AE 4 p10.03 m2 1105 GPa2 We recall from (1) that RA 5 0.4RB and write d1 5 dD 1 dByD 5 3 4.7410.4RB 2 1 4.04RB 4 1029 5 5.94 3 1029RB x But dT 5 d1: 188.1 3 1026 m 5 5.94 3 1029 R B Stress in Cylinder: sB 5 RB 31.7 kN 51 2 A 4 p10.03 m2 R B 5 31.7 kN sB 5 44.8 MPa ◀ 87 bee80288_ch02_052-139.indd Page 88 9/4/10 5:17:22 PM user-f499 /Users/user-f499/Desktop/Temp Work/Don't Delete Job/MHDQ251:Beer:201/ch02 PROBLEMS 25 mm Brass core E  105 GPa 2.33 An axial force of 200 kN is applied to the assembly shown by means of rigid end plates. Determine (a) the normal stress in the aluminum shell, (b) the corresponding deformation of the assembly. 2.34 The length of the assembly shown decreases by 0.40 mm when an axial force is applied by means of rigid end plates. Determine (a) the magnitude of the applied force, (b) the corresponding stress in the brass core. 300 mm Aluminium shell E  70 GPa 2.35 A 4-ft concrete post is reinforced with four steel bars, each with a 34-in. diameter. Knowing that Es 5 29 3 106 psi and Ec 5 3.6 3 106 psi, determine the normal stresses in the steel and in the concrete when a 150-kip axial centric force P is applied to the post. P 60 mm Fig. P2.33 and P2.34 4 ft 8 in. 8 in. Fig. P2.35 2.36 A 250-mm bar of 150 3 30-mm rectangular cross section consists of two aluminum layers, 5 mm thick, brazed to a center brass layer of the same thickness. If it is subjected to centric forces of magnitude P 5 30 kN, and knowing that Ea 5 70 GPa and Eb 5 105 GPa, determine the normal stress (a) in the aluminum layers, (b) in the brass layer. P' 250 mm 5 mm 5 mm Aluminum Brass Aluminum 5 mm 30 mm P Fig. P2.36 2.37 Determine the deformation of the composite bar of Prob. 2.36 if it is subjected to centric forces of magnitude P 5 45 kN. 88 bee80288_ch02_052-139.indd Page 89 9/4/10 5:17:29 PM user-f499 /Users/user-f499/Desktop/Temp Work/Don't Delete Job/MHDQ251:Beer:201/ch02 Problems 2.38 Compressive centric forces of 40 kips are applied at both ends of the assembly shown by means of rigid end plates. Knowing that Es 5 29 3 106 psi and Ea 5 10.1 3 106 psi, determine (a) the normal stresses in the steel core and the aluminum shell, (b) the deformation of the assembly. 2.39 Three wires are used to suspend the plate shown. Aluminum wires of 18-in. diameter are used at A and B while a steel wire of 121 -in. diameter is used at C. Knowing that the allowable stress for aluminum (Ea 5 10.4 3 106 psi) is 14 ksi and that the allowable stress for steel (Es 5 29 3 106 psi) is 18 ksi, determine the maximum load P that can be applied. L 10 in. 1 in. Aluminum shell Steel core 2.5 in. Fig. P2.38 A B L C A 25 in. P 1.25 in. 6 kips 6 kips Fig. P2.39 B 2.40 A polystyrene rod consisting of two cylindrical portions AB and BC is restrained at both ends and supports two 6-kip loads as shown. Knowing that E 5 0.45 3 106 psi, determine (a) the reactions at A and C, (b) the normal stress in each portion of the rod. 2 in. 15 in. C Fig. P2.40 2.41 Two cylindrical rods, one of steel and the other of brass, are joined at C and restrained by rigid supports at A and E. For the loading shown and knowing that Es 5 200 GPa and Eb 5 105 GPa, determine (a) the reactions at A and E, (b) the deflection of point C. Dimensions in mm 180 100 120 A C Steel B 60 kN 40-mm diam. 100 D Brass E 40 kN 30-mm diam. Fig. P2.41 A B C 2.42 Solve Prob. 2.41, assuming that rod AC is made of brass and rod CE is made of steel. 2.43 The rigid bar ABCD is suspended from four identical wires. Determine the tension in each wire caused by the load P shown. D P L Fig. P2.43 L L 89 bee80288_ch02_052-139.indd Page 90 11/1/10 11:30:16 PM user-f499 90 2.44 The rigid bar AD is supported by two steel wires of 161 -in. diameter (E 5 29 3 106 psi) and a pin and bracket at D. Knowing that the wires were initially taut, determine (a) the additional tension in each wire when a 120-lb load P is applied at B, (b) the corresponding deflection of point B. Stress and Strain—Axial Loading E F 15 in. 8 in. A B C 8 in. 8 in. /Users/user-f499/Desktop/Temp Work/Don't Delete Job/MHDQ251:Beer:201/ch02 D 2.45 The steel rods BE and CD each have a 16-mm diameter (E 5 200 GPa); the ends of the rods are single-threaded with a pitch of 2.5 mm. Knowing that after being snugly fitted, the nut at C is tightened one full turn, determine (a) the tension in rod CD, (b) the deflection of point C of the rigid member ABC. 8 in. A P 150 mm Fig. P2.44 B 100 mm D E C 2m 3m Fig. P2.45 A 2.46 Links BC and DE are both made of steel (E 5 29 3 106 psi) and are 12 in. wide and 14 in. thick. Determine (a) the force in each link when a 600-lb force P is applied to the rigid member AF shown, (b) the corresponding deflection of point A. P 4 in. B C 2 in. 2.47 The concrete post (Ec 5 3.6 3 106 psi and ac 5 5.5 3 1026/ 8F) is reinforced with six steel bars, each of 78-in diameter (Es 5 29 3 106 psi and as 5 6.5 3 1026/ 8F). Determine the normal stresses induced in the steel and in the concrete by a temperature rise of 658F. E D 2 in. F 4 in. 5 in. Fig. P2.46 6 ft 8 in. Aluminum shell 1.25 in. Fig. P2.48 10 in. 0.75 in. Steel core 10 in. Fig. P2.47 2.48 The assembly shown consists of an aluminum shell (Ea 5 10.6 3 106 psi, aa 5 12.9 3 1026/ 8F) fully bonded to a steel core (Es 5 29 3 106 psi, as 5 6.5 3 1026/ 8F) and is unstressed. Determine (a) the largest allowable change in temperature if the stress in the aluminum shell is not to exceed 6 ksi, (b) the corresponding change in length of the assembly. bee80288_ch02_052-139.indd Page 91 9/4/10 5:17:48 PM user-f499 /Users/user-f499/Desktop/Temp Work/Don't Delete Job/MHDQ251:Beer:201/ch02 2.49 The aluminum shell is fully bonded to the brass core and the assembly is unstressed at a temperature of 158C. Considering only axial deformations, determine the stress in the aluminum when the temperature reaches 1958C. Problems 25 mm Brass core E  105 GPa   20.9  10–6/C 2.50 Solve Prob. 2.49, assuming that the core is made of steel (Es 5 200 GPa, as 5 11.7 3 1026/8C) instead of brass. 2.51 A rod consisting of two cylindrical portions AB and BC is restrained at both ends. Portion AB is made of steel (Es 5 200 GPa, as 5 11.7 3 1026/8C) and portion BC is made of brass (Eb 5 105 GPa, ab 5 20.9 3 1026/8C). Knowing that the rod is initially unstressed, determine the compressive force induced in ABC when there is a temperature rise of 508C. A 30-mm diameter 250 mm B 50-mm diameter 300 mm C Fig. P2.51 2.52 A steel railroad track (Es 5 200 GPa, as 5 11.7 3 1026/8C) was laid out at a temperature of 68C. Determine the normal stress in the rails when the temperature reaches 488C, assuming that the rails (a) are welded to form a continuous track, (b) are 10 m long with 3-mm gaps between them. 2.53 A rod consisting of two cylindrical portions AB and BC is restrained at both ends. Portion AB is made of steel (Es 5 29 3 106 psi, as 5 6.5 3 1026/ 8F) and portion BC is made of aluminum (Ea 5 10.4 3 106 psi, aa 5 13.3 3 1026/ 8F). Knowing that the rod is initially unstressed, determine (a) the normal stresses induced in portions AB and BC by a temperature rise of 708F, (b) the corresponding deflection of point B. 24 in. A 32 in. B C 1 2 14 -in. diameter 1 2 -in. diameter Fig. P2.53 2.54 Solve Prob. 2.53, assuming that portion AB of the composite rod is made of aluminum and portion BC is made of steel. Aluminum shell E  70 GPa   23.6  10–6/C 60 mm Fig. P2.49 91 bee80288_ch02_052-139.indd Page 92 11/1/10 11:30:28 PM user-f499 92 /Users/user-f499/Desktop/Temp Work/Don't Delete Job/MHDQ251:Beer:201/ch02 2.55 A brass link (Eb 5 105 GPa, ab 5 20.9 3 1026/8C) and a steel rod (Es 5 200 GPa, as 5 11.7 3 1026/8C) have the dimensions shown at a temperature of 208C. The steel rod is cooled until it fits freely into the link. The temperature of the whole assembly is then raised to 458C. Determine (a) the final normal stress in the steel rod, (b) the final length of the steel rod. Stress and Strain—Axial Loading A 50 mm Brass 37.5 mm 37.5 mm 0.12 mm 250 mm 30-mm diameter Steel A Section A-A Fig. P2.55 P⬘ 2m 15 mm Steel 5 mm Brass P Steel 40 mm Fig. P2.56 0.02 in. 14 in. Bronze A  2.4 in2 E  15  106 psi  12  10 –6/F 18 in. Aluminum A  2.8 in2 E  10.6  106 psi  12.9  10 –6/F Fig. P2.58 and P2.59 2.56 Two steel bars (Es 5 200 GPa and as 5 11.7 3 1026/ 8C) are used to reinforce a brass bar (Eb 5 105 GPa, ab 5 20.9 3 1026/ 8C) that is subjected to a load P 5 25 kN. When the steel bars were fabricated, the distance between the centers of the holes that were to fit on the pins was made 0.5 mm smaller than the 2 m needed. The steel bars were then placed in an oven to increase their length so that they would just fit on the pins. Following fabrication, the temperature in the steel bars dropped back to room temperature. Determine (a) the increase in temperature that was required to fit the steel bars on the pins, (b) the stress in the brass bar after the load is applied to it. 2.57 Determine the maximum load P that can be applied to the brass bar of Prob. 2.56 if the allowable stress in the steel bars is 30 MPa and the allowable stress in the brass bar is 25 MPa. 2.58 Knowing that a 0.02-in. gap exists when the temperature is 758F, determine (a) the temperature at which the normal stress in the aluminum bar will be equal to 211 ksi, (b) the corresponding exact length of the aluminum bar. 2.59 Determine (a) the compressive force in the bars shown after a temperature rise of 1808F, (b) the corresponding change in length of the bronze bar. 2.60 At room temperature (208C) a 0.5-mm gap exists between the ends of the rods shown. At a later time when the temperature has reached 1408C, determine (a) the normal stress in the aluminum rod, (b) the change in length of the aluminum rod. 0.5 mm 300 mm A Aluminum A  2000 mm2 E  75 GPa  23  16–6/C Fig. P2.60 250 mm B Stainless steel A  800 mm2 E  190 GPa  17.3  10–6/C bee80288_ch02_052-139.indd Page 93 11/8/10 8:06:36 PM user-f499 2.11 /Users/user-f499/Desktop/Temp Work/Don't Delete Job/MHDQ251:Beer:201/ch02 2.11 Poisson’s Ratio POISSON’S RATIO We saw in the earlier part of this chapter that, when a homogeneous slender bar is axially loaded, the resulting stress and strain satisfy Hooke’s law, as long as the elastic limit of the material is not exceeded. Assuming that the load P is directed along the x axis (Fig. 2.35a), we have sx 5 PyA, where A is the cross-sectional area of the bar, and, from Hooke’s law, y A z (2.24) Px 5 sxyE where E is the modulus of elasticity of the material. We also note that the normal stresses on faces respectively perpendicular to the y and z axes are zero: sy 5 sz 5 0 (Fig. 2.35b). It would be tempting to conclude that the corresponding strains Py and Pz are also zero. This, however, is not the case. In all engineering materials, the elongation produced by an axial tensile force P in the direction of the force is accompanied by a contraction in any transverse direction (Fig. 2.36).† In this section and the following sections (Secs. 2.12 through 2.15), all materials considered will be assumed to be both homogeneous and isotropic, i.e., their mechanical properties will be assumed independent of both position and direction. It follows that the strain must have the same value for any transverse direction. Therefore, for the loading shown in Fig. 2.35 we must have Py 5 Pz. This common value is referred to as the lateral strain. An important constant for a given material is its Poisson’s ratio, named after the French mathematician Siméon Denis Poisson (1781–1840) and denoted by the Greek letter n (nu). It is defined as P (a) ␴y ⫽ 0 ␴z ⫽ 0 ␴x ⫽ P A (b) Fig. 2.35 Stresses in an axiallyloaded bar. P' lateral strain n52 axial strain (2.25) or n52 Py Px 52 P Pz Px (2.26) for the loading condition represented in Fig. 2.35. Note the use of a minus sign in the above equations to obtain a positive value for n, the axial and lateral strains having opposite signs for all engineering materials.‡ Solving Eq. (2.26) for Py and Pz, and recalling (2.24), we write the following relations, which fully describe the condition of strain under an axial load applied in a direction parallel to the x axis: Px 5 sx E Py 5 Pz 5 2 nsx E (2.27) †It would also be tempting, but equally wrong, to assume that the volume of the rod remains unchanged as a result of the combined effect of the axial elongation and transverse contraction (see Sec. 2.13). ‡However, some experimental materials, such as polymer foams, expand laterally when stretched. Since the axial and lateral strains have then the same sign, the Poisson’s ratio of these materials is negative. (See Roderic Lakes, “Foam Structures with a Negative Poisson’s Ratio,” Science, 27 February 1987, Volume 235, pp. 1038–1040.) Fig. 2.36 Transverse contraction of bar under axial tensile force. x 93 bee80288_ch02_052-139.indd Page 94 11/8/10 9:32:56 PM user-f494 EXAMPLE 2.07 y L ⫽ 500 mm ␦ x ⫽ 300 ␮m volume 201/FREE048/work%0/indd%0/ A 500-mm-long, 16-mm-diameter rod made of a homogenous, isotropic material is observed to increase in length by 300 mm, and to decrease in diameter by 2.4 mm when subjected to an axial 12-kN load. Determine the modulus of elasticity and Poisson’s ratio of the material. The cross-sectional area of the rod is A 5 pr 2 5 p18 3 1023 m2 2 5 201 3 1026 m 2 Choosing the x axis along the axis of the rod (Fig. 2.37), we write z d ⫽ 16 mm ␦y ⫽ – 2.4 ␮m x 12 kN P 12 3 103 N 5 5 59.7 MPa A 201 3 1026 m2 300 mm dx Px 5 5 5 600 3 1026 500 mm L dy 22.4 mm 5 2150 3 1026 Py 5 5 16 mm d sx 5 Fig. 2.37 From Hooke’s law, sx 5 EPx, we obtain E5 sx 59.7 MPa 5 5 99.5 GPa Px 600 3 1026 and, from Eq. (2.26), n52 2.12 ␴y ␴x ␴z ␴z ␴x ␴y Fig. 2.38 Stress state for multiaxial loading. 94 Py Px 52 2150 3 1026 5 0.25 600 3 1026 MULTIAXIAL LOADING; GENERALIZED HOOKE’S LAW All the examples considered so far in this chapter have dealt with slender members subjected to axial loads, i.e., to forces directed along a single axis. Choosing this axis as the x axis, and denoting by P the internal force at a given location, the corresponding stress components were found to be sx 5 PyA, sy 5 0, and sz 5 0. Let us now consider structural elements subjected to loads acting in the directions of the three coordinate axes and producing normal stresses sx, sy, and sz which are all different from zero (Fig. 2.38). This condition is referred to as a multiaxial loading. Note that this is not the general stress condition described in Sec. 1.12, since no shearing stresses are included among the stresses shown in Fig. 2.38. Consider an element of an isotropic material in the shape of a cube (Fig. 2.39a). We can assume the side of the cube to be equal to unity, since it is always possible to select the side of the cube as a unit of length. Under the given multiaxial loading, the element will deform into a rectangular parallelepiped of sides equal, respectively, to 1 1 Px, 1 1 Py, and 1 1 Pz, where Px, Py, and Pz denote the values of the normal strain in the directions of the three coordinate axes (Fig. 2.39b). You should note that, as a result of the deformations of bee80288_ch02_052-139.indd Page 95 11/8/10 8:06:59 PM user-f499 /Users/user-f499/Desktop/Temp Work/Don't Delete Job/MHDQ251:Beer:201/ch02 the other elements of the material, the element under consideration could also undergo a translation, but we are concerned here only with the actual deformation of the element, and not with any possible superimposed rigid-body displacement. In order to express the strain components Px, Py, Pz in terms of the stress components sx, sy, sz, we will consider separately the effect of each stress component and combine the results obtained. The approach we propose here will be used repeatedly in this text, and is based on the principle of superposition. This principle states that the effect of a given combined loading on a structure can be obtained by determining separately the effects of the various loads and combining the results obtained, provided that the following conditions are satisfied: 2.12 Multiaxial Loading; Generalized Hooke’s Law y 1 1 1 z y ␴y not affect the conditions of application of the other loads. In the case of a multiaxial loading, the first condition will be satisfied if the stresses do not exceed the proportional limit of the material, and the second condition will also be satisfied if the stress on any given face does not cause deformations of the other faces that are large enough to affect the computation of the stresses on those faces. Considering first the effect of the stress component sx, we recall from Sec. 2.11 that sx causes a strain equal to sxyE in the x direction, and strains equal to 2nsxyE in each of the y and z directions. Similarly, the stress component sy, if applied separately, will cause a strain syyE in the y direction and strains 2nsyyE in the other two directions. Finally, the stress component sz causes a strain szyE in the z direction and strains 2nszyE in the x and y directions. Combining the results obtained, we conclude that the components of strain corresponding to the given multiaxial loading are ns y sx ns z 2 2 E E E s nsx nsz y 1 2 Py 5 2 E E E nsy nsx sz Pz 5 2 2 1 E E E Px 5 1 (2.28) The relations (2.28) are referred to as the generalized Hooke’s law for the multiaxial loading of a homogeneous isotropic material. As we indicated earlier, the results obtained are valid only as long as the stresses do not exceed the proportional limit, and as long as the deformations involved remain small. We also recall that a positive value for a stress component signifies tension, and a negative value compression. Similarly, a positive value for a strain component indicates expansion in the corresponding direction, and a negative value contraction. x (a) 1. Each effect is linearly related to the load that produces it. 2. The deformation resulting from any given load is small and does 1 ⫹ ⑀x 1 ⫹ ⑀y ␴z ␴x 1 ⫹ ⑀z z (b) x Fig. 2.39 Deformation of cube under multiaxial loading. 95 bee80288_ch02_052-139.indd Page 96 11/8/10 8:07:08 PM user-f499 EXAMPLE 2.08 y z 2 in. C A D 3 in. 4 in. B Fig. 2.40 /Users/user-f499/Desktop/Temp Work/Don't Delete Job/MHDQ251:Beer:201/ch02 The steel block shown (Fig. 2.40) is subjected to a uniform pressure on all its faces. Knowing that the change in length of edge AB is 21.2 3 1023 in., determine (a) the change in length of the other two edges, (b) the pressure p applied to the faces of the block. Assume E 5 29 3 106 psi and n 5 0.29. (a) Change in Length of Other Edges. Substituting sx 5 sy 5 sz 5 2p into the relations (2.28), we find that the three strain components have the common value x Px 5 Py 5 Pz 5 2 p E 11 2 2n2 (2.29) Since Px 5 dxyAB 5 121.2 3 1023 in.2y14 in.2 5 2300 3 1026 in./in. we obtain Py 5 Pz 5 Px 5 2300 3 1026 in./in. from which it follows that dy 5 Py 1BC2 5 12300 3 1026 2 12 in.2 5 2600 3 1026 in. dz 5 Pz 1BD2 5 12300 3 1026 2 13 in.2 5 2900 3 1026 in. (b) Pressure. p52 *2.13 Solving Eq. (2.29) for p, we write 129 3 106 psi2 12300 3 1026 2 EPx 52 1 2 2n 1 2 0.58 p 5 20.7 ksi DILATATION; BULK MODULUS In this section you will examine the effect of the normal stresses sx, sy, and sz on the volume of an element of isotropic material. Consider the element shown in Fig. 2.39. In its unstressed state, it is in the shape of a cube of unit volume; and under the stresses sx, sy, sz, it deforms into a rectangular parallelepiped of volume v 5 (1 1 Px)(1 1 Py)(1 1 Pz) Since the strains Px, Py, Pz are much smaller than unity, their products will be even smaller and may be omitted in the expansion of the product. We have, therefore, v 5 1 1 Px 1 Py 1 Pz Denoting by e the change in volume of our element, we write e 5 v 2 1 5 1 1 Px 1 Py 1 Pz 2 1 or e 5 Px 1 Py 1 Pz 96 (2.30) bee80288_ch02_052-139.indd Page 97 11/8/10 9:34:11 PM user-f494 volume 201/FREE048/work%0/indd%0/ Since the element had originally a unit volume, the quantity e represents the change in volume per unit volume; it is referred to as the dilatation of the material. Substituting for Px, Py, and Pz from Eqs. (2.28) into (2.30), we write sx 1 s y 1 s z 2n1sx 1 sy 1 sz 2 e5 2 E E 1 2 2n 1sx 1 sy 1 sz 2 E e5 (2.31)† A case of special interest is that of a body subjected to a uniform hydrostatic pressure p. Each of the stress components is then equal to 2p and Eq. (2.31) yields e52 311 2 2n2 p E (2.32) Introducing the constant k5 E 311 2 2n2 (2.33) we write Eq. (2.32) in the form e52 p (2.34) k The constant k is known as the bulk modulus or modulus of compression of the material. It is expressed in the same units as the modulus of elasticity E, that is, in pascals or in psi. Observation and common sense indicate that a stable material subjected to a hydrostatic pressure can only decrease in volume; thus the dilatation e in Eq. (2.34) is negative, from which it follows that the bulk modulus k is a positive quantity. Referring to Eq. (2.33), we conclude that 1 2 2n . 0, or n , 12. On the other hand, we recall from Sec. 2.11 that n is positive for all engineering materials. We thus conclude that, for any engineering material, 0,n, 1 2 (2.35) We note that an ideal material having a value of v equal to zero could be stretched in one direction without any lateral contraction. On the other hand, an ideal material for which n 5 12, and thus k 5 `, would be perfectly incompressible (e 5 0). Referring to Eq. (2.31) we also note that, since n , 12 in the elastic range, stretching an engineering material in one direction, for example in the x direction (sx . 0, sy 5 sz 5 0), will result in an increase of its volume (e . 0).‡ †Since the dilatation e represents a change in volume, it must be independent of the orientation of the element considered. It then follows from Eqs. (2.30) and (2.31) that the quantities Px 1 Py 1 Pz and sx 1 sy 1 sz are also independent of the orientation of the element. This property will be verified in Chap. 7. ‡However, in the plastic range, the volume of the material remains nearly constant. *2.13 Dilatation; Bulk Modulus 97 bee80288_ch02_052-139.indd Page 98 11/8/10 8:07:24 PM user-f499 EXAMPLE 2.09 /Users/user-f499/Desktop/Temp Work/Don't Delete Job/MHDQ251:Beer:201/ch02 Determine the change in volume DV of the steel block shown in Fig. 2.40, when it is subjected to the hydrostatic pressure p 5 180 MPa. Use E 5 200 GPa and n 5 0.29. From Eq. (2.33), we determine the bulk modulus of steel, k5 E 200 GPa 5 5 158.7 GPa 311 2 2n2 311 2 0.582 and, from Eq. (2.34), the dilatation, e52 p k 52 180 MPa 5 21.134 3 1023 158.7 GPa Since the volume V of the block in its unstressed state is V 5 (80 mm)(40 mm)(60 mm) 5 192 3 103 mm3 and since e represents the change in volume per unit volume, e 5 DVyV, we have DV 5 eV 5 (21.134 3 1023)(192 3 103 mm3) DV 5 2218 mm3 2.14 SHEARING STRAIN When we derived in Sec. 2.12 the relations (2.28) between normal stresses and normal strains in a homogeneous isotropic material, we assumed that no shearing stresses were involved. In the more general stress situation represented in Fig. 2.41, shearing stresses txy, tyz, and tzx will be present (as well, of course, as the corresponding shearing stresses tyx, tzy, and txz). These stresses have no direct effect on the normal strains and, as long as all the deformations involved remain small, they will not affect the derivation nor the validity of the relations (2.28). The shearing stresses, however, will tend to deform a cubic element of material into an oblique parallelepiped. y ␴y ␶yx ␶yz ␶zy ␴z ␶xy Q ␶zx ␶xz ␴x z x Fig. 2.41 98 General state of stress. bee80288_ch02_052-139.indd Page 99 9/4/10 5:18:22 PM user-f499 /Users/user-f499/Desktop/Temp Work/Don't Delete Job/MHDQ251:Beer:201/ch02 Consider first a cubic element of side one (Fig. 2.42) subjected to no other stresses than the shearing stresses txy and tyx applied to faces of the element respectively perpendicular to the x and y axes. (We recall from Sec. 1.12 that txy 5 tyx.) The element is observed to deform into a rhomboid of sides equal to one (Fig. 2.43). Two of the angles formed by the four faces under stress are reduced from p p p p 2 to 2 2 gxy, while the other two are increased from 2 to 2 1 gxy, The small angle gxy (expressed in radians) defines the shearing strain corresponding to the x and y directions. When the deformation involves a reduction of the angle formed by the two faces oriented respectively toward the positive x and y axes (as shown in Fig. 2.43), the shearing strain gxy is said to be positive; otherwise, it is said to be negative. We should note that, as a result of the deformations of the other elements of the material, the element under consideration can also undergo an overall rotation. However, as was the case in our study of normal strains, we are concerned here only with the actual deformation of the element, and not with any possible superimposed rigid-body displacement.† Plotting successive values of txy against the corresponding values of gxy, we obtain the shearing stress-strain diagram for the material under consideration. This can be accomplished by carrying out a torsion test, as you will see in Chap. 3. The diagram obtained is similar to the normal stress-strain diagram obtained for the same material from the tensile test described earlier in this chapter. However, the values obtained for the yield strength, ultimate strength, etc., of a given material are only about half as large in shear as they are in tension. As was the case for normal stresses and strains, the initial portion of the shearing stress-strain diagram is a straight line. For values of the shearing stress that do not exceed the proportional †In defining the strain gxy, some authors arbitrarily assume that the actual deformation of the element is accompanied by a rigid-body rotation such that the horizontal faces of the element do not rotate. The strain gxy is then represented by the angle through which the other two faces have rotated (Fig. 2.44). Others assume a rigid-body rotation such that the horizontal faces rotate through 12 gxy counterclockwise and the vertical faces through 1 2 gxy clockwise (Fig. 2.45). Since both assumptions are unnecessary and may lead to confusion, we prefer in this text to associate the shearing strain gxy with the change in the angle formed by the two faces, rather than with the rotation of a given face under restrictive conditions. y y xy 2 2   xy 1 2 xy xy x x Fig. 2.44 1 2 xy Fig. 2.45 2.14 Shearing Strain y 1 yx 1 xy xy 1 yx z x Fig. 2.42 Cubic element subjected to shearing stresses. y 2 yx xy 1 xy 2  xy 1 z x Fig. 2.43 Deformation of cubic element due to shearing stresses. 99 bee80288_ch02_052-139.indd Page 100 11/8/10 8:07:33 PM user-f499 100 /Users/user-f499/Desktop/Temp Work/Don't Delete Job/MHDQ251:Beer:201/ch02 limit in shear, we can therefore write for any homogeneous isotropic material, Stress and Strain—Axial Loading y txy 5 Ggxy ␶yz ␶zy z x (a) y ␶zx tyz 5 Ggyz x (b) Fig. 2.46 This relation is known as Hooke’s law for shearing stress and strain, and the constant G is called the modulus of rigidity or shear modulus of the material. Since the strain gxy was defined as an angle in radians, it is dimensionless, and the modulus G is expressed in the same units as txy, that is, in pascals or in psi. The modulus of rigidity G of any given material is less than one-half, but more than one-third of the modulus of elasticity E of that material.† Considering now a small element of material subjected to shearing stresses tyz and tzy (Fig. 2.46a), we define the shearing strain gyz as the change in the angle formed by the faces under stress. The shearing strain gzx is defined in a similar way by considering an element subjected to shearing stresses tzx and txz (Fig. 2.46b). For values of the stress that do not exceed the proportional limit, we can write the two additional relations ␶xz z (2.36) tzx 5 Ggzx (2.37) where the constant G is the same as in Eq. (2.36). For the general stress condition represented in Fig. 2.41, and as long as none of the stresses involved exceeds the corresponding proportional limit, we can apply the principle of superposition and combine the results obtained in this section and in Sec. 2.12. We obtain the following group of equations representing the generalized Hooke’s law for a homogeneous isotropic material under the most general stress condition. ns y sx ns z 2 2 E E E sy nsx nsz Py 5 2 1 2 E E E nsx nsy sz 1 2 Pz 5 2 E E E txy tyz tzx gyz 5 gzx 5 gxy 5 G G G Px 5 1 (2.38) An examination of Eqs. (2.38) might lead us to believe that three distinct constants, E, n, and G, must first be determined experimentally, if we are to predict the deformations caused in a given material by an arbitrary combination of stresses. Actually, only two of these constants need be determined experimentally for any given material. As you will see in the next section, the third constant can then be obtained through a very simple computation. †See Prob. 2.91. bee80288_ch02_052-139.indd Page 101 11/2/10 1:11:22 AM user-f499 /Users/user-f499/Desktop/Temp Work/Don't Delete Job/MHDQ251:Beer:201/ch02 A rectangular block of a material with a modulus of rigidity G 5 90 ksi is bonded to two rigid horizontal plates. The lower plate is fixed, while the upper plate is subjected to a horizontal force P (Fig. 2.47). Knowing that the upper plate moves through 0.04 in. under the action of the force, determine (a) the average shearing strain in the material, (b) the force P exerted on the upper plate. (a) Shearing Strain. We select coordinate axes centered at the midpoint C of edge AB and directed as shown (Fig. 2.48). According to its definition, the shearing strain gxy is equal to the angle formed by the vertical and the line CF joining the midpoints of edges AB and DE. Noting that this is a very small angle and recalling that it should be expressed in radians, we write g xy < tan g xy 5 0.04 in. 2 in. EXAMPLE 2.10 2.5 in. 8 in. 2 in. P Fig. 2.47 y g xy 5 0.020 rad (b) Force Exerted on Upper Plate. We first determine the shearing stress txy in the material. Using Hooke’s law for shearing stress and strain, we have D 2 in. F E P A C ␥xy B z txy 5 Gg xy 5 190 3 103 psi2 10.020 rad2 5 1800 psi The force exerted on the upper plate is thus 0.04 in. x Fig. 2.48 P 5 txy A 5 11800 psi2 18 in.2 12.5 in.2 5 36.0 3 103 lb P 5 36.0 kips 2.15 FURTHER DISCUSSION OF DEFORMATIONS UNDER AXIAL LOADING; RELATION AMONG E, N, AND G We saw in Sec. 2.11 that a slender bar subjected to an axial tensile load P directed along the x axis will elongate in the x direction and contract in both of the transverse y and z directions. If Px denotes the axial strain, the lateral strain is expressed as Py 5 Pz 5 2nPx, where n is Poisson’s ratio. Thus, an element in the shape of a cube of side equal to one and oriented as shown in Fig. 2.49a will deform into a rectangular parallelepiped of sides 1 1 Px, 1 2 nPx, and 1 2 nPx. (Note that only one face of the element is shown in the figure.) On the other hand, if the element is oriented at 458 to the axis of the load (Fig. 2.49b), the face shown in the figure is observed to deform into a rhombus. We conclude that the axial load P causes in this element a shearing strain g9 equal to the amount by which each of the angles shown in Fig. 2.49b increases or decreases.† y 1 P' P 1 x 1 ⫺ ␯⑀ x 1⫹ ⑀x (a) P' P ␲ ⫹␥ ' ␲ ⫺␥ ' 2 2 (b) Fig. 2.49 Representations of strain in an axially-loaded bar. †Note that the load P also produces normal strains in the element shown in Fig. 2.49b (see Prob. 2.73). 101 bee80288_ch02_052-139.indd Page 102 9/4/10 5:18:40 PM user-f499 102 Stress and Strain—Axial Loading y P' P x x ⫽ P A z x (a) P' ' ' m⫽ 45 P 2A ' P m ' ⫽ P 2A (b) Fig. 1.38 (repeated ) /Users/user-f499/Desktop/Temp Work/Don't Delete Job/MHDQ251:Beer:201/ch02 The fact that shearing strains, as well as normal strains, result from an axial loading should not come to us as a surprise, since we already observed at the end of Sec. 1.12 that an axial load P causes normal and shearing stresses of equal magnitude on four of the faces of an element oriented at 458 to the axis of the member. This was illustrated in Fig. 1.38, which, for convenience, has been repeated here. It was also shown in Sec. 1.11 that the shearing stress is maximum on a plane forming an angle of 458 with the axis of the load. It follows from Hooke’s law for shearing stress and strain that the shearing strain g9 associated with the element of Fig. 2.49b is also maximum: g9 5 gm. While a more detailed study of the transformations of strain will be postponed until Chap. 7, we will derive in this section a relation between the maximum shearing strain g9 5 gm associated with the element of Fig. 2.49b and the normal strain Px in the direction of the load. Let us consider for this purpose the prismatic element obtained by intersecting the cubic element of Fig. 2.49a by a diagonal plane (Fig. 2.50a and b). Referring to Fig. 2.49a, we conclude that this new element will deform into the element shown in Fig. 2.50c, which has horizontal and vertical sides respectively equal to 1 1 Px and 1 2 nPx. But the angle formed by the oblique and horizontal faces of the element of Fig. 2.50b is precisely half of one of the right angles of the cubic element considered in Fig. 2.49b. The angle b into which this angle deforms must therefore be equal to half of py2 2 gm. We write b5 1 1 1 gm p 2 4 2 1 1 4 1 (a) x  1 (b) x (c) Fig. 2.50 Applying the formula for the tangent of the difference of two angles, we obtain gm gm p 1 2 tan tan 2 tan 4 2 2 5 tan b 5 gm gm p 1 1 tan tan 1 1 tan 4 2 2 or, since gmy2 is a very small angle, gm 2 tan b 5 gm 11 2 12 (2.39) bee80288_ch02_052-139.indd Page 103 11/2/10 1:11:29 AM user-f499 /Users/user-f499/Desktop/Temp Work/Don't Delete Job/MHDQ251:Beer:201/ch02 *2.16 Stress-Strain Relationships for FiberReinforced Composite Materials But, from Fig. 2.50c, we observe that tan b 5 1 2 nPx 1 1 Px (2.40) Equating the right-hand members of (2.39) and (2.40), and solving for gm, we write gm 5 11 1 n2Px 12n 11 Px 2 Since Px V 1, the denominator in the expression obtained can be assumed equal to one; we have, therefore, gm 5 (1 1 n)Px (2.41) which is the desired relation between the maximum shearing strain gm and the axial strain Px. To obtain a relation among the constants E, n, and G, we recall that, by Hooke’s law, gm 5 tmyG, and that, for an axial loading, Px 5 sx yE. Equation (2.41) can therefore be written as sx tm 5 11 1 n2 G E or sx E 5 11 1 n2 tm G (2.42) We now recall from Fig. 1.38 that sx 5 PyA and tm 5 Py2A, where A is the cross-sectional area of the member. It thus follows that sxytm 5 2. Substituting this value into (2.42) and dividing both members by 2, we obtain the relation E 511n 2G (2.43) which can be used to determine one of the constants E, n, or G from the other two. For example, solving Eq. (2.43) for G, we write G5 *2.16 E 211 1 n2 (2.439) STRESS-STRAIN RELATIONSHIPS FOR FIBER-REINFORCED COMPOSITE MATERIALS Fiber-reinforced composite materials were briefly discussed in Sec. 2.5. It was shown at that time that these materials are obtained by embedding fibers of a strong, stiff material into a weaker, softer material, referred to as a matrix. It was also shown that the relationship between the normal stress and the corresponding normal strain created in a lamina, or layer, of a composite material depends upon the direction in which the load is applied. Different moduli of elasticity, Ex, Ey, and Ez, are therefore required to describe the relationship between normal stress and normal strain, according to whether the 103 bee80288_ch02_052-139.indd Page 104 11/8/10 8:07:43 PM user-f499 104 Stress and Strain—Axial Loading y Load Layer of material Load z Fibers (a) y' ␴x ␴x z' x' (b) Fig. 2.51 Fiber-reinforced composite material under uniaxial tensile load. x /Users/user-f499/Desktop/Temp Work/Don't Delete Job/MHDQ251:Beer:201/ch02 load is applied in a direction parallel to the fibers, in a direction perpendicular to the layer, or in a transverse direction. Let us consider again the layer of composite material discussed in Sec. 2.5 and let us subject it to a uniaxial tensile load parallel to its fibers, i.e., in the x direction (Fig. 2.51a). To simplify our analysis, it will be assumed that the properties of the fibers and of the matrix have been combined, or “smeared,” into a fictitious equivalent homogeneous material possessing these combined properties. We now consider a small element of that layer of smeared material (Fig. 2.51b). We denote by sx the corresponding normal stress and observe that sy 5 sz 5 0. As indicated earlier in Sec. 2.5, the corresponding normal strain in the x direction is Px 5 sxyEx, where Ex is the modulus of elasticity of the composite material in the x direction. As we saw for isotropic materials, the elongation of the material in the x direction is accompanied by contractions in the y and z directions. These contractions depend upon the placement of the fibers in the matrix and will generally be different. It follows that the lateral strains Py and Pz will also be different, and so will the corresponding Poisson’s ratios: Py Pz nxy 5 2 nxz 5 2 and (2.44) Px Px Note that the first subscript in each of the Poisson’s ratios nxy and nxz in Eqs. (2.44) refers to the direction of the load, and the second to the direction of the contraction. It follows from the above that, in the case of the multiaxial loading of a layer of a composite material, equations similar to Eqs. (2.28) of Sec. 2.12 can be used to describe the stress-strain relationship. In the present case, however, three different values of the modulus of elasticity and six different values of Poisson’s ratio will be involved. We write nyxsy sx nzxsz 2 2 Px 5 Ex Ey Ez Py 5 2 Pz 5 2 nxysx Ex 1 sy 2 Ey nyzsy nzysz Ez (2.45) nxzsx sz 2 1 Ex Ey Ez Equations (2.45) may be considered as defining the transformation of stress into strain for the given layer. It follows from a general property of such transformations that the coefficients of the stress components are symmetric, i.e., that nyx nzy nyz nxy nzx nxz 5 5 5 (2.46) Ex Ey Ey Ez Ez Ex These equations show that, while different, the Poisson’s ratios nxy and nyx are not independent; either of them can be obtained from the other if the corresponding values of the modulus of elasticity are known. The same is true of nyz and nzy, and of nzx and nxz. Consider now the effect of the presence of shearing stresses on the faces of a small element of smeared layer. As pointed out in bee80288_ch02_052-139.indd Page 105 11/8/10 9:37:51 PM user-f494 volume 201/FREE048/work%0/indd%0/ 2.16 Stress-Strain Relationships for FiberReinforced Composite Materials Sec. 2.14 in the case of isotropic materials, these stresses come in pairs of equal and opposite vectors applied to opposite sides of the given element and have no effect on the normal strains. Thus, Eqs. (2.45) remain valid. The shearing stresses, however, will create shearing strains which are defined by equations similar to the last three of the equations (2.38) of Sec. 2.14, except that three different values of the modulus of rigidity, Gxy, Gyz, and Gzx, must now be used. We have txy tyz tzx gxy 5 gyz 5 gzx 5 (2.47) Gxy Gyz Gzx The fact that the three components of strain Px, Py, and Pz can be expressed in terms of the normal stresses only and do not depend upon any shearing stresses characterizes orthotropic materials and distinguishes them from other anisotropic materials. As we saw in Sec. 2.5, a flat laminate is obtained by superposing a number of layers or laminas. If the fibers in all layers are given the same orientation to better withstand an axial tensile load, the laminate itself will be orthotropic. If the lateral stability of the laminate is increased by positioning some of its layers so that their fibers are at a right angle to the fibers of the other layers, the resulting laminate will also be orthotropic. On the other hand, if any of the layers of a laminate are positioned so that their fibers are neither parallel nor perpendicular to the fibers of other layers, the lamina, generally, will not be orthotropic.† A 60-mm cube is made from layers of graphite epoxy with fibers aligned in the x direction. The cube is subjected to a compressive load of 140 kN in the x direction. The properties of the composite material are: Ex 5 155.0 GPa, Ey 5 12.10 GPa, Ez 5 12.10 GPa, nxy 5 0.248, nxz 5 0.248, and nyz 5 0.458. Determine the changes in the cube dimensions, knowing that (a) the cube is free to expand in the y and z directions (Fig. 2.52); (b) the cube is free to expand in the z direction, but is restrained from expanding in the y direction by two fixed frictionless plates (Fig. 2.53). (a) Free in y and z Directions. in the direction of loading. We have y 140 kN 60 mm We first determine the stress sx 3 sx 5 EXAMPLE 2.11 P 2140 3 10 N 5 5 238.89 MPa A 10.060 m2 10.060 m2 Since the cube is not loaded or restrained in the y and z directions, we have sy 5 sz 5 0. Thus, the right-hand members of Eqs. (2.45) reduce to their first terms. Substituting the given data into these equations, we write sx 238.89 MPa 5 5 2250.9 3 1026 Ex 155.0 GPa nxysx 10.2482 1238.89 MPa2 Py 5 2 52 5 162.22 3 1026 Ex 155.0 GPa 10.2482 1238.69 MPa2 nxzsx Pz 5 2 52 5 162.22 3 1026 Ex 155.0 GPa Px 5 60 mm 140 kN 60 mm x z Fig. 2.52 y 140 kN Fixed frictionless plates z †For more information on fiber-reinforced composite materials, see Hyer, M. W., Stress Analysis of Fiber-Reinforced Composite Materials, McGraw-Hill, New York, 1998. Fig. 2.53 60 mm 140 kN 60 mm 60 mm x 105 bee80288_ch02_052-139.indd Page 106 11/2/10 1:11:30 AM user-f499 /Users/user-f499/Desktop/Temp Work/Don't Delete Job/MHDQ251:Beer:201/ch02 The changes in the cube dimensions are obtained by multiplying the corresponding strains by the length L 5 0.060 m of the side of the cube: dx 5 PxL 5 12250.9 3 1026 2 10.060 m2 5 215.05 mm dy 5 PyL 5 1162.2 3 1026 2 10.060 m2 5 13.73 mm dz 5 PzL 5 1162.2 3 1026 2 10.060 m2 5 13.73 mm (b) Free in z Direction, Restrained in y Direction. The stress in the x direction is the same as in part a, namely, sx 5 238.89 MPa. Since the cube is free to expand in the z direction as in part a, we again have sz 5 0. But since the cube is now restrained in the y direction, we should expect a stress sy different from zero. On the other hand, since the cube cannot expand in the y direction, we must have dy 5 0 and, thus, Py 5 dyyL 5 0. Making sz 5 0 and Py 5 0 in the second of Eqs. (2.45), solving that equation for sy, and substituting the given data, we have sy 5 a Ey Ex 12.10 b10.2482 1238.89 MPa2 155.0 5 2752.9 kPa b nxysx 5 a Now that the three components of stress have been determined, we can use the first and last of Eqs. (2.45) to compute the strain components Px and Pz. But the first of these equations contains Poisson’s ratio nyx and, as we saw earlier, this ratio is not equal to the ratio nxy which was among the given data. To find nyx we use the first of Eqs. (2.46) and write nyx 5 a Ey Ex b nxy 5 a 12.10 b10.2482 5 0.01936 155.0 Making sz 5 0 in the first and third of Eqs. (2.45) and substituting in these equations the given values of Ex, Ey, nxz, and nyz, as well as the values obtained for sx, sy, and nyx, we have nyxsy 10.019362 12752.9 kPa2 sx 238.89 MPa 2 2 5 Ex Ey 155.0 GPa 12.10 GPa 5 2249.7 3 1026 nyzsy 10.2482 1238.89 MPa2 10.4582 12752.9 kPa2 nxzsx Pz 5 2 2 52 2 Ex Ey 155.0 GPa 12.10 GPa 26 5 190.72 3 10 Px 5 The changes in the cube dimensions are obtained by multiplying the corresponding strains by the length L 5 0.060 m of the side of the cube: dx 5 PxL 5 12249.7 3 1026 2 10.060 m2 5 214.98 mm dy 5 PyL 5 102 10.060 m2 5 0 dz 5 PzL 5 1190.72 3 1026 2 10.060 m2 5 15.44 mm Comparing the results of parts a and b, we note that the difference between the values obtained for the deformation dx in the direction of the fibers is negligible. However, the difference between the values obtained for the lateral deformation dz is not negligible. This deformation is clearly larger when the cube is restrained from deforming in the y direction. 106 bee80288_ch02_052-139.indd Page 107 11/1/10 11:30:48 PM user-f499 y SAMPLE PROBLEM 2.5 15 in. 15 in. A D z z /Users/user-f499/Desktop/Temp Work/Don't Delete Job/MHDQ251:Beer:201/ch02 C B x x A circle of diameter d 5 9 in. is scribed on an unstressed aluminum plate of thickness t 5 34 in. Forces acting in the plane of the plate later cause normal stresses sx 5 12 ksi and sz 5 20 ksi. For E 5 10 3 106 psi and n 5 13, determine the change in (a) the length of diameter AB, (b) the length of diameter CD, (c) the thickness of the plate, (d) the volume of the plate. SOLUTION Hooke’s Law. We note that sy 5 0. Using Eqs. (2.28) we find the strain in each of the coordinate directions. Px 5 1 ns y sx ns z 2 2 E E E 1 1 c 112 ksi2 2 0 2 120 ksi2 d 5 10.533 3 1023 in./in. 3 10 3 106 psi sy ns z ns x 1 2 Py 5 2 E E E 5 1 1 1 c 2 112 ksi2 1 0 2 120 ksi2 d 5 21.067 3 1023 in./in. 6 3 3 10 3 10 psi ns y sz ns x Pz 5 2 2 1 E E E 5 5 1 1 c 2 112 ksi2 2 0 1 120 ksi2 d 5 11.600 3 1023 in./in. 6 3 10 3 10 psi a. Diameter AB. The change in length is dByA 5 Px d. dByA 5 Pxd 5 110.533 3 1023 in./in.2 19 in.2 dByA 5 14.8 3 1023 in. ◀ b. Diameter CD. dCyD 5 Pzd 5 111.600 3 1023 in./in.2 19 in.2 dCyD 5 114.4 3 1023 in. ◀ c. Thickness. Recalling that t 5 34 in., we have dt 5 Pyt 5 121.067 3 1023 in./in.2 1 34 in.2 dt 5 20.800 3 1023 in. ◀ d. Volume of the Plate. Using Eq. (2.30), we write e 5 Px 1 Py 1 Pz 5 110.533 2 1.067 1 1.60021023 5 11.067 3 1023 ¢V 5 eV 5 11.067 3 1023 3 115 in.2 115 in.2 1 34 in.2 4 ¢V 5 10.187 3 in3 ◀ 107 bee80288_ch02_052-139.indd Page 108 11/1/10 11:30:52 PM user-f499 /Users/user-f499/Desktop/Temp Work/Don't Delete Job/MHDQ251:Beer:201/ch02 PROBLEMS 2.61 A 600-lb tensile load is applied to a test coupon made from 161 -in. flat steel plate (E 5 29 3 106 psi, n 5 0.30). Determine the resulting change (a) in the 2-in. gage length, (b) in the width of portion AB of the test coupon, (c) in the thickness of portion AB, (d) in the cross-sectional area of portion AB. 2 in. 600 lb 600 lb A 1 2 B in. Fig. P2.61 22-mm diameter 75 kN 75 kN 200 mm 2.62 In a standard tensile test a steel rod of 22-mm diameter is subjected to a tension force of 75 kN. Knowing that n 5 0.3 and E 5 200 GPa, determine (a) the elongation of the rod in a 200-mm gage length, (b) the change in diameter of the rod. Fig. P2.62 2.63 A 20-mm-diameter rod made of an experimental plastic is subjected to a tensile force of magnitude P 5 6 kN. Knowing that an elongation of 14 mm and a decrease in diameter of 0.85 mm are observed in a 150-mm length, determine the modulus of elasticity, the modulus of rigidity, and Poisson’s ratio for the material. 2.64 The change in diameter of a large steel bolt is carefully measured as the nut is tightened. Knowing that E 5 29 3 106 psi and n 5 0.30, determine the internal force in the bolt, if the diameter is observed to decrease by 0.5 3 1023 in. 2.5 in. 700 kN Fig. P2.64 2.65 A 2.5-m length of a steel pipe of 300-mm outer diameter and 15-mm wall thickness is used as a column to carry a 700-kN centric axial load. Knowing that E 5 200 GPa and n 5 0.30, determine (a) the change in length of the pipe, (b) the change in its outer diameter, (c) the change in its wall thickness. 2.5 m 2.66 An aluminum plate (E 5 74 GPa, n 5 0.33) is subjected to a centric axial load that causes a normal stress s. Knowing that, before loading, a line of slope 2:1 is scribed on the plate, determine the slope of the line when s 5 125 MPa.  2 1 108 Fig. P2.65 Fig. P2.66  bee80288_ch02_052-139.indd Page 109 11/1/10 11:31:14 PM user-f499 /Users/user-f499/Desktop/Temp Work/Don't Delete Job/MHDQ251:Beer:201/ch02 Problems 2.67 The block shown is made of a magnesium alloy for which E 5 45 GPa and n 5 0.35. Knowing that sx 5 2180 MPa, determine (a) the magnitude of sy for which the change in the height of the block will be zero, (b) the corresponding change in the area of the face ABCD, (c) the corresponding change in the volume of the block. y y 25 mm 40 mm D A B G C E z 100 mm y  40 MPa x A x F x  80 MPa 30 mm Fig. P2.67 2.68 A 30-mm square was scribed on the side of a large steel pressure vessel. After pressurization the biaxial stress condition at the square is as shown. For E 5 200 GPa and n 5 0.30, determine the change in length of (a) side AB, (b) side BC, (c) diagonal AC. 2.69 The aluminum rod AD is fitted with a jacket that is used to apply a hydrostatic pressure of 6000 psi to the 12-in. portion BC of the rod. Knowing that E 5 10.1 3 106 psi and n 5 0.36, determine (a) the change in the total length AD, (b) the change in diameter at the middle of the rod. 2.71 In many situations physical constraints prevent strain from occurring in a given direction. For example, Pz 5 0 in the case shown, where longitudinal movement of the long prism is prevented at every point. Plane sections perpendicular to the longitudinal axis remain plane and the same distance apart. Show that for this situation, which is known as plane strain, we can express sz, Px, and Py as follows: s z 5 n1s x 1 s y 2 1 Px 5 3 11 2 n2 2s x 2 n11 1 n2s y 4 E 1 Py 5 3 11 2 n2 2s y 2 n11 1 n2s x 4 E y 30 mm Fig. P2.68 A x x (a) Fig. P2.71 z (b) B 12 in. C D 1.5 in. Fig. P2.69 y C D 2.70 For the rod of Prob. 2.69, determine the forces that should be applied to the ends A and D of the rod (a) if the axial strain in portion BC of the rod is to remain zero as the hydrostatic pressure is applied, (b) if the total length AD of the rod is to remain unchanged. z B 20 in. 109 bee80288_ch02_052-139.indd Page 110 9/4/10 5:19:15 PM user-f499 110 /Users/user-f499/Desktop/Temp Work/Don't Delete Job/MHDQ251:Beer:201/ch02 2.72 In many situations it is known that the normal stress in a given direction is zero. For example, sz 5 0 in the case of the thin plate shown. For this case, which is known as plane stress, show that if the strains Px and Py have been determined experimentally, we can express sx, sy and Pz as follows: Stress and Strain—Axial Loading y x sx 5 E sy 5 E 2.73 For a member under axial loading, express the normal strain P9 in a direction forming an angle of 458 with the axis of the load in terms of the axial strain Px by (a) comparing the hypotenuses of the triangles shown in Fig. 2.50, which represent respectively an element before and after deformation, (b) using the values of the corresponding stresses s9 and sx shown in Fig. 1.38, and the generalized Hooke’s law. y A B D x x C z Fig. P2.74 P 100 mm B 30 mm 30 mm Fig. P2.75 and P2.76 2.74 The homogeneous plate ABCD is subjected to a biaxial loading as shown. It is known that sz 5 s0 and that the change in length of the plate in the x direction must be zero, that is, Px 5 0. Denoting by E the modulus of elasticity and by n Poisson’s ratio, determine (a) the required magnitude of sx, (b) the ratio s0yPz. 2.75 A vibration isolation unit consists of two blocks of hard rubber bonded to a plate AB and to rigid supports as shown. Knowing that a force of magnitude P 5 25 kN causes a deflection d 5 1.5 mm of plate AB, determine the modulus of rigidity of the rubber used. A 150 mm 1 2 n2 Py 1 nPx 1 2 n2 n Pz 5 2 1Px 1 Py 2 12n Fig. P2.72 z Px 1 nPy 2.76 A vibration isolation unit consists of two blocks of hard rubber with a modulus of rigidity G 5 19 MPa bonded to a plate AB and to rigid supports as shown. Denoting by P the magnitude of the force applied to the plate and by d the corresponding deflection, determine the effective spring constant, k 5 Pyd, of the system. 2.77 The plastic block shown is bonded to a fixed base and to a horizontal rigid plate to which a force P is applied. Knowing that for the plastic used G 5 55 ksi, determine the deflection of the plate when P 5 9 kips. 3.5 in. P 5.5 in. Fig. P2.77 2.2 in. bee80288_ch02_052-139.indd Page 111 9/4/10 5:19:23 PM user-f499 /Users/user-f499/Desktop/Temp Work/Don't Delete Job/MHDQ251:Beer:201/ch02 Problems 2.78 A vibration isolation unit consists of two blocks of hard rubber bonded to plate AB and to rigid supports as shown. For the type and grade of rubber used tall 5 220 psi and G 5 1800 psi. Knowing that a centric vertical force of magnitude P 5 3.2 kips must cause a 0.1-in. vertical deflection of the plate AB, determine the smallest allowable dimensions a and b of the block. P 3.0 in. A 2.79 The plastic block shown is bonded to a rigid support and to a vertical plate to which a 55-kip load P is applied. Knowing that for the plastic used G 5 150 ksi, determine the deflection of the plate. b 3.2 in. B a a Fig. P2.78 4.8 in. 2 in. P Fig. P2.79 2.80 What load P should be applied to the plate of Prob. 2.79 to produce a 161 -in. deflection? 2.81 Two blocks of rubber with a modulus of rigidity G 5 12 MPa are bonded to rigid supports and to a plate AB. Knowing that c 5 100 mm and P 5 45 kN, determine the smallest allowable dimensions a and b of the blocks if the shearing stress in the rubber is not to exceed 1.4 MPa and the deflection of the plate is to be at least 5 mm. 2.82 Two blocks of rubber with a modulus of rigidity G 5 10 MPa are bonded to rigid supports and to a plate AB. Knowing that b 5 200 mm and c 5 125 mm, determine the largest allowable load P and the smallest allowable thickness a of the blocks if the shearing stress in the rubber is not to exceed 1.5 MPa and the deflection of the plate is to be at least 6 mm. *2.83 Determine the dilatation e and the change in volume of the 200-mm length of the rod shown if (a) the rod is made of steel with E 5 200 GPa and n 5 0.30, (b) the rod is made of aluminum with E 5 70 GPa and n 5 0.35. *2.84 Determine the change in volume of the 2-in. gage length segment AB in Prob. 2.61 (a) by computing the dilatation of the material, (b) by subtracting the original volume of portion AB from its final volume. *2.85 A 6-in.-diameter solid steel sphere is lowered into the ocean to a point where the pressure is 7.1 ksi (about 3 miles below the surface). Knowing that E 5 29 3 106 psi and n 5 0.30, determine (a) the decrease in diameter of the sphere, (b) the decrease in volume of the sphere, (c) the percent increase in the density of the sphere. a a b B A P c Figs. P2.81 and P2.82 22-mm diameter 46 kN 46 kN 200 mm Fig. P2.83 111 bee80288_ch02_052-139.indd Page 112 9/4/10 5:19:31 PM user-f499 112 /Users/user-f499/Desktop/Temp Work/Don't Delete Job/MHDQ251:Beer:201/ch02 *2.86 (a) For the axial loading shown, determine the change in height and the change in volume of the brass cylinder shown. (b) Solve part a, assuming that the loading is hydrostatic with sx 5 sy 5 sz 5 270 MPa. Stress and Strain—Axial Loading 85 mm y  58 MPa E  105 GPa v  0.33 *2.87 A vibration isolation support consists of a rod A of radius R1 5 10 mm and a tube B of inner radius R2 5 25 mm bonded to an 80-mm-long hollow rubber cylinder with a modulus of rigidity G 5 12 MPa. Determine the largest allowable force P that can be applied to rod A if its deflection is not to exceed 2.50 mm. 135 mm P A R1 R2 Fig. P2.86 B 80 mm Fig. P2.87 and P2.88 y E x  50 GPa E y  15.2 GPa E z  15.2 GPa xz  0.254 xy  0.254 zy  0.428 *2.88 A vibration isolation support consists of a rod A of radius R1 and a tube B of inner radius R2 bonded to an 80-mm-long hollow rubber cylinder with a modulus of rigidity G 5 10.93 MPa. Determine the required value of the ratio R2yR1 if a 10-kN force P is to cause a 2-mm deflection of rod A. *2.89 The material constants E, G, k, and n are related by Eqs. (2.33) and (2.43). Show that any one of the constants may be expressed in terms of any other two constants. For example, show that (a) k 5 GEy(9G 2 3E) and (b) n 5 (3k 2 2G)y(6k 1 2G). *2.90 Show that for any given material, the ratio G/E of the modulus of rigidity over the modulus of elasticity is always less than 12 but more than 13. [Hint: Refer to Eq. (2.43) and to Sec. 2.13.] z x Fig. P2.91 *2.91 A composite cube with 40-mm sides and the properties shown is made with glass polymer fibers aligned in the x direction. The cube is constrained against deformations in the y and z directions and is subjected to a tensile load of 65 kN in the x direction. Determine (a) the change in the length of the cube in the x direction, (b) the stresses sx, sy, and sz. *2.92 The composite cube of Prob. 2.91 is constrained against deformation in the z direction and elongated in the x direction by 0.035 mm due to a tensile load in the x direction. Determine (a) the stresses sx, sy, and sz, (b) the change in the dimension in the y direction. bee80288_ch02_052-139.indd Page 113 9/7/10 4:34:35 PM user-f499 2.17 /Users/user-f499/Desktop/Temp Work/Don't Delete Job/MHDQ251:Beer:201/ch07 STRESS AND STRAIN DISTRIBUTION UNDER AXIAL LOADING; SAINT-VENANT’S PRINCIPLE We have assumed so far that, in an axially loaded member, the normal stresses are uniformly distributed in any section perpendicular to the axis of the member. As we saw in Sec. 1.5, such an assumption may be quite in error in the immediate vicinity of the points of application of the loads. However, the determination of the actual stresses in a given section of the member requires the solution of a statically indeterminate problem. In Sec. 2.9, you saw that statically indeterminate problems involving the determination of forces can be solved by considering the deformations caused by these forces. It is thus reasonable to conclude that the determination of the stresses in a member requires the analysis of the strains produced by the stresses in the member. This is essentially the approach found in advanced textbooks, where the mathematical theory of elasticity is used to determine the distribution of stresses corresponding to various modes of application of the loads at the ends of the member. Given the more limited mathematical means at our disposal, our analysis of stresses will be restricted to the particular case when two rigid plates are used to transmit the loads to a member made of a homogeneous isotropic material (Fig. 2.54). If the loads are applied at the center of each plate,† the plates will move toward each other without rotating, causing the member to get shorter, while increasing in width and thickness. It is reasonable to assume that the member will remain straight, that plane sections will remain plane, and that all elements of the member will deform in the same way, since such an assumption is clearly compatible with the given end conditions. This is illustrated in Fig. 2.55, P P' (a) (b) Fig. 2.55 Axial load applied by rigid plates to rubber model. †More precisely, the common line of action of the loads should pass through the centroid of the cross section (cf. Sec. 1.5). 2.17 Stress and Strain Distribution under Axial Loading; Saint-Venant’s Principle P P' Fig. 2.54 Axial load applied by rigid plates to a member. 113 bee80288_ch02_052-139.indd Page 114 9/7/10 4:34:35 PM user-f499 114 /Users/user-f499/Desktop/Temp Work/Don't Delete Job/MHDQ251:Beer:201/ch07 which shows a rubber model before and after loading.† Now, if all elements deform in the same way, the distribution of strains throughout the member must be uniform. In other words, the axial strain Py and the lateral strain Px 5 2nPy are constant. But, if the stresses do not exceed the proportional limit, Hooke’s law applies and we may write sy 5 EPy, from which it follows that the normal stress sy is also constant. Thus, the distribution of stresses is uniform throughout the member and, at any point, Stress and Strain—Axial Loading P s y 5 1s y 2 ave 5 P' Fig. 2.56 Concentrated axial load applied to rubber model. P P b b  min  0.973 ave  max  1.027 ave P' P 1 2 P 1 4 b b  min P A On the other hand, if the loads are concentrated, as illustrated in Fig. 2.56, the elements in the immediate vicinity of the points of application of the loads are subjected to very large stresses, while other elements near the ends of the member are unaffected by the loading. This may be verified by observing that strong deformations, and thus large strains and large stresses, occur near the points of application of the loads, while no deformation takes place at the corners. As we consider elements farther and farther from the ends, however, we note a progressive equalization of the deformations involved, and thus a more nearly uniform distribution of the strains and stresses across a section of the member. This is further illustrated in Fig. 2.57, which shows the result of the calculation by advanced mathematical methods of the distribution of stresses across various sections of a thin rectangular plate subjected to concentrated loads. We note that at a distance b from either end, where b is the width of the plate, the stress distribution is nearly uniform across the section, and the value of the stress sy at any point of that section can be assumed equal to the average value PyA. Thus, at a distance equal to, or greater than, the width of the member, the distribution of stresses across a given section is the same, whether the member is loaded as shown in Fig. 2.54 or Fig. 2.56. In other words, except in the immediate vicinity of the points of application of the loads, the stress distribution may be assumed independent of the actual mode of application of the loads. This statement, which applies not only to axial loadings, but to practically any type of load, is known as SaintVenant’s principle, after the French mathematician and engineer Adhémar Barré de Saint-Venant (1797–1886). While Saint-Venant’s principle makes it possible to replace a given loading by a simpler one for the purpose of computing the stresses in a structural member, you should keep in mind two important points when applying this principle: 1. The actual loading and the loading used to compute the stresses P  ave  A  max  min  0.668 ave  max  1.387 ave  min  0.198 ave  max  2.575 ave Fig. 2.57 Stress distributions in a plate under concentrated axial loads. must be statically equivalent. 2. Stresses cannot be computed in this manner in the immediate vicinity of the points of application of the loads. Advanced theoretical or experimental methods must be used to determine the distribution of stresses in these areas. †Note that for long, slender members, another configuration is possible, and indeed will prevail, if the load is sufficiently large; the member buckles and assumes a curved shape. This will be discussed in Chap. 10. bee80288_ch02_052-139.indd Page 115 9/4/10 5:19:47 PM user-f499 /Users/user-f499/Desktop/Temp Work/Don't Delete Job/MHDQ251:Beer:201/ch02 2.18 Stress Concentrations You should also observe that the plates used to obtain a uniform stress distribution in the member of Fig. 2.55 must allow the member to freely expand laterally. Thus, the plates cannot be rigidly attached to the member; you must assume them to be just in contact with the member, and smooth enough not to impede the lateral expansion of the member. While such end conditions can actually be achieved for a member in compression, they cannot be physically realized in the case of a member in tension. It does not matter, however, whether or not an actual fixture can be realized and used to load a member so that the distribution of stresses in the member is uniform. The important thing is to imagine a model that will allow such a distribution of stresses, and to keep this model in mind so that you may later compare it with the actual loading conditions. 2.18 STRESS CONCENTRATIONS As you saw in the preceding section, the stresses near the points of application of concentrated loads can reach values much larger than the average value of the stress in the member. When a structural member contains a discontinuity, such as a hole or a sudden change in cross section, high localized stresses can also occur near the discontinuity. Figures 2.58 and 2.59 show the distribution of stresses in critical sections corresponding to two such situations. Figure 2.58 refers to a flat bar with a circular hole and shows the stress distribution in a section passing through the center of the hole. Figure 2.59 refers to a flat bar consisting of two portions of different widths connected by fillets; it shows the stress distribution in the narrowest part of the connection, where the highest stresses occur. These results were obtained experimentally through the use of a photoelastic method. Fortunately for the engineer who has to design a given member and cannot afford to carry out such an analysis, the results obtained are independent of the size of the member and of the material used; they depend only upon the ratios of the geometric parameters involved, i.e., upon the ratio ryd in the case of a circular hole, and upon the ratios ryd and Dyd in the case of fillets. Furthermore, the designer is more interested in the maximum value of the stress in a given section, than in the actual distribution of stresses in that section, since the main concern is to determine whether the allowable stress will be exceeded under a given loading, and not where this value will be exceeded. For this reason, one defines the ratio K5 s max s ave P' 1 2d r P D 1 2d P'  max  ave Fig. 2.58 Stress distribution near circular hole in flat bar under axial loading. r P' D P d (2.48) of the maximum stress over the average stress computed in the critical (narrowest) section of the discontinuity. This ratio is referred to as the stress-concentration factor of the given discontinuity. Stressconcentration factors can be computed once and for all in terms of the ratios of the geometric parameters involved, and the results obtained can be expressed in the form of tables or of graphs, as  max P'  ave Fig. 2.59 Stress distribution near fillets in flat bar under axial loading. 115 bee80288_ch02_052-139.indd Page 116 11/1/10 11:31:28 PM user-f499 3.4 P' 3.2 1 2d r 3.4 P D 1 2d 3.0 /Users/user-f499/Desktop/Temp Work/Don't Delete Job/MHDQ251:Beer:201/ch02 3.0 2.8 2.6 2.6 2.4 2.4 K 2.2 K 2.0 1.8 1.8 1.6 1.6 1.4 1.4 1.2 1.2 0.1 0.2 0.3 2r/D 0.4 0.5 (a) Flat bars with holes 0.7 P D/d  2 1.5 1.3 1.2 1.0 1.1 0 0.02 0.04 0.06 0.08 0.10 0.12 0.14 0.16 0.18 0.20 0.22 0.24 0.26 0.28 0.30 r/d (b) Flat bars with fillets Fig. 2.60 Stress concentration factors for flat bars under axial loading† Note that the average stress must be computed across the narrowest section: save 5 P/td, where t is the thickness of the bar. EXAMPLE 2.12 116 0.6 d 2.2 2.0 0 D 3.2 2.8 1.0 r P' shown in Fig. 2.60. To determine the maximum stress occurring near a discontinuity in a given member subjected to a given axial load P, the designer needs only to compute the average stress save 5 PyA in the critical section, and multiply the result obtained by the appropriate value of the stress-concentration factor K. You should note, however, that this procedure is valid only as long as smax does not exceed the proportional limit of the material, since the values of K plotted in Fig. 2.60 were obtained by assuming a linear relation between stress and strain. Determine the largest axial load P that can be safely supported by a flat steel bar consisting of two portions, both 10 mm thick and, respectively, 40 and 60 mm wide, connected by fillets of radius r 5 8 mm. Assume an allowable normal stress of 165 MPa. We first compute the ratios D 60 mm r 8 mm 5 5 1.50 5 5 0.20 d 40 mm d 40 mm Using the curve in Fig. 2.60b corresponding to Dyd 5 1.50, we find that the value of the stress-concentration factor corresponding to ryd 5 0.20 is K 5 1.82 Carrying this value into Eq. (2.48) and solving for save, we have s max s ave 5 1.82 But smax cannot exceed the allowable stress sall 5 165 MPa. Substituting this value for smax, we find that the average stress in the narrower portion (d 5 40 mm) of the bar should not exceed the value 165 MPa 5 90.7 MPa save 5 1.82 Recalling that save 5 PyA, we have P 5 Asave 5 140 mm2 110 mm2 190.7 MPa2 5 36.3 3 103 N P 5 36.3 kN †W. D. Pilkey, Peterson’s Stress Concentration Factors, 2nd ed., John Wiley & Sons, New York, 1997. bee80288_ch02_052-139.indd Page 117 11/2/10 2:59:54 PM user-f499 2.19 /Users/user-f499/Desktop/Temp Work/Don't Delete Job/MHDQ251:Beer:201/ch02 2.19 Plastic Deformations PLASTIC DEFORMATIONS The results obtained in the preceding sections were based on the assumption of a linear stress-strain relationship. In other words, we assumed that the proportional limit of the material was never exceeded. This is a reasonable assumption in the case of brittle materials, which rupture without yielding. In the case of ductile materials, however, this assumption implies that the yield strength of the material is not exceeded. The deformations will then remain within the elastic range and the structural member under consideration will regain its original shape after all loads have been removed. If, on the other hand, the stresses in any part of the member exceed the yield strength of the material, plastic deformations occur and most of the results obtained in earlier sections cease to be valid. A more involved analysis, based on a nonlinear stress-strain relationship, must then be carried out. While an analysis taking into account the actual stress-strain relationship is beyond the scope of this text, we gain considerable insight into plastic behavior by considering an idealized elastoplastic material for which the stress-strain diagram consists of the two straight-line segments shown in Fig. 2.61. We may note that the stress-strain diagram for mild steel in the elastic and plastic ranges is similar to this idealization. As long as the stress s is less than the yield strength sY, the material behaves elastically and obeys Hooke’s law, s 5 EP. When s reaches the value sY, the material starts yielding and keeps deforming plastically under a constant load. If the load is removed, unloading takes place along a straightline segment CD parallel to the initial portion AY of the loading curve. The segment AD of the horizontal axis represents the strain corresponding to the permanent set or plastic deformation resulting from the loading and unloading of the specimen. While no actual material behaves exactly as shown in Fig. 2.61, this stress-strain diagram will prove useful in discussing the plastic deformations of ductile materials such as mild steel. A rod of length L 5 500 mm and cross-sectional area A 5 60 mm2 is made of an elastoplastic material having a modulus of elasticity E 5 200 GPa in its elastic range and a yield point sY 5 300 MPa. The rod is subjected to an axial load until it is stretched 7 mm and the load is then removed. What is the resulting permanent set? Referring to the diagram of Fig. 2.61, we find that the maximum strain, represented by the abscissa of point C, is dC 7 mm 5 5 14 3 1023 PC 5 L 500 mm On the other hand, the yield strain, represented by the abscissa of point Y, is sY 300 3 106 Pa PY 5 5 1.5 3 1023 5 E 200 3 109 Pa The strain after unloading is represented by the abscissa PD of point D. We note from Fig. 2.61 that PD 5 AD 5 YC 5 PC 2 PY 5 14 3 1023 2 1.5 3 1023 5 12.5 3 1023 The permanent set is the deformation dD corresponding to the strain PD. We have dD 5 PDL 5 112.5 3 1023 2 1500 mm2 5 6.25 mm ␴ ␴Y Y C Rupture A D ⑀ Fig. 2.61 Stress-strain diagram for an idealized elastoplastic material. EXAMPLE 2.13 117 bee80288_ch02_052-139.indd Page 118 11/2/10 1:11:33 AM user-f499 EXAMPLE 2.14 /Users/user-f499/Desktop/Temp Work/Don't Delete Job/MHDQ251:Beer:201/ch02 A 30-in.-long cylindrical rod of cross-sectional area Ar 5 0.075 in2 is placed inside a tube of the same length and of cross-sectional area At 5 0.100 in2. The ends of the rod and tube are attached to a rigid support on one side, and to a rigid plate on the other, as shown in the longitudinal section of Fig. 2.62. The rod and tube are both assumed to be elastoplastic, with moduli of elasticity Er 5 30 3 106 psi and Et 5 15 3 106 psi, and yield strengths (sr)Y 5 36 ksi and (st)Y 5 45 ksi. Draw the load-deflection diagram of the rod-tube assembly when a load P is applied to the plate as shown. Tube Plate Rod P 30 in. Fig. 2.62 Pr (kips) 2.7 Yr We first determine the internal force and the elongation of the rod as it begins to yield: 0 ␦ r (10–3 in.) 36 (a) Pt (kips) Yt 4.5 1Pr 2 Y 5 1sr 2 YAr 5 136 ksi2 10.075 in2 2 5 2.7 kips 36 3 103 psi 1sr 2 Y L5 130 in.2 1dr 2 Y 5 1Pr 2 YL 5 Er 30 3 106 psi 5 36 3 1023 in. Since the material is elastoplastic, the force-elongation diagram of the rod alone consists of an oblique straight line and of a horizontal straight line, as shown in Fig. 2.63a. Following the same procedure for the tube, we have 1.8 0 36 (b) P (kips) 90 ␦ t (10–3 in.) Yt 7.2 Yr 4.5 1Pt 2 Y 5 1st 2 YAt 5 145 ksi2 10.100 in2 2 5 4.5 kips 45 3 103 psi 1st 2 Y L5 130 in.2 1dt 2 Y 5 1Pt 2 YL 5 Et 15 3 106 psi 5 90 3 1023 in. The load-deflection diagram of the tube alone is shown in Fig. 2.63b. Observing that the load and deflection of the rod-tube combination are, respectively, P 5 Pr 1 Pt 0 Fig. 2.63 118 36 (c) 90 ␦ (10–3 in.) d 5 dr 5 dt we draw the required load-deflection diagram by adding the ordinates of the diagrams obtained for the rod and for the tube (Fig. 2.63c). Points Yr and Yt correspond to the onset of yield in the rod and in the tube, respectively. bee80288_ch02_052-139.indd Page 119 11/8/10 8:15:38 PM user-f499 /Users/user-f499/Desktop/Temp Work/Don't Delete Job/MHDQ251:Beer:201/ch02 If the load P applied to the rod-tube assembly of Example 2.14 is increased from zero to 5.7 kips and decreased back to zero, determine (a) the maximum elongation of the assembly, (b) the permanent set after the load has been removed. (a) Maximum Elongation. Referring to Fig. 2.63c, we observe that the load Pmax 5 5.7 kips corresponds to a point located on the segment YrYt of the load-deflection diagram of the assembly. Thus, the rod has reached the plastic range, with Pr 5 (Pr)Y 5 2.7 kips and sr 5 (sr)Y 5 36 ksi, while the tube is still in the elastic range, with Pt 5 P 2 Pr 5 5.7 kips 2 2.7 kips 5 3.0 kips 3.0 kips Pt 5 5 30 ksi st 5 At 0.1 in2 30 3 103 psi st 130 in.2 5 60 3 1023 in. dt 5 PtL 5 L 5 Et 15 3 106 psi EXAMPLE 2.15 Pr (kips) Yr 2.7 C D 0 (a) Pt (kips) Yt The maximum elongation of the assembly, therefore, is dmax 5 dt 5 60 3 1023 in. (b) Permanent Set. As the load P decreases from 5.7 kips to zero, the internal forces Pr and Pt both decrease along a straight line, as shown in Fig. 2.64a and b, respectively. The force Pr decreases along line CD parallel to the initial portion of the loading curve, while the force Pt decreases along the original loading curve, since the yield stress was not exceeded in the tube. Their sum P, therefore, will decrease along a line CE parallel to the portion 0Yr of the load-deflection curve of the assembly (Fig. 2.64c). Referring to Fig. 2.63c, we find that the slope of 0Yr, and thus of CE, is m5 4.5 kips 36 3 1023 in. 5 125 kips/in. C 3.0 0 5.7 kips Pmax 5 245.6 3 1023 in. 52 m 125 kips/in. The permanent set is thus dP 5 dmax 1 d¿ 5 60 3 10 23 2 45.6 3 1023 5 14.4 3 1023 in.  t (10–3 in.) 60 (b) P (kips) Yt C 5.7 Yr 4.5 Pmax The segment of line FE in Fig. 2.64c represents the deformation d9 of the assembly during the unloading phase, and the segment 0E the permanent set dp after the load P has been removed. From triangle CEF we have d¿ 5 2  r (10–3 in.) 60 E 0 p F  (10–3 in.) '  max  60  10–3 in. (c) Fig. 2.64 We recall that the discussion of stress concentrations of Sec. 2.18 was carried out under the assumption of a linear stress-strain relationship. The stress distributions shown in Figs. 2.58 and 2.59, and the values of the stress-concentration factors plotted in Fig. 2.60 cannot be used, therefore, when plastic deformations take place, i.e., when the value of smax obtained from these figures exceeds the yield strength sY. Let us consider again the flat bar with a circular hole of Fig. 2.58, and let us assume that the material is elastoplastic, i.e., that its stressstrain diagram is as shown in Fig. 2.61. As long as no plastic deformation takes place, the distribution of stresses is as indicated in Sec. 2.18 119 bee80288_ch02_052-139.indd Page 120 9/4/10 5:20:08 PM user-f499 120 Stress and Strain—Axial Loading /Users/user-f499/Desktop/Temp Work/Don't Delete Job/MHDQ251:Beer:201/ch02 max P Y (a) PY max  Y (b) P (c) PU ave   Y (d) Fig. 2.65 Distribution of stresses in elastoplastic material under increasing load. (Fig. 2.65a). We observe that the area under the stress-distribution curve represents the integral e s dA, which is equal to the load P. Thus this area, and the value of smax, must increase as the load P increases. As long as smax # sY, all the successive stress distributions obtained as P increases will have the shape shown in Fig. 2.58 and repeated in Fig. 2.65a. However, as P is increased beyond the value PY corresponding to smax 5 sY (Fig. 2.65b), the stress-distribution curve must flatten in the vicinity of the hole (Fig. 2.65c), since the stress in the material considered cannot exceed the value sY. This indicates that the material is yielding in the vicinity of the hole. As the load P is further increased, the plastic zone where yield takes place keeps expanding, until it reaches the edges of the plate (Fig. 2.65d). At that point, the distribution of stresses across the plate is uniform, s 5 sY, and the corresponding value P 5 PU of the load is the largest that can be applied to the bar without causing rupture. It is interesting to compare the maximum value PY of the load that can be applied if no permanent deformation is to be produced in the bar, with the value PU that will cause rupture. Recalling the definition of the average stress, save 5 PyA, where A is the net cross-sectional area, and the definition of the stress concentration factor, K 5 smax ysave, we write P 5 save A 5 smax A K (2.49) for any value of smax that does not exceed sY. When smax 5 sY (Fig. 2.65b), we have P 5 PY, and Eq. (2.49) yields PY 5 sYA K (2.50) bee80288_ch02_052-139.indd Page 121 11/1/10 11:31:37 PM user-f499 /Users/user-f499/Desktop/Temp Work/Don't Delete Job/MHDQ251:Beer:201/ch02 *2.20 Residual Stresses On the other hand, when P 5 PU (Fig. 2.65d), we have save 5 sY and PU 5 sYA (2.51) Comparing Eqs. (2.50) and (2.51), we conclude that PU PY 5 (2.52) K *2.20 121 RESIDUAL STRESSES In Example 2.13 of the preceding section, we considered a rod that was stretched beyond the yield point. As the load was removed, the rod did not regain its original length; it had been permanently deformed. However, after the load was removed, all stresses disappeared. You should not assume that this will always be the case. Indeed, when only some of the parts of an indeterminate structure undergo plastic deformations, as in Example 2.15, or when different parts of the structure undergo different plastic deformations, the stresses in the various parts of the structure will not, in general, return to zero after the load has been removed. Stresses, called residual stresses, will remain in the various parts of the structure. While the computation of the residual stresses in an actual structure can be quite involved, the following example will provide you with a general understanding of the method to be used for their determination. Determine the residual stresses in the rod and tube of Examples 2.14 and 2.15 after the load P has been increased from zero to 5.7 kips and decreased back to zero. We observe from the diagrams of Fig. 2.66 that after the load P has returned to zero, the internal forces Pr and Pt are not equal to zero. Their values have been indicated by point E in parts a and b, respectively, of Fig. 2.66. It follows that the corresponding stresses are not equal to zero either after the assembly has been unloaded. To determine these residual stresses, we shall determine the reverse stresses s9r and s9t caused by the unloading and add them to the maximum stresses sr 5 36 ksi and st 5 30 ksi found in part a of Example 2.15. The strain caused by the unloading is the same in the rod and in the tube. It is equal to d9yL, where d9 is the deformation of the assembly during unloading, which was found in Example 2.15. We have P¿ 5 EXAMPLE 2.16 Pr (kips) C (a) D 0 E  r (10–3 in.) 60 Pt (kips) Yt C 3.0 (b) E 245.6 3 1023 in. d¿ 5 5 21.52 3 1023 in./in. L 30 in. 0 The corresponding reverse stresses in the rod and tube are  t (10–3 in.) 60 Yt P (kips) 6 s¿r 5 P¿Er 5 121.52 3 10 2 130 3 10 psi2 5 245.6 ksi s¿t 5 P¿Et 5 121.52 3 1023 2 115 3 106 psi2 5 222.8 ksi 23 Yr 2.7 C 5.7 The residual stresses are found by superposing the stresses due to loading and the reverse stresses due to unloading. We have Yr 4.5 (c) Pmax 1sr 2 res 5 sr 1 s¿r 5 36 ksi 2 45.6 ksi 5 29.6 ksi 1st 2 res 5 st 1 s¿t 5 30 ksi 2 22.8 ksi 5 17.2 ksi E 0 Fig. 2.66 p F '  (10–3 in.) bee80288_ch02_052-139.indd Page 122 11/8/10 8:16:38 PM user-f499 122 Stress and Strain—Axial Loading /Users/user-f499/Desktop/Temp Work/Don't Delete Job/MHDQ251:Beer:201/ch02 Plastic deformations caused by temperature changes can also result in residual stresses. For example, consider a small plug that is to be welded to a large plate. For discussion purposes the plug will be considered as a small rod AB that is to be welded across a small hole in the plate (Fig. 2.67). During the welding process the temperature of the rod will be raised to over 10008C, at which temperature its modulus of elasticity, and hence its stiffness and stress, will be almost zero. Since the plate is large, its temperature will not be increased significantly above room temperature (208C). Thus, when the welding is completed, we have rod AB at T 5 10008C, with no stress, attached to the plate, which is at 208C. A B Fig. 2.67 Small rod welded to a large plate. As the rod cools, its modulus of elasticity increases and, at about 5008C, will approach its normal value of about 200 GPa. As the temperature of the rod decreases further, we have a situation similar to that considered in Sec. 2.10 and illustrated in Fig. 2.31. Solving Eq. (2.23) for DT and making s equal to the yield strength, sY 5 300 MPa, of average steel, and a 5 12 3 1026/8C, we find the temperature change that will cause the rod to yield: ¢T 5 2 s 300 MPa 5 2125°C 52 Ea 1200 GPa2 112 3 1026/°C2 This means that the rod will start yielding at about 3758C and will keep yielding at a fairly constant stress level, as it cools down to room temperature. As a result of the welding operation, a residual stress approximately equal to the yield strength of the steel used is thus created in the plug and in the weld. Residual stresses also occur as a result of the cooling of metals that have been cast or hot rolled. In these cases, the outer layers cool more rapidly than the inner core. This causes the outer layers to reacquire their stiffness (E returns to its normal value) faster than the inner core. When the entire specimen has returned to room temperature, the inner core will have contracted more than the outer layers. The result is residual longitudinal tensile stresses in the inner core and residual compressive stresses in the outer layers. Residual stresses due to welding, casting, and hot rolling can be quite large (of the order of magnitude of the yield strength). These stresses can be removed, when necessary, by reheating the entire specimen to about 6008C, and then allowing it to cool slowly over a period of 12 to 24 hours. bee80288_ch02_052-139.indd Page 123 11/1/10 11:31:40 PM user-f499 Areas: AD  400 mm2 CE  500 mm2 SAMPLE PROBLEM 2.6 E 5m D 2m A C B Q 2m PAD PCE SOLUTION C Statics. Q 2m PAD (kN) 120 H Y 120 J 0 3 11 14 mm Rod AD Since Q is applied at the midpoint of the beam, we have PAD 5 PCE 2m PCE (kN) Y The rigid beam ABC is suspended from two steel rods as shown and is initially horizontal. The midpoint B of the beam is deflected 10 mm downward by the slow application of the force Q, after which the force is slowly removed. Knowing that the steel used for the rods is elastoplastic with E 5 200 GPa and sY 5 300 MPa, determine (a) the required maximum value of Q and the corresponding position of the beam, (b) the final position of the beam. 2m B A /Users/user-f499/Desktop/Temp Work/Don't Delete Job/MHDQ251:Beer:201/ch02 0 6 mm Rod CE Load-deflection diagrams 4.5 mm 6 mm B1 C1 Q = 240 kN 14 mm A2 10 mm 6 mm C1 B2 Q = 240 kN (a) Deflections for B  10 mm C = 0 11 mm A3 3 mm A2 C3 B3 B2 6 mm C2 Q=0 (b) Final deflections Q 5 2PAD Elastic Action. The maximum value of Q and the maximum elastic deflection of point A occur when s 5 sY in rod AD. 1PAD 2 max 5 sYA 5 1300 MPa2 1400 mm2 2 5 120 kN Qmax 5 240 kN Qmax 5 21PAD 2 max 5 21120 kN2 sY 300 MPa dA1 5 PL 5 L5a b 12 m2 5 3 mm E 200 GPa ◀ Since PCE 5 PAD 5 120 kN, the stress in rod CE is sCE 5 3 mm A1 and PCE 120 kN 5 5 240 MPa A 500 mm2 The corresponding deflection of point C is dC1 5 PL 5 sCE 240 MPa L5a b15 m2 5 6 mm E 200 GPa The corresponding deflection of point B is dB1 5 12 1dA1 1 dC1 2 5 12 13 mm 1 6 mm2 5 4.5 mm Since we must have dB 5 10 mm, we conclude that plastic deformation will occur. Plastic Deformation. For Q 5 240 kN, plastic deformation occurs in rod AD, where sAD 5 sY 5 300 MPa. Since the stress in rod CE is within the elastic range, dC remains equal to 6 mm. The deflection dA for which dB 5 10 mm is obtained by writing dB2 5 10 mm 5 12 1dA2 1 6 mm2 dA2 5 14 mm Unloading. As force Q is slowly removed, the force PAD decreases along line HJ parallel to the initial portion of the load-deflection diagram of rod AD. The final deflection of point A is dA3 5 14 mm 2 3 mm 5 11 mm Since the stress in rod CE remained within the elastic range, we note that the final deflection of point C is zero. 123 bee80288_ch02_052-139.indd Page 124 9/4/10 5:20:18 PM user-f499 /Users/user-f499/Desktop/Temp Work/Don't Delete Job/MHDQ251:Beer:201/ch02 PROBLEMS 1 2 1 2 3 in. 2.93 Two holes have been drilled through a long steel bar that is subjected to a centric axial load as shown. For P 5 6.5 kips, determine the maximum value of the stress (a) at A, (b) at B. in. A in. 2.94 Knowing that sall 5 16 ksi, determine the maximum allowable value of the centric axial load P. B 1 12 in. P Fig. P2.93 and P2.94 2.95 Knowing that the hole has a diameter of 9 mm, determine (a) the radius rf of the fillets for which the same maximum stress occurs at the hole A and at the fillets, (b) the corresponding maximum allowable load P if the allowable stress is 100 MPa. 9 mm rf 96 mm A 9 mm 60 mm P 9 mm Fig. P2.95 2.96 For P 5 100 kN, determine the minimum plate thickness t required if the allowable stress is 125 MPa. 88 mm rA  20 mm P 150 A rB  15 mm B 75 15 64 mm 300 P 60 r6 150 75 P' Dimensions in mm Fig. P2.97 124 t Fig. P2.96 2.97 The aluminum test specimen shown is subjected to two equal and opposite centric axial forces of magnitude P. (a) Knowing that E 5 70 GPa and sall 5 200 MPa, determine the maximum allowable value of P and the corresponding total elongation of the specimen. (b) Solve part a, assuming that the specimen has been replaced by an aluminum bar of the same length and a uniform 60 3 15-mm rectangular cross section. bee80288_ch02_052-139.indd Page 125 9/4/10 5:20:28 PM user-f499 /Users/user-f499/Desktop/Temp Work/Don't Delete Job/MHDQ251:Beer:201/ch02 Problems 2.98 For the test specimen of Prob. 2.97, determine the maximum value of the normal stress corresponding to a total elongation of 0.75 mm. 2.99 A hole is to be drilled in the plate at A. The diameters of the bits available to drill the hole range from 12 to 112 in. in 14-in. increments. If the allowable stress in the plate is 21 ksi, determine (a) the diameter d of the largest bit that can be used if the allowable load P at the hole is to exceed that at the fillets, (b) the corresponding allowable load P. 411 16 d in. A 1 2 in. rf  3 8 in. 3 18 in. P Figs. P2.99 and P2.100 2.100 (a) For P 5 13 kips and d 5 12 in., determine the maximum stress in the plate shown. (b) Solve part a, assuming that the hole at A is not drilled. 2.101 Rod ABC consists of two cylindrical portions AB and BC; it is made of a mild steel that is assumed to be elastoplastic with E 5 200 GPa and sY 5 250 MPa. A force P is applied to the rod and then removed to give it a permanent set dp 5 2 mm. Determine the maximum value of the force P and the maximum amount dm by which the rod should be stretched to give it the desired permanent set. 2.102 Rod ABC consists of two cylindrical portions AB and BC; it is made of a mild steel that is assumed to be elastoplastic with E 5 200 GPa and sY 5 250 MPa. A force P is applied to the rod until its end A has moved down by an amount dm 5 5 mm. Determine the maximum value of the force P and the permanent set of the rod after the force has been removed. 2.103 The 30-mm-square bar AB has a length L 5 2.2 m; it is made of a mild steel that is assumed to be elastoplastic with E 5 200 GPa and sY 5 345 MPa. A force P is applied to the bar until end A has moved down by an amount dm. Determine the maximum value of the force P and the permanent set of the bar after the force has been removed, knowing that (a) dm 5 4.5 mm, (b) dm 5 8 mm. 2.104 The 30-mm-square bar AB has a length L 5 2.5 m; it is made of a mild steel that is assumed to be elastoplastic with E 5 200 GPa and sY 5 345 MPa. A force P is applied to the bar and then removed to give it a permanent set dp. Determine the maximum value of the force P and the maximum amount dm by which the bar should be stretched if the desired value of dp is (a) 3.5 mm, (b) 6.5 mm. C 40-mm diameter 1.2 m B 30-mm diameter 0.8 m A P Fig. P2.101 and P2.102 B L A P Fig. P2.103 and P2.104 125 bee80288_ch02_052-139.indd Page 126 9/4/10 5:20:36 PM user-f499 126 Stress and Strain—Axial Loading A 3 -in. 8 diameter 60 in. C 3 8 D 2.107 Each cable has a cross-sectional area of 100 mm2 and is made of an elastoplastic material for which sY 5 345 MPa and E 5 200 GPa. A force Q is applied at C to the rigid bar ABC and is gradually increased from 0 to 50 kN and then reduced to zero. Knowing that the cables were initially taut, determine (a) the maximum stress that occurs in cable BD, (b) the maximum deflection of point C, (c) the final displacement of point C. (Hint: In part c, cable CE is not taut.) 1 C' 11 in. 2.105 Rod AB is made of a mild steel that is assumed to be elastoplastic with E 5 29 3 106 psi and sY 5 36 ksi. After the rod has been attached to the rigid lever CD, it is found that end C is 38 in. too high. A vertical force Q is then applied at C until this point has moved to position C9. Determine the required magnitude of Q and the deflection d1 if the lever is to snap back to a horizontal position after Q is removed. 2.106 Solve Prob. 2.105, assuming that the yield point of the mild steel is 50 ksi. B in. /Users/user-f499/Desktop/Temp Work/Don't Delete Job/MHDQ251:Beer:201/ch02 22 in. Fig. P2.105 E D 2m C B A Q 1m 1m Fig. P2.107 2.108 Solve Prob. 2.107, assuming that the cables are replaced by rods of the same cross-sectional area and material. Further assume that the rods are braced so that they can carry compressive forces. A 190 mm C 190 mm P Fig. P2.109 B 2.109 Rod AB consists of two cylindrical portions AC and BC, each with a cross-sectional area of 1750 mm2. Portion AC is made of a mild steel with E 5 200 GPa and sY 5 250 MPa, and portion CB is made of a high-strength steel with E 5 200 GPa and sY 5 345 MPa. A load P is applied at C as shown. Assuming both steels to be elastoplastic, determine (a) the maximum deflection of C if P is gradually increased from zero to 975 kN and then reduced back to zero, (b) the maximum stress in each portion of the rod, (c) the permanent deflection of C. 2.110 For the composite rod of Prob. 2.109, if P is gradually increased from zero until the deflection of point C reaches a maximum value of dm 5 0.3 mm and then decreased back to zero, determine (a) the maximum value of P, (b) the maximum stress in each portion of the rod, (c) the permanent deflection of C after the load is removed. bee80288_ch02_052-139.indd Page 127 9/4/10 5:20:44 PM user-f499 /Users/user-f499/Desktop/Temp Work/Don't Delete Job/MHDQ251:Beer:201/ch02 2.111 Two tempered-steel bars, each 163 -in. thick, are bonded to a 12-in. mild-steel bar. This composite bar is subjected as shown to a centric axial load of magnitude P. Both steels are elastoplastic with E 5 29 3 106 psi and with yield strengths equal to 100 ksi and 50 ksi, respectively, for the tempered and mild steel. The load P is gradually increased from zero until the deformation of the bar reaches a maximum value dm 5 0.04 in. and then decreased back to zero. Determine (a) the maximum value of P, (b) the maximum stress in the tempered-steel bars, (c) the permanent set after the load is removed. Problems P' 3 16 2.112 For the composite bar of Prob. 2.111, if P is gradually increased from zero to 98 kips and then decreased back to zero, determine (a) the maximum deformation of the bar, (b) the maximum stress in the tempered-steel bars, (c) the permanent set after the load is removed. 2.113 The rigid bar ABC is supported by two links, AD and BE, of uniform 37.5 3 6-mm rectangular cross section and made of a mild steel that is assumed to be elastoplastic with E 5 200 GPa and sY 5 250 MPa. The magnitude of the force Q applied at B is gradually increased from zero to 260 kN. Knowing that a 5 0.640 m, determine (a) the value of the normal stress in each link, (b) the maximum deflection of point B. 1 2 14 in. in. 3 16 in. 2.0 in. P Fig. P2.111 D E 1.7 m 1m C A B a Q 2.64 m Fig. P2.113 2.114 Solve Prob. 2.113, knowing that a 5 1.76 m and that the magnitude of the force Q applied at B is gradually increased from zero to 135 kN. *2.115 Solve Prob. 2.113, assuming that the magnitude of the force Q applied at B is gradually increased from zero to 260 kN and then decreased back to zero. Knowing that a 5 0.640 m, determine (a) the residual stress in each link, (b) the final deflection of point B. Assume that the links are braced so that they can carry compressive forces without buckling. 2.116 A uniform steel rod of cross-sectional area A is attached to rigid supports and is unstressed at a temperature of 458F. The steel is assumed to be elastoplastic with sY 5 36 ksi and E 5 29 3 106 psi. Knowing that a 5 6.5 3 1026/8F, determine the stress in the bar (a) when the temperature is raised to 3208F, (b) after the temperature has returned to 458F. A B L Fig. P2.116 in. 127 bee80288_ch02_052-139.indd Page 128 9/7/10 4:34:37 PM user-f499 128 Stress and Strain—Axial Loading /Users/user-f499/Desktop/Temp Work/Don't Delete Job/MHDQ251:Beer:201/ch07 2.117 The steel rod ABC is attached to rigid supports and is unstressed at a temperature of 258C. The steel is assumed elastoplastic with E 5 200 GPa and sY 5 250 MPa. The temperature of both portions of the rod is then raised to 1508C. Knowing that a 5 11.7 3 1026/8C, determine (a) the stress in both portions of the rod, (b) the deflection of point C. A  500 mm2 A 300 mm2 A C B 150 mm 250 mm Fig. P2.117 *2.118 Solve Prob. 2.117, assuming that the temperature of the rod is raised to 1508C and then returned to 258C. *2.119 For the composite bar of Prob. 2.111, determine the residual stresses in the tempered-steel bars if P is gradually increased from zero to 98 kips and then decreased back to zero. *2.120 For the composite bar in Prob. 2.111, determine the residual stresses in the tempered-steel bars if P is gradually increased from zero until the deformation of the bar reaches a maximum value dm 5 0.04 in. and is then decreased back to zero. Fig. P2.121 *2.121 Narrow bars of aluminum are bonded to the two sides of a thick steel plate as shown. Initially, at T1 5 708F, all stresses are zero. Knowing that the temperature will be slowly raised to T2 and then reduced to T1, determine (a) the highest temperature T2 that does not result in residual stresses, (b) the temperature T2 that will result in a residual stress in the aluminum equal to 58 ksi. Assume aa 5 12.8 3 1026/8F for the aluminum and as 5 6.5 3 1026/8F for the steel. Further assume that the aluminum is elastoplastic with E 5 10.9 3 106 psi and sY 5 58 ksi. (Hint: Neglect the small stresses in the plate.) *2.122 Bar AB has a cross-sectional area of 1200 mm2 and is made of a steel that is assumed to be elastoplastic with E 5 200 GPa and sY 5 250 MPa. Knowing that the force F increases from 0 to 520 kN and then decreases to zero, determine (a) the permanent deflection of point C, (b) the residual stress in the bar. A C B F a  120 mm 440 mm Fig. P2.122 *2.123 Solve Prob. 2.122, assuming that a 5 180 mm. bee80288_ch02_052-139.indd Page 129 9/7/10 4:34:38 PM user-f499 /Users/user-f499/Desktop/Temp Work/Don't Delete Job/MHDQ251:Beer:201/ch07 REVIEW AND SUMMARY This chapter was devoted to the introduction of the concept of strain, to the discussion of the relationship between stress and strain in various types of materials, and to the determination of the deformations of structural components under axial loading. Considering a rod of length L and uniform cross section and denoting by d its deformation under an axial load P (Fig. 2.68), we defined the normal strain P in the rod as the deformation per unit length [Sec. 2.2]: P5 d L Normal strain B B (2.1) In the case of a rod of variable cross section, the normal strain was defined at any given point Q by considering a small element of rod at Q. Denoting by Dx the length of the element and by Dd its deformation under the given load, we wrote ¢d dd P 5 lim 5 ¢xy0 ¢x dx (2.2) L C  C A P (a) Plotting the stress s versus the strain P as the load increased, we obtained a stress-strain diagram for the material used [Sec. 2.3]. From such a diagram, we were able to distinguish between brittle and ductile materials: A specimen made of a brittle material ruptures without any noticeable prior change in the rate of elongation (Fig. 2.69), while a specimen made of a ductile material yields after a critical stress sY, called the yield strength, has been reached, i.e., the specimen undergoes a large deformation before rupturing, with a relatively small increase in the applied load (Fig. 2.70). An example of brittle material with different properties in tension and in compression was provided by concrete. Rupture 60 U  (ksi) U  B Stress-strain diagram 40 Y B Fig. 2.69 Rupture 40 Y B 20 20  60 U Rupture  (ksi)  (b) Fig. 2.68 Yield Strain-hardening Necking 0.02 0.2 0.0012 (a) Low-carbon steel 0.25  0.004 0.2  (b) Aluminum alloy Fig. 2.70 129 bee80288_ch02_052-139.indd Page 130 9/4/10 5:21:10 PM user-f499 130 Stress and Strain—Axial Loading Hooke’s law Modulus of elasticity y Layer of material z x Fibers Fig. 2.71 Elastic limit. Plastic deformation  C Rupture B A D Fig. 2.72 Fatigue. Endurance limit Elastic deformation under axial loading B B L C We noted in Sec. 2.5 that the initial portion of the stress-strain diagram is a straight line. This means that for small deformations, the stress is directly proportional to the strain: s 5 EP  C P (2.4) This relation is known as Hooke’s law and the coefficient E as the modulus of elasticity of the material. The largest stress for which Eq. (2.4) applies is the proportional limit of the material. Materials considered up to this point were isotropic, i.e., their properties were independent of direction. In Sec. 2.5 we also considered a class of anisotropic materials, i.e., materials whose properties depend upon direction. They were fiber-reinforced composite materials, made of fibers of a strong, stiff material embedded in layers of a weaker, softer material (Fig. 2.71). We saw that different moduli of elasticity had to be used, depending upon the direction of loading. If the strains caused in a test specimen by the application of a given load disappear when the load is removed, the material is said to behave elastically, and the largest stress for which this occurs is called the elastic limit of the material [Sec. 2.6]. If the elastic limit is exceeded, the stress and strain decrease in a linear fashion when the load is removed and the strain does not return to zero (Fig. 2.72), indicating that a permanent set or plastic deformation of the material has taken place. In Sec. 2.7, we discussed the phenomenon of fatigue, which causes the failure of structural or machine components after a very large number of repeated loadings, even though the stresses remain in the elastic range. A standard fatigue test consists in determining the number n of successive loading-and-unloading cycles required to cause the failure of a specimen for any given maximum stress level s, and plotting the resulting s-n curve. The value of s for which failure does not occur, even for an indefinitely large number of cycles, is known as the endurance limit of the material used in the test. Section 2.8 was devoted to the determination of the elastic deformations of various types of machine and structural components under various conditions of axial loading. We saw that if a rod of length L and uniform cross section of area A is subjected at its end to a centric axial load P (Fig. 2.73), the corresponding deformation is d5 A Fig. 2.73 /Users/user-f499/Desktop/Temp Work/Don't Delete Job/MHDQ251:Beer:201/ch02 PL AE (2.7) If the rod is loaded at several points or consists of several parts of various cross sections and possibly of different materials, the deformation d of the rod must be expressed as the sum of the deformations of its component parts [Example 2.01]: P iL i d5 a A i iE i (2.8) bee80288_ch02_052-139.indd Page 131 9/4/10 5:21:16 PM user-f499 /Users/user-f499/Desktop/Temp Work/Don't Delete Job/MHDQ251:Beer:201/ch02 Review and Summary Tube (A2, E2) 131 P Rod (A1, E1) Statically indeterminate problems End plate L Fig. 2.74 RA Section 2.9 was devoted to the solution of statically indeterminate problems, i.e., problems in which the reactions and the internal forces cannot be determined from statics alone. The equilibrium equations derived from the free-body diagram of the member under consideration were complemented by relations involving deformations and obtained from the geometry of the problem. The forces in the rod and in the tube of Fig. 2.74, for instance, were determined by observing, on one hand, that their sum is equal to P, and on the other, that they cause equal deformations in the rod and in the tube [Example 2.02]. Similarly, the reactions at the supports of the bar of Fig. 2.75 could not be obtained from the free-body diagram of the bar alone [Example 2.03]; but they could be determined by expressing that the total elongation of the bar must be equal to zero. In Sec. 2.10, we considered problems involving temperature changes. We first observed that if the temperature of an unrestrained rod AB of length L is increased by DT, its elongation is A A C L1 C L L2 P P B B RB (a) (b) Fig. 2.75 Problems with temperature changes (2.21) dT 5 a1 ¢T2 L where a is the coefficient of thermal expansion of the material. We noted that the corresponding strain, called thermal strain, is (2.22) PT 5 a¢T and that no stress is associated with this strain. However, if the rod AB is restrained by fixed supports (Fig. 2.76), stresses develop in the L L A A B Fig. 2.76 rod as the temperature increases, because of the reactions at the supports. To determine the magnitude P of the reactions, we detached the rod from its support at B (Fig. 2.77) and considered separately the deformation dT of the rod as it expands freely because of the temperature change, and the deformation dP caused by the force P required to bring it back to its original length, so that it may be reattached to the support at B. Writing that the total deformation d 5 dT 1 dP is equal to zero, we obtained an equation that could be solved for P. While the final strain in rod AB is clearly zero, this will generally not be the case for rods and bars consisting of elements of different cross sections or materials, since the deformations of the various elements will usually not be zero [Example 2.06]. B (a) T A B (b) P A B P L (c) Fig. 2.77 bee80288_ch02_052-139.indd Page 132 9/4/10 5:21:25 PM user-f499 132 /Users/user-f499/Desktop/Temp Work/Don't Delete Job/MHDQ251:Beer:201/ch02 y Stress and Strain—Axial Loading A z P x Fig. 2.78 Lateral strain. Poisson’s ratio When an axial load P is applied to a homogeneous, slender bar (Fig. 2.78), it causes a strain, not only along the axis of the bar but in any transverse direction as well [Sec. 2.11]. This strain is referred to as the lateral strain, and the ratio of the lateral strain over the axial strain is called Poisson’s ratio and is denoted by n (Greek letter nu). We wrote lateral strain n52 (2.25) axial strain Recalling that the axial strain in the bar is Px 5 sxyE, we expressed as follows the condition of strain under an axial loading in the x direction: sx ns x Py 5 Pz 5 2 Px 5 (2.27) E E Multiaxial loading y x z z x y Fig. 2.79 Dilatation This result was extended in Sec. 2.12 to the case of a multiaxial loading causing the state of stress shown in Fig. 2.79. The resulting strain condition was described by the following relations, referred to as the generalized Hooke’s law for a multiaxial loading. ns y sx ns z Px 5 1 2 2 E E E s y ns z ns x Py 5 2 1 2 (2.28) E E E ns y sz ns x Pz 5 2 2 1 E E E If an element of material is subjected to the stresses sx, sy, sz, it will deform and a certain change of volume will result [Sec. 2.13]. The change in volume per unit volume is referred to as the dilatation of the material and is denoted by e. We showed that e5 1 2 2n 1sx 1 sy 1 sz 2 E (2.31) When a material is subjected to a hydrostatic pressure p, we have p e52 (2.34) k Bulk modulus where k is known as the bulk modulus of the material: k5 E 311 2 2n2 (2.33) bee80288_ch02_052-139.indd Page 133 11/1/10 11:31:43 PM user-f499 y /Users/user-f499/Desktop/Temp Work/Don't Delete Job/MHDQ251:Beer:201/ch02 y y 2 zy z xy yx yz zx 1 xy xy Q xz 133 Review and Summary yx x z 2  xy 1 z x Fig. 2.80 x Fig. 2.81 As we saw in Chap. 1, the state of stress in a material under the most general loading condition involves shearing stresses, as well as normal stresses (Fig. 2.80). The shearing stresses tend to deform a cubic element of material into an oblique parallelepiped [Sec. 2.14]. Considering, for instance, the stresses txy and tyx shown in Fig. 2.81 (which, we recall, are equal in magnitude), we noted that they cause the angles formed by the faces on which they act to either increase or decrease by a small angle gxy; this angle, expressed in radians, defines the shearing strain corresponding to the x and y directions. Defining in a similar way the shearing strains gyz and gzx, we wrote the relations txy 5 Ggxy tyz 5 Ggyz tzx 5 Ggzx y 1 P' P 1 (2.36, 37) which are valid for any homogeneous isotropic material within its proportional limit in shear. The constant G is called the modulus of rigidity of the material and the relations obtained express Hooke’s law for shearing stress and strain. Together with Eqs. (2.28), they form a group of equations representing the generalized Hooke’s law for a homogeneous isotropic material under the most general stress condition. We observed in Sec. 2.15 that while an axial load exerted on a slender bar produces only normal strains—both axial and transverse— on an element of material oriented along the axis of the bar, it will produce both normal and shearing strains on an element rotated through 458 (Fig. 2.82). We also noted that the three constants E, n, and G are not independent; they satisfy the relation. E 511n 2G Shearing strain. Modulus of rigidity x 1   x 1 (a) x P' P 2 ' 2  ' (b) Fig. 2.82 (2.43) which may be used to determine any of the three constants in terms of the other two. Stress-strain relationships for fiber-reinforced composite materials were discussed in an optional section (Sec. 2.16). Equations similar to Eqs. (2.28) and (2.36, 37) were derived for these materials, but we noted that direction-dependent moduli of elasticity, Poisson’s ratios, and moduli of rigidity had to be used. Fiber-reinforced composite materials bee80288_ch02_052-139.indd Page 134 11/2/10 3:00:01 PM user-f499 134 Stress and Strain—Axial Loading Saint-Venant’s principle Stress concentrations /Users/user-f499/Desktop/Temp Work/Don't Delete Job/MHDQ251:Beer:201/ch02 In Sec. 2.17, we discussed Saint-Venant’s principle, which states that except in the immediate vicinity of the points of application of the loads, the distribution of stresses in a given member is independent of the actual mode of application of the loads. This principle makes it possible to assume a uniform distribution of stresses in a member subjected to concentrated axial loads, except close to the points of application of the loads, where stress concentrations will occur. Stress concentrations will also occur in structural members near a discontinuity, such as a hole or a sudden change in cross section [Sec. 2.18]. The ratio of the maximum value of the stress occurring near the discontinuity over the average stress computed in the critical section is referred to as the stress-concentration factor of the discontinuity and is denoted by K: K5 s max s ave (2.48) Values of K for circular holes and fillets in flat bars were given in Fig. 2.64 on p. 108. Plastic deformotions In Sec. 2.19, we discussed the plastic deformations which occur in structural members made of a ductile material when the stresses in some part of the member exceed the yield strength of the material. Our analysis was carried out for an idealized elastoplastic material characterized by the stress-strain diagram shown in Fig. 2.83 [Examples 2.13, 2.14, and 2.15]. Finally, in Sec. 2.20, we observed that when an indeterminate structure undergoes plastic deformations, the stresses do not, in general, return to zero after the load has been removed. The stresses remaining in the various parts of the structure are called residual stresses and may be determined by adding the maximum stresses reached during the loading phase and the reverse stresses corresponding to the unloading phase [Example 2.16]. ␴ ␴Y A Fig. 2.83 Y C D Rupture ⑀ bee80288_ch02_052-139.indd Page 135 9/4/10 5:22:24 PM user-f499 /Users/user-f499/Desktop/Temp Work/Don't Delete Job/MHDQ251:Beer:201/ch02 REVIEW PROBLEMS 2.124 Rod BD is made of steel (E 5 29 3 10 6 psi) and is used to brace the axially compressed member ABC. The maximum force that can be developed in member BD is 0.02P. If the stress must not exceed 18 ksi and the maximum change in length of BD must not exceed 0.001 times the length of ABC, determine the smallest-diameter rod that can be used for member BD. P  130 kips A 72 in. D 2.125 Two solid cylindrical rods are joined at B and loaded as shown. Rod AB is made of steel (E 5 200 GPa) and rod BC of brass (E 5 105 GPa). Determine (a) the total deformation of the composite rod ABC, (b) the deflection of point B. B 72 in. C P  30 kN 54 in. A Fig. P2.124 30 mm 250 mm 40 kN B 50 mm 300 mm C Fig. P2.125 2.126 Two solid cylindrical rods are joined at B and loaded as shown. Rod AB is made of steel (E 5 29 3 10 6 psi), and rod BC of brass (E 5 15 3 10 6 psi). Determine (a) the total deformation of the composite rod ABC, (b) the deflection of point B. C 2.127 The uniform wire ABC, of unstretched length 2l, is attached to the supports shown and a vertical load P is applied at the midpoint B. Denoting by A the cross-sectional area of the wire and by E the modulus of elasticity, show that, for d V l, the deflection at the midpoint B is P d5l 3 B AE l 3 in. 30 in. B 40 in. 30 kips 30 kips 2 in. A l P  40 kips  A C Fig. P2.126 B P Fig. P2.127 135 bee80288_ch02_052-139.indd Page 136 9/4/10 5:22:35 PM user-f499 136 /Users/user-f499/Desktop/Temp Work/Don't Delete Job/MHDQ251:Beer:201/ch02 2.128 The brass strip AB has been attached to a fixed support at A Stress and Strain—Axial Loading and rests on a rough support at B. Knowing that the coefficient of friction is 0.60 between the strip and the support at B, determine the decrease in temperature for which slipping will impend. Brass strip: E  105 GPa   20  106/C A 3 mm C B h 40 mm 20 mm 100 kg B Fig. P2.128 A D 3 ft Fig. P2.129 2.129 Members AB and CD are 118-in.-diameter steel rods, and members BC and AD are 78-in.-diameter steel rods. When the turnbuckle is tightened, the diagonal member AC is put in tension. Knowing that E 5 29 3 10 6 psi and h 5 4 ft, determine the largest allowable tension in AC so that the deformations in members AB and CD do not exceed 0.04 in. 2.130 The 1.5-m concrete post is reinforced with six steel bars, each with P a 28-mm diameter. Knowing that Es 5 200 GPa and Ec 5 25 GPa, determine the normal stresses in the steel and in the concrete when a 1550-kN axial centric force P is applied to the post. 450 mm 1.5 m 2.131 The brass shell (ab 5 11.6 3 1026/8F) is fully bonded to the steel core (as 5 6.5 3 1026/8F). Determine the largest allowable increase in temperature if the stress in the steel core is not to exceed 8 ksi. 1 4 1 4 Fig. P2.130 1 in. in. in. 1 4 in. 1 in. 1 4 Steel core E  29  106 psi Brass shell E  15  106 psi Fig. P2.131 12 in. in. bee80288_ch02_052-139.indd Page 137 9/4/10 5:22:48 PM user-f499 /Users/user-f499/Desktop/Temp Work/Don't Delete Job/MHDQ251:Beer:201/ch02 Review Problems 2.132 A fabric used in air-inflated structures is subjected to a biax- ial loading that results in normal stresses sx 5 120 MPa and sz 5 160 MPa. Knowing that the properties of the fabric can be approximated as E 5 87 GPa and n 5 0.34, determine the change in length of (a) side AB, (b) side BC, (c) diagonal AC. y 100 mm 75 mm 2.133 An elastomeric bearing (G 5 0.9 MPa) is used to support a bridge girder as shown to provide flexibility during earthquakes. The beam must not displace more than 10 mm when a 22-kN lateral load is applied as shown. Knowing that the maximum allowable shearing stress is 420 kPa, determine (a) the smallest allowable dimension b, (b) the smallest required thickness a. A B D z z Fig. P2.132 P 2.50 in. r a b 200 mm Fig. P2.133 5.0 in. 3 4 (a) r 5 0.50 in., (b) r 5 0.625 in. 2.135 The uniform rod BC has cross-sectional area A and is made of a mild steel that can be assumed to be elastoplastic with a modulus of elasticity E and a yield strength sY. Using the block-and-spring system shown, it is desired to simulate the deflection of end C of the rod as the axial force P is gradually applied and removed, that is, the deflection of points C and C9 should be the same for all values of P. Denoting by m the coefficient of friction between the block and the horizontal surface, derive an expression for (a) the required mass m of the block, (b) the required constant k of the spring. L B C B' m Fig. P2.135 k C' P P Fig. P2.134 x x C P 2.134 Knowing that P 5 10 kips, determine the maximum stress when 137 in. bee80288_ch02_052-139.indd Page 138 9/4/10 5:22:56 PM user-f499 /Users/user-f499/Desktop/Temp Work/Don't Delete Job/MHDQ251:Beer:201/ch02 COMPUTER PROBLEMS The following problems are designed to be solved with a computer. Write each program so that it can be used with either SI or U.S. customary units and in such a way that solid cylindrical elements may be defined by either their diameter or their cross-sectional area. Element 1 Element n P1 Pn Fig. P2.C1 2.C2 Rod AB is horizontal with both ends fixed; it consists of n elements, each of which is homogeneous and of uniform cross section, and is subjected to the loading shown. The length of element i is denoted by Li, its cross-sectional area by Ai, its modulus of elasticity by Ei, and the load applied to its right end by Pi, the magnitude Pi of this load being assumed to be positive if Pi is directed to the right and negative otherwise. (Note that P1 5 0.) (a) Write a computer program that can be used to determine the reactions at A and B, the average normal stress in each element, and the deformation of each element. (b) Use this program to solve Probs. 2.41 and 2.42. Element 1 Element n A Pn B P2 Fig. P2.C2 Element n Element 1 0 2.C3 Rod AB consists of n elements, each of which is homogeneous and of uniform cross section. End A is fixed, while initially there is a gap d0 between end B and the fixed vertical surface on the right. The length of element i is denoted by Li, its cross-sectional area by Ai, its modulus of elasticity by Ei, and its coefficient of thermal expansion by ai. After the temperature of the rod has been increased by DT, the gap at B is closed and the vertical surfaces exert equal and opposite forces on the rod. (a) Write a computer program that can be used to determine the magnitude of the reactions at A and B, the normal stress in each element, and the deformation of each element. (b) Use this program to solve Probs. 2.51, 2.59, and 2.60. B A Fig. P2.C3 A 1, E1, (Y)1 L P A 2 , E2 , ( Y)2 Fig. P2.C4 138 Plate 2.C1 A rod consisting of n elements, each of which is homogeneous and of uniform cross section, is subjected to the loading shown. The length of element i is denoted by Li, its cross-sectional area by Ai, modulus of elasticity by Ei, and the load applied to its right end by Pi, the magnitude Pi of this load being assumed to be positive if Pi is directed to the right and negative otherwise. (a) Write a computer program that can be used to determine the average normal stress in each element, the deformation of each element, and the total deformation of the rod. (b) Use this program to solve Probs. 2.20 and 2.126. 2.C4 Bar AB has a length L and is made of two different materials of given cross-sectional area, modulus of elasticity, and yield strength. The bar is subjected as shown to a load P that is gradually increased from zero until the deformation of the bar has reached a maximum value dm and then decreased back to zero. (a) Write a computer program that, for each of 25 values of dm equally spaced over a range extending from 0 to a value equal to 120% of the deformation causing both materials to yield, can be used to determine the maximum value Pm of the load, the maximum normal stress in each material, the permanent deformation dp of the bar, and the residual stress in each material. (b) Use this program to solve Probs. 2.111 and 2.112. bee80288_ch02_052-139.indd Page 139 9/4/10 5:23:10 PM user-f499 /Users/user-f499/Desktop/Temp Work/Don't Delete Job/MHDQ251:Beer:201/ch02 Computer Problems 2.C5 The plate has a hole centered across the width. The stress concentration factor for a flat bar under axial loading with a centric hole is: K 5 3.00 2 3.13 a 2r 2r 2 2r 3 b 1 3.66 a b 2 1.53 a b D D D where r is the radius of the hole and D is the width of the bar. Write a computer program to determine the allowable load P for the given values of r, D, the thickness t of the bar, and the allowable stress sall of the material. Knowing that t 5 14 in., D 5 3.0 in. and sall 5 16 ksi, determine the allowable load P for values of r from 0.125 in. to 0.75 in., using 0.125 in. increments. 2.C6 A solid truncated cone is subjected to an axial force P as shown. The exact elongation is (PL)y(2pc2E). By replacing the cone by n circular cylinders of equal thickness, write a computer program that can be used to calculate the elongation of the truncated cone. What is the percentage error in the answer obtained from the program using (a) n 5 6, (b) n 5 12, (c) n 5 60? L A B P 2c c Fig. P2.C6 P' 1 2 d 1 2 d r D Fig. P2.C5 P 139 bee80288_ch03_140-219.indd Page 140 9/21/10 2:58:29 PM user-f499 This chapter is devoted to the study of torsion and of the stresses and deformations it causes. In the jet engine shown here, the central shaft links the components of the engine to develop the thrust that propels the plane. 140 /Users/user-f499/Desktop/Temp Work/Don't Delete Job/MHDQ251:Beer:201/ch03 bee80288_ch03_140-219.indd Page 141 9/21/10 3:03:30 PM user-f499 /Users/user-f499/Desktop/Temp Work/Don't Delete Job/MHDQ251:Beer:201/ch03 C H A P T E R Torsion 141 bee80288_ch03_140-219.indd Page 142 11/2/10 12:48:11 AM user-f499 3.1 Chapter 3 Torsion 3.1 3.2 3.3 3.4 3.5 3.6 3.7 3.8 *3.9 *3.10 *3.11 *3.12 *3.13 Introduction Preliminary Discussion of the Stresses in a Shaft Deformations in a Circular Shaft Stresses in the Elastic Range Angle of Twist in the Elastic Range Statically Indeterminate Shafts Design of Transmission Shafts Stress Concentrations in Circular Shafts Plastic Deformations in Circular Shafts Circular Shafts Made of an Elastoplastic Material Residual Stresses in Circular Shafts Torsion of Noncircular Members Thin-Walled Hollow Shafts /Users/user-f499/Desktop/Temp Work/Don't Delete Job/MHDQ251:Beer:201/ch03 INTRODUCTION In the two preceding chapters you studied how to calculate the stresses and strains in structural members subjected to axial loads, that is, to forces directed along the axis of the member. In this chapter structural members and machine parts that are in torsion will be considered. More specifically, you will analyze the stresses and strains in members of circular cross section subjected to twisting couples, or torques, T and T9 (Fig. 3.1). These couples have a common magnitude T, and opposite senses. They are vector quantities and can be represented either by curved arrows as in Fig. 3.1a, or by couple vectors as in Fig. 3.1b. Members in torsion are encountered in many engineering applications. The most common application is provided by transmission shafts, which are used to transmit power from one point to another. For example, the shaft shown in Photo 3.1 is used to transmit power from the engine to the rear wheels of an automobile. These shafts can be either solid, as shown in Fig. 3.1, or hollow. B T T' A (a) T' B T (b) Fig. 3.1 A Shaft subject to torsion. Photo 3.1 In the automotive power train shown, the shaft transmits power from the engine to the rear wheels. Consider the system shown in Fig. 3.2a, which consists of a steam turbine A and an electric generator B connected by a transmission shaft AB. By breaking the system into its three component parts (Fig. 3.2b), you can see that the turbine exerts a twisting couple or torque T on the shaft and that the shaft exerts an equal torque on the generator. The generator reacts by exerting the equal and opposite torque T9 on the shaft, and the shaft by exerting the torque T9 on the turbine. You will first analyze the stresses and deformations that take place in circular shafts. In Sec. 3.3, an important property of circular shafts is demonstrated: When a circular shaft is subjected to torsion, 142 bee80288_ch03_140-219.indd Page 143 9/21/10 3:03:48 PM user-f499 /Users/user-f499/Desktop/Temp Work/Don't Delete Job/MHDQ251:Beer:201/ch03 Generator 3.1 Introduction B Rotation Turbine A (a) T B T T' A T' (b) Fig. 3.2 Transmission shaft. every cross section remains plane and undistorted. In other words, while the various cross sections along the shaft rotate through different angles, each cross section rotates as a solid rigid slab. This property will enable you to determine the distribution of shearing strains in a circular shaft and to conclude that the shearing strain varies linearly with the distance from the axis of the shaft. Considering deformations in the elastic range and using Hooke’s law for shearing stress and strain, you will determine the distribution of shearing stresses in a circular shaft and derive the elastic torsion formulas (Sec. 3.4). In Sec. 3.5, you will learn how to find the angle of twist of a circular shaft subjected to a given torque, assuming again elastic deformations. The solution of problems involving statically indeterminate shafts is considered in Sec. 3.6. In Sec. 3.7, you will study the design of transmission shafts. In order to accomplish the design, you will learn to determine the required physical characteristics of a shaft in terms of its speed of rotation and the power to be transmitted. The torsion formulas cannot be used to determine stresses near sections where the loading couples are applied or near a section where an abrupt change in the diameter of the shaft occurs. Moreover, these formulas apply only within the elastic range of the material. 143 bee80288_ch03_140-219.indd Page 144 9/21/10 3:03:50 PM user-f499 144 In Sec. 3.8, you will learn how to account for stress concentrations where an abrupt change in diameter of the shaft occurs. In Secs. 3.9 to 3.11, you will consider stresses and deformations in circular shafts made of a ductile material when the yield point of the material is exceeded. You will then learn how to determine the permanent plastic deformations and residual stresses that remain in a shaft after it has been loaded beyond the yield point of the material. In the last sections of this chapter, you will study the torsion of noncircular members (Sec. 3.12) and analyze the distribution of stresses in thin-walled hollow noncircular shafts (Sec. 3.13). Torsion 3.2 B C T A T' Fig. 3.3 /Users/user-f499/Desktop/Temp Work/Don't Delete Job/MHDQ251:Beer:201/ch03 Shaft subject to torques. B C ␳ T' dF (a) Considering a shaft AB subjected at A and B to equal and opposite torques T and T9, we pass a section perpendicular to the axis of the shaft through some arbitrary point C (Fig. 3.3). The free-body diagram of the portion BC of the shaft must include the elementary shearing forces dF, perpendicular to the radius of the shaft, that portion AC exerts on BC as the shaft is twisted (Fig. 3.4a). But the conditions of equilibrium for BC require that the system of these elementary forces be equivalent to an internal torque T, equal and opposite to T9 (Fig. 3.4b). Denoting by r the perpendicular distance from the force dF to the axis of the shaft, and expressing that the sum of the moments of the shearing forces dF about the axis of the shaft is equal in magnitude to the torque T, we write erdF 5 T B T C T' (b) Fig. 3.4 ␶ Axis of shaft Fig. 3.5 PRELIMINARY DISCUSSION OF THE STRESSES IN A SHAFT Element in shaft. or, since dF 5 t dA, where t is the shearing stress on the element of area dA, er(t dA) 5 T (3.1) While the relation obtained expresses an important condition that must be satisfied by the shearing stresses in any given cross section of the shaft, it does not tell us how these stresses are distributed in the cross section. We thus observe, as we already did in Sec. 1.5, that the actual distribution of stresses under a given load is statically indeterminate, i.e., this distribution cannot be determined by the methods of statics. However, having assumed in Sec. 1.5 that the normal stresses produced by an axial centric load were uniformly distributed, we found later (Sec. 2.17) that this assumption was justified, except in the neighborhood of concentrated loads. A similar assumption with respect to the distribution of shearing stresses in an elastic shaft would be wrong. We must withhold any judgment regarding the distribution of stresses in a shaft until we have analyzed the deformations that are produced in the shaft. This will be done in the next section. One more observation should be made at this point. As was indicated in Sec. 1.12, shear cannot take place in one plane only. Consider the very small element of shaft shown in Fig. 3.5. We know that the torque applied to the shaft produces shearing stresses t on bee80288_ch03_140-219.indd Page 145 9/21/10 3:03:56 PM user-f499 /Users/user-f499/Desktop/Temp Work/Don't Delete Job/MHDQ251:Beer:201/ch03 3.3 Deformations in a Circular Shaft T' Fig. 3.6 (a) Model of shaft. T (b) the faces perpendicular to the axis of the shaft. But the conditions of equilibrium discussed in Sec. 1.12 require the existence of equal stresses on the faces formed by the two planes containing the axis of the shaft. That such shearing stresses actually occur in torsion can be demonstrated by considering a “shaft” made of separate slats pinned at both ends to disks as shown in Fig. 3.6a. If markings have been painted on two adjoining slats, it is observed that the slats slide with respect to each other when equal and opposite torques are applied to the ends of the “shaft” (Fig. 3.6b). While sliding will not actually take place in a shaft made of a homogeneous and cohesive material, the tendency for sliding will exist, showing that stresses occur on longitudinal planes as well as on planes perpendicular to the axis of the shaft.† 3.3 B A (a) L DEFORMATIONS IN A CIRCULAR SHAFT Consider a circular shaft that is attached to a fixed support at one end (Fig. 3.7a). If a torque T is applied to the other end, the shaft will twist, with its free end rotating through an angle f called the angle of twist (Fig. 3.7b). Observation shows that, within a certain range of values of T, the angle of twist f is proportional to T. It also shows that f is proportional to the length L of the shaft. In other words, the angle of twist for a shaft of the same material and same cross section, but twice as long, will be twice as large under the same torque T. One purpose of our analysis will be to find the specific relation existing among f, L, and T; another purpose will be to determine the distribution of shearing stresses in the shaft, which we were unable to obtain in the preceding section on the basis of statics alone. At this point, an important property of circular shafts should be noted: When a circular shaft is subjected to torsion, every cross section remains plane and undistorted. In other words, while the various cross sections along the shaft rotate through different amounts, each cross section rotates as a solid rigid slab. This is illustrated in Fig. 3.8a, which shows the deformations in a rubber model subjected to torsion. The property we are discussing is characteristic of circular shafts, whether solid or hollow; it is not enjoyed by members of noncircular cross section. For example, when a bar of square cross section is subjected to torsion, its various cross sections warp and do not remain plane (Fig. 3.8b). †The twisting of a cardboard tube that has been slit lengthwise provides another demonstration of the existence of shearing stresses on longitudinal planes. B A' A (b) Fig. 3.7 T ␾ Shaft with fixed support. T T' (a) T T' (b) Fig. 3.8 Comparison of deformations in circular and square shafts. 145 bee80288_ch03_140-219.indd Page 146 9/21/10 3:04:03 PM user-f499 146 Torsion B D' C' T' D T A C (a) B D' C' T' D T C A (b) Fig. 3.9 Shaft subject to twisting. B T A T' Fig. 3.10 Concentric circles. /Users/user-f499/Desktop/Temp Work/Don't Delete Job/MHDQ251:Beer:201/ch03 The cross sections of a circular shaft remain plane and undistorted because a circular shaft is axisymmetric, i.e., its appearance remains the same when it is viewed from a fixed position and rotated about its axis through an arbitrary angle. (Square bars, on the other hand, retain the same appearance only if they are rotated through 908 or 1808.) As we will see presently, the axisymmetry of circular shafts may be used to prove theoretically that their cross sections remain plane and undistorted. Consider the points C and D located on the circumference of a given cross section of the shaft, and let C9 and D9 be the positions they will occupy after the shaft has been twisted (Fig. 3.9a). The axisymmetry of the shaft and of the loading requires that the rotation which would have brought D into D9 should now bring C into C9. Thus C9 and D9 must lie on the circumference of a circle, and the arc C9D9 must be equal to the arc CD (Fig. 3.9b). We will now examine whether the circle on which C9 and D9 lie is different from the original circle. Let us assume that C9 and D9 do lie on a different circle and that the new circle is located to the left of the original circle, as shown in Fig. 3.9b. The same situation will prevail for any other cross section, since all the cross sections of the shaft are subjected to the same internal torque T, and an observer looking at the shaft from its end A will conclude that the loading causes any given circle drawn on the shaft to move away. But an observer located at B, to whom the given loading looks the same (a clockwise couple in the foreground and a counterclockwise couple in the background) will reach the opposite conclusion, i.e., that the circle moves toward him. This contradiction proves that our assumption is wrong and that C9 and D9 lie on the same circle as C and D. Thus, as the shaft is twisted, the original circle just rotates in its own plane. Since the same reasoning may be applied to any smaller, concentric circle located in the cross section under consideration, we conclude that the entire cross section remains plane (Fig. 3.10). The above argument does not preclude the possibility for the various concentric circles of Fig. 3.10 to rotate by different amounts when the shaft is twisted. But if that were so, a given diameter of the cross section would be distorted into a curve which might look as shown in Fig. 3.11a. An observer looking at this curve from A would conclude that the outer layers of the shaft get more twisted than the inner ones, while an observer looking from B would reach the opposite conclusion (Fig. 3.11b). This inconsistency leads us to conclude that any diameter of a given cross section remains straight (Fig. 3.11c) and, therefore, that any given cross section of a circular shaft remains plane and undistorted. B T' T T' A A T T' A B (a) Fig. 3.11 B T Potential deformations of cross section. (b) (c) bee80288_ch03_140-219.indd Page 147 9/21/10 3:04:11 PM user-f499 /Users/user-f499/Desktop/Temp Work/Don't Delete Job/MHDQ251:Beer:201/ch03 Our discussion so far has ignored the mode of application of the twisting couples T and T9. If all sections of the shaft, from one end to the other, are to remain plane and undistorted, we must make sure that the couples are applied in such a way that the ends of the shaft themselves remain plane and undistorted. This may be accomplished by applying the couples T and T9 to rigid plates, which are solidly attached to the ends of the shaft (Fig. 3.12a). We can then be sure that all sections will remain plane and undistorted when the loading is applied, and that the resulting deformations will occur in a uniform fashion throughout the entire length of the shaft. All of the equally spaced circles shown in Fig. 3.12a will rotate by the same amount relative to their neighbors, and each of the straight lines will be transformed into a curve (helix) intersecting the various circles at the same angle (Fig. 3.12b). The derivations given in this and the following sections will be based on the assumption of rigid end plates. Loading conditions encountered in practice may differ appreciably from those corresponding to the model of Fig. 3.12. The chief merit of this model is that it helps us define a torsion problem for which we can obtain an exact solution, just as the rigid-end-plates model of Sec. 2.17 made it possible for us to define an axial-load problem which could be easily and accurately solved. By virtue of Saint-Venant’s principle, the results obtained for our idealized model may be extended to most engineering applications. However, we should keep these results associated in our mind with the specific model shown in Fig. 3.12. We will now determine the distribution of shearing strains in a circular shaft of length L and radius c that has been twisted through an angle f (Fig. 3.13a). Detaching from the shaft a cylinder of radius r, we consider the small square element formed by two adjacent circles and two adjacent straight lines traced on the surface of the cylinder before any load is applied (Fig. 3.13b). As the shaft is subjected to a torsional load, the element deforms into a rhombus (Fig. 3.13c). We now recall from Sec. 2.14 that the shearing strain g in a given element is measured by the change in the angles formed by the sides of that element. Since the circles defining two of the sides of the element considered here remain unchanged, the shearing strain g must be equal to the angle between lines AB and A9B. (We recall that g should be expressed in radians.) We observe from Fig. 3.13c that, for small values of g, we can express the arc length AA9 as AA9 5 Lg. But, on the other hand, we have AA9 5 rf. It follows that Lg 5 rf, or rf g5 L 3.3 Deformations in a Circular Shaft (a) T' T (b) Fig. 3.12 Deformation of shaft subject to twisting couples. c O ␾ (a) L B (b) ␳ A' ␳ L B ␥ A (3.2) where g and f are both expressed in radians. The equation obtained shows, as we could have anticipated, that the shearing strain g at a given point of a shaft in torsion is proportional to the angle of twist f. It also shows that g is proportional to the distance r from the axis of the shaft to the point under consideration. Thus, the shearing strain in a circular shaft varies linearly with the distance from the axis of the shaft. O A (c) Fig. 3.13 L Shearing strain. O ␾ 147 bee80288_ch03_140-219.indd Page 148 9/21/10 3:04:16 PM user-f499 148 Torsion /Users/user-f499/Desktop/Temp Work/Don't Delete Job/MHDQ251:Beer:201/ch03 It follows from Eq. (3.2) that the shearing strain is maximum on the surface of the shaft, where r 5 c. We have gmax 5 cf L (3.3) Eliminating f from Eqs. (3.2) and (3.3), we can express the shearing strain g at a distance r from the axis of the shaft as g5 3.4 r g c max (3.4) STRESSES IN THE ELASTIC RANGE No particular stress-strain relationship has been assumed so far in our discussion of circular shafts in torsion. Let us now consider the case when the torque T is such that all shearing stresses in the shaft remain below the yield strength tY. We know from Chap. 2 that, for all practical purposes, this means that the stresses in the shaft will remain below the proportional limit and below the elastic limit as well. Thus, Hooke’s law will apply and there will be no permanent deformation. Recalling Hooke’s law for shearing stress and strain from Sec. 2.14, we write t 5 Gg (3.5) where G is the modulus of rigidity or shear modulus of the material. Multiplying both members of Eq. (3.4) by G, we write Gg 5 r Ggmax c or, making use of Eq. (3.5), t5 r t c max (3.6) The equation obtained shows that, as long as the yield strength (or proportional limit) is not exceeded in any part of a circular shaft, the shearing stress in the shaft varies linearly with the distance r from the axis of the shaft. Figure 3.14a shows the stress distribution in a solid circular shaft of radius c, and Fig. 3.14b in a hollow circular shaft of inner radius c1 and outer radius c2. From Eq. (3.6), we find that, in the latter case, tmin 5 c1 t c2 max (3.7) We now recall from Sec. 3.2 that the sum of the moments of the elementary forces exerted on any cross section of the shaft must be equal to the magnitude T of the torque exerted on the shaft: er(t dA) 5 T (3.1) bee80288_ch03_140-219.indd Page 149 9/21/10 3:04:17 PM user-f499 ␶ ␶ ␶max O c ␶min O ␳ c1 3.4 Stresses in the Elastic Range ␶max c2 ␳ (b) (a) Fig. 3.14 /Users/user-f499/Desktop/Temp Work/Don't Delete Job/MHDQ251:Beer:201/ch03 Distribution of shearing stresses. Substituting for t from (3.6) into (3.1), we write tmax 2 T 5 e rt dA 5 r dA c e But the integral in the last member represents the polar moment of inertia J of the cross section with respect to its center O. We have therefore T5 tmax J c (3.8) Tc J (3.9) or, solving for tmax, tmax 5 Substituting for tmax from (3.9) into (3.6), we express the shearing stress at any distance r from the axis of the shaft as t5 Tr J (3.10) Equations (3.9) and (3.10) are known as the elastic torsion formulas. We recall from statics that the polar moment of inertia of a circle of radius c is J 5 12 pc4. In the case of a hollow circular shaft of inner radius c1 and outer radius c2, the polar moment of inertia is J 5 12 pc42 2 12 pc41 5 12 p 1c42 2 c41 2 (3.11) We note that, if SI metric units are used in Eq. (3.9) or (3.10), T will be expressed in N ? m, c or r in meters, and J in m4; we check that the resulting shearing stress will be expressed in N/m2, that is, pascals (Pa). If U.S. customary units are used, T should be expressed in lb ? in., c or r in inches, and J in in4, with the resulting shearing stress expressed in psi. 149 bee80288_ch03_140-219.indd Page 150 11/2/10 12:48:16 AM user-f499 EXAMPLE 3.01 T 60 mm 40 mm 1.5 m /Users/user-f499/Desktop/Temp Work/Don't Delete Job/MHDQ251:Beer:201/ch03 A hollow cylindrical steel shaft is 1.5 m long and has inner and outer diameters respectively equal to 40 and 60 mm (Fig. 3.15). (a) What is the largest torque that can be applied to the shaft if the shearing stress is not to exceed 120 MPa? (b) What is the corresponding minimum value of the shearing stress in the shaft? (a) Largest Permissible Torque. The largest torque T that can be applied to the shaft is the torque for which tmax 5 120 MPa. Since this value is less than the yield strength for steel, we can use Eq. (3.9). Solving this equation for T, we have T5 Fig. 3.15 Jtmax c (3.12) Recalling that the polar moment of inertia J of the cross section is given by Eq. (3.11), where c1 5 12 140 mm2 5 0.02 m and c2 5 12 160 mm2 5 0.03 m, we write J 5 12 p 1c42 2 c41 2 5 12 p10.034 2 0.024 2 5 1.021 3 1026 m4 Substituting for J and tmax into (3.12), and letting c 5 c2 5 0.03 m, we have T5 Jtmax c 5 11.021 3 1026 m 4 2 1120 3 106 Pa2 0.03 m 5 4.08 kN ? m (b) Minimum Shearing Stress. The minimum value of the shearing stress occurs on the inner surface of the shaft. It is obtained from Eq. (3.7), which expresses that tmin and tmax are respectively proportional to c1 and c2: tmin 5 E TE S TC B TB A C TA (a) E TE B TB T (b) Fig. 3.16 section. 150 S Shaft with variable cross c1 0.02 m t 5 1120 MPa2 5 80 MPa c2 max 0.03 m The torsion formulas (3.9) and (3.10) were derived for a shaft of uniform circular cross section subjected to torques at its ends. However, they can also be used for a shaft of variable cross section or for a shaft subjected to torques at locations other than its ends (Fig. 3.16a). The distribution of shearing stresses in a given cross section S of the shaft is obtained from Eq. (3.9), where J denotes the polar moment of inertia of that section, and where T represents the internal torque in that section. The value of T is obtained by drawing the free-body diagram of the portion of shaft located on one side of the section (Fig. 3.16b) and writing that the sum of the torques applied to that portion, including the internal torque T, is zero (see Sample Prob. 3.1). Up to this point, our analysis of stresses in a shaft has been limited to shearing stresses. This is due to the fact that the element we had selected was oriented in such a way that its faces were either parallel or perpendicular to the axis of the shaft (Fig. 3.5). We know from earlier discussions (Secs. 1.11 and 1.12) that normal stresses, shearing stresses, or a combination of both may be found under the same loading condition, depending upon the orientation of the element that has been chosen. Consider the two elements a and b located on the surface of a circular shaft subjected to torsion bee80288_ch03_140-219.indd Page 151 9/21/10 3:04:24 PM user-f499 /Users/user-f499/Desktop/Temp Work/Don't Delete Job/MHDQ251:Beer:201/ch03 (Fig. 3.17). Since the faces of element a are respectively parallel and perpendicular to the axis of the shaft, the only stresses on the element will be the shearing stresses defined by formula (3.9), namely tmax 5 TcyJ. On the other hand, the faces of element b, which form arbitrary angles with the axis of the shaft, will be subjected to a combination of normal and shearing stresses. Let us consider the stresses and resulting forces on faces that are at 458 to the axis of the shaft. In order to determine the stresses on the faces of this element, we consider the two triangular elements shown in Fig. 3.18 and draw their free-body diagrams. In the case of the element of Fig. 3.18a, we know that the stresses exerted on the faces BC and BD are the shearing stresses tmax 5 TcyJ. The magnitude of the corresponding shearing forces is thus tmax A0, where A0 denotes the area of the face. Observing that the components along DC of the two shearing forces are equal and opposite, we conclude that the force F exerted on DC must be perpendicular to that face. It is a tensile force, and its magnitude is F 5 21tmax A0 2cos 45° 5 tmax A0 12 tmax A0 12 F 5 5 tmax A A0 12 T ␶max T' a b Fig. 3.17 Circular shaft with elements at different orientations. F D ␶max A0 E B ␶max A0 C B C ␶max A0 (a) Fig. 3.18 shaft axis. ␶max A0 45⬚ (b) Forces on faces at 458 to (3.14) A similar analysis of the element of Fig. 3.18b shows that the stress on the face BE is s 5 2tmax. We conclude that the stresses exerted on the faces of an element c at 458 to the axis of the shaft (Fig. 3.19) are normal stresses equal to 6tmax. Thus, while the element a in Fig. 3.19 is in pure shear, the element c in the same figure is subjected to a tensile stress on two of its faces, and to a compressive stress on the other two. We also note that all the stresses involved have the same magnitude, TcyJ.† As you learned in Sec. 2.3, ductile materials generally fail in shear. Therefore, when subjected to torsion, a specimen J made of a ductile material breaks along a plane perpendicular to its longitudinal axis (Photo 3.2a). On the other hand, brittle materials are weaker in tension than in shear. Thus, when subjected to torsion, a specimen made of a brittle material tends to break along surfaces that are perpendicular to the direction in which tension is maximum, i.e., along surfaces forming a 458 angle with the longitudinal axis of the specimen (Photo 3.2b). T T' a ␶max ⫽ Tc J c ␴45⬚ ⫽⫾ Tc J Fig. 3.19 Shaft with elements with only shear stresses or normal stresses. T T' (a) Ductile failure F' 45⬚ (3.13) The corresponding stress is obtained by dividing the force F by the area A of face DC. Observing that A 5 A0 12, we write s5 3.4 Stresses in the Elastic Range T T' Photo 3.2 Shear failure of shaft subject to torque. †Stresses on elements of arbitrary orientation, such as element b of Fig. 3.18, will be discussed in Chap. 7. (b) Brittle failure 151 bee80288_ch03_140-219.indd Page 152 9/21/10 3:04:31 PM user-f499 /Users/user-f499/Desktop/Temp Work/Don't Delete Job/MHDQ251:Beer:201/ch03 SAMPLE PROBLEM 3.1 0.9 m 0.7 m d 0.5 m A 120 mm Shaft BC is hollow with inner and outer diameters of 90 mm and 120 mm, respectively. Shafts AB and CD are solid and of diameter d. For the loading shown, determine (a) the maximum and minimum shearing stress in shaft BC, (b) the required diameter d of shafts AB and CD if the allowable shearing stress in these shafts is 65 MPa. d TA  6 kN · m B TB  14 kN · m C TC  26 kN · m D TD  6 kN · m TA  6 kN · m SOLUTION Equations of Statics. Denoting by TAB the torque in shaft AB, we pass a section through shaft AB and, for the free body shown, we write A TAB 16 kN ? m2 2 TAB 5 0 ©M x 5 0: x TAB 5 6 kN ? m We now pass a section through shaft BC and, for the free body shown, we have TA  6 kN · m ©M x 5 0: TB  14 kN · m 16 kN ? m2 1 114 kN ? m2 2 TBC 5 0 TBC 5 20 kN ? m a. Shaft BC. For this hollow shaft we have A J5 TBC B x p 4 p 1c2 2 c41 2 5 3 10.0602 4 2 10.0452 4 4 5 13.92 3 1026 m 4 2 2 Maximum Shearing Stress. tmax 5 t2 5 2 1 c1  45 mm c2  60 mm 6 kN · m B 152 120 kN ? m2 10.060 m2 TBC c2 5 J 13.92 3 1026 m4 tmax 5 86.2 MPa b Minimum Shearing Stress. We write that the stresses are proportional to the distance from the axis of the shaft. c1 tmin 5 tmax c2 tmin 45 mm 5 86.2 MPa 60 mm tmin 5 64.7 MPa b b. Shafts AB and CD. We note that in both of these shafts the magnitude of the torque is T 5 6 kN ? m and tall 5 65 MPa. Denoting by c the radius of the shafts, we write 6 kN · m A On the outer surface, we have 16 kN ? m2c p 4 c 2 c3 5 58.8 3 1026 m 3 c 5 38.9 3 1023 m d 5 2c 5 2138.9 mm2 d 5 77.8 mm b t5 Tc J 65 MPa 5 bee80288_ch03_140-219.indd Page 153 9/21/10 9:15:26 PM user-f499 T' 4 in. 6 in. T /Users/user-f499/Desktop/Temp Work/Don't Delete Job/MHDQ251:Beer:201/ch03 SAMPLE PROBLEM 3.2 The preliminary design of a large shaft connecting a motor to a generator calls for the use of a hollow shaft with inner and outer diameters of 4 in. and 6 in., respectively. Knowing that the allowable shearing stress is 12 ksi, determine the maximum torque that can be transmitted (a) by the shaft as designed, (b) by a solid shaft of the same weight, (c) by a hollow shaft of the same weight and of 8-in. outer diameter. 8 ft SOLUTION a. Hollow Shaft as Designed. For the hollow shaft we have c2 ⫽ 3 in. J5 c1 ⫽ 2 in. p 4 p 1c2 2 c41 2 5 3 13 in.2 4 2 12 in.2 4 4 5 102.1 in4 2 2 Using Eq. (3.9), we write tmax 5 T Tc2 J 12 ksi 5 T 13 in.2 102.1 in4 T 5 408 kip ? in. b b. Solid Shaft of Equal Weight. For the shaft as designed and this solid shaft to have the same weight and length, their cross-sectional areas must be equal. A 1a2 5 A 1b2 p 3 13 in.2 2 2 12 in.2 2 4 5 pc23 c3 5 2.24 in. Since tall 5 12 ksi, we write c3 tmax 5 T Tc3 J 12 ksi 5 T 12.24 in.2 p 12.24 in.2 4 2 T 5 211 kip ? in. b c. Hollow Shaft of 8-in. Diameter. For equal weight, the crosssectional areas again must be equal. We determine the inside diameter of the shaft by writing A 1a2 5 A 1c2 p 3 13 in.2 2 12 in.2 2 4 5 p 3 14 in.2 2 2 c25 4 2 c4 ⫽ 4 in. c5 5 3.317 in. For c5 5 3.317 in. and c4 5 4 in., c5 T J5 p 3 14 in.2 4 2 13.317 in.2 4 4 5 212 in4 2 With tall 5 12 ksi and c4 5 4 in., tmax 5 Tc4 J 12 ksi 5 T14 in.2 212 in4 T 5 636 kip ? in. b 153 bee80288_ch03_140-219.indd Page 154 9/21/10 3:04:51 PM user-f499 /Users/user-f499/Desktop/Temp Work/Don't Delete Job/MHDQ251:Beer:201/ch03 PROBLEMS 3.1 (a) Determine the maximum shearing stress caused by a 4.6-kN ? m torque T in the 76-mm-diameter solid aluminum shaft shown. (b) Solve part a, assuming that the solid shaft has been replaced by a hollow shaft of the same outer diameter and of 24-mm inner diameter. 1.2 m 76 mm T 3.2 (a) Determine the torque T that causes a maximum shearing stress of 45 MPa in the hollow cylindrical steel shaft shown. (b) Determine the maximum shearing stress caused by the same torque T in a solid cylindrical shaft of the same cross-sectional area. Fig. P3.1 30 mm 45 mm T 2.4 m Fig. P3.2 3.3 Knowing that d 5 1.2 in., determine the torque T that causes a maximum shearing stress of 7.5 ksi in the hollow shaft shown. 3.4 Knowing that the internal diameter of the hollow shaft shown is d 5 0.9 in., determine the maximum shearing stress caused by a torque of magnitude T 5 9 kip ? in. T d 1.6 in. 3.5 A torque T 5 3 kN ? m is applied to the solid bronze cylinder shown. Determine (a) the maximum shearing stress, (b) the shearing stress at point D, which lies on a 15-mm-radius circle drawn on the end of the cylinder, (c) the percent of the torque carried by the portion of the cylinder within the 15-mm radius. Fig. P3.3 and P3.4 30 mm T  3 kN · m 60 mm D 200 mm Fig. P3.5 3.6 (a) Determine the torque that can be applied to a solid shaft of 20-mm diameter without exceeding an allowable shearing stress of 80 MPa. (b) Solve part a, assuming that the solid shaft has been replaced by a hollow shaft of the same cross-sectional area and with an inner diameter equal to half of its outer diameter. 154 bee80288_ch03_140-219.indd Page 155 9/21/10 3:05:08 PM user-f499 /Users/user-f499/Desktop/Temp Work/Don't Delete Job/MHDQ251:Beer:201/ch03 Problems 3.7 The solid spindle AB has a diameter ds 5 1.5 in. and is made of a steel with an allowable shearing stress of 12 ksi, while sleeve CD is made of a brass with an allowable shearing stress of 7 ksi. Determine the largest torque T that can be applied at A. 3.8 The solid spindle AB is made of a steel with an allowable shearing stress of 12 ksi, and sleeve CD is made of a brass with an allowable shearing stress of 7 ksi. Determine (a) the largest torque T that can be applied at A if the allowable shearing stress is not to be exceeded in sleeve CD, (b) the corresponding required value of the diameter ds of spindle AB. 3.9 The torques shown are exerted on pulleys A and B. Knowing that both shafts are solid, determine the maximum shearing stress in (a) in shaft AB, (b) in shaft BC. TA  300 N · m A 30 mm B TB  400 N · m 46 mm C Fig. P3.9 3.10 In order to reduce the total mass of the assembly of Prob. 3.9, a new design is being considered in which the diameter of shaft BC will be smaller. Determine the smallest diameter of shaft BC for which the maximum value of the shearing stress in the assembly will not increase. 3.11 Knowing that each of the shafts AB, BC, and CD consists of a solid circular rod, determine (a) the shaft in which the maximum shearing stress occurs, (b) the magnitude of that stress. 60 N · m 144 N · m D 48 N · m dCD  21 mm C dBC  18 mm B A dAB  15 mm Fig. P3.11 and P3.12 3.12 Knowing that an 8-mm-diameter hole has been drilled through each of the shafts AB, BC, and CD, determine (a) the shaft in which the maximum shearing stress occurs, (b) the magnitude of that stress. C B 3 in. 8 in. 4 in. t D ds A Fig. P3.7 and P3.8 T 1 4 in. 155 bee80288_ch03_140-219.indd Page 156 9/21/10 3:05:18 PM user-f499 156 /Users/user-f499/Desktop/Temp Work/Don't Delete Job/MHDQ251:Beer:201/ch03 3.13 Under normal operating conditions, the electric motor exerts a 12-kip ? in. torque at E. Knowing that each shaft is solid, determine the maximum shearing stress in (a) shaft BC, (b) shaft CD, (c) shaft DE. Torsion E 5 kip · in. 4 kip · in. 3 kip · in. D 2.25 in. C 2 in. B 1.75 in. A 1.50 in. Fig. P3.13 T A Steel B Brass C Fig. P3.15 and P3.16 3.14 Solve Prob. 3.13, assuming that a 1-in.-diameter hole has been drilled into each shaft. 3.15 The allowable shearing stress is 15 ksi in the 1.5-in.-diameter steel rod AB and 8 ksi in the 1.8-in.-diameter brass rod BC. Neglecting the effect of stress concentrations, determine the largest torque that can be applied at A. 3.16 The allowable shearing stress is 15 ksi in the steel rod AB and 8 ksi in the brass rod BC. Knowing that a torque of magnitude T 5 10 kip ? in. is applied at A, determine the required diameter of (a) rod AB, (b) rod BC. 3.17 The allowable stress is 50 MPa in the brass rod AB and 25 MPa in the aluminum rod BC. Knowing that a torque of magnitude T 5 1250 N ? m is applied at A, determine the required diameter of (a) rod AB, (b) rod BC. Aluminum Brass C T B A Fig. P3.17 and P3.18 3.18 The solid rod BC has a diameter of 30 mm and is made of an aluminum for which the allowable shearing stress is 25 MPa. Rod AB is hollow and has an outer diameter of 25 mm; it is made of a brass for which the allowable shearing stress is 50 MPa. Determine (a) the largest inner diameter of rod AB for which the factor of safety is the same for each rod, (b) the largest torque that can be applied at A. bee80288_ch03_140-219.indd Page 157 9/21/10 3:05:21 PM user-f499 /Users/user-f499/Desktop/Temp Work/Don't Delete Job/MHDQ251:Beer:201/ch03 Problems 3.19 The solid rod AB has a diameter dAB 5 60 mm. The pipe CD has an outer diameter of 90 mm and a wall thickness of 6 mm. Knowing that both the rod and the pipe are made of steel for which the allowable shearing stress is 75 MPa, determine the largest torque T that can be applied at A. 3.20 The solid rod AB has a diameter dAB 5 60 mm and is made of a steel for which the allowable shearing stress is 85 MPa. The pipe CD, which has an outer diameter of 90 mm and a wall thickness of 6 mm, is made of an aluminum for which the allowable shearing stress is 54 MPa. Determine the largest torque T that can be applied at A. 3.21 A torque of magnitude T 5 1000 N ? m is applied at D as shown. Knowing that the diameter of shaft AB is 56 mm and that the diameter of shaft CD is 42 mm, determine the maximum shearing stress in (a) shaft AB, (b) shaft CD. C 40 mm T  1000 N · m A B D 100 mm Fig. P3.21 and P3.22 3.22 A torque of magnitude T 5 1000 N ? m is applied at D as shown. Knowing that the allowable shearing stress is 60 MPa in each shaft, determine the required diameter of (a) shaft AB, (b) shaft CD. 3.23 Under normal operating conditions a motor exerts a torque of magnitude TF 5 1200 lb ? in. at F. Knowing that rD 5 8 in., rG 5 3 in., and the allowable shearing stress is 10.5 ksi in each shaft, determine the required diameter of (a) shaft CDE, (b) shaft FGH. A F C TF D rG rD B G H TE E Fig. P3.23 and P3.24 3.24 Under normal operating conditions a motor exerts a torque of magnitude TF at F. The shafts are made of a steel for which the allowable shearing stress is 12 ksi and have diameters dCDE 5 0.900 in. and dFGH 5 0.800 in. Knowing that rD 5 6.5 in. and rG 5 4.5 in., determine the largest allowable value of TF. 90 mm D C dAB B A T Fig. P3.19 and P3.20 157 bee80288_ch03_140-219.indd Page 158 11/2/10 3:06:37 PM user-f499 158 3.25 The two solid shafts are connected by gears as shown and are made of a steel for which the allowable shearing stress is 8500 psi. Knowing that a torque of magnitude TC 5 5 kip ? in. is applied at C and that the assembly is in equilibrium, determine the required diameter of (a) shaft BC, (b) shaft EF. Torsion A 4 in. B 2.5 in. /Users/user-f499/Desktop/Temp Work/Don't Delete Job/MHDQ251:Beer:201/ch03 C D TC E F G H Fig. P3.25 and P3.26 TF 3.26 The two solid shafts are connected by gears as shown and are made of a steel for which the allowable shearing stress is 7000 psi. Knowing the diameters of the two shafts are, respectively, dBC 5 1.6 in. and dEF 5 1.25 in., determine the largest torque TC that can be applied at C. 3.27 A torque of magnitude T 5 100 N ? m is applied to shaft AB of the gear train shown. Knowing that the diameters of the three solid shafts are, respectively, dAB 5 21 mm, dCD 5 30 mm, and dEF 5 40 mm, determine the maximum shearing stress in (a) shaft AB, (b) shaft CD, (c) shaft EF. 75 mm 30 mm D E A T F C B 60 mm 25 mm Fig. P3.27 and P3.28 3.28 A torque of magnitude T 5 120 N ? m is applied to shaft AB of the gear train shown. Knowing that the allowable shearing stress is 75 MPa in each of the three solid shafts, determine the required diameter of (a) shaft AB, (b) shaft CD, (c) shaft EF. c2 c1 Fig. P3.29 ␶max ␶0 O c1 (a) Fig. P3.30 c2 O rm (b) 3.29 (a) For a given allowable shearing stress, determine the ratio Tyw of the maximum allowable torque T and the weight per unit length w for the hollow shaft shown. (b) Denoting by (Tyw)0 the value of this ratio for a solid shaft of the same radius c2, express the ratio Tyw for the hollow shaft in terms of (Tyw)0 and c1yc2. 3.30 While the exact distribution of the shearing stresses in a hollow cylindrical shaft is as shown in Fig. P3.30a, an approximate value can be obtained for tmax by assuming that the stresses are uniformly distributed over the area A of the cross section, as shown in Fig. P3.30b, and then further assuming that all of the elementary shearing forces act at a distance from O equal to the mean radius 12(c1 1 c2) of the cross section. This approximate value t0 5 TyArm, where T is the applied torque. Determine the ratio tmax yt0 of the true value of the maximum shearing stress and its approximate value t0 for values of c1 yc2 respectively equal to 1.00, 0.95, 0.75, 0.50 and 0. bee80288_ch03_140-219.indd Page 159 9/21/10 3:05:30 PM user-f499 3.5 /Users/user-f499/Desktop/Temp Work/Don't Delete Job/MHDQ251:Beer:201/ch03 3.5 Angle of Twist in the Elastic Range ANGLE OF TWIST IN THE ELASTIC RANGE In this section, a relation will be derived between the angle of twist f of a circular shaft and the torque T exerted on the shaft. The entire shaft will be assumed to remain elastic. Considering first the case of a shaft of length L and of uniform cross section of radius c subjected to a torque T at its free end (Fig. 3.20), we recall from Sec. 3.3 that the angle of twist f and the maximum shearing strain gmax are related as follows: cf gmax 5 L Equating the right-hand members of Eqs. (3.3) and (3.15), and solving for f, we write TL JG (3.16) where f is expressed in radians. The relation obtained shows that, within the elastic range, the angle of twist f is proportional to the torque T applied to the shaft. This is in accordance with the experimental evidence cited at the beginning of Sec. 3.3. Equation (3.16) provides us with a convenient method for determining the modulus of rigidity of a given material. A specimen of the material, in the form of a cylindrical rod of known diameter and length, is placed in a torsion testing machine (Photo 3.3). Torques of increasing magnitude T are applied to the specimen, and the corresponding values of the angle of twist f in a length L of the specimen are recorded. As long as the yield stress of the material is not exceeded, the points obtained by plotting f against T will fall on a straight line. The slope of this line represents the quantity JGyL, from which the modulus of rigidity G may be computed. Photo 3.3 Torsion testing machine. c  (3.3) But, in the elastic range, the yield stress is not exceeded anywhere in the shaft, Hooke’s law applies, and we have gmax 5 tmaxyG or, recalling Eq. (3.9), tmax Tc 5 gmax 5 (3.15) G JG f5 max L Fig. 3.20 Angle of twist f. T 159 bee80288_ch03_140-219.indd Page 160 11/2/10 12:48:17 AM user-f499 EXAMPLE 3.02 /Users/user-f499/Desktop/Temp Work/Don't Delete Job/MHDQ251:Beer:201/ch03 What torque should be applied to the end of the shaft of Example 3.01 to produce a twist of 28? Use the value G 5 77 GPa for the modulus of rigidity of steel. Solving Eq. (3.16) for T, we write T5 JG L f Substituting the given values G 5 77 3 109 Pa L 5 1.5 m 2p rad f 5 2°a b 5 34.9 3 1023 rad 360° and recalling from Example 3.01 that, for the given cross section, J 5 1.021 3 1026 m4 we have T5 EXAMPLE 3.03 JG L f5 11.021 3 1026 m4 2 177 3 109 Pa2 134.9 3 1023 rad2 1.5 m T 5 1.829 3 103 N ? m 5 1.829 kN ? m What angle of twist will create a shearing stress of 70 MPa on the inner surface of the hollow steel shaft of Examples 3.01 and 3.02? The method of attack for solving this problem that first comes to mind is to use Eq. (3.10) to find the torque T corresponding to the given value of t, and Eq. (3.16) to determine the angle of twist f corresponding to the value of T just found. A more direct solution, however, may be used. From Hooke’s law, we first compute the shearing strain on the inner surface of the shaft: g min 5 tmin 70 3 106 Pa 5 909 3 1026 5 G 77 3 109 Pa Recalling Eq. (3.2), which was obtained by expressing the length of arc AA9 in Fig. 3.13c in terms of both g and f, we have f5 Lg min 1500 mm 5 1909 3 1026 2 5 68.2 3 1023 rad c1 20 mm To obtain the angle of twist in degrees, we write f 5 168.2 3 1023 rad2a 360° b 5 3.91° 2p rad Formula (3.16) for the angle of twist can be used only if the shaft is homogeneous (constant G), has a uniform cross section, and is loaded only at its ends. If the shaft is subjected to torques at locations other than its ends, or if it consists of several portions with various cross sections and possibly of different materials, we must divide it into component parts that satisfy individually the required 160 bee80288_ch03_140-219.indd Page 161 9/21/10 3:05:36 PM user-f499 /Users/user-f499/Desktop/Temp Work/Don't Delete Job/MHDQ251:Beer:201/ch03 3.5 Angle of Twist in the Elastic Range TD B TC TB E A D C Fig. 3.21 torques. TA Multiple sections and multiple conditions for the application of formula (3.16). In the case of the shaft AB shown in Fig. 3.21, for example, four different parts should be considered: AC, CD, DE, and EB. The total angle of twist of the shaft, i.e., the angle through which end A rotates with respect to end B, is obtained by adding algebraically the angles of twist of each component part. Denoting, respectively, by Ti, Li, Ji, and Gi the internal torque, length, cross-sectional polar moment of inertia, and modulus of rigidity corresponding to part i, the total angle of twist of the shaft is expressed as Ti Li f5 a Ji G i i (3.17) The internal torque Ti in any given part of the shaft is obtained by passing a section through that part and drawing the free-body diagram of the portion of shaft located on one side of the section. This procedure, which has already been explained in Sec. 3.4 and illustrated in Fig. 3.16, is applied in Sample Prob. 3.3. In the case of a shaft with a variable circular cross section, as shown in Fig. 3.22, formula (3.16) may be applied to a disk of thickness dx. The angle by which one face of the disk rotates with respect to the other is thus df 5 x T dx JG dx B T T' A L Fig. 3.22 Shaft with variable cross section. 161 bee80288_ch03_140-219.indd Page 162 9/21/10 3:05:40 PM user-f499 162 Torsion /Users/user-f499/Desktop/Temp Work/Don't Delete Job/MHDQ251:Beer:201/ch03 where J is a function of x, which may be determined. Integrating in x from 0 to L, we obtain the total angle of twist of the shaft: f5 # L 0 T dx JG (3.18) The shaft shown in Fig. 3.20, which was used to derive formula (3.16), and the shaft of Fig. 3.15, which was discussed in Examples 3.02 and 3.03, both had one end attached to a fixed support. In each case, therefore, the angle of twist f of the shaft was equal to the angle of rotation of its free end. When both ends of a shaft rotate, however, the angle of twist of the shaft is equal to the angle through which one end of the shaft rotates with respect to the other. Consider, for instance, the assembly shown in Fig. 3.23a, consisting of two elastic shafts AD and BE, each of length L, radius c, and modulus of rigidity G, which are attached to gears meshed at C. If a torque T is applied at E (Fig. 3.23b), both shafts will be twisted. Since the end D of shaft AD is fixed, the angle of twist of AD is measured by the angle of rotation fA of end A. On the other hand, since both ends of shaft BE rotate, the angle of twist of BE is equal to the difference between the angles of rotation fB and fE, i.e., the angle of twist is equal to the angle through which end E rotates with respect to end B. Denoting this relative angle of rotation by fEyB, we write fEyB 5 fE 2 fB 5 TL JG Fixed support Fixed end E D T E D E L L A rA C B rB A A C C' B (a) Fig. 3.23 Gear assembly. (b) B C'' bee80288_ch03_140-219.indd Page 163 9/21/10 9:15:47 PM user-f499 /Users/user-f499/Desktop/Temp Work/Don't Delete Job/MHDQ251:Beer:201/ch03 For the assembly of Fig. 3.23, knowing that rA 5 2rB, determine the angle of rotation of end E of shaft BE when the torque T is applied at E. We first determine the torque TAD exerted on shaft AD. Observing that equal and opposite forces F and F9 are applied on the two gears at C (Fig. 3.24), and recalling that rA 5 2rB, we conclude that the torque exerted on shaft AD is twice as large as the torque exerted on shaft BE; thus, TAD 5 2T. Since the end D of shaft AD is fixed, the angle of rotation fA of gear A is equal to the angle of twist of the shaft and is obtained by writing fA 5 T AD L 2TL 5 JG JG EXAMPLE 3.04 F rA C rB B A F' Fig. 3.24 Observing that the arcs CC9 and CC0 in Fig. 3.26b must be equal, we write rAfA 5 rBfB and obtain f B 5 1rAyrB 2f A 5 2f A We have, therefore, f B 5 2f A 5 4TL JG Considering now shaft BE, we recall that the angle of twist of the shaft is equal to the angle fEyB through which end E rotates with respect to end B. We have f EyB 5 T BEL TL 5 JG JG The angle of rotation of end E is obtained by writing f E 5 f B 1 f EyB 5 3.6 TL 5TL 4TL 1 5 JG JG JG STATICALLY INDETERMINATE SHAFTS You saw in Sec. 3.4 that, in order to determine the stresses in a shaft, it was necessary to first calculate the internal torques in the various parts of the shaft. These torques were obtained from statics by drawing the free-body diagram of the portion of shaft located on one side of a given section and writing that the sum of the torques exerted on that portion was zero. There are situations, however, where the internal torques cannot be determined from statics alone. In fact, in such cases the external torques themselves, i.e., the torques exerted on the shaft by the supports and connections, cannot be determined from the free-body diagram of the entire shaft. The equilibrium equations must be complemented by relations involving the deformations of the shaft and obtained by considering the geometry of the problem. Because statics is not sufficient to determine the external and internal torques, the shafts are said to be statically indeterminate. The following example, as well as Sample Prob. 3.5, will show how to analyze statically indeterminate shafts. 163 bee80288_ch03_140-219.indd Page 164 9/21/10 3:05:46 PM user-f499 EXAMPLE 3.05 /Users/user-f499/Desktop/Temp Work/Don't Delete Job/MHDQ251:Beer:201/ch03 A circular shaft AB consists of a 10-in.-long, 78 -in.-diameter steel cylinder, in which a 5-in.-long, 58 -in.-diameter cavity has been drilled from end B. The shaft is attached to fixed supports at both ends, and a 90 lb ? ft torque is applied at its midsection (Fig. 3.25). Determine the torque exerted on the shaft by each of the supports. 5 in. 5 in. A 90 lb · ft B Fig. 3.25 Drawing the free-body diagram of the shaft and denoting by TA and TB the torques exerted by the supports (Fig. 3.26a), we obtain the equilibrium equation TA 1 TB 5 90 lb ? ft Since this equation is not sufficient to determine the two unknown torques TA and TB, the shaft is statically indeterminate. However, TA and TB can be determined if we observe that the total angle of twist of shaft AB must be zero, since both of its ends are restrained. Denoting by f1 and f2, respectively, the angles of twist of portions AC and CB, we write f 5 f1 1 f2 5 0 TA C A TB 90 lb · ft (a) From the free-body diagram of a small portion of shaft including end A (Fig. 3.26b), we note that the internal torque T1 in AC is equal to TA; from the free-body diagram of a small portion of shaft including end B (Fig. 3.26c), we note that the internal torque T2 in CB is equal to TB. Recalling Eq. (3.16) and observing that portions AC and CB of the shaft are twisted in opposite senses, we write B f 5 f1 1 f2 5 TA Solving for TB , we have A TB 5 T1 (b) TB T2 (c) TAL1 TBL2 2 50 J 1G J 2G L 1 J2 L 2 J1 TA Substituting the numerical data gives B L1 5 L2 5 5 in. J1 5 12 p 1 167 in.2 4 5 57.6 3 1023 in4 J2 5 12 p 3 1 167 in.2 4 2 1 165 in.2 4 4 5 42.6 3 1023 in4 Fig. 3.26 we obtain TB 5 0.740 TA Substituting this expression into the original equilibrium equation, we write 1.740 TA 5 90 lb ? ft TA 5 51.7 lb ? ft 164 TB 5 38.3 lb ? ft bee80288_ch03_140-219.indd Page 165 11/2/10 1:29:33 AM user-f499 /Users/user-f499/Desktop/Temp Work/Don't Delete Job/MHDQ251:Beer:201/ch03 SAMPLE PROBLEM 3.3 60 mm 44 mm The horizontal shaft AD is attached to a fixed base at D and is subjected to the torques shown. A 44-mm-diameter hole has been drilled into portion CD of the shaft. Knowing that the entire shaft is made of steel for which 2000 N · m G 5 77 GPa, determine the angle of twist at end A. D 250 N · m C 0.6 m B 30 mm A 0.2 m 0.4 m TAB SOLUTION 250 N · m Since the shaft consists of three portions AB, BC, and CD, each of uniform cross section and each with a constant internal torque, Eq. (3.17) may be used. Statics. Passing a section through the shaft between A and B and using the free body shown, we find x A ©M x 5 0: TBC 2000 N · m 1250 N ? m2 2 T AB 5 0 T AB 5 250 N ? m Passing now a section between B and C, we have ©M x 5 0: 1250 N ? m2 1 12000 N ? m2 2 T BC 5 0 250 N · m T BC 5 2250 N ? m Since no torque is applied at C, T CD 5 T BC 5 2250 N ? m B A 15 mm BC CD p 4 p c 5 10.015 m2 4 5 0.0795 3 1026 m 4 2 2 p 4 p JBC 5 c 5 10.030 m2 4 5 1.272 3 1026 m 4 2 2 p 4 p JCD 5 1c2 2 c41 2 5 3 10.030 m2 4 2 10.022 m2 4 4 5 0.904 3 1026 m 4 2 2 JAB 5 30 mm 30 mm AB Polar Moments of Inertia x 22 mm Angle of Twist. Using Eq. (3.17) and recalling that G 5 77 GPa for the entire shaft, we have ␾A TiLi TBCLBC TCDLCD 1 TABLAB 5 a 1 1 b fA 5 a JiG G JAB JBC JCD i 122502 10.22 122502 10.62 1250 N ? m2 10.4 m2 1 1 1 d c 77 GPa 0.0795 3 1026 m4 1.272 3 1026 0.904 3 1026 5 0.01634 1 0.00459 1 0.01939 5 0.0403 rad fA 5 D C B fA 5 10.0403 rad2 A 360° 2p rad f A 5 2.31° b 165 bee80288_ch03_140-219.indd Page 166 9/21/10 9:15:56 PM user-f499 /Users/user-f499/Desktop/Temp Work/Don't Delete Job/MHDQ251:Beer:201/ch03 SAMPLE PROBLEM 3.4 36 in. D 1 in. A 0.75 in. C 2.45 in. T0 SOLUTION 24 in. B 0.875 in. TCD TAB  T0 F C B Statics. Denoting by F the magnitude of the tangential force between gear teeth, we have F10.875 in.2 2 T 0 5 0 Gear B. oM B 5 0: (1) T 5 2.8T 0 Gear C. oM C 5 0: F12.45 in.2 2 T CD 5 0 CD Kinematics. we write F rC  2.45 in. Two solid steel shafts are connected by the gears shown. Knowing that for each shaft G 5 11.2 3 106 psi and that the allowable shearing stress is 8 ksi, determine (a) the largest torque T0 that may be applied to end A of shaft AB, (b) the corresponding angle through which end A of shaft AB rotates. rB f B 5 rC f C rB  0.875 in. C fB 5 fC rC 2.45 in. 5 2.8f C 5 fC rB 0.875 in. (2) a. Torque T0 B Shaft AB. With TAB 5 T0 and c 5 0.375 in., together with a maximum permissible shearing stress of 8000 psi, we write B C rB  0.875 in. rC  2.45 in. TAB  T0 A c  0.375 in. t5 B 24 in. TAB c J 8000 psi 5 T0 10.375 1 2 p10.375 TCD c J 8000 psi 5 1 2 p10.5 T0 5 663 lb ? in. in.2 4 T0 5 561 lb ? in. ◀ b. Angle of Rotation at End A. for each shaft. ◀ We choose the smaller value obtained T 0 5 561 lb ? in. TCD c  0.5 in. in.2 4 2.8T0 10.5 in.2 Maximum Permissible Torque. for T0 D in.2 Shaft CD. From (1) we have TCD 5 2.8T0. With c 5 0.5 in. and tall 5 8000 psi, we write t5 TAB  T0 Noting that the peripheral motions of the gears are equal, ◀ We first compute the angle of twist For TAB 5 T0 5 561 lb ? in., we have 1561 lb ? in.2 124 in.2 T ABL 5 51 5 0.0387 rad 5 2.22° 4 6 JG 2 p 10.375 in.2 111.2 3 10 psi2 Shaft AB. C 36 in. f AyB TCD TCD 5 2.8T0 5 2.8(561 lb ? in.) 2.81561 lb ? in.2 136 in.2 TCDL 51 fCyD 5 5 0.0514 rad 5 2.95° JG p10.5 in.2 4 111.2 3 106 psi2 2 Since end D of shaft CD is fixed, we have fC 5 fC@D 5 2.958. Using (2), we find the angle of rotation of gear B to be Shaft CD.  C  2.95 D  B  8.26 A A  10.48 C B 166 fB 5 2.8fC 5 2.812.95°2 5 8.26° For end A of shaft AB, we have fA 5 fB 1 fAyB 5 8.26° 1 2.22° f A 5 10.48° ◀ bee80288_ch03_140-219.indd Page 167 9/21/10 3:06:10 PM user-f499 8 mm /Users/user-f499/Desktop/Temp Work/Don't Delete Job/MHDQ251:Beer:201/ch03 SAMPLE PROBLEM 3.5 50 mm 76 mm 500 mm T1 A steel shaft and an aluminum tube are connected to a fixed support and to a rigid disk as shown in the cross section. Knowing that the initial stresses are zero, determine the maximum torque T0 that can be applied to the disk if the allowable stresses are 120 MPa in the steel shaft and 70 MPa in the aluminum tube. Use G 5 77 GPa for steel and G 5 27 GPa for aluminum. SOLUTION Statics. Free Body of Disk. Denoting by T1 the torque exerted by the tube on the disk and by T2 the torque exerted by the shaft, we find T0 T2 Deformations. rigid disk, we have Since both the tube and the shaft are connected to the f 1 5 f 2: T 1 10.5 m2 0.5 m T1 T 2L 2 T 1L 1 5 J1G 1 J2G 2 T 2 10.5 m2 5 12.003 3 1026 m 4 2 127 GPa2 10.614 3 1026 m 4 2 177 GPa2 T 2 5 0.874T 1 (2) Shearing Stresses. We assume that the requirement talum # 70 MPa is critical. For the aluminum tube, we have 38 mm 30 mm T1 5 1 (1) T0 5 T1 1 T2 Aluminum G1  27 GPa  J1  2 (38 mm)4  (30 mm)4  2.003 106m4 talum J1 c1 170 MPa2 12.003 3 1026 m 4 2 5 0.038 m 5 3690 N ? m Using Eq. (2), we compute the corresponding value T2 and then find the maximum shearing stress in the steel shaft. T 2 5 0.874T 1 5 0.874 136902 5 3225 N ? m 13225 N ? m2 10.025 m2 T 2c2 5 5 131.3 MPa tsteel 5 J2 0.614 3 1026 m 4 We note that the allowable steel stress of 120 MPa is exceeded; our assumption was wrong. Thus the maximum torque T0 will be obtained by making tsteel 5 120 MPa. We first determine the torque T2. T2 5 0.5 m T2 25 mm 2 tsteel J2 c2 5 1120 MPa2 10.614 3 1026 m 4 2 0.025 m 5 2950 N ? m From Eq. (2), we have 2950 N ? m 5 0.874T 1 T 1 5 3375 N ? m Steel Using Eq. (1), we obtain the maximum permissible torque G1  77 GPa  4 J1  2 (25 mm) T 0 5 T 1 1 T 2 5 3375 N ? m 1 2950 N ? m  0.614 106m4 T 0 5 6.325 kN ? m ◀ 167 bee80288_ch03_140-219.indd Page 168 9/21/10 3:06:20 PM user-f499 /Users/user-f499/Desktop/Temp Work/Don't Delete Job/MHDQ251:Beer:201/ch03 PROBLEMS 3.31 (a) For the solid steel shaft shown (G 5 77 GPa), determine the angle of twist at A. (b) Solve part a, assuming that the steel shaft is hollow with a 30-mm-outer diameter and a 20-mm-inner diameter. 1.8 m 30 mm 250 N · m A Fig. P3.31 1.25 m 3.33 Determine the largest allowable diameter of a 10-ft-long steel rod (G 5 11.2 3 106 psi) if the rod is to be twisted through 308 without exceeding a shearing stress of 12 ksi. T 18 mm 12 mm Fig. P3.32 3.32 For the aluminum shaft shown (G 5 27 GPa), determine (a) the torque T that causes an angle of twist of 48, (b) the angle of twist caused by the same torque T in a solid cylindrical shaft of the same length and cross-sectional area. 3.34 While an oil well is being drilled at a depth of 6000 ft, it is observed that the top of the 8-in.-diameter steel drill pipe rotates though two complete revolutions before the drilling bit starts to rotate. Using G 5 11.2 3 106 psi, determine the maximum shearing stress in the pipe caused by torsion. 3.35 The electric motor exerts a 500 N ? m-torque on the aluminum shaft ABCD when it is rotating at a constant speed. Knowing that G 5 27 GPa and that the torques exerted on pulleys B and C are as shown, determine the angle of twist between (a) B and C, (b) B and D. 300 N · m D 200 N · m C 48 mm B 0.9 m 44 mm A 1.2 m 40 mm 1m Fig. P3.35 168 bee80288_ch03_140-219.indd Page 169 9/21/10 9:16:47 PM user-f499 /Users/user-f499/Desktop/Temp Work/Don't Delete Job/MHDQ251:Beer:201/ch03 Problems 3.36 The torques shown are exerted on pulleys B, C, and D. Knowing that the entire shaft is made of aluminum (G 5 27 GPa), determine the angle of twist between (a) C and B, (b) D and B. 30 mm 30 mm 400 N · m A 900 N · m 36 mm 36 mm A 500 N · m B 0.6 m 200 mm C 0.8 m Brass E D B 1m 0.5 m Aluminum 300 mm Fig. P3.36 3.37 The aluminum rod BC (G 5 26 GPa) is bonded to the brass rod AB (G 5 39 GPa). Knowing that each rod is solid and has a diameter of 12 mm, determine the angle of twist (a) at B, (b) at C. C 100 N · m Fig. P3.37 3.38 The aluminum rod AB (G 5 27 GPa) is bonded to the brass rod BD (G 5 39 GPa). Knowing that portion CD of the brass rod is hollow and has an inner diameter of 40 mm, determine the angle of twist at A. 60 mm TB ⫽ 1600 N · m D 36 mm C TA ⫽ 800 N · m B 250 mm 375 mm A 400 mm Fig. P3.38 3.39 The solid spindle AB has a diameter ds 5 1.5 in. and is made of a steel with G 5 11.2 3 106 psi and tall 5 12 ksi, while sleeve CD is made of a brass with G 5 5.6 3 106 psi and tall 5 7 ksi. Determine the largest angle through which end A can be rotated. 3.40 The solid spindle AB has a diameter ds 5 1.75 in. and is made of a steel with G 5 11.2 3 106 psi and tall 5 12 ksi, while sleeve CD is made of a brass with G 5 5.6 3 106 psi and tall 5 7 ksi. Determine (a) the largest torque T that can be applied at A if the given allowable stresses are not to be exceeded and if the angle of twist of sleeve CD is not to exceed 0.3758, (b) the corresponding angle through which end A rotates. C B 3 in. 8 in. 4 in. t⫽ D ds A T Fig. P3.39 and P3.40 1 4 in. 169 bee80288_ch03_140-219.indd Page 170 9/21/10 3:06:34 PM user-f499 170 Torsion /Users/user-f499/Desktop/Temp Work/Don't Delete Job/MHDQ251:Beer:201/ch03 3.41 Two shafts, each of 78 -in. diameter, are connected by the gears shown. Knowing that G 5 11.2 3 106 psi and that the shaft at F is fixed, determine the angle through which end A rotates when a 1.2 kip ? in. torque is applied at A. C 4.5 in. F B 6 in. E T 12 in. A D 8 in. 6 in. Fig. P3.41 3.42 Two solid shafts are connected by gears as shown. Knowing that G 5 77.2 GPa for each shaft, determine the angle through which end A rotates when TA 5 1200 N ? m. 240 mm C 60 mm D 80 mm 42 mm B 1.2 m A TA Fig. P3.42 1.6 m bee80288_ch03_140-219.indd Page 171 9/21/10 3:06:38 PM user-f499 /Users/user-f499/Desktop/Temp Work/Don't Delete Job/MHDQ251:Beer:201/ch03 3.43 A coder F, used to record in digital form the rotation of shaft A, is connected to the shaft by means of the gear train shown, which consists of four gears and three solid steel shafts each of diameter d. Two of the gears have a radius r and the other two a radius nr. If the rotation of the coder F is prevented, determine in terms of T, l, G, J, and n the angle through which end A rotates. F nr r D l nr l r B E C TA l A Fig. P3.43 3.44 For the gear train described in Prob. 3.43, determine the angle through which end A rotates when T 5 5 lb ? in., l 5 2.4 in., d 5 1 6 16 in., G 5 11.2 3 10 psi, and n 5 2. 3.45 The design of the gear-and-shaft system shown requires that steel shafts of the same diameter be used for both AB and CD. It is further required that tmax # 60 MPa and that the angle fD through which end D of shaft CD rotates not exceed 1.58. Knowing that G 5 77 GPa, determine the required diameter of the shafts. C 40 mm T  1000 N · m A B 100 mm 400 mm 600 mm Fig. P3.45 D Problems 171 bee80288_ch03_140-219.indd Page 172 9/21/10 3:06:43 PM user-f499 172 Torsion /Users/user-f499/Desktop/Temp Work/Don't Delete Job/MHDQ251:Beer:201/ch03 3.46 The electric motor exerts a torque of 800 N ? m on the steel shaft ABCD when it is rotating at a constant speed. Design specifications require that the diameter of the shaft be uniform from A to D and that the angle of twist between A and D not exceed 1.58. Knowing that tmax # 60 MPa and G 5 77 GPa, determine the minimum diameter shaft that can be used. A 300 N · m 500 N · m B D 0.4 m C 0.6 m 0.3 m Fig. P3.46 3.47 The design specifications of a 2-m-long solid circular transmission shaft require that the angle of twist of the shaft not exceed 38 when a torque of 9 kN ? m is applied. Determine the required diameter of the shaft, knowing that the shaft is made of (a) a steel with an allowable shearing stress of 90 MPa and a modulus of rigidity of 77 GPa, (b) a bronze with an allowable shearing stress of 35 MPa and a modulus of rigidity of 42 GPa. 3.48 A hole is punched at A in a plastic sheet by applying a 600-N force P to end D of lever CD, which is rigidly attached to the solid cylindrical shaft BC. Design specifications require that the displacement of D should not exceed 15 mm from the time the punch first touches the plastic sheet to the time it actually penetrates it. Determine the required diameter of shaft BC if the shaft is made of a steel with G 5 77 GPa and tall 5 80 MPa. B 500 mm A 300 mm P D Fig. P3.48 C bee80288_ch03_140-219.indd Page 173 9/21/10 3:06:47 PM user-f499 /Users/user-f499/Desktop/Temp Work/Don't Delete Job/MHDQ251:Beer:201/ch03 3.49 The design specifications for the gear-and-shaft system shown require that the same diameter be used for both shafts and that the angle through which pulley A will rotate when subjected to a 2-kip ? in. torque TA while pulley D is held fixed will not exceed 7.58. Determine the required diameter of the shafts if both shafts are made of a steel with G 5 11.2 3 106 psi and tall 5 12 ksi. 6 in. 16 in. B 2 in. 8 in. TA C A 5 in. TD D Fig. P3.49 3.50 Solve Prob. 3.49, assuming that both shafts are made of a brass with G 5 5.6 3 106 psi and tall 5 8 ksi. 3.51 A torque of magnitude T 5 4 kN ? m is applied at end A of the composite shaft shown. Knowing that the modulus of rigidity is 77 GPa for the steel and 27 GPa for the aluminum, determine (a) the maximum shearing stress in the steel core, (b) the maximum shearing stress in the aluminum jacket, (c) the angle of twist at A. B 72 mm 54 mm A Steel core Aluminum jacket T 25 m Fig. P3.51 and P3.52 3.52 The composite shaft shown is to be twisted by applying a torque T at end A. Knowing that the modulus of rigidity is 77 GPa for the steel and 27 GPa for the aluminum, determine the largest angle through which end A can be rotated if the following allowable stresses are not to be exceeded: tsteel 5 60 MPa and taluminum 5 45 MPa. Problems 173 bee80288_ch03_140-219.indd Page 174 9/21/10 3:06:52 PM user-f499 174 Torsion /Users/user-f499/Desktop/Temp Work/Don't Delete Job/MHDQ251:Beer:201/ch03 3.53 The solid cylinders AB and BC are bonded together at B and are attached to fixed supports at A and C. Knowing that the modulus of rigidity is 3.7 3 106 psi for aluminum and 5.6 3 106 psi for brass, determine the maximum shearing stress (a) in cylinder AB, (b) in cylinder BC. A Aluminum 12 in. 1.5 in. B T  12.5 kip · in. Brass 2.0 in. 18 in. C Fig. P3.53 3.54 Solve Prob. 3.53, assuming that cylinder AB is made of steel, for which G 5 11.2 3 106 psi. 3.55 and 3.56 Two solid steel shafts are fitted with flanges that are then connected by bolts as shown. The bolts are slightly undersized and permit a 1.58 rotation of one flange with respect to the other before the flanges begin to rotate as a single unit. Knowing that G 5 11.2 3 106 psi, determine the maximum shearing stress in each shaft when a torque of T of magnitude 420 kip ? ft is applied to the flange indicated. 3.55 The torque T is applied to flange B. 3.56 The torque T is applied to flange C. 1.5 in. D 1.25 in. T  350 lb · ft C B A 2 ft Fig. P3.55 and P3.56 3 ft bee80288_ch03_140-219.indd Page 175 9/21/10 3:06:57 PM user-f499 /Users/user-f499/Desktop/Temp Work/Don't Delete Job/MHDQ251:Beer:201/ch03 Problems 3.57 Ends A and D of the two solid steel shafts AB and CD are fixed, while ends B and C are connected to gears as shown. Knowing that a 4-kN ? m torque T is applied to gear B, determine the maximum shearing stress (a) in shaft AB, (b) in shaft CD. 60 mm C 40 mm A 45 mm B T 300 mm 100 mm D 500 mm Fig. P3.57 and P3.58 3.58 Ends A and D of the two solid steel shafts AB and CD are fixed, while ends B and C are connected to gears as shown. Knowing that the allowable shearing stress is 50 MPa in each shaft, determine the largest torque T that can be applied to gear B. 3.59 The steel jacket CD has been attached to the 40-mm-diameter steel shaft AE by means of rigid flanges welded to the jacket and to the rod. The outer diameter of the jacket is 80 mm and its wall thickness is 4 mm. If 500 N ? m-torques are applied as shown, determine the maximum shearing stress in the jacket. T' D E C B A T Fig. P3.59 3.60 A solid shaft and a hollow shaft are made of the same material and are of the same weight and length. Denoting by n the ratio c1yc2, show that the ratio TsyTh of the torque Ts in the solid shaft to the torque Th in the hollow shaft is (a) 211 2 n2 2y11 1 n2 2 if the maximum shearing stress is the same in each shaft, (b) (1 2 n2)y (1 1 n2) if the angle of twist is the same for each shaft. 3.61 A torque T is applied as shown to a solid tapered shaft AB. Show by integration that the angle of twist at A is f5 7TL 12pGc4 T A L B Fig. P3.61 22c c 175 bee80288_ch03_140-219.indd Page 176 9/21/10 3:07:02 PM user-f499 176 Torsion /Users/user-f499/Desktop/Temp Work/Don't Delete Job/MHDQ251:Beer:201/ch03 3.62 The mass moment of inertia of a gear is to be determined experimentally by using a torsional pendulum consisting of a 6-ft steel wire. Knowing that G 5 11.2 3 106 psi, determine the diameter of the wire for which the torsional spring constant will be 4.27 lb ? ft/rad. Fig. P3.62 3.63 An annular plate of thickness t and modulus G is used to connect shaft AB of radius r1 to tube CD of radius r2. Knowing that a torque T is applied to end A of shaft AB and that end D of tube CD is fixed, (a) determine the magnitude and location of the maximum shearing stress in the annular plate, (b) show that the angle through which end B of the shaft rotates with respect to end C of the tube is fBC 5 T 1 1 a 2 2b 4pGt r 21 r2 L2 D L1 C B A r2 T r1 t Fig. P3.63 3.7 DESIGN OF TRANSMISSION SHAFTS The principal specifications to be met in the design of a transmission shaft are the power to be transmitted and the speed of rotation of the shaft. The role of the designer is to select the material and the dimensions of the cross section of the shaft, so that the maximum bee80288_ch03_140-219.indd Page 177 11/2/10 12:48:17 AM user-f499 /Users/user-f499/Desktop/Temp Work/Don't Delete Job/MHDQ251:Beer:201/ch03 shearing stress allowable in the material will not be exceeded when the shaft is transmitting the required power at the specified speed. To determine the torque exerted on the shaft, we recall from elementary dynamics that the power P associated with the rotation of a rigid body subjected to a torque T is P 5 Tv (3.19) where v is the angular velocity of the body expressed in radians per second. But v 5 2pf, where f is the frequency of the rotation, i.e., the number of revolutions per second. The unit of frequency is thus 1 s21 and is called a hertz (Hz). Substituting for v into Eq. (3.19), we write P 5 2p f T (3.20) If SI units are used we verify that, with f expressed in Hz and T in N ? m, the power will be expressed in N ? m/s, that is, in watts (W). Solving Eq. (3.20) for T, we obtain the torque exerted on a shaft transmitting the power P at a frequency of rotation f, T5 P 2p f (3.21) where P, f, and T are expressed in the units indicated above. After having determined the torque T that will be applied to the shaft and having selected the material to be used, the designer will carry the values of T and of the maximum allowable stress into the elastic torsion formula (3.9). Solving for Jyc, we have J T 5 tmax c (3.22) and obtain in this way the minimum value allowable for the parameter Jyc. We check that, if SI units are used, T will be expressed in N ? m, tmax in Pa (or N/m2), and Jyc will be obtained in m3. In the case of a solid circular shaft, J 5 12pc4, and Jyc 5 12pc3; substituting this value for Jyc into Eq. (3.22) and solving for c yields the minimum allowable value for the radius of the shaft. In the case of a hollow circular shaft, the critical parameter is Jyc2, where c2 is the outer radius of the shaft; the value of this parameter may be computed from Eq. (3.11) of Sec. 3.4 to determine whether a given cross section will be acceptable. When U.S. customary units are used, the frequency is usually expressed in rpm and the power in horsepower (hp). It is then necessary, before applying formula (3.21), to convert the frequency into revolutions per second (i.e., hertzes) and the power into ft ? lb/s or in ? lb/s through the use of the following relations: 1 21 1 s 5 Hz 60 60 1 hp 5 550 ft ? lb/s 5 6600 in ? lb/s 1 rpm 5 If we express the power in in ? lb/s, formula (3.21) will yield the value of the torque T in lb ? in. Carrying this value of T into Eq. (3.22), and expressing tmax in psi, we obtain the value of the parameter Jyc in in3. 3.7 Design of Transmission Shafts 177 bee80288_ch03_140-219.indd Page 178 11/2/10 12:48:19 AM user-f499 EXAMPLE 3.06 /Users/user-f499/Desktop/Temp Work/Don't Delete Job/MHDQ251:Beer:201/ch03 What size of shaft should be used for the rotor of a 5-hp motor operating at 3600 rpm if the shearing stress is not to exceed 8500 psi in the shaft? We first express the power of the motor in in ? lb/s and its frequency in cycles per second (or hertzes). 6600 in ? lb/s b 5 33,000 in ? lb/s 1 hp 1 Hz 5 60 Hz 5 60 s21 f 5 13600 rpm2 60 rpm P 5 15 hp2a The torque exerted on the shaft is given by Eq. (3.21): T5 33,000 in ? lb/s P 5 5 87.54 lb ? in. 2p f 2p 160 s21 2 Substituting for T and tmax into Eq. (3.22), we write J T 87.54 lb ? in. 5 10.30 3 1023 in3 5 5 tmax c 8500 psi But Jyc 5 12pc3 for a solid shaft. We have, therefore, 1 3 2 pc 5 10.30 3 1023 in3 c 5 0.1872 in. d 5 2c 5 0.374 in. A 38-in. shaft should be used. EXAMPLE 3.07 A shaft consisting of a steel tube of 50-mm outer diameter is to transmit 100 kW of power while rotating at a frequency of 20 Hz. Determine the tube thickness that should be used if the shearing stress is not to exceed 60 MPa. The torque exerted on the shaft is given by Eq. (3.21): T5 P 100 3 103 W 5 5 795.8 N ? m 2p f 2p 120 Hz2 From Eq. (3.22) we conclude that the parameter Jyc2 must be at least equal to J T 795.8 N ? m (3.23) 5 5 5 13.26 3 1026 m3 tmax c2 60 3 106 N/m2 But, from Eq. (3.10) we have J p 4 p 5 1c2 2 c41 2 5 3 10.0252 4 2 c41 4 c2 2c2 0.050 (3.24) Equating the right-hand members of Eqs. (3.23) and (3.24), we obtain: 10.0252 4 2 c41 5 0.050 113.26 3 1026 2 p c41 5 390.6 3 1029 2 211.0 3 1029 5 179.6 3 1029 m4 c1 5 20.6 3 1023 m 5 20.6 mm The corresponding tube thickness is c2 2 c1 5 25 mm 2 20.6 mm 5 4.4 mm A tube thickness of 5 mm should be used. 178 bee80288_ch03_140-219.indd Page 179 9/21/10 3:07:09 PM user-f499 3.8 /Users/user-f499/Desktop/Temp Work/Don't Delete Job/MHDQ251:Beer:201/ch03 STRESS CONCENTRATIONS IN CIRCULAR SHAFTS The torsion formula tmax 5 TcyJ was derived in Sec. 3.4 for a circular shaft of uniform cross section. Moreover, we had assumed earlier in Sec. 3.3 that the shaft was loaded at its ends through rigid end plates solidly attached to it. In practice, however, the torques are usually applied to the shaft through flange couplings (Fig. 3.27a) or through gears connected to the shaft by keys fitted into keyways (Fig. 3.27b). In both cases one should expect the distribution of stresses, in and near the section where the torques are applied, to be different from that given by the torsion formula. High concentrations of stresses, for example, will occur in the neighborhood of the keyway shown in Fig. 3.27b. The determination of these localized stresses may be carried out by experimental stress analysis methods or, in some cases, through the use of the mathematical theory of elasticity. As we indicated in Sec. 3.4, the torsion formula can also be used for a shaft of variable circular cross section. In the case of a shaft with an abrupt change in the diameter of its cross section, however, stress concentrations will occur near the discontinuity, with the highest stresses occurring at A (Fig. 3.28). These stresses may 3.8 Stress Concentrations in Circular Shafts (a) Fig. 3.27 (b) Shaft examples. A D d Fig. 3.28 Shaft with change in diameter. be reduced through the use of a fillet, and the maximum value of the shearing stress at the fillet can be expressed as tmax 5 K Tc J (3.25) where the stress TcyJ is the stress computed for the smaller-diameter shaft, and where K is a stress-concentration factor. Since the factor K depends only upon the ratio of the two diameters and the ratio of the radius of the fillet to the diameter of the smaller shaft, it may be computed once and for all and recorded in the form of a table or a graph, as shown in Fig. 3.29. We should note, however, that this procedure for determining localized shearing stresses is valid only as long as the value of tmax given by Eq. (3.25) does not exceed the proportional limit of the material, since the values of K plotted in Fig. 3.29 were obtained under the assumption of a linear relation between shearing stress and shearing strain. If plastic deformations occur, they will result in values of the maximum stress lower than those indicated by Eq. (3.25). †W. D. Pilkey, Peterson’s Stress Concentration Factors, 2nd ed., John Wiley & Sons, New York, 1997. 1.8 r 1.7 d D  1.111 d 1.6 D  d 1.5 D 1.25 D  1.666 d K 1.4 D 2 d 1.3 D  2.5 d 1.2 1.1 1.0 0 0.05 0.10 0.15 0.20 0.25 0.30 r/d Fig. 3.29 Stress-concentration factors for fillets in circular shafts.† 179 bee80288_ch03_140-219.indd Page 180 11/2/10 1:37:52 AM user-f499 /Users/user-f499/Desktop/Temp Work/Don't Delete Job/MHDQ251:Beer:201/ch03 SAMPLE PROBLEM 3.6 7.50 in. 3.75 in. The stepped shaft shown is to rotate at 900 rpm as it transmits power from a turbine to a generator. The grade of steel specified in the design has an allowable shearing stress of 8 ksi. (a) For the preliminary design shown, determine the maximum power that can be transmitted. (b) If in the final design the radius of the fillet is increased so that r 5 15 16 in., what will be the percent change, relative to the preliminary design, in the power that can be transmitted? 9 r ⫽ 16 in. SOLUTION a. Preliminary Design. Using the notation of Fig. 3.32, we have: D 5 7.50 in., d 5 3.75 in., r 5 169 in. 5 0.5625 in. 7.50 in. D 5 52 d 3.75 in. 0.5625 in. r 5 5 0.15 d 3.75 in. A stress-concentration factor K 5 1.33 is found from Fig. 3.29. Torque. Recalling Eq. (3.25), we write tmax 5 K ␶m ⫽ ␶max ⫽ 6.02 ksi K 9 r ⫽ 16 in. T5 J tmax c K (1) where Jyc refers to the smaller-diameter shaft: Jyc 5 12pc3 5 12p11.875 in.2 3 5 10.35 in3 tmax 8 ksi 5 5 6.02 ksi K 1.33 and where T a ⫽ 62.3 kip · in. Tc J Substituting into Eq. (1), we find T 5 (10.35 in3)(6.02 ksi) 5 62.3 kip ? in. Power. Since f 5 1900 rpm2 1 Hz 5 15 Hz 5 15 s21, we write 60 rpm Pa 5 2p f T 5 2p(15 s21)(62.3 kip ? in.) 5 5.87 3 10 6 in. ? lb/s Pa 5 (5.87 3 10 6 in. ? lb/s)(1 hp/6600 in. ? lb/s) Pa 5 890 hp ◀ 15 b. Final Design. For r 5 16 in. 5 0.9375 in., D 52 d ␶m ⫽ ␶max ⫽ 6.67 ksi K r 0.9375 in. 5 5 0.250 d 3.75 in. K 5 1.20 Following the procedure used above, we write tmax 8 ksi 5 5 6.67 ksi K 1.20 J tmax 5 110.35 in3 2 16.67 ksi2 5 69.0 kip ? in. c K Pb 5 2p f T 5 2p115 s21 2 169.0 kip ? in.2 5 6.50 3 106 in. ? lb/s Pb 5 16.50 3 106 in. ? lb/s2 11 hp/6600 in. ? lb/s2 5 985 hp T5 T b ⫽ 69.0 kip · in. r ⫽ 15 16 in. Percent Change in Power Percent change 5 100 180 Pb 2 Pa 985 2 890 5 100 5 111% Pa 890 ◀ bee80288_ch03_140-219.indd Page 181 9/21/10 3:07:25 PM user-f499 /Users/user-f499/Desktop/Temp Work/Don't Delete Job/MHDQ251:Beer:201/ch03 PROBLEMS 3.64 Determine the maximum shearing stress in a solid shaft of 12-mm diameter as it transmits 2.5 kW at a frequency of (a) 25 Hz, (b) 50 Hz. 3.65 Determine the maximum shearing stress in a solid shaft of 1.5-in. diameter as it transmits 75 hp at a speed of (a) 750 rpm, (b) 1500 rpm. 3.66 Design a solid steel shaft to transmit 0.375 kW at a frequency of 29 Hz, if the shearing stress in the shaft is not to exceed 35 MPa. 3.67 Design a solid steel shaft to transmit 100 hp at a speed of 1200 rpm, if the maximum shearing stress is not to exceed 7500 psi. 3.68 Determine the required thickness of the 50-mm tubular shaft of Example 3.07, if it is to transmit the same power while rotating at a frequency of 30 Hz. 3.69 While a steel shaft of the cross section shown rotates at 120 rpm, a stroboscopic measurement indicates that the angle of twist is 28 in a 12-ft length. Using G 5 11.2 3 106 psi, determine the power being transmitted. 1.2 in. 3 in. Fig. P3.69 3.70 The hollow steel shaft shown (G 5 77.2 GPa, tall 5 50 MPa) rotates at 240 rpm. Determine (a) the maximum power that can be transmitted, (b) the corresponding angle of twist of the shaft. 5m T' 60 mm T 25 mm Fig. P3.70 and P3.71 3.71 As the hollow steel shaft shown rotates at 180 rpm, a stroboscopic measurement indicates that the angle of twist of the shaft is 38. Knowing that G 5 77.2 GPa, determine (a) the power being transmitted, (b) the maximum shearing stress in the shaft. 3.72 The design of a machine element calls for a 40-mm-outer-diameter shaft to transmit 45 kW. (a) If the speed of rotation is 720 rpm, determine the maximum shearing stress in shaft a. (b) If the speed of rotation can be increased 50% to 1080 rpm, determine the largest inner diameter of shaft b for which the maximum shearing stress will be the same in each shaft. 40 mm (a) Fig. P3.72 d2 (b) 181 bee80288_ch03_140-219.indd Page 182 9/21/10 3:07:32 PM user-f499 182 Torsion t 3.5 in. /Users/user-f499/Desktop/Temp Work/Don't Delete Job/MHDQ251:Beer:201/ch03 3.73 A steel pipe of 3.5-in. outer diameter is to be used to transmit a torque of 3000 lb ? ft without exceeding an allowable shearing stress of 8 ksi. A series of 3.5-in.-outer-diameter pipes is available for use. Knowing that the wall thickness of the available pipes varies from 0.25 in. to 0.50 in. in 0.0625-in. increments, choose the lightest pipe that can be used. 3.74 The two solid shafts and gears shown are used to transmit 16 hp from the motor at A operating at a speed of 1260 rpm to a machine tool at D. Knowing that the maximum allowable shearing stress is 8 ksi, determine the required diameter (a) of shaft AB, (b) of shaft CD. Fig. P3.73 D C 5 in. B A 3 in. Fig. P3.74 and P3.75 3.75 The two solid shafts and gears shown are used to transmit 16 hp from the motor at A operating at a speed of 1260 rpm to a machine tool at D. Knowing that each shaft has a diameter of 1 in., determine the maximum shearing stress (a) in shaft AB, (b) in shaft CD. 150 mm F E C 150 mm 3.76 Three shafts and four gears are used to form a gear train that will transmit 7.5 kW from the motor at A to a machine tool at F. (Bearings for the shafts are omitted in the sketch.) Knowing that the frequency of the motor is 30 Hz and that the allowable stress for each shaft is 60 MPa, determine the required diameter of each shaft. B 3.77 Three shafts and four gears are used to form a gear train that will transmit power from the motor at A to a machine tool at F. (Bearings for the shafts are omitted in the sketch.) The diameter of each shaft is as follows: dAB 5 16 mm, dCD 5 20 mm, dEF 5 28 mm. Knowing that the frequency of the motor is 24 Hz and that the allowable shearing stress for each shaft is 75 MPa, determine the maximum power that can be transmitted. Fig. P3.76 and P3.77 3.78 A 1.5-m-long solid steel shaft of 48-mm diameter is to transmit 36 kW between a motor and a machine tool. Determine the lowest speed at which the shaft can rotate, knowing that G 5 77.2 GPa, that the maximum shearing stress must not exceed 60 MPa, and the angle of twist must not exceed 2.58. D 60 mm 60 mm A bee80288_ch03_140-219.indd Page 183 9/21/10 3:07:39 PM user-f499 /Users/user-f499/Desktop/Temp Work/Don't Delete Job/MHDQ251:Beer:201/ch03 Problems 3.79 A 2.5-m-long steel shaft of 30-mm diameter rotates at a frequency of 30 Hz. Determine the maximum power that the shaft can transmit, knowing that G 5 77.2 GPa, that the allowable shearing stress is 50 MPa, and that the angle of twist must not exceed 7.58. 3.80 A steel shaft must transmit 210 hp at a speed of 360 rpm. Knowing that G 5 11.2 3 106 psi, design a solid shaft so that the maximum shearing stress will not exceed 12 ksi and the angle of twist in an 8.2-ft length will not exceed 38. r  1 18 in. 5 8 in. A 3.81 The shaft-disk-belt arrangement shown is used to transmit 3 hp from point A to point D. (a) Using an allowable shearing stress of 9500 psi, determine the required speed of shaft AB. (b) Solve part a, assuming that the diameters of shafts AB and CD are, respectively, 0.75 in. and 0.625 in. 3.82 A 1.6-m-long tubular steel shaft of 42-mm outer diameter d1 is to be made of a steel for which tall 5 75 MPa and G 5 77.2 GPa. Knowing that the angle of twist must not exceed 48 when the shaft is subjected to a torque of 900 N ? m, determine the largest inner diameter d2 that can be specified in the design. B C 3 4 in. D Fig. P3.81 r  4 12 in. d2 d1  42 mm Fig. P3.82 and P3.83 3.83 A 1.6-m-long tubular steel shaft (G 5 77.2 GPa) of 42-mm outer diameter d1 and 30-mm inner diameter d2 is to transmit 120 kW between a turbine and a generator. Knowing that the allowable shearing stress is 65 MPa and that the angle of twist must not exceed 38, determine the minimum frequency at which the shaft can rotate. 3.84 Knowing that the stepped shaft shown transmits a torque of magnitude T 5 2.50 kip ? in., determine the maximum shearing stress in the shaft when the radius of the fillet is (a) r 5 18 in., (b) r 5 163 in. 3.85 Knowing that the allowable shearing stress is 8 ksi for the stepped shaft shown, determine the magnitude T of the largest torque that can be transmitted by the shaft when the radius of the fillet is (a) r 5 163 in., (b) r 5 14 in. 3.86 The stepped shaft shown must transmit 40 kW at a speed of 720 rpm. Determine the minimum radius r of the fillet if an allowable stress of 36 MPa is not to be exceeded. 90 mm 45 mm r Fig. P3.86 T' 2 in. r 1.5 in. T Fig. P3.84 and P3.85 183 bee80288_ch03_140-219.indd Page 184 11/2/10 3:06:45 PM user-f499 184 3.87 The stepped shaft shown must transmit 45 kW. Knowing that the allowable shearing stress in the shaft is 40 MPa and that the radius of the fillet is r 5 6 mm, determine the smallest permissible speed of the shaft. Torsion T' /Users/user-f499/Desktop/Temp Work/Don't Delete Job/MHDQ251:Beer:201/ch03 60 mm 30 mm T 3.89 In the stepped shaft shown, which has a full quarter-circular fillet, D 5 1.25 in. and d 5 1 in. Knowing that the speed of the shaft is 2400 rpm and that the allowable shearing stress is 7500 psi, determine the maximum power that can be transmitted by the shaft. Fig. P3.87 and P3.88 d r⫽ 3.88 The stepped shaft shown must rotate at a frequency of 50 Hz. Knowing that the radius of the fillet is r 5 8 mm and the allowable shearing stress is 45 MPa, determine the maximum power that can be transmitted. 1 2 (D ⫺ d) 3.90 A torque of magnitude T 5 200 lb ? in. is applied to the stepped shaft shown, which has a full quarter-circular fillet. Knowing that D 5 1 in., determine the maximum shearing stress in the shaft when (a) d 5 0.8 in., (b) d 5 0.9 in. D Full quarter-circular fillet extends to edge of larger shaft. Fig. P3.89, P3.90, and P3.91 3.91 In the stepped shaft shown, which has a full quarter-circular fillet, the allowable shearing stress is 80 MPa. Knowing that D 5 30 mm, determine the largest allowable torque that can be applied to the shaft if (a) d 5 26 mm, (b) d 5 24 mm. *3.9 ␥ ␥max O c ␳ PLASTIC DEFORMATIONS IN CIRCULAR SHAFTS When we derived Eqs. (3.10) and (3.16), which define, respectively, the stress distribution and the angle of twist for a circular shaft subjected to a torque T, we assumed that Hooke’s law applied throughout the shaft. If the yield strength is exceeded in some portion of the shaft, or if the material involved is a brittle material with a nonlinear shearing-stress-strain diagram, these relations cease to be valid. The purpose of this section is to develop a more general method—which may be used when Hooke’s law does not apply—for determining the distribution of stresses in a solid circular shaft, and for computing the torque required to produce a given angle of twist. We first recall that no specific stress-strain relationship was assumed in Sec. 3.3, when we proved that the shearing strain g varies linearly with the distance r from the axis of the shaft (Fig. 3.30). Thus, we may still use this property in our present analysis and write g5 Fig. 3.30 Shearing strain variation. r g c max where c is the radius of the shaft. (3.4) bee80288_ch03_140-219.indd Page 185 11/2/10 3:06:52 PM user-f499 /Users/user-f499/Desktop/Temp Work/Don't Delete Job/MHDQ251:Beer:201/ch03 Assuming that the maximum value tmax of the shearing stress t has been specified, the plot of t versus r may be obtained as follows. We first determine from the shearing-stress-strain diagram the value of gmax corresponding to tmax (Fig. 3.31), and carry this value into Eq. (3.4). Then, for each value of r, we determine the corresponding value of g from Eq. (3.4) or Fig. 3.30 and obtain from the stress-strain diagram of Fig. 3.31 the shearing stress t corresponding to this value of g. Plotting t against r yields the desired distribution of stresses (Fig. 3.32). We now recall that, when we derived Eq. (3.1) in Sec. 3.2, we assumed no particular relation between shearing stress and strain. We may therefore use Eq. (3.1) to determine the torque T corresponding to the shearing-stress distribution obtained in Fig. 3.32. Considering an annular element of radius r and thickness dr, we express the element of area in Eq. (3.1) as dA 5 2pr dr and write 3.9 Plastic Deformations in Circular Shafts    f( ) max max  Fig. 3.31 Nonlinear, shear stressstrain diagram.  max c T5 # rt12pr dr2 0 O or c  c T 5 2p # r t dr 2 (3.26) 0 Fig. 3.32 Shearing strain variation for shaft with nonlinear stress-strain diagram. where t is the function of r plotted in Fig. 3.32. If t is a known analytical function of g, Eq. (3.4) may be used to express t as a function of r, and the integral in (3.26) may be determined analytically. Otherwise, the torque T may be obtained through a numerical integration. This computation becomes more meaningful if we note that the integral in Eq. (3.26) represents the second moment, or moment of inertia, with respect to the vertical axis of the area in Fig. 3.32 located above the horizontal axis and bounded by the stress-distribution curve. An important value of the torque is the ultimate torque TU which causes failure of the shaft. This value may be determined from the ultimate shearing stress tU of the material by choosing tmax 5 tU and carrying out the computations indicated earlier. However, it is found more convenient in practice to determine TU experimentally by twisting a specimen of a given material until it breaks. Assuming a fictitious linear distribution of stresses, Eq. (3.9) is then used to determine the corresponding maximum shearing stress RT: RT 5 TU c J (3.27) The fictitious stress RT is called the modulus of rupture in torsion of the given material. It may be used to determine the ultimate torque TU of a shaft made of the same material, but of different dimensions, by solving Eq. (3.27) for TU. Since the actual and the fictitious linear stress distributions shown in Fig. 3.33 must yield the same value TU for the ultimate torque, the areas they define must have the same moment of inertia with respect to the vertical axis. It is thus clear that the modulus of rupture RT will always be larger than the actual ultimate shearing stress tU.  RT U O Fig. 3.33 c  Shaft at failure. 185 bee80288_ch03_140-219.indd Page 186 9/21/10 3:08:00 PM user-f499 186 /Users/user-f499/Desktop/Temp Work/Don't Delete Job/MHDQ251:Beer:201/ch03 In some cases, we may wish to determine the stress distribution and the torque T corresponding to a given angle of twist f. This may be done by recalling the expression obtained in Sec. 3.3 for the shearing strain g in terms of f, r, and the length L of the shaft: Torsion g5 rf L (3.2) With f and L given, we may determine from Eq. (3.2) the value of g corresponding to any given value of r. Using the stress-strain diagram of the material, we may then obtain the corresponding value of the shearing stress t and plot t against r. Once the shearing-stress distribution has been obtained, the torque T may be determined analytically or numerically as explained earlier. *3.10 CIRCULAR SHAFTS MADE OF AN ELASTOPLASTIC MATERIAL Further insight into the plastic behavior of a shaft in torsion is obtained by considering the idealized case of a solid circular shaft made of an elastoplastic material. The shearing-stress-strain diagram of such a material is shown in Fig. 3.34. Using this diagram, we can proceed as indicated earlier and find the stress distribution across a section of the shaft for any value of the torque T. As long as the shearing stress t does not exceed the yield strength tY, Hooke’s law applies, and the stress distribution across the section is linear (Fig. 3.35a), with tmax given by Eq. (3.9):  Y  Fig. 3.34 Elastoplastic stressstrain diagram. Tc J tmax 5 max  Y max O (a) Fig. 3.35 c (3.9) Y Y Y O c (b) O Y c O c (d) (c) Stress-strain diagrams for shaft made of elastoplastic material. As the torque increases, tmax eventually reaches the value tY (Fig. 3.35b). Substituting this value into Eq. (3.9), and solving for the corresponding value of T, we obtain the value TY of the torque at the onset of yield: J TY 5 tY c (3.28) bee80288_ch03_140-219.indd Page 187 9/21/10 3:08:03 PM user-f499 /Users/user-f499/Desktop/Temp Work/Don't Delete Job/MHDQ251:Beer:201/ch03 The value obtained is referred to as the maximum elastic torque, since it is the largest torque for which the deformation remains fully elastic. Recalling that for a solid circular shaft Jyc 5 12 pc3, we have TY 5 12 pc3tY (3.29) As the torque is further increased, a plastic region develops in the shaft, around an elastic core of radius rY (Fig. 3.35c). In the plastic region the stress is uniformly equal to tY, while in the elastic core the stress varies linearly with r and may be expressed as t5 tY r rY (3.30) As T is increased, the plastic region expands until, at the limit, the deformation is fully plastic (Fig. 3.35d). Equation (3.26) will be used to determine the value of the torque T corresponding to a given radius rY of the elastic core. Recalling that t is given by Eq. (3.30) for 0 # r # rY, and is equal to tY for rY # r # c, we write c tY rb dr 1 2p r2tY dr rY rY 0 1 3 2 3 2 3 5 prY tY 1 pc tY 2 prY tY 2 3 3 3 2 1 rY T 5 pc3tY a1 2 b 3 4 c3 T 5 2p # rY # r2 a (3.31) or, in view of Eq. (3.29), T5 4 1 r3Y TY a1 2 b 3 4 c3 (3.32) where TY is the maximum elastic torque. We note that, as r Y approaches zero, the torque approaches the limiting value Tp 5 4 TY 3 (3.33) This value of the torque, which corresponds to a fully plastic deformation (Fig. 3.35d), is called the plastic torque of the shaft considered. We note that Eq. (3.33) is valid only for a solid circular shaft made of an elastoplastic material. Since the distribution of strain across the section remains linear after the onset of yield, Eq. (3.2) remains valid and can be used to express the radius rY of the elastic core in terms of the angle of twist f. If f is large enough to cause a plastic deformation, the radius rY of the elastic core is obtained by making g equal to the yield strain gY in Eq. (3.2) and solving for the corresponding value rY of the distance r. We have rY 5 LgY f (3.34) 3.10 Circular Shafts Made of an Elastoplastic Material 187 bee80288_ch03_140-219.indd Page 188 9/21/10 3:08:04 PM user-f499 188 /Users/user-f499/Desktop/Temp Work/Don't Delete Job/MHDQ251:Beer:201/ch03 Let us denote by fY the angle of twist at the onset of yield, i.e., when rY 5 c. Making f 5 fY and rY 5 c in Eq. (3.34), we have Torsion c5 LgY fY (3.35) Dividing (3.34) by (3.35), member by member, we obtain the following relation:† rY fY 5 c f (3.36) If we carry into Eq. (3.32) the expression obtained for rYyc, we express the torque T as a function of the angle of twist f, 4 1 f3Y T 5 TY a1 2 b 3 4 f3 T Tp  4 3 TY TY Y 0 3 Y  Y 2 Y Fig. 3.36 Load displacement relation for elastoplastic material. (3.37) where TY and fY represent, respectively, the torque and the angle of twist at the onset of yield. Note that Eq. (3.37) may be used only for values of f larger than f Y. For f , f Y, the relation between T and f is linear and given by Eq. (3.16). Combining both equations, we obtain the plot of T against f represented in Fig. 3.39. We check that, as f increases indefinitely, T approaches the limiting value Tp 5 43 TY corresponding to the case of a fully developed plastic zone (Fig. 3.35d). While the value Tp cannot actually be reached, we note from Eq. (3.37) that it is rapidly approached as f increases. For f 5 2fY, T is within about 3% of Tp, and for f 5 3fY within about 1%. Since the plot of T against f that we have obtained for an idealized elastoplastic material (Fig. 3.36) differs greatly from the shearingstress-strain diagram of that material (Fig. 3.34), it is clear that the shearing-stress-strain diagram of an actual material cannot be obtained directly from a torsion test carried out on a solid circular rod made of that material. However, a fairly accurate diagram may be obtained from a torsion test if the specimen used incorporates a portion consisting of a thin circular tube.‡ Indeed, we may assume that the shearing stress will have a constant value t in that portion. Equation (3.1) thus reduces to T 5 rAt where r denotes the average radius of the tube and A its crosssectional area. The shearing stress is thus proportional to the torque, and successive values of t can be easily computed from the corresponding values of T. On the other hand, the values of the shearing strain g may be obtained from Eq. (3.2) and from the values of f and L measured on the tubular portion of the specimen. †Equation (3.36) applies to any ductile material with a well-defined yield point, since its derivation is independent of the shape of the stress-strain diagram beyond the yield point. ‡In order to minimize the possibility of failure by buckling, the specimen should be made so that the length of the tubular portion is no longer than its diameter. bee80288_ch03_140-219.indd Page 189 9/21/10 3:08:06 PM user-f499 /Users/user-f499/Desktop/Temp Work/Don't Delete Job/MHDQ251:Beer:201/ch03 A solid circular shaft, 1.2 m long and 50 mm in diameter, is subjected to a 4.60 kN ? m torque at each end (Fig. 3.37). Assuming the shaft to be made of an elastoplastic material with a yield strength in shear of 150 MPa and a modulus of rigidity of 77 GPa, determine (a) the radius of the elastic core, (b) the angle of twist of the shaft. EXAMPLE 3.08 4.60 kN · m 4.60 kN · m (a) Radius of Elastic Core. We first determine the torque TY at the onset of yield. Using Eq. (3.28) with tY 5 150 MPa, c 5 25 mm, and J5 1 4 2 pc 5 1 2 p125 23 3 10 4 29 m2 5 614 3 10 m 50 mm 4 1.2 m we write TY 5 JtY 1614 3 1029 m4 2 1150 3 106 Pa2 5 3.68 kN ? m 5 c 25 3 1023 m Fig. 3.37 Solving Eq. (3.32) for (rYyc)3 and substituting the values of T and TY, we have a 314.60 kN ? m2 rY 3 3T 542 b 542 5 0.250 c TY 3.68 kN ? m rY 5 0.630 c rY 5 0.630125 mm2 5 15.8 mm (b) Angle of Twist. We first determine the angle of twist fY at the onset of yield from Eq. (3.16): fY 5 13.68 3 103 N ? m2 11.2 m2 TYL 5 5 93.4 3 1023 rad JG 1614 3 1029 m4 2 177 3 109 Pa2 Solving Eq. (3.36) for f and substituting the values obtained for fY and rYyc, we write f5 fY 93.4 3 1023 rad 5 148.3 3 1023 rad 5 rYyc 0.630 or f 5 1148.3 3 1023 rad2a *3.11 360° b 5 8.50° 2p rad RESIDUAL STRESSES IN CIRCULAR SHAFTS In the two preceding sections, we saw that a plastic region will develop in a shaft subjected to a large enough torque, and that the shearing stress t at any given point in the plastic region may be obtained from the shearing-stress-strain diagram of Fig. 3.31. If the torque is removed, the resulting reduction of stress and strain at the point considered will take place along a straight line (Fig. 3.38). As you will see further in this section, the final value of the stress will not, in general, be zero. There will be a residual stress at most points, and that stress may be either positive or negative. We note that, as was the case for the normal stress, the shearing stress will keep decreasing until it has reached a value equal to its maximum value at C minus twice the yield strength of the material. Consider again the idealized case of the elastoplastic material characterized by the shearing-stress-strain diagram of Fig. 3.34.  Y C Y 2 Y 0  Fig. 3.38 Unloading of shaft with nonlinear stress-strain diagram. 189 bee80288_ch03_140-219.indd Page 190 9/21/10 3:08:11 PM user-f499 190 /Users/user-f499/Desktop/Temp Work/Don't Delete Job/MHDQ251:Beer:201/ch03 Assuming that the relation between t and g at any point of the shaft remains linear as long as the stress does not decrease by more than 2tY, we can use Eq. (3.16) to obtain the angle through which the shaft untwists as the torque decreases back to zero. As a result, the unloading of the shaft will be represented by a straight line on the T-f diagram (Fig. 3.39). We note that the angle of twist does not return to zero after the torque has been removed. Indeed, the loading and unloading of the shaft result in a permanent deformation characterized by the angle Torsion T TY T (3.38) fp 5 f 2 f9 0  p where f corresponds to the loading phase and may be obtained from T by solving Eq. (3.38), and where f9 corresponds to the unloading phase and may be obtained from Eq. (3.16). The residual stresses in an elastoplastic material are obtained by applying the principle of superposition in a manner similar to that described in Sec. 2.20 for an axial loading. We consider, on one hand, the stresses due to the application of the given torque T and, on the other, the stresses due to the equal and opposite torque which is applied to unload the shaft. The first group of stresses reflects the elastoplastic behavior of the material during the loading phase (Fig. 3.40a), and the second group the linear behavior of the same material during the unloading phase (Fig. 3.40b). Adding the two groups of stresses, we obtain the distribution of the residual stresses in the shaft (Fig. 3.40c).   Fig. 3.39 Unloading of shaft with elastoplastic material. Y    Y Y 0 c (a) Fig. 3.40  0  c (b) 0  'm  Tc J c  (c) Stress distributions for unloading of shaft with elastoplastic material. We note from Fig. 3.40c that some residual stresses have the same sense as the original stresses, while others have the opposite sense. This was to be expected since, according to Eq. (3.1), the relation e r1t dA2 5 0 (3.39) must be verified after the torque has been removed. EXAMPLE 3.09 For the shaft of Example 3.08 determine (a) the permanent twist, (b) the distribution of residual stresses, after the 4.60 kN ? m torque has been removed. (a) Permanent Twist. We recall from Example 3.08 that the angle of twist corresponding to the given torque is f 5 8.508. The angle f9 bee80288_ch03_140-219.indd Page 191 9/21/10 3:08:16 PM user-f499 /Users/user-f499/Desktop/Temp Work/Don't Delete Job/MHDQ251:Beer:201/ch03 through which the shaft untwists as the torque is removed is obtained from Eq. (3.16). Substituting the given data, T 5 4.60 3 103 N ? m L 5 1.2 m G 5 77 3 109 Pa and the value J 5 614 3 1029 m4 obtained in the solution of Example 3.08, we have 14.60 3 103 N ? m2 11.2 m2 TL 5 f¿ 5 JG 1614 3 1029 m4 2 177 3 109 Pa2 5 116.8 3 1023 rad or 360° 5 6.69° f¿ 5 1116.8 3 1023 rad2 2p rad The permanent twist is therefore fp 5 f 2 f¿ 5 8.50° 2 6.69° 5 1.81° (b) Residual Stresses. We recall from Example 3.08 that the yield strength is tY 5 150 MPa and that the radius of the elastic core corresponding to the given torque is rY 5 15.8 mm. The distribution of the stresses in the loaded shaft is thus as shown in Fig. 3.41a. The distribution of stresses due to the opposite 4.60 kN ? m torque required to unload the shaft is linear and as shown in Fig. 3.41b. The maximum stress in the distribution of the reverse stresses is obtained from Eq. (3.9): 14.60 3 103 N ? m2 125 3 1023 m2 Tc 5 t¿ max 5 J 614 3 1029 m4 5 187.3 MPa Superposing the two distributions of stresses, we obtain the residual stresses shown in Fig. 3.41c. We check that, even though the reverse stresses exceed the yield strength tY, the assumption of a linear distribution of these stresses is valid, since they do not exceed 2tY.  (MPa)  (MPa)  (MPa) 150 31.6 0  0  0  –37.3 15.8 mm 15.8 mm –118.4 25 mm –187.3 (a) (b) (c) Fig. 3.41 191 bee80288_ch03_140-219.indd Page 192 11/2/10 3:07:04 PM user-f499 2.25 in. T´ A SAMPLE PROBLEM 3.7 Shaft AB is made of a mild steel that is assumed to be elastoplastic with G 5 11.2 3 106 psi and tY 5 21 ksi. A torque T is applied and gradually increased in magnitude. Determine the magnitude of T and the corresponding angle of twist (a) when yield first occurs, (b) when the deformation has become fully plastic. B 1.5 in. /Users/user-f499/Desktop/Temp Work/Don't Delete Job/MHDQ251:Beer:201/ch03 60 in. T ␶ (ksi) SOLUTION 21 Geometric Properties The geometric properties of the cross section are ␥ TY ␶Y 37.7 kip · in. 21 ksi c2 5 12 12.25 in.2 5 1.125 in. c1 5 12 11.5 in.2 5 0.75 in. J 5 12p1c42 2 c41 2 5 12p 3 11.125 in.2 4 2 10.75 in.2 4 4 5 2.02 in4 a. Onset of Yield. For tmax 5 tY 5 21 ksi, we find TY 5 tYJ 121 ksi2 12.02 in4 2 5 c2 1.125 in. T Y 5 37.7 kip ? in. ◀ c2 1.125 in. c1 Making r 5 c2 and g 5 gY in Eq. (3.2) and solving for f, we obtain the value of fY: 0.75 in. ␾Y Tp 44.1 kip · in. 121 3 103 psi2 160 in.2 gYL tYL 5 5 5 0.100 rad fY 5 c2 c2G 11.125 in.2 111.2 3 106 psi2 5.73 Y fY 5 5.738 21 ksi ◀ b. Fully Plastic Deformation. When the plastic zone reaches the inner surface, the stresses are uniformly distributed as shown. Using Eq. (3.26), we write Tp 5 2ptY # c2 r2 dr 5 23ptY 1c32 2 c31 2 c1 5 23p121 ksi2 3 11.125 in.2 3 2 10.75 in.2 3 4 Tp 5 44.1 kip ? in. ◀ 8.59 ␾f When yield first occurs on the inner surface, the deformation is fully plastic; we have from Eq. (3.2): T Tp ff 5 TY 121 3 103 psi2 160 in.2 gYL tYL 5 5 5 0.150 rad c1 c1G 10.75 in.2 111.2 3 106 psi2 ff 5 8.598 ␾Y 192 ␾f ␾ ◀ For larger angles of twist, the torque remains constant; the T-f diagram of the shaft is as shown. bee80288_ch03_140-219.indd Page 193 11/2/10 9:56:55 PM user-f494 /207/BSMB003/work%0/indd%0 SAMPLE PROBLEM 3.8 For the shaft of Sample Prob. 3.7, determine the residual stresses and the permanent angle of twist after the torque Tp 5 44.1 kip ? in. has been removed. SOLUTION Referring to Sample Prob. 3.7, we recall that when the plastic zone first reached the inner surface, the applied torque was Tp 5 44.1 kip ? in. and the corresponding angle of twist was ff 5 8.598. These values are shown in Fig. 1. Elastic Unloading. We unload the shaft by applying a 44.1 kip ? in. torque in the sense shown in Fig. 2. During this unloading, the behavior of the material is linear. Recalling from Sample Prob. 3.7 the values found for c1, c2, and J, we obtain the following stresses and angle of twist: 144.1 kip ? in.2 11.125 in.2 Tc2 5 5 24.56 ksi J 2.02 in4 c1 0.75 in. 5 16.37 ksi tmin 5 tmax 5 124.56 ksi2 c2 1.125 in. 144.1 3 103 psi2 160 in.2 TL 5 5 0.1170 rad 5 6.70° f¿ 5 JG 12.02 in4 2 111.2 3 106 psi2 tmax 5 Residual Stresses and Permanent Twist. The results of the loading (Fig. 1) and the unloading (Fig. 2) are superposed (Fig. 3) to obtain the residual stresses and the permanent angle of twist fp. 44.1 kip · in. 44.1 kip · in. 44.1 kip · in. Tp ⫽ 44.1 kip · in. (2) (1) Y 21 ksi (3) 16.37 ksi 4.63 ksi 1 2 3.56 ksi 44.1 kip · in. Tp ⫽ 44.1 kip · in. ␾ f ⫽ 8.59⬚ ␾' ⫽ 6.70⬚ 24.56 ksi ␾ p ⫽ 1.89⬚ 193 bee80288_ch03_140-219.indd Page 194 9/21/10 9:17:11 PM user-f499 /Users/user-f499/Desktop/Temp Work/Don't Delete Job/MHDQ251:Beer:201/ch03 PROBLEMS 3.92 A 30-mm diameter solid rod is made of an elastoplastic material with tY 5 3.5 MPa. Knowing that the elastic core of the rod is 25 mm in diameter, determine the magnitude of the applied torque T. c ⫽ 1.5 in. T' T 3.93 The solid circular shaft shown is made of a steel that is assumed to be elastoplastic with tY 5 21 ksi. Determine the magnitude T of the applied torques when the plastic zone is (a) 0.8 in. deep, (b) 1.2 in. deep. 3.94 The solid circular shaft shown is made of a steel that is assumed to be elastoplastic with tY 5 145 MPa. Determine the magnitude T of the applied torque when the plastic zone is (a) 16 mm deep, (b) 24 mm deep. Fig. P3.93 3.95 The solid shaft shown is made of a mild steel that is assumed to be elastoplastic with G 5 11.2 3 106 psi and tY 5 21 ksi. Determine the maximum shearing stress and the radius of the elastic core caused by the application of a torque of magnitude (a) T 5 100 kip ? in., (b) T 5 140 kip ? in. 4 ft 3 in. Fig. P3.95 1.2 m T 30 mm Fig. P3.97 T 3.96 It is observed that a straightened paper clip can be twisted through several revolutions by the application of a torque of approximately 60 mN ? m. Knowing that the diameter of the wire in the paper clip is 0.9 mm, determine the approximate value of the yield stress of the steel. 3.97 The solid shaft shown is made of a mild steel that is assumed to be elastoplastic with tY 5 145 MPa. Determine the radius of the elastic core caused by the application of a torque equal to 1.1 TY, where TY is the magnitude of the torque at the onset of yield. 3.98 For the solid circular shaft of Prob. 3.95, determine the angle of twist caused by the application of a torque of magnitude (a) T 5 80 kip ? in., (b) T 5 130 kip ? in. 3.99 The solid shaft shown is made of a mild steel that is assumed to be elastoplastic with G 5 77.2 GPa and tY 5 145 MPa. Determine the angle of twist caused by the application of a torque of magnitude (a) T 5 600 N ? m, (b) T 5 1000 N ? m. A 15 mm 1.2 m Fig. P3.99 194 B T bee80288_ch03_140-219.indd Page 195 9/21/10 3:08:30 PM user-f499 /Users/user-f499/Desktop/Temp Work/Don't Delete Job/MHDQ251:Beer:201/ch03 Problems 3.100 A 3-ft-long solid shaft has a diameter of 2.5 in. and is made of a mild steel that is assumed to be elastoplastic with tY 5 21 ksi and G 5 11.2 3 106 psi. Determine the torque required to twist the shaft through an angle of (a) 2.58, (b) 58. 195 3.101 For the solid shaft of Prob. 3.99, determine (a) the magnitude of the torque T required to twist the shaft through an angle of 158, (b) the radius of the corresponding elastic core. A 3.102 The shaft AB is made of a material that is elastoplastic with tY 5 12 ksi and G 5 4.5 3 106 psi. For the loading shown, determine (a) the radius of the elastic core of the shaft, (b) the angle of twist at end B. 3.103 A 1.25-in.-diameter solid circular shaft is made of a material that is assumed to be elastoplastic with tY 5 18 ksi and G 5 11.2 3 106 psi. For an 8-ft length of the shaft, determine the maximum shearing stress and the angle of twist caused by a 7.5-kip ? in. torque. 1 2 6.4 ft Fig. P3.102 3.104 An 18-mm-diameter solid circular shaft is made of a material that is assumed to be elastoplastic with tY 5 145 MPa and G 5 77 GPa. For an 1.2-m length of the shaft, determine the maximum shearing stress and the angle of twist caused by a 200 N ? m-torque. 3.105 A solid circular rod is made of a material that is assumed to be elastoplastic. Denoting by TY and fY, respectively, the torque and the angle of twist at the onset of yield, determine the angle of twist if the torque is increased to (a) T 5 1.1 TY, (b) T 5 1.25 TY, (c) T 5 1.3 TY. 3.106 The hollow shaft shown is made of steel that is assumed to be elastoplastic with tY 5 145 MPa and G 5 77.2 GPa. Determine the magnitude T of the torque and the corresponding angle of twist (a) at the onset of yield, (b) when the plastic zone is 10 mm deep. 5m T' T 60 mm 25 mm Fig. P3.106 3.107 For the shaft of Prob. 3.106, determine (a) angle of twist at which the section first becomes fully plastic, (b) the corresponding magnitude T of the applied torque. Sketch the T-f curve for the shaft. 3.108 A steel rod is machined to the shape shown to form a tapered solid shaft to which torques of magnitude T 5 75 kip ? in. are applied. Assuming the steel to be elastoplastic with tY 5 21 ksi and G 5 11.2 3 106 psi, determine (a) the radius of the elastic core in portion AB of the shaft, (b) the length of portion CD that remains fully elastic. 3.109 If the torques applied to the tapered shaft of Prob. 3.108 are slowly increased, determine (a) the magnitude T of the largest torques that can be applied to the shaft, (b) the length of the portion CD that remains fully elastic. T A 2.5 in. B C x D 3 in. E T' Fig. P3.108 5 in. B in. T  2560 lb · in. bee80288_ch03_140-219.indd Page 196 9/21/10 9:16:30 PM user-f499 196 /Users/user-f499/Desktop/Temp Work/Don't Delete Job/MHDQ251:Beer:201/ch03 3.110 A hollow shaft of outer and inner diameters respectively equal to 0.6 in. and 0.2 in. is fabricated from an aluminum alloy for which the stress-strain diagram is given in the diagram shown. Determine the torque required to twist a 9-in. length of the shaft through 108. Torsion  (ksi) 16 12 3.111 Using the stress-strain diagram shown, determine (a) the torque that causes a maximum shearing stress of 15 ksi in a 0.8-in.-diameter solid rod, (b) the corresponding angle of twist in a 20-in. length of the rod. 8 4 0 0.002 0.004 0.006 0.008 0.010  Fig. P3.110 and P3.111 T' d ⫽ 50 mm  (MPa) 100 3.113 Three points on the nonlinear stress-strain diagram used in Prob. 3.112 are (0, 0), (0.0015, 55 MPa), and (0.003, 80MPa). By fitting the polynomial T 5 A 1 Bg 1 Cg2 through these points, the following approximate relation has been obtained. 725 mm T 3.112 A 50-mm-diameter cylinder is made of a brass for which the stressstrain diagram is as shown. Knowing that the angle of twist is 58 in a 725-mm length, determine by approximate means the magnitude T of torque applied to the shaft. T 5 46.7 3 109g 2 6.67 3 1012g2 80 60 Solve Prob. 3.112 using this relation, Eq. (3.2), and Eq. (3.26). 40 20 0 0.001 Fig. P3.112 0.002 0.003  3.114 The solid circular drill rod AB is made of a steel that is assumed to be elastoplastic with tY 5 22 ksi and G 5 11.2 3 106 psi. Knowing that a torque T 5 75 kip ? in. is applied to the rod and then removed, determine the maximum residual shearing stress in the rod. A T 1.2 in. 35 ft B Fig. P3.114 0.6 m 3.115 In Prob. 3.114, determine the permanent angle of twist of the rod. A B 16 mm Fig. P3.116 T 3.116 The solid shaft shown is made of a steel that is assumed to be elastoplastic with tY 5 145 MPa and G 5 77.2 GPa. The torque is increased in magnitude until the shaft has been twisted through 68; the torque is then removed. Determine (a) the magnitude and location of the maximum residual shearing stress, (b) the permanent angle of twist. bee80288_ch03_140-219.indd Page 197 11/2/10 3:07:29 PM user-f499 /Users/user-f499/Desktop/Temp Work/Don't Delete Job/MHDQ251:Beer:201/ch03 3.117 After the solid shaft of Prob. 3.116 has been loaded and unloaded as described in that problem, a torque T1 of sense opposite to the original torque T is applied to the shaft. Assuming no change in the value of fY, determine the angle of twist f1 for which yield is initiated in this second loading and compare it with the angle f Y for which the shaft started to yield in the original loading. 3.12 Torsion of Noncircular Members 3.118 The hollow shaft shown is made of a steel that is assumed to be elastoplastic with tY 5 145 MPa and G 5 77.2 GPa. The magnitude T of the torques is slowly increased until the plastic zone first reaches the inner surface of the shaft; the torques are then removed. Determine the magnitude and location of the maximum residual shearing stress in the rod. T 5m T' 60 mm 25 mm Fig. P3.118 ␶Y 3.119 In Prob. 3.118, determine the permanent angle of twist of the rod. 3.120 A torque T applied to a solid rod made of an elastoplastic material is increased until the rod is fully plastic and then removed. (a) Show that the distribution of residual shearing stresses is as represented in the figure. (b) Determine the magnitude of the torque due to the stresses acting on the portion of the rod located within a circle of radius c0. c c0 1 3 ␶Y Fig. P3.120 *3.12 TORSION OF NONCIRCULAR MEMBERS The formulas obtained in Secs. 3.3 and 3.4 for the distributions of strain and stress under a torsional loading apply only to members with a circular cross section. Indeed, their derivation was based on the assumption that the cross section of the member remained plane and undistorted, and we saw in Sec. 3.3 that the validity of this assumption depends upon the axisymmetry of the member, i.e., upon the fact that its appearance remains the same when it is viewed from a fixed position and rotated about its axis through an arbitrary angle. A square bar, on the other hand, retains the same appearance only when it is rotated through 908 or 1808. Following a line of reasoning similar to that used in Sec. 3.3, one could show that the diagonals of the square cross section of the bar and the lines joining the midpoints of the sides of that section remain straight (Fig. 3.42). However, because of the lack of axisymmetry of the bar, any other line drawn in its cross section will deform when the bar is twisted, and the cross section itself will be warped out of its original plane. It follows that Eqs. (3.4) and (3.6), which define, respectively, the distributions of strain and stress in an elastic circular shaft, cannot be used for noncircular members. For example, it would be wrong to assume that the shearing stress in the cross section of a square bar varies linearly with the distance from the axis of the bar and is, therefore, largest at the corners of the cross section. As you will see presently, the shearing stress is actually zero at these points. T T' Fig. 3.42 Twisting of shaft with square cross section. 197 bee80288_ch03_140-219.indd Page 198 11/2/10 3:07:38 PM user-f499 198 /Users/user-f499/Desktop/Temp Work/Don't Delete Job/MHDQ251:Beer:201/ch03 Consider a small cubic element located at a corner of the cross section of a square bar in torsion and select coordinate axes parallel to the edges of the element (Fig. 3.43a). Since the face of the element perpendicular to the y axis is part of the free surface of the bar, all stresses on this face must be zero. Referring to Fig. 3.43b, we write Torsion y x z tyx 5 0 tyz 5 0 For the same reason, all stresses on the face of the element perpendicular to the z axis must be zero, and we write (a) tzx 5 0 y ␶xz ␶zx txy 5 0 x z tzy 5 0 (3.41) It follows from the first of Eqs. (3.40) and the first of Eqs. (3.41) that ␶yz ␶yx ␶zy (3.40) ␶xy (b) Fig. 3.43 Corner element. txz 5 0 (3.42) Thus, both components of the shearing stress on the face of the element perpendicular to the axis of the bar are zero. We conclude that there is no shearing stress at the corners of the cross section of the bar. By twisting a rubber model of a square bar, one easily verifies that no deformations—and, thus, no stresses—occur along the edges of the bar, while the largest deformations—and, thus, the largest stresses—occur along the center line of each of the faces of the bar (Fig. 3.44). ␶max T' Fig. 3.44 a T' ␶max T b L Fig. 3.45 Shaft with rectangular cross section. T ␶max Deformation of square bar. The determination of the stresses in noncircular members subjected to a torsional loading is beyond the scope of this text. However, results obtained from the mathematical theory of elasticity for straight bars with a uniform rectangular cross section will be indicated here for convenience.† Denoting by L the length of the bar, by a and b, respectively, the wider and narrower side of its cross section, and by T the magnitude of the torques applied to the bar (Fig. 3.45), we find that the maximum shearing stress occurs along the center line of the wider face of the bar and is equal to tmax 5 T c1ab2 (3.43) The angle of twist, on the other hand, may be expressed as f5 TL c2ab3G (3.44) †See S. P. Timoshenko and J. N. Goodier, Theory of Elasticity, 3d ed., McGraw-Hill, New York, 1969, sec. 109. bee80288_ch03_140-219.indd Page 199 11/2/10 12:48:32 AM user-f499 /Users/user-f499/Desktop/Temp Work/Don't Delete Job/MHDQ251:Beer:201/ch03 The coefficients c1 and c2 depend only upon the ratio ayb and are given in Table 3.1 for a number of values of that ratio. Note that Eqs. (3.43) and (3.44) are valid only within the elastic range. We note from Table 3.1 that for ayb $ 5, the coefficients c1 and c2 are equal. It may be shown that for such values of ayb, we have c1 5 c2 5 13 11 2 0.630bya2 (for ayb % 5 only) (3.45) The distribution of shearing stresses in a noncircular member may be visualized more easily by using the membrane analogy. A homogeneous elastic membrane attached to a fixed frame and subjected to a uniform pressure on one of its sides happens to constitute an analog of the bar in torsion, i.e., the determination of the deformation of the membrane depends upon the solution of the same partial differential equation as the determination of the shearing stresses in the bar.† More specifically, if Q is a point of the cross section of the bar and Q9 the corresponding point of the membrane (Fig. 3.46), the shearing stress t at Q will have the same direction as the horizontal tangent to the membrane at Q9, and its magnitude will be proportional to the maximum slope of the membrane at Q9.‡ Furthermore, the applied torque will be proportional to the volume between the membrane and the plane of the fixed frame. In the case of the membrane of Fig. 3.46, which is attached to a rectangular frame, the steepest slope occurs at the midpoint N9 of the larger side of the frame. Thus, we verify that the maximum shearing stress in a bar of rectangular cross section will occur at the midpoint N of the larger side of that section. The membrane analogy may be used just as effectively to visualize the shearing stresses in any straight bar of uniform, noncircular cross section. In particular, let us consider several thin-walled members with the cross sections shown in Fig. 3.47, which are subjected TABLE 3.1. Coefficients for Rectangular Bars in Torsion a/b c1 c2 1.0 1.2 1.5 2.0 2.5 3.0 4.0 5.0 10.0 ` 0.208 0.219 0.231 0.246 0.258 0.267 0.282 0.291 0.312 0.333 0.1406 0.1661 0.1958 0.229 0.249 0.263 0.281 0.291 0.312 0.333 Tangent of max. slope Rectangular frame Membrane Q' b N' a b Horizontal tangent T Q N  a a Fig. 3.47 b b a a b Various thin-walled members. †See ibid. Sec. 107. ‡This is the slope measured in a direction perpendicular to the horizontal tangent at Q9. 199 3.12 Torsion of Noncircular Members Fig. 3.46 Application of membrane analogy to shaft with rectangular cross section. bee80288_ch03_140-219.indd Page 200 9/21/10 3:09:04 PM user-f499 200 /Users/user-f499/Desktop/Temp Work/Don't Delete Job/MHDQ251:Beer:201/ch03 to the same torque. Using the membrane analogy to help us visualize the shearing stresses, we note that, since the same torque is applied to each member, the same volume will be located under each membrane, and the maximum slope will be about the same in each case. Thus, for a thin-walled member of uniform thickness and arbitrary shape, the maximum shearing stress is the same as for a rectangular bar with a very large value of ayb and may be determined from Eq. (3.43) with c1 5 0.333.† Torsion *3.13 T' x T t B A x Fig. 3.48 Thin-walled hollow shaft. FB B tB A FA tA x x Fig. 3.49 Segment of thin-walled hollow shaft. THIN-WALLED HOLLOW SHAFTS In the preceding section we saw that the determination of stresses in noncircular members generally requires the use of advanced mathematical methods. In the case of thin-walled hollow noncircular shafts, however, a good approximation of the distribution of stresses in the shaft can be obtained by a simple computation. Consider a hollow cylindrical member of noncircular section subjected to a torsional loading (Fig. 3.48).‡ While the thickness t of the wall may vary within a transverse section, it will be assumed that it remains small compared to the other dimensions of the member. We now detach from the member the colored portion of wall AB bounded by two transverse planes at a distance Dx from each other, and by two longitudinal planes perpendicular to the wall. Since the portion AB is in equilibrium, the sum of the forces exerted on it in the longitudinal x direction must be zero (Fig. 3.49). But the only forces involved are the shearing forces FA and FB exerted on the ends of portion AB. We have therefore oFx 5 0: FA 2 FB 5 0 (3.46) We now express FA as the product of the longitudinal shearing stress tA on the small face at A and of the area tA Dx of that face: FA 5 tA(tA Dx) We note that, while the shearing stress is independent of the x coordinate of the point considered, it may vary across the wall; thus, tA represents the average value of the stress computed across the wall. Expressing FB in a similar way and substituting for FA and FB into (3.46), we write tA(tA Dx) 2 tB(tB Dx) 5 0 or tAtA 5 tBtB (3.47) Since A and B were chosen arbitrarily, Eq. (3.47) expresses that the product tt of the longitudinal shearing stress t and of the wall thickness t is constant throughout the member. Denoting this product by q, we have q 5 tt 5 constant (3.48) †It could also be shown that the angle of twist may be determined from Eq. (3.44) with c2 5 0.333. ‡The wall of the member must enclose a single cavity and must not be slit open. In other words, the member should be topologically equivalent to a hollow circular shaft. bee80288_ch03_140-219.indd Page 201 9/21/10 3:09:08 PM user-f499 /Users/user-f499/Desktop/Temp Work/Don't Delete Job/MHDQ251:Beer:201/ch03 We now detach a small element from the wall portion AB (Fig. 3.50). Since the upper and lower faces of this element are part of the free surface of the hollow member, the stresses on these faces are equal to zero. Recalling relations (1.21) and (1.22) of Sec. 1.12, it follows that the stress components indicated on the other faces by dashed arrows are also zero, while those represented by solid arrows are equal. Thus, the shearing stress at any point of a transverse section of the hollow member is parallel to the wall surface (Fig. 3.51) and its average value computed across the wall satisfies Eq. (3.48). At this point we can note an analogy between the distribution of the shearing stresses t in the transverse section of a thin-walled hollow shaft and the distribution of the velocities v in water flowing through a closed channel of unit depth and variable width. While the velocity v of the water varies from point to point on account of the variation in the width t of the channel, the rate of flow, q 5 vt, remains constant throughout the channel, just as tt in Eq. (3.48). Because of this analogy, the product q 5 tt is referred to as the shear flow in the wall of the hollow shaft. We will now derive a relation between the torque T applied to a hollow member and the shear flow q in its wall. We consider a small element of the wall section, of length ds (Fig. 3.52). The area of the element is dA 5 t ds, and the magnitude of the shearing force dF exerted on the element is dF 5 t dA 5 t(t ds) 5 (tt) ds 5 q ds t s (3.50) x x Fig. 3.50 Small element from segment. t  Fig. 3.51 Direction of shearing stress on cross section. ds t p O dF Fig. 3.52 But the product p ds is equal to twice the area dA of the colored triangle in Fig. 3.53. We thus have dMO 5 q(2dA)   (3.49) The moment dMO of this force about an arbitrary point O within the cavity of the member may be obtained by multiplying dF by the perpendicular distance p from O to the line of action of dF. We have dMO 5 p dF 5 p(q ds) 5 q(p ds) 3.13 Thin-Walled Hollow Shafts p ds (3.51) Since the integral around the wall section of the left-hand member of Eq. (3.51) represents the sum of the moments of all the elementary shearing forces exerted on the wall section, and since this sum is equal to the torque T applied to the hollow member, we have O dF d Fig. 3.53 T 5 A dMO 5 A q12dA2 The shear flow q being a constant, we write T 5 2qA (3.52) where A is the area bounded by the center line of the wall cross section (Fig. 3.54). The shearing stress t at any given point of the wall may be expressed in terms of the torque T if we substitute for q from (3.48) into (3.52) and solve for t the equation obtained. We have T t5 2tA t  (3.53) Fig. 3.54 Area for shear flow. 201 bee80288_ch03_140-219.indd Page 202 11/2/10 12:48:33 AM user-f499 202 /Users/user-f499/Desktop/Temp Work/Don't Delete Job/MHDQ251:Beer:201/ch03 where t is the wall thickness at the point considered and A the area bounded by the center line. We recall that t represents the average value of the shearing stress across the wall. However, for elastic deformations the distribution of stresses across the wall may be assumed uniform, and Eq. (3.53) will yield the actual value of the shearing stress at a given point of the wall. The angle of twist of a thin-walled hollow shaft may be obtained by using the method of energy (Chap. 11). Assuming an elastic deformation, it may be shown† that the angle of twist of a thin-walled shaft of length L and modulus of rigidity G is Torsion f5 ds TL 2 4A G C t (3.54) where the integral is computed along the center line of the wall section. EXAMPLE 3.10 4 in. A B 0.160 in. 2.5 in. (a) Tubing of Uniform Wall Thickness. the center line (Fig. 3.56) is 0.160 in. C Structural aluminum tubing of 2.5 3 4-in. rectangular cross section was fabricated by extrusion. Determine the shearing stress in each of the four walls of a portion of such tubing when it is subjected to a torque of 24 kip ? in., assuming (a) a uniform 0.160-in. wall thickness (Fig. 3.55a), (b) that, as a result of defective fabrication, walls AB and AC are 0.120-in. thick, and walls BD and CD are 0.200-in. thick (Fig. 3.55b). D (a) A 5 (3.84 in.)(2.34 in.) 5 8.986 in2 Since the thickness of each of the four walls is t 5 0.160 in., we find from Eq. (3.53) that the shearing stress in each wall is 4 in. A t5 B 24 kip ? in. T 5 5 8.35 ksi 2tA 210.160 in.2 18.986 in2 2 0.120 in. 2.5 in. The area bounded by 3.84 in. A B 0.200 in. t  0.160 in. 2.34 in. D C t  0.160 in. (b) Fig. 3.55 C D Fig. 3.56 (b) Tubing with Variable Wall Thickness. Observing that the area A bounded by the center line is the same as in part a, and substituting successively t 5 0.120 in. and t 5 0.200 in. into Eq. (3.53), we have tAB 5 tAC 5 and tBD 5 tCD 5 24 kip ? in. 210.120 in.2 18.986 in2 2 24 kip ? in. 210.200 in.2 18.986 in2 2 5 11.13 ksi 5 6.68 ksi We note that the stress in a given wall depends only upon its thickness. †See Prob. 11.70. bee80288_ch03_140-219.indd Page 203 9/21/10 3:09:23 PM user-f499 /Users/user-f499/Desktop/Temp Work/Don't Delete Job/MHDQ251:Beer:201/ch03 SAMPLE PROBLEM 3.9 T1 T2 40 mm T3 40 mm 25 mm 6 mm t 64 mm 40 mm 40 mm Using tall 5 40 MPa, determine the largest torque that may be applied to each of the brass bars and to the brass tube shown. Note that the two solid bars have the same crosssectional area, and that the square bar and square tube have the same outside dimensions. (1) (2) (3) SOLUTION 1. Bar with Square Cross Section. For a solid bar of rectangular cross section the maximum shearing stress is given by Eq. (3.43) tmax 5 T c1ab2 where the coefficient c1 is obtained from Table 3.1 in Sec. 3.12. We have a 5 1.00 b a 5 b 5 0.040 m T c1 5 0.208 For tmax 5 tall 5 40 MPa, we have tmax 5 a T1 2 c1ab 40 MPa 5 T1 0.20810.040 m2 3 2. Bar with Rectangular Cross Section. b L a 5 0.064 m b 5 0.025 m T1 5 532 N ? m ◀ We now have a 5 2.56 b Interpolating in Table 3.1: c1 5 0.259 tmax 5 T2 2 c1ab 40 MPa 5 3. Square Tube. by Eq. (3.53) t  6 mm T2 0.25910.064 m2 10.025 m2 2 For a tube of thickness t, the shearing stress is given t5 40 mm 34 mm T2 5 414 N ? m ◀ T 2tA where A is the area bounded by the center line of the cross section. We have A 5 10.034 m2 10.034 m2 5 1.156 3 1023 m2 We substitute t 5 tall 5 40 MPa and t 5 0.006 m and solve for the allowable torque: 34 mm 40 mm t5 T 2tA 40 MPa 5 T3 210.006 m2 11.156 3 1023 m2 2 T3 5 555 N ? m ◀ 203 bee80288_ch03_140-219.indd Page 204 9/21/10 3:09:28 PM user-f499 /Users/user-f499/Desktop/Temp Work/Don't Delete Job/MHDQ251:Beer:201/ch03 PROBLEMS T 3.121 Determine the largest torque T that can be applied to each of the two brass bars shown and the corresponding angle of twist at B, knowing that tall 5 12 ksi and G 5 5.6 3 106 psi. 1 in. B T 1.6 in. B 4 in. 2.4 in. 25 in. A 3.122 Each of the two brass bars shown is subjected to a torque of magnitude T 5 12.5 kip ? in. Knowing that G 5 5.6 3 106 psi, determine for each bar the maximum shearing stress and the angle of twist at B. 3.123 Each of the two aluminum bars shown is subjected to a torque of magnitude T 5 1800 N ? m. Knowing that G 5 26 GPa, determine for each bar the maximum shearing stress and the angle of twist at B. (a) A A (b) Fig. P3.121 and P3.122 60 mm (a) A 60 mm B T 38 mm (b) 95 mm 300 mm B T Fig. P3.123 and P3.124 3.124 Determine the largest torque T that can be applied to each of the two aluminum bars shown and the corresponding angle of twist at B, knowing that tall 5 50 MPa and G 5 26 GPa. 3.125 Determine the largest allowable square cross section of a steel shaft of length 20 ft if the maximum shearing stress is not to exceed 10 ksi when the shaft is twisted through one complete revolution. Use G 5 11.2 3 106 psi. 3.126 Determine the largest allowable length of a stainless steel shaft of 3 3 8 3 4 -in. cross section if the shearing stress is not to exceed 15 ksi when the shaft is twisted through 158. Use G 5 11.2 3 106 psi. 204 bee80288_ch03_140-219.indd Page 205 9/21/10 9:17:25 PM user-f499 /Users/user-f499/Desktop/Temp Work/Don't Delete Job/MHDQ251:Beer:201/ch03 Problems 3.127 The torque T causes a rotation of 28 at end B of the stainless steel bar shown. Knowing that b 5 20 mm and G 5 75 GPa, determine the maximum shearing stress in the bar. 3.128 The torque T causes a rotation of 0.68 at end B of the aluminum bar shown. Knowing that b 5 15 mm and G 5 26 GPa, determine the maximum shearing stress in the bar. 3.129 Two shafts are made of the same material. The cross section of shaft A is a square of side b and that of shaft B is a circle of diameter b. Knowing that the shafts are subjected to the same torque, determine the ratio tAytB of maximum shearing stresses occurring in the shafts. A b B 750 mm 30 mm Fig. P3.127 and P3.128 b b b A Fig. P3.129 B 3.130 Shafts A and B are made of the same material and have the same cross-sectional area, but A has a circular cross section and B has a square cross section. Determine the ratio of the maximum shearing stresses occurring in A and B, respectively, when the two shafts are subjected to the same torque (TA 5 TB). Assume both deformations to be elastic. A B TA TB Fig. P3.130, P3.131 and P3.132 3.131 Shafts A and B are made of the same material and have the same cross-sectional area, but A has a circular cross section and B has a square cross section. Determine the ratio of the maximum torques TA and TB that can be safely applied to A and B, respectively. 3.132 Shafts A and B are made of the same material and have the same length and cross-sectional area, but A has a circular cross section and B has a square cross section. Determine the ratio of the maximum values of the angles fA and fB through which shafts A and B, respectively, can be twisted. T 205 bee80288_ch03_140-219.indd Page 206 9/21/10 3:09:38 PM user-f499 206 /Users/user-f499/Desktop/Temp Work/Don't Delete Job/MHDQ251:Beer:201/ch03 3.133 Each of the three aluminum bars shown is to be twisted through an angle of 28. Knowing that b 5 30 mm, tall 5 50 MPa, and G 5 27 GPa, determine the shortest allowable length of each bar. Torsion T b 3.134 Each of the three steel bars is subjected to a torque as shown. Knowing that the allowable shearing stress is 8 ksi and that b 5 1.4 in., determine the maximum torque T that can be applied to each bar. b T b T 1.2b 3.135 A 36-kip ? in. torque is applied to a 10-ft-long steel angle with an L8 3 8 3 1 cross section. From Appendix C we find that the thickness of the section is 1 in. and that its area is 15 in2. Knowing that G 5 11.2 3 106 psi, determine (a) the maximum shearing stress along line a-a, (b) the angle of twist. 1 in. (a) (b) L8  8  1 8 in. a (c) a 8 in. Fig. P3.133 and P3.134 Fig. P3.135 3.136 A 3-m-long steel angle has an L203 3 152 3 12.7 cross section. From Appendix C we find that the thickness of the section is 12.7 mm and that its area is 4350 mm2. Knowing that tall 5 50 MPa and that G 5 77.2 GPa, and ignoring the effect of stress concentrations, determine (a) the largest torque T that can be applied, (b) the corresponding angle of twist. 3m L203  152  12.7 Fig. P3.136 a a b b W8  31 Fig. P3.137 T 3.137 An 8-ft-long steel member with a W8 3 31 cross section is subjected to a 5-kip ? in. torque. The properties of the rolled-steel section are given in Appendix C. Knowing that G 5 11.2 3 106 psi, determine (a) the maximum shearing stress along line a-a, (b) the maximum shearing stress along line b-b, (c) the angle of twist. (Hint: consider the web and flanges separately and obtain a relation between the torques exerted on the web and a flange, respectively, by expressing that the resulting angles of twist are equal.) bee80288_ch03_140-219.indd Page 207 9/21/10 3:09:47 PM user-f499 /Users/user-f499/Desktop/Temp Work/Don't Delete Job/MHDQ251:Beer:201/ch03 3.138 A 4-m-long steel member has a W310 3 60 cross section. Knowing that G 5 77.2 GPa and that the allowable shearing stress is 40 MPa, determine (a) the largest torque T that can be applied, (b) the corresponding angle of twist. Refer to Appendix C for the dimensions of the cross section and neglect the effect of stress concentrations. (See hint of Prob. 3.137.) Problems T W310  60 3.139 A torque T 5 750 kN ? m is applied to the hollow shaft shown that has a uniform 8-mm wall thickness. Neglecting the effect of stress concentrations, determine the shearing stress at points a and b. 90 mm a 60 b Fig. P3.138 Fig. P3.139 3.140 A torque T 5 5 kN ? m is applied to a hollow shaft having the cross section shown. Neglecting the effect of stress concentrations, determine the shearing stress at points a and b. 10 mm 6 mm a 125 mm 6 mm 10 mm b 75 mm Fig. P3.140 3.141 A 90-N ? m torque is applied to a hollow shaft having the cross section shown. Neglecting the effect of stress concentrations, determine the shearing stress at points a and b. 2 mm 4 mm b 40 mm 4 mm 55 mm a 55 mm Fig. P3.141 207 bee80288_ch03_140-219.indd Page 208 9/21/10 3:09:56 PM user-f499 208 Torsion a 5 mm /Users/user-f499/Desktop/Temp Work/Don't Delete Job/MHDQ251:Beer:201/ch03 3.142 A 5.6 kN ? m-torque is applied to a hollow shaft having the cross section shown. Neglecting the effect of stress concentrations, determine the shearing stress at points a and b. 3.143 A hollow member having the cross section shown is formed from sheet metal of 2-mm thickness. Knowing that the shearing stress must not exceed 3 MPa, determine the largest torque that can be applied to the member. 50 mm 100 mm 5 mm 50 mm 20 mm b 50 mm 8 mm Fig. P3.142 20 mm Fig. P3.143 0.5 in. 0.2 in. 1.5 in. 6 in. 1.5 in. 0.5 in. 0.2 in. 0.2 in. 0.2 in. 5 in. Fig. P3.144 3.144 A hollow brass shaft has the cross section shown. Knowing that the shearing stress must not exceed 12 ksi and neglecting the effect of stress concentrations, determine the largest torque that can be applied to the shaft. 3.145 and 3.146 A hollow member having the cross section shown is to be formed from sheet metal of 0.06-in. thickness. Knowing that a 1250 lb ? in.-torque will be applied to the member, determine the smallest dimension d that can be used if the shearing stress is not to exceed 750 psi. 2 in. 2 in. d 2 in. 0.08 in. a d 2 in. 2 in. 3 in. Fig. P3.145 2.4 in. 2 in. 3 in. Fig. P3.146 1.1 in. 0.12 in. Fig. P3.147 b 3.147 A hollow cylindrical shaft was designed to have a uniform wall thickness of 0.1 in. Defective fabrication, however, resulted in the shaft having the cross section shown. Knowing that a 15 kip ? in.torque is applied to the shaft, determine the shearing stresses at points a and b. bee80288_ch03_140-219.indd Page 209 9/21/10 3:10:06 PM user-f499 /Users/user-f499/Desktop/Temp Work/Don't Delete Job/MHDQ251:Beer:201/ch03 3.148 A cooling tube having the cross section shown is formed from a sheet of stainless steel of 3-mm thickness. The radii c1 5 150 mm and c2 5 100 mm are measured to the center line of the sheet metal. Knowing that a torque of magnitude T 5 3 kN ? m is applied to the tube, determine (a) the maximum shearing stress in the tube, (b) the magnitude of the torque carried by the outer circular shell. Neglect the dimension of the small opening where the outer and inner shells are connected. 3.149 A hollow cylindrical shaft of length L, mean radius cm, and uniform thickness t is subjected to a torque of magnitude T. Consider, on the one hand, the values of the average shearing stress tave and the angle of twist f obtained from the elastic torsion formulas developed in Secs. 3.4 and 3.5 and, on the other hand, the corresponding values obtained from the formulas developed in Sec. 3.13 for thin-walled shafts. (a) Show that the relative error introduced by using the thin-walled-shaft formulas rather than the elastic torsion formulas is the same for tave and f and that the relative error is positive and proportional to the ratio tycm. (b) Compare the percent error corresponding to values of the ratio tycm of 0.1, 0.2, and 0.4. T' L cm T t Fig. P3.149 3.150 Equal torques are applied to thin-walled tubes of the same length L, same thickness t, and same radius c. One of the tubes has been slit lengthwise as shown. Determine (a) the ratio tbyta of the maximum shearing stresses in the tubes, (b) the ratio fbyfa of the angles of twist of the tubes. T' T (a) Fig. P3.150 T' T (b) Problems c1 O c2 Fig. P3.148 209 bee80288_ch03_140-219.indd Page 210 9/21/10 3:10:12 PM user-f499 /Users/user-f499/Desktop/Temp Work/Don't Delete Job/MHDQ251:Beer:201/ch03 REVIEW AND SUMMARY Deformations in circular shafts c O  (a) This chapter was devoted to the analysis and design of shafts subjected to twisting couples, or torques. Except for the last two sections of the chapter, our discussion was limited to circular shafts. In a preliminary discussion [Sec. 3.2], it was pointed out that the distribution of stresses in the cross section of a circular shaft is statically indeterminate. The determination of these stresses, therefore, requires a prior analysis of the deformations occurring in the shaft [Sec. 3.3]. Having demonstrated that in a circular shaft subjected to torsion, every cross section remains plane and undistorted, we derived the following expression for the shearing strain in a small element with sides parallel and perpendicular to the axis of the shaft and at a distance r from that axis: L g5 B A O  L (b) B A' A (c)  O  L Fig. 3.57 Shearing stresses in elastic range (3.2) where f is the angle of twist for a length L of the shaft (Fig. 3.57). Equation (3.2) shows that the shearing strain in a circular shaft varies linearly with the distance from the axis of the shaft. It follows that the strain is maximum at the surface of the shaft, where r is equal to the radius c of the shaft. We wrote cf L gmax 5  rf L g5 r g c max (3.3, 4) Considering shearing stresses in a circular shaft within the elastic range [Sec. 3.4] and recalling Hooke’s law for shearing stress and strain, t 5 Gg, we derived the relation t5 r t c max (3.6) which shows that within the elastic range, the shearing stress t in a circular shaft also varies linearly with the distance from the axis of the shaft. Equating the sum of the moments of the elementary forces exerted on any section of the shaft to the magnitude T of the torque applied to the shaft, we derived the elastic torsion formulas tmax 5 Tc J t5 Tr J (3.9, 10) where c is the radius of the cross section and J its centroidal polar moment of inertia. We noted that J 5 12 pc4 for a solid shaft and J 5 12 p1c42 2 c41 2 for a hollow shaft of inner radius c1 and outer radius c2. 210 bee80288_ch03_140-219.indd Page 211 9/21/10 3:10:21 PM user-f499 /Users/user-f499/Desktop/Temp Work/Don't Delete Job/MHDQ251:Beer:201/ch03 We noted that while the element a in Fig. 3.58 is in pure shear, the element c in the same figure is subjected to normal stresses of the same magnitude, TcyJ, two of the normal stresses being tensile and two compressive. This explains why in a torsion test ductile materials, which generally fail in shear, will break along a plane perpendicular to the axis of the specimen, while brittle materials, which are weaker in tension than in shear, will break along surfaces forming a 458 angle with that axis. In Sec. 3.5, we found that within the elastic range, the angle of twist f of a circular shaft is proportional to the torque T applied to it (Fig. 3.59). Expressing f in radians, we wrote f5 where TL JG Review and Summary T T' a c max  Tc J Tc J Angle of twist (3.16) max If the shaft is subjected to torques at locations other than its ends or consists of several parts of various cross sections and possibly of different materials, the angle of twist of the shaft must be expressed as the algebraic sum of the angles of twist of its component parts [Sample Prob. 3.3]: (3.17) We observed that when both ends of a shaft BE rotate (Fig. 3.60), the angle of twist of the shaft is equal to the difference between the angles of rotation fB and fE of its ends. We also noted that when two shafts AD and BE are connected by gears A and B, the torques applied, respectively, by gear A on shaft AD and by gear B on shaft BE are directly proportional to the radii rA and rB of the two gears— since the forces applied on each other by the gear teeth at C are equal and opposite. On the other hand, the angles fA and fB through which the two gears rotate are inversely proportional to rA and rB— since the arcs CC9 and CC0 described by the gear teeth are equal [Example 3.04 and Sample Prob. 3.4]. If the reactions at the supports of a shaft or the internal torques cannot be determined from statics alone, the shaft is said to be statically indeterminate [Sec. 3.6]. The equilibrium equations obtained from free-body diagrams must then be complemented by relations involving the deformations of the shaft and obtained from the geometry of the problem [Example 3.05, Sample Prob. 3.5]. In Sec. 3.7, we discussed the design of transmission shafts. We first observed that the power P transmitted by a shaft is P 5 2p f T  Fig. 3.58 L 5 length of shaft J 5 polar moment of inertia of cross section G 5 modulus of rigidity of material T iL i f5 a i J iG i 45 (3.20) where T is the torque exerted at each end of the shaft and f the frequency or speed of rotation of the shaft. The unit of frequency is T c  L Fig. 3.59 Fixed end T E D E L A A C C' B C'' B Fig. 3.60 Statically indeterminate shafts Transmission shafts 211 bee80288_ch03_140-219.indd Page 212 11/2/10 9:56:48 PM user-f494 212 /207/BSMB003/work%0/indd%0 the revolution per second (s21) or hertz (Hz). If SI units are used, T is expressed in newton-meters (N ? m) and P in watts (W). If U.S. customary units are used, T is expressed in lb ? ft or lb ? in., and P in ft ? lb/s or in ? lb/s; the power may then be converted into horsepower (hp) through the use of the relation Torsion 1 hp 5 550 ft ? lb/s 5 6600 in ? lb/s To design a shaft to transmit a given power P at a frequency f, you should first solve Eq. (3.20) for T. Carrying this value and the maximum allowable value of t for the material used into the elastic formula (3.9), you will obtain the corresponding value of the parameter Jyc, from which the required diameter of the shaft may be calculated [Examples 3.06 and 3.07]. Stress concentrations A D In Sec. 3.8, we discussed stress concentrations in circular shafts. We saw that the stress concentrations resulting from an abrupt change in the diameter of a shaft can be reduced through the use of a fillet (Fig. 3.61). The maximum value of the shearing stress at the fillet is tmax 5 K d Fig. 3.61 Tc J (3.25) where the stress TcyJ is computed for the smaller-diameter shaft, and where K is a stress-concentration factor. Values of K were plotted in Fig. 3.29 on p. 179 against the ratio ryd, where r is the radius of the fillet, for various values of Dyd. Plastic deformations ␶ O ␶max c ␳ Fig. 3.62 Sections 3.9 through 3.11 were devoted to the discussion of plastic deformations and residual stresses in circular shafts. We first recalled that even when Hooke’s law does not apply, the distribution of strains in a circular shaft is always linear [Sec. 3.9]. If the shearing-stressstrain diagram for the material is known, it is then possible to plot the shearing stress t against the distance r from the axis of the shaft for any given value of tmax (Fig. 3.62). Summing the contributions to the torque of annular elements of radius r and thickness dr, we expressed the torque T as T5 # c 0 Modulus of rupture ␶ RT ␶U O Fig. 3.63 c ␳ rt12pr dr2 5 2p c # r t dr 2 (3.26) 0 where t is the function of r plotted in Fig. 3.62. An important value of the torque is the ultimate torque TU which causes failure of the shaft. This value can be determined, either experimentally, or by carrying out the computations indicated above with tmax chosen equal to the ultimate shearing stress tU of the material. From TU, and assuming a linear stress distribution (Fig 3.63), we determined the corresponding fictitious stress RT 5 TU cyJ, known as the modulus of rupture in torsion of the given material. Considering the idealized case of a solid circular shaft made of an elastoplastic material [Sec. 3.10], we first noted that, as long as bee80288_ch03_140-219.indd Page 213 9/21/10 3:10:34 PM user-f499 /Users/user-f499/Desktop/Temp Work/Don't Delete Job/MHDQ251:Beer:201/ch03 Review and Summary tmax does not exceed the yield strength tY of the material, the stress distribution across a section of the shaft is linear (Fig. 3.64a). The torque TY corresponding to tmax 5 tY (Fig. 3.64b) is known as the maximum elastic torque; for a solid circular shaft of radius c, we have TY 5 12pc3tY (3.29) max  Y max O Y Y Y O c O c (b) (a) Fig. 3.64 Y (c) As the torque increases, a plastic region develops in the shaft around an elastic core of radius rY. The torque T corresponding to a given value of rY was found to be T5 4 1 r3Y TY a1 2 b 3 4 c3 (3.32) We noted that as rY approaches zero, the torque approaches a limiting value Tp, called the plastic torque of the shaft considered: Tp 5 4 TY 3 (3.33) Plotting the torque T against the angle of twist f of a solid circular shaft (Fig. 3.65), we obtained the segment of straight line 0Y defined by Eq. (3.16), followed by a curve approaching the straight line T 5 Tp and defined by the equation 4 1 f3Y T 5 TY a1 2 b 3 4 f3 (3.37) T Tp  213 4 3 TY Y TY 0 Fig. 3.65 Y 2 Y 3 Y  c O c (d) Solid shaft of elastoplastic material bee80288_ch03_140-219.indd Page 214 9/21/10 3:10:39 PM user-f499 214 Torsion Permanent deformation. Residual stresses /Users/user-f499/Desktop/Temp Work/Don't Delete Job/MHDQ251:Beer:201/ch03 Loading a circular shaft beyond the onset of yield and unloading it [Sec. 3.11] results in a permanent deformation characterized by the angle of twist fp 5 f 2 f9, where f corresponds to the loading phase described in the previous paragraph, and f9 to the unloading phase represented by a straight line in Fig. 3.66. There will also be residual stresses in the shaft, which can be determined by adding the maximum stresses reached during the loading phase and the reverse stresses corresponding to the unloading phase [Example 3.09]. T TY T 0  p   Fig. 3.66 Torsion of noncircular members T T' Fig. 3.67 Bars of rectangular cross section max a T' T b L Fig. 3.68 Thin-walled hollow shafts t  The last two sections of the chapter dealt with the torsion of noncircular members. We first recalled that the derivation of the formulas for the distribution of strain and stress in circular shafts was based on the fact that due to the axisymmetry of these members, cross sections remain plane and undistorted. Since this property does not hold for noncircular members, such as the square bar of Fig. 3.67, none of the formulas derived earlier can be used in their analysis [Sec. 3.12]. It was indicated in Sec. 3.12 that in the case of straight bars with a uniform rectangular cross section (Fig. 3.68), the maximum shearing stress occurs along the center line of the wider face of the bar. Formulas for the maximum shearing stress and the angle of twist were given without proof. The membrane analogy for visualizing the distribution of stresses in a noncircular member was also discussed. We next analyzed the distribution of stresses in noncircular thin-walled hollow shafts [Sec. 3.13]. We saw that the shearing stress is parallel to the wall surface and varies both across the wall and along the wall cross section. Denoting by t the average value of the shearing stress computed across the wall at a given point of the cross section, and by t the thickness of the wall at that point (Fig. 3.69), we showed that the product q 5 tt, called the shear flow, is constant along the cross section. Furthermore, denoting by T the torque applied to the hollow shaft and by A the area bounded by the center line of the wall cross section, we expressed as follows the average shearing stress t at any given point of the cross section: t5 Fig. 3.69 T 2tA (3.53) bee80288_ch03_140-219.indd Page 215 9/21/10 3:11:46 PM user-f499 /Users/user-f499/Desktop/Temp Work/Don't Delete Job/MHDQ251:Beer:201/ch03 REVIEW PROBLEMS 3.151 The ship at A has just started to drill for oil on the ocean floor at a depth of 5000 ft. Knowing that the top of the 8-in.-diameter steel drill pipe (G 5 11.2 3 10 6 psi) rotates through two complete revolutions before the drill bit at B starts to operate, determine the maximum shearing stress caused in the pipe by torsion. A 5000 ft 3.152 The shafts of the pulley assembly shown are to be designed. Knowing that the allowable shearing stress in each shaft is 8.5 ksi, determine the smallest allowable diameter of (a) shaft AB, (b) shaft BC. B Fig. P3.151 6.8 kip · in. 10.4 kip · in. C 3.6 kip · in. 72 in. B A 48 in. Fig. P3.152 3.153 A steel pipe of 12-in. outer diameter is fabricated from 14-in.-thick plate by welding along a helix that forms an angle of 458 with a plane perpendicular to the axis of the pipe. Knowing that the maximum allowable tensile stress in the weld is 12 ksi, determine the largest torque that can be applied to the pipe. T' 12 in. 45⬚ 1 4 in. T Fig. P3.153 215 bee80288_ch03_140-219.indd Page 216 9/21/10 3:12:38 PM user-f499 216 /Users/user-f499/Desktop/Temp Work/Don't Delete Job/MHDQ251:Beer:201/ch03 3.154 For the gear train shown, the diameters of the three solid shafts Torsion are: dAB 5 20 mm dCD 5 25 mm dEF 5 40 mm Knowing that for each shaft the allowable shearing stress is 60 MPa, determine the largest torque T that can be applied. B 30 mm A C T 75 mm D 30 mm F 90 mm E 250 mm 200 mm C Fig. P3.154 B 38 mm A 50 mm Fig. P3.155 1.4 kN · m 3.155 Two solid steel shafts (G 5 77.2 GPa) are connected to a cou- pling disk B and to fixed supports at A and C. For the loading shown, determine (a) the reaction at each support, (b) the maximum shearing stress in shaft AB, (c) the maximum shearing stress in shaft BC. 3.156 In the bevel-gear system shown, a 5 18.438. Knowing that the allowable shearing stress is 8 ksi in each shaft and that the system is in equilibrium, determine the largest torque TA that can be applied at A. 0.5 in. ␣ ␣ C 0.625 in. B TB Fig. P3.156 A TA bee80288_ch03_140-219.indd Page 217 9/21/10 9:21:14 PM user-f499 /Users/user-f499/Desktop 3.157 Three solid shafts, each of 34-in. diameter, are connected by the Review Problems gears as shown. Knowing that G 5 11.2 3 106 psi, determine (a) the angle through which end A of shaft AB rotates, (b) the angle through which end E of shaft EF rotates. 3 ft 4 ft B r  1.5 in. D TA ⫽ 100 lb · in. 6 in. A C4 in. 2 in. F TE ⫽ 200 lb · in. E Fig. P3.157 3.158 The design specifications of a 1.2-m-long solid transmission shaft require that the angle of twist of the shaft not exceed 48 when a torque of 750 N ? m is applied. Determine the required diameter of the shaft, knowing that the shaft is made of a steel with an allowable shearing stress of 90 MPa and a modulus of rigidity of 77.2 GPa. 5 in. 6 in. r Fig. P3.159 a 3.159 The stepped shaft shown rotates at 450 rpm. Knowing that r 5 0.5 in., determine the maximum power that can be transmitted without exceeding an allowable shearing stress of 7500 psi. 30 mm 3.160 A 750-N ? m torque is applied to a hollow shaft having the cross section shown and a uniform 6-mm wall thickness. Neglecting the effect of stress concentrations, determine the shearing stress at points a and b. 60 mm b 3.161 The composite shaft shown is twisted by applying a torque T at end A. Knowing that the maximum shearing stress in the steel shell is 150 MPa, determine the corresponding maximum shearing stress in the aluminum core. Use G 5 77.2 GPa for steel and G 5 27 GPa for aluminum. 3.162 Two solid brass rods AB and CD are brazed to a brass sleeve EF. 30 mm Fig. P3.160 Determine the ratio d2yd1 for which the same maximum shearing stress occurs in the rods and in the sleeve. F d2 d1 D E T' 30 mm A T C B T Aluminum A Fig. P3.162 B 40 mm Fig. P3.161 Steel 2m 217 bee80288_ch03_140-219.indd Page 218 9/21/10 3:13:00 PM user-f499 /Users/user-f499/Desktop/Temp Work/Don't Delete Job/MHDQ251:Beer:201/ch03 COMPUTER PROBLEMS The following problems are designed to be solved with a computer. Write each program so that it can be used with either SI or U.S. customary units. 3.C1 Shaft AB consists of n homogeneous cylindrical elements, which can be solid or hollow. Its end A is fixed, while its end B is free, and it is subjected to the loading shown. The length of element i is denoted by Li, its outer diameter by ODi, its inner diameter by IDi, its modulus of rigidity by Gi, and the torque applied to its right end by Ti, the magnitude Ti of this torque being assumed to be positive if Ti is observed as counterclockwise from end B and negative otherwise. (Note that IDi 5 0 if the element is solid.) (a) Write a computer program that can be used to determine the maximum shearing stress in each element, the angle of twist of each element, and the angle of twist of the entire shaft. (b) Use this program to solve Probs. 3.35 and 3.38. Element n Bn A an Tn An Element 1 bn –1 a2 B A2 Fig. P3.C1 B1 T0 B2 b1 A1 Fig. P3.C2 Element n A Element 1 Tn T2 Fig. P3.C3 218 T1 B 3.C2 The assembly shown consists of n cylindrical shafts, which can be solid or hollow, connected by gears and supported by brackets (not shown). End A1 of the first shaft is free and is subjected to a torque T0, while end Bn of the last shaft is fixed. The length of shaft AiBi is denoted by Li, its outer diameter by ODi, its inner diameter by IDi, and its modulus of rigidity by Gi. (Note that IDi 5 0 if the element is solid.) The radius of gear Ai is denoted by ai, and the radius of gear Bi by bi. (a) Write a computer program that can be used to determine the maximum shearing stress in each shaft, the angle of twist of each shaft, and the angle through which end Ai rotates. (b) Use this program to solve Probs. 3.41 and 3.44. 3.C3 Shaft AB consists of n homogeneous cylindrical elements, which can be solid or hollow. Both of its ends are fixed, and it is subjected to the loading shown. The length of element i is denoted by Li, its outer diameter by ODi, its inner diameter by IDi, its modulus of rigidity by Gi, and the torque applied to its right end by Ti, the magnitude Ti of this torque being assumed to be positive if Ti is observed as counterclockwise from end B and negative otherwise. Note that IDi 5 0 if the element is solid and also that T1 5 0. Write a computer program that can be used to determine the reactions at A and B, the maximum shearing stress in each element, and the angle of twist of each element. Use this program (a) to solve Prob. 3.155, (b) to determine the maximum shearing stress in the shaft of Example 3.05. bee80288_ch03_140-219.indd Page 219 9/21/10 3:13:10 PM user-f499 /Users/user-f499/Desktop/Temp Work/Don't Delete Job/MHDQ251:Beer:201/ch03 Computer Problems 3.C4 The homogeneous, solid cylindrical shaft AB has a length L, a diameter d, a modulus of rigidity G, and a yield strength tY. It is subjected to a torque T that is gradually increased from zero until the angle of twist of the shaft has reached a maximum value fm and then decreased back to zero. (a) Write a computer program that, for each of 16 values of fm equally spaced over a range extending from 0 to a value 3 times as large as the angle of twist at the onset of yield, can be used to determine the maximum value Tm of the torque, the radius of the elastic core, the maximum shearing stress, the permanent twist, and the residual shearing stress both at the surface of the shaft and at the interface of the elastic core and the plastic region. (b) Use this program to obtain approximate answers to Probs. 3.114, 3.115, 3.116. L A B T Fig. P3.C4 3.C5 The exact expression is given in Prob. 3.61 for the angle of twist of the solid tapered shaft AB when a torque T is applied as shown. Derive an approximate expression for the angle of twist by replacing the tapered shaft by n cylindrical shafts of equal length and of radius ri 5 1n 1 i 2 12 2(cyn), where i 5 1, 2, . . ., n. Using for T, L, G, and c values of your choice, determine the percentage error in the approximate expression when (a) n 5 4, (b) n 5 8, (c) n 5 20, (d) n 5 100. T T A c A A L/n L c r1 ri L 2c B T c t rn B A 2c Fig. P3.C5 3.C6 A torque T is applied as shown to the long, hollow, tapered shaft AB of uniform thickness t. Derive an approximate expression for the angle of twist by replacing the tapered shaft by n cylindrical rings of equal length and of radius ri 5 1n 1 i 2 12 2(cyn), where i 5 1, 2, . . ., n. Using for T, L, G, c, and t values of your choice, determine the percentage error in the approximate expression when (a) n 5 4, (b) n 5 8, (c) n 5 20, (d) n 5 100. L 2c B Fig. P3.C6 219 bee80288_ch04_220-313.indd Page 220 10/26/10 4:21:38 PM user-f499 The athlete shown holds the barbell with his hands placed at equal distances from the weights. This results in pure bending in the center portion of the bar. The normal stresses and the curvature resulting from pure bending will be determined in this chapter. 220 /Volumes/201/MHDQ251/bee80288_disk1of1/0073380288/bee80288_pagefiles bee80288_ch04_220-313.indd Page 221 10/26/10 4:24:22 PM user-f499 /Volumes/201/MHDQ251/bee80288_disk1of1/0073380288/bee80288_pagefiles C H A P T E R Pure Bending 221 bee80288_ch04_220-313.indd Page 222 11/11/10 3:03:05 PM user-f499 4.1 Chapter 4 Pure Bending 4.1 4.2 4.3 4.4 4.5 4.6 4.7 *4.8 *4.9 *4.10 *4.11 4.12 4.13 4.14 *4.15 Introduction Symmetric Member in Pure Bending Deformations in a Symmetric Member in Pure Bending Stresses and Deformations in the Elastic Range Deformations in a Transverse Cross Section Bending of Members Made of Several Materials Stress Concentrations Plastic Deformations Members Made of Elastoplastic Material Plastic Deformations of Members with a Single Plane of Symmetry Residual Stresses Eccentric Axial Loading in a Plane of Symmetry Unsymmetric Bending General Case of Eccentric Axial Loading Bending of Curved Members 80 lb 80 lb 12 in. 26 in. C A RC = 80 lb 12 in. D (a) B RD = 80 lb D C M = 960 lb · in. /Users/user-f499/Desktop/Temp Work/Don't Delete Job/MHDQ251:Beer:201/ch04 INTRODUCTION In the preceding chapters you studied how to determine the stresses in prismatic members subjected to axial loads or to twisting couples. In this chapter and in the following two you will analyze the stresses and strains in prismatic members subjected to bending. Bending is a major concept used in the design of many machine and structural components, such as beams and girders. This chapter will be devoted to the analysis of prismatic members subjected to equal and opposite couples M and M9 acting in the same longitudinal plane. Such members are said to be in pure bending. In most of the chapter, the members will be assumed to possess a plane of symmetry and the couples M and M9 to be acting in that plane (Fig. 4.1). M' M A B Fig. 4.1 Member in pure bending. An example of pure bending is provided by the bar of a typical barbell as it is held overhead by a weight lifter as shown in the opening photo for this chapter. The bar carries equal weights at equal distances from the hands of the weight lifter. Because of the symmetry of the free-body diagram of the bar (Fig. 4.2a), the reactions at the hands must be equal and opposite to the weights. Therefore, as far as the middle portion CD of the bar is concerned, the weights and the reactions can be replaced by two equal and opposite 960-lb ? in. couples (Fig. 4.2b), showing that the middle portion of the bar is in pure bending. A similar analysis of the axle of a small sport buggy (Photo 4.1) would show that, between the two points where it is attached to the frame, the axle is in pure bending. M' = 960 lb · in. (b) Fig. 4.2 Beam in which portion CD is in pure bending. Photo 4.1 For the sport buggy shown, the center portion of the rear axle is in pure bending. 222 bee80288_ch04_220-313.indd Page 223 10/26/10 4:24:41 PM user-f499 /Volumes/201/MHDQ251/bee80288_disk1of1/0073380288/bee80288_pagefiles 4.1 Introduction As interesting as the direct applications of pure bending may be, devoting an entire chapter to its study would not be justified if it were not for the fact that the results obtained will be used in the analysis of other types of loadings as well, such as eccentric axial loadings and transverse loadings. Photo 4.2 shows a 12-in. steel bar clamp used to exert 150-lb forces on two pieces of lumber as they are being glued together. Figure 4.3a shows the equal and opposite forces exerted by the lumber on the clamp. These forces result in an eccentric loading of the straight portion of the clamp. In Fig. 4.3b a section CC9 has been passed through the clamp and a free-body diagram has been drawn of the upper half of the clamp, from which we conclude that the internal forces in the section are equivalent to a 150-lb axial tensile force P and a 750-lb ? in. couple M. We can thus combine our knowledge of the stresses under a centric load and the results of our forthcoming analysis of stresses in pure bending to obtain the distribution of stresses under an eccentric load. This will be further discussed in Sec. 4.12. 5 in. C C' 5 in. P'  150 lb P  150 lb P'  150 lb C C' M  750 lb · in. P  150 lb (a) Fig. 4.3 Photo 4.2 Clamp used to glue lumber pieces together. (b) Forces exerted on clamp. The study of pure bending will also play an essential role in the study of beams, i.e., the study of prismatic members subjected to various types of transverse loads. Consider, for instance, a cantilever beam AB supporting a concentrated load P at its free end (Fig. 4.4a). If we pass a section through C at a distance x from A, we observe from the free-body diagram of AC (Fig. 4.4b) that the internal forces in the section consist of a force P9 equal and opposite to P and a couple M of magnitude M 5 Px. The distribution of normal stresses in the section can be obtained from the couple M as if the beam were in pure bending. On the other hand, the shearing stresses in the section depend on the force P9, and you will learn in Chap. 6 how to determine their distribution over a given section. The first part of the chapter is devoted to the analysis of the stresses and deformations caused by pure bending in a homogeneous member possessing a plane of symmetry and made of a material following Hooke’s law. In a preliminary discussion of the stresses due to bending (Sec. 4.2), the methods of statics will be used to derive P L C A B (a) P x C M A (b) P' Fig. 4.4 Cantilever beam, not in pure bending. 223 bee80288_ch04_220-313.indd Page 224 10/26/10 4:24:49 PM user-f499 224 Pure Bending M' M A C B /Volumes/201/MHDQ251/bee80288_disk1of1/0073380288/bee80288_pagefiles three fundamental equations which must be satisfied by the normal stresses in any given cross section of the member. In Sec. 4.3, it will be proved that transverse sections remain plane in a member subjected to pure bending, while in Sec. 4.4 formulas will be developed that can be used to determine the normal stresses, as well as the radius of curvature for that member within the elastic range. In Sec. 4.6, you will study the stresses and deformations in composite members made of more than one material, such as reinforcedconcrete beams, which utilize the best features of steel and concrete and are extensively used in the construction of buildings and bridges. You will learn to draw a transformed section representing the section of a member made of a homogeneous material that undergoes the same deformations as the composite member under the same loading. The transformed section will be used to find the stresses and deformations in the original composite member. Section 4.7 is devoted to the determination of the stress concentrations occurring at locations where the cross section of a member undergoes a sudden change. In the next part of the chapter you will study plastic deformations in bending, i.e., the deformations of members which are made of a material which does not follow Hooke’s law and are subjected to bending. After a general discussion of the deformations of such members (Sec. 4.8), you will investigate the stresses and deformations in members made of an elastoplastic material (Sec. 4.9). Starting with the maximum elastic moment MY, which corresponds to the onset of yield, you will consider the effects of increasingly larger moments until the plastic moment Mp is reached, at which time the member has yielded fully. You will also learn to determine the permanent deformations and residual stresses that result from such loadings (Sec. 4.11). It should be noted that during the past half-century the elastoplastic property of steel has been widely used to produce designs resulting in both improved safety and economy. In Sec. 4.12, you will learn to analyze an eccentric axial loading in a plane of symmetry, such as the one shown in Fig. 4.4, by superposing the stresses due to pure bending and the stresses due to a centric axial loading. Your study of the bending of prismatic members will conclude with the analysis of unsymmetric bending (Sec. 4.13), and the study of the general case of eccentric axial loading (Sec. 4.14). The final section of the chapter will be devoted to the determination of the stresses in curved members (Sec. 4.15). (a) 4.2 M' M A C (b) Fig. 4.5 Member in pure bending. SYMMETRIC MEMBER IN PURE BENDING Consider a prismatic member AB possessing a plane of symmetry and subjected to equal and opposite couples M and M9 acting in that plane (Fig. 4.5a). We observe that if a section is passed through the member AB at some arbitrary point C, the conditions of equilibrium of the portion AC of the member require that the internal forces in the section be equivalent to the couple M (Fig. 4.5b). Thus, the internal forces in any cross section of a symmetric member in pure bending are equivalent to a couple. The moment M of that couple bee80288_ch04_220-313.indd Page 225 10/26/10 4:24:51 PM user-f499 /Volumes/201/MHDQ251/bee80288_disk1of1/0073380288/bee80288_pagefiles is referred to as the bending moment in the section. Following the usual convention, a positive sign will be assigned to M when the member is bent as shown in Fig. 4.5a, i.e., when the concavity of the beam faces upward, and a negative sign otherwise. Denoting by sx the normal stress at a given point of the cross section and by txy and txz the components of the shearing stress, we express that the system of the elementary internal forces exerted on the section is equivalent to the couple M (Fig. 4.6). y y  xydA  xzdA z M = z xdA x x z y Fig. 4.6 We recall from statics that a couple M actually consists of two equal and opposite forces. The sum of the components of these forces in any direction is therefore equal to zero. Moreover, the moment of the couple is the same about any axis perpendicular to its plane, and is zero about any axis contained in that plane. Selecting arbitrarily the z axis as shown in Fig. 4.6, we express the equivalence of the elementary internal forces and of the couple M by writing that the sums of the components and of the moments of the elementary forces are equal to the corresponding components and moments of the couple M: x components: esx dA 5 0 (4.1) moments about y axis: ezsx dA 5 0 (4.2) moments about z axis: e(2ysx dA) 5 M (4.3) Three additional equations could be obtained by setting equal to zero the sums of the y components, z components, and moments about the x axis, but these equations would involve only the components of the shearing stress and, as you will see in the next section, the components of the shearing stress are both equal to zero. Two remarks should be made at this point: (1) The minus sign in Eq. (4.3) is due to the fact that a tensile stress (sx . 0) leads to a negative moment (clockwise) of the normal force sx dA about the z axis. (2) Equation (4.2) could have been anticipated, since the application of couples in the plane of symmetry of member AB will result in a distribution of normal stresses that is symmetric about the y axis. Once more, we note that the actual distribution of stresses in a given cross section cannot be determined from statics alone. It is statically indeterminate and may be obtained only by analyzing the deformations produced in the member. 4.2 Symmetric Member in Pure Bending 225 bee80288_ch04_220-313.indd Page 226 10/26/10 4:24:54 PM user-f499 226 4.3 Pure Bending /Volumes/201/MHDQ251/bee80288_disk1of1/0073380288/bee80288_pagefiles DEFORMATIONS IN A SYMMETRIC MEMBER IN PURE BENDING Let us now analyze the deformations of a prismatic member possessing a plane of symmetry and subjected at its ends to equal and opposite couples M and M9 acting in the plane of symmetry. The member will bend under the action of the couples, but will remain symmetric with respect to that plane (Fig. 4.7). Moreover, since the bending moment M is the same in any cross section, the member will bend uniformly. Thus, the line AB along which the upper face of the member intersects the plane of the couples will have a constant curvature. In other words, the line AB, which was originally a straight line, will be transformed into a circle of center C, and so will the line A9B9 (not shown in the figure) along which the lower face of the member intersects the plane of symmetry. We also note that the line AB will decrease in length when the member is bent as shown in the figure, i.e., when M . 0, while A9B9 will become longer. C Mⴕ M B A D B⬘ Fig. 4.7 Deformation of member in pure bending. D A B E E⬘ E E⬘ (a) C M' M B A D EE⬘ (b) Fig. 4.8 Next we will prove that any cross section perpendicular to the axis of the member remains plane, and that the plane of the section passes through C. If this were not the case, we could find a point E of the original section through D (Fig. 4.8a) which, after the member has been bent, would not lie in the plane perpendicular to the plane of symmetry that contains line CD (Fig. 4.8b). But, because of the symmetry of the member, there would be another point E9 that would be transformed exactly in the same way. Let us assume that, after the beam has been bent, both points would be located to the left of the plane defined by CD, as shown in Fig. 4.8b. Since the bending moment M is the same throughout the member, a similar situation would prevail in any other cross section, and the points corresponding to E and E9 would also move to the left. Thus, an observer at A would conclude that the loading causes the points E and E9 in the various cross sections to move forward (toward the observer). But an observer at B, to whom the loading looks the same, and who observes the points E and E9 in the same positions (except that they are now inverted) would reach the opposite conclusion. This inconsistency leads us to conclude that E and E9 will lie in the plane defined by CD and, therefore, that the section remains plane and passes through C. We should note, bee80288_ch04_220-313.indd Page 227 10/26/10 4:24:59 PM user-f499 /Volumes/201/MHDQ251/bee80288_disk1of1/0073380288/bee80288_pagefiles however, that this discussion does not rule out the possibility of deformations within the plane of the section (see Sec. 4.5). Suppose that the member is divided into a large number of small cubic elements with faces respectively parallel to the three coordinate planes. The property we have established requires that these elements be transformed as shown in Fig. 4.9 when the member is subjected to the couples M and M9. Since all the faces represented in the two projections of Fig. 4.9 are at 908 to each other, we conclude that gxy 5 gzx 5 0 and, thus, that txy 5 txz 5 0. Regarding the three stress components that we have not yet discussed, namely, sy, sz, and tyz, we note that they must be zero on the surface of the member. Since, on the other hand, the deformations involved do not require any interaction between the elements of a given transverse cross section, we can assume that these three stress components are equal to zero throughout the member. This assumption is verified, both from experimental evidence and from the theory of elasticity, for slender members undergoing small deformations.† We conclude that the only nonzero stress component exerted on any of the small cubic elements considered here is the normal component sx. Thus, at any point of a slender member in pure bending, we have a state of uniaxial stress. Recalling that, for M . 0, lines AB and A9B9 are observed, respectively, to decrease and increase in length, we note that the strain Px and the stress sx are negative in the upper portion of the member (compression) and positive in the lower portion (tension). It follows from the above that there must exist a surface parallel to the upper and lower faces of the member, where Px and sx are zero. This surface is called the neutral surface. The neutral surface intersects the plane of symmetry along an arc of circle DE (Fig. 4.10a), and it intersects a transverse section along a straight line called the neutral axis of the section (Fig. 4.10b). The origin of coordinates will now be selected on the neutral surface, rather than on the lower face of the member as done earlier, so that the distance from any point to the neutral surface will be measured by its coordinate y. C   –y y y B K A J D A⬘ O x Neutral axis y E B⬘ z (a) Longitudinal, vertical section (plane of symmetry) Fig. 4.10 Deformation with respect to neutral axis. †Also see Prob. 4.32. c O y (b) Transverse section 4.3 Deformations in a Symmetric Member in Pure Bending y C M' A B A⬘ M B⬘ x (a) Longitudinal, vertical section (plane of symmetry) M' x M z (b) Longitudinal, horizontal section Fig. 4.9 bending. Member subject to pure 227 bee80288_ch04_220-313.indd Page 228 11/11/10 3:03:06 PM user-f499 228 Pure Bending /Users/user-f499/Desktop/Temp Work/Don't Delete Job/MHDQ251:Beer:201/ch04 Denoting by r the radius of arc DE (Fig. 4.10a), by u the central angle corresponding to DE, and observing that the length of DE is equal to the length L of the undeformed member, we write L 5 ru (4.4) Considering now the arc JK located at a distance y above the neutral surface, we note that its length L9 is L9 5 (r 2 y)u (4.5) Since the original length of arc JK was equal to L, the deformation of JK is (4.6) d 5 L9 2 L or, if we substitute from (4.4) and (4.5) into (4.6), d 5 (r 2 y)u 2 ru 5 2yu (4.7) The longitudinal strain Px in the elements of JK is obtained by dividing d by the original length L of JK. We write Px 5 2yu d 5 ru L or Px 5 2 y r (4.8) The minus sign is due to the fact that we have assumed the bending moment to be positive and, thus, the beam to be concave upward. Because of the requirement that transverse sections remain plane, identical deformations will occur in all planes parallel to the plane of symmetry. Thus the value of the strain given by Eq. (4.8) is valid anywhere, and we conclude that the longitudinal normal strain Px varies linearly with the distance y from the neutral surface. The strain Px reaches its maximum absolute value when y itself is largest. Denoting by c the largest distance from the neutral surface (which corresponds to either the upper or the lower surface of the member), and by Pm the maximum absolute value of the strain, we have Pm 5 c r (4.9) Solving (4.9) for r and substituting the value obtained into (4.8), we can also write y Px 5 2 Pm (4.10) c We conclude our analysis of the deformations of a member in pure bending by observing that we are still unable to compute the strain or stress at a given point of the member, since we have not yet located the neutral surface in the member. In order to locate this surface, we must first specify the stress-strain relation of the material used.† †Let us note, however, that if the member possesses both a vertical and a horizontal plane of symmetry (e.g., a member with a rectangular cross section), and if the stress-strain curve is the same in tension and compression, the neutral surface will coincide with the plane of symmetry (cf. Sec. 4.8). bee80288_ch04_220-313.indd Page 229 11/11/10 3:03:07 PM user-f499 4.4 /Users/user-f499/Desktop/Temp Work/Don't Delete Job/MHDQ251:Beer:201/ch04 4.4 Stresses and Deformations in the Elastic Range STRESSES AND DEFORMATIONS IN THE ELASTIC RANGE We now consider the case when the bending moment M is such that the normal stresses in the member remain below the yield strength sY. This means that, for all practical purposes, the stresses in the member will remain below the proportional limit and the elastic limit as well. There will be no permanent deformation, and Hooke’s law for uniaxial stress applies. Assuming the material to be homogeneous, and denoting by E its modulus of elasticity, we have in the longitudinal x direction (4.11) sx 5 EPx Recalling Eq. (4.10), and multiplying both members of that equation by E, we write y EPx 5 2 1EPm 2 c or, using (4.11), y sx 5 2 sm c (4.12) where sm denotes the maximum absolute value of the stress. This result shows that, in the elastic range, the normal stress varies linearly with the distance from the neutral surface (Fig. 4.11). It should be noted that, at this point, we do not know the location of the neutral surface, nor the maximum value sm of the stress. Both can be found if we recall the relations (4.1) and (4.3) which were obtained earlier from statics. Substituting first for sx from (4.12) into (4.1), we write #s x dA 5 # y sm a2 s m b dA 5 2 c c # y dA 5 0 from which it follows that # y dA 5 0 (4.13) This equation shows that the first moment of the cross section about its neutral axis must be zero.† In other words, for a member subjected to pure bending, and as long as the stresses remain in the elastic range, the neutral axis passes through the centroid of the section. We now recall Eq. (4.3), which was derived in Sec. 4.2 with respect to an arbitrary horizontal z axis, # 12ys dA2 5 M x (4.3) Specifying that the z axis should coincide with the neutral axis of the cross section, we substitute for sx from (4.12) into (4.3) and write y # 12y2 a2 c s mb dA 5 M †See Appendix A for a discussion of the moments of areas. m y c Neutral surface Fig. 4.11 Bending stresses. x 229 bee80288_ch04_220-313.indd Page 230 11/11/10 3:03:07 PM user-f499 230 Pure Bending /Users/user-f499/Desktop/Temp Work/Don't Delete Job/MHDQ251:Beer:201/ch04 or sm c # y dA 5 M 2 (4.14) Recalling that in the case of pure bending the neutral axis passes through the centroid of the cross section, we note that I is the moment of inertia, or second moment, of the cross section with respect to a centroidal axis perpendicular to the plane of the couple M. Solving (4.14) for sm, we write therefore† sm 5 Mc I (4.15) Substituting for sm from (4.15) into (4.12), we obtain the normal stress sx at any distance y from the neutral axis: sx 5 2 My (4.16) I Equations (4.15) and (4.16) are called the elastic flexure formulas, and the normal stress sx caused by the bending or “flexing” of the member is often referred to as the flexural stress. We verify that the stress is compressive (sx , 0) above the neutral axis (y . 0) when the bending moment M is positive, and tensile (sx . 0) when M is negative. Returning to Eq. (4.15), we note that the ratio Iyc depends only upon the geometry of the cross section. This ratio is called the elastic section modulus and is denoted by S. We have Elastic section modulus 5 S 5 I c (4.17) Substituting S for Iyc into Eq. (4.15), we write this equation in the alternative form sm 5 M S (4.18) Since the maximum stress sm is inversely proportional to the elastic section modulus S, it is clear that beams should be designed with as large a value of S as practicable. For example, in the case of a wooden beam with a rectangular cross section of width b and depth h, we have S5 1 3 I 12 bh 5 5 16 bh2 5 16 Ah c hy2 (4.19) †We recall that the bending moment was assumed to be positive. If the bending moment is negative, M should be replaced in Eq. (4.15) by its absolute value |M|. bee80288_ch04_220-313.indd Page 231 10/28/10 9:25:20 PM user-f499 /Volumes/201/MHDQ251/bee80288_disk1of1/0073380288/bee80288_pagefiles 4.4 Stresses and Deformations in the Elastic Range where A is the cross-sectional area of the beam. This shows that, of two beams with the same cross-sectional area A (Fig. 4.12), the beam with the larger depth h will have the larger section modulus and, thus, will be the more effective in resisting bending.† In the case of structural steel, American standard beams (S-beams) and wide-flange beams (W-beams), Photo 4.3, are preferred A ⫽ 24 in2 h ⫽ 8 in. h ⫽ 6 in. b ⫽ 4 in. Fig. 4.12 Photo 4.3 Wide-flange steel beams form the frame of many buildings. to other shapes because a large portion of their cross section is located far from the neutral axis (Fig. 4.13). Thus, for a given crosssectional area and a given depth, their design provides large values c N. A. c (a) S-beam Fig. 4.13 (b) W-beam Steel beam cross sections. †However, large values of the ratio hyb could result in lateral instability of the beam. b ⫽ 3 in. Wood beam cross sections. 231 bee80288_ch04_220-313.indd Page 232 10/27/10 5:46:11 PM user-f499 /Users/user-f499/Desktop/Temp Work/Don't Delete Job/MHDQ251:Beer:201/ch04 of I and, consequently, of S. Values of the elastic section modulus of commonly manufactured beams can be obtained from tables listing the various geometric properties of such beams. To determine the maximum stress sm in a given section of a standard beam, the engineer needs only to read the value of the elastic section modulus S in a table, and divide the bending moment M in the section by S. The deformation of the member caused by the bending moment M is measured by the curvature of the neutral surface. The curvature is defined as the reciprocal of the radius of curvature r, and can be obtained by solving Eq. (4.9) for 1yr: Pm 1 5 r c (4.20) But, in the elastic range, we have Pm 5 smyE. Substituting for Pm into (4.20), and recalling (4.15), we write sm 1 1 Mc 5 5 r Ec Ec I or 1 M 5 r EI EXAMPLE 4.01 0.8 in. M' M 2.5 in. Fig. 4.14 (4.21) A steel bar of 0.8 3 2.5-in. rectangular cross section is subjected to two equal and opposite couples acting in the vertical plane of symmetry of the bar (Fig. 4.14). Determine the value of the bending moment M that causes the bar to yield. Assume sY 5 36 ksi. Since the neutral axis must pass through the centroid C of the cross section, we have c 5 1.25 in. (Fig. 4.15). On the other hand, the centroidal moment of inertia of the rectangular cross section is I5 1 3 12 bh 5 1 12 10.8 in.2 12.5 in.2 3 5 1.042 in4 Solving Eq. (4.15) for M, and substituting the above data, we have I 1.042 in4 M 5 sm 5 136 ksi2 c 1.25 in. M 5 30 kip ? in. 0.8 in. 1.25 in. 2.5 in. Fig. 4.15 232 C N. A. bee80288_ch04_220-313.indd Page 233 11/11/10 3:03:08 PM user-f499 /Users/user-f499/Desktop/Temp Work/Don't Delete Job/MHDQ251:Beer:201/ch04 An aluminum rod with a semicircular cross section of radius r 5 12 mm (Fig. 4.16) is bent into the shape of a circular arc of mean radius r 5 2.5 m. Knowing that the flat face of the rod is turned toward the center of curvature of the arc, determine the maximum tensile and compressive stress in the rod. Use E 5 70 GPa. EXAMPLE 4.02 We could use Eq. (4.21) to determine the bending moment M corresponding to the given radius of curvature r, and then Eq. (4.15) to determine sm. However, it is simpler to use Eq. (4.9) to determine Pm, and Hooke’s law to obtain sm. The ordinate y of the centroid C of the semicircular cross section is Fig. 4.16 y5 4112 mm2 4r 5 5.093 mm 5 3p 3p The neutral axis passes through C (Fig. 4.17) and the distance c to the point of the cross section farthest away from the neutral axis is c 5 r 2 y 5 12 mm 2 5.093 mm 5 6.907 mm c C N. A. y Fig. 4.17 Using Eq. (4.9), we write Pm 5 r  12 mm 6.907 3 1023 m c 5 5 2.763 3 1023 r 2.5 m and, applying Hooke’s law, s m 5 EPm 5 170 3 109 Pa2 12.763 3 1023 2 5 193.4 MPa Since this side of the rod faces away from the center of curvature, the stress obtained is a tensile stress. The maximum compressive stress occurs on the flat side of the rod. Using the fact that the stress is proportional to the distance from the neutral axis, we write y 5.093 mm 1193.4 MPa2 scomp 5 2 sm 5 2 c 6.907 mm 5 2142.6 MPa 4.5 DEFORMATIONS IN A TRANSVERSE CROSS SECTION When we proved in Sec. 4.3 that the transverse cross section of a member in pure bending remains plane, we did not rule out the possibility of deformations within the plane of the section. That such deformations will exist is evident, if we recall from Sec. 2.11 that elements in a state of uniaxial stress, sx ? 0, sy 5 sz 5 0, are deformed in the transverse y and z directions, as well as in the axial x direction. The normal strains Py and Pz depend upon Poisson’s ratio n for the material used and are expressed as Py 5 2nPx Pz 5 2nPx or, recalling Eq. (4.8), Py 5 ny r Pz 5 ny r (4.22) 233 bee80288_ch04_220-313.indd Page 234 10/27/10 5:46:13 PM user-f499 234 Pure Bending y C  Neutral surface  x z Neutral axis of transverse section  ⬘   / /Users/user-f499/Desktop/Temp Work/Don't Delete Job/MHDQ251:Beer:201/ch04 The relations we have obtained show that the elements located above the neutral surface (y . 0) will expand in both the y and z directions, while the elements located below the neutral surface (y , 0) will contract. In the case of a member of rectangular cross section, the expansion and contraction of the various elements in the vertical direction will compensate, and no change in the vertical dimension of the cross section will be observed. As far as the deformations in the horizontal transverse z direction are concerned, however, the expansion of the elements located above the neutral surface and the corresponding contraction of the elements located below that surface will result in the various horizontal lines in the section being bent into arcs of circle (Fig. 4.18). The situation observed here is similar to that observed earlier in a longitudinal cross section. Comparing the second of Eqs. (4.22) with Eq. (4.8), we conclude that the neutral axis of the transverse section will be bent into a circle of radius r9 5 ryn. The center C9 of this circle is located below the neutral surface (assuming M . 0), i.e., on the side opposite to the center of curvature C of the member. The reciprocal of the radius of curvature r9 represents the curvature of the transverse cross section and is called the anticlastic curvature. We have Anticlastic curvature 5 1 n 5 r r¿ (4.23) C⬘ Fig. 4.18 Deformation of transverse cross section. M' Fig. 4.19 segment. M Deformation of longitudinal In our discussion of the deformations of a symmetric member in pure bending, in this section and in the preceding ones, we have ignored the manner in which the couples M and M9 were actually applied to the member. If all transverse sections of the member, from one end to the other, are to remain plane and free of shearing stresses, we must make sure that the couples are applied in such a way that the ends of the member themselves remain plane and free of shearing stresses. This can be accomplished by applying the couples M and M9 to the member through the use of rigid and smooth plates (Fig. 4.19). The elementary forces exerted by the plates on the member will be normal to the end sections, and these sections, while remaining plane, will be free to deform as described earlier in this section. We should note that these loading conditions cannot be actually realized, since they require each plate to exert tensile forces on the corresponding end section below its neutral axis, while allowing the section to freely deform in its own plane. The fact that the rigid-endplates model of Fig. 4.19 cannot be physically realized, however, does not detract from its importance, which is to allow us to visualize the loading conditions corresponding to the relations derived in the preceding sections. Actual loading conditions may differ appreciably from this idealized model. By virtue of Saint-Venant’s principle, however, the relations obtained can be used to compute stresses in engineering situations, as long as the section considered is not too close to the points where the couples are applied. bee80288_ch04_220-313.indd Page 235 11/11/10 3:03:09 PM user-f499 /Users/user-f499/Desktop/Temp Work/Don't Delete Job/MHDQ251:Beer:201/ch04 SAMPLE PROBLEM 4.1 The rectangular tube shown is extruded from an aluminum alloy for which sY 5 40 ksi, sU 5 60 ksi, and E 5 10.6 3 106 psi. Neglecting the effect of fillets, determine (a) the bending moment M for which the factor of safety will be 3.00, (b) the corresponding radius of curvature of the tube. t 5 in. x C t t M t  0.25 in. t 3.25 in. x SOLUTION C = Moment of Inertia. Considering the cross-sectional area of the tube as the difference between the two rectangles shown and recalling the for4.5 in. mula for the centroidal moment of inertia of a rectangle, we write − 5 in. x I5 3.25 in. 2.75 in. 1 12 13.252 152 3 2 Allowable Stress. of 60 ksi, we have 1 12 12.752 14.52 3 I 5 12.97 in4 For a factor of safety of 3.00 and an ultimate stress sall 5 sU 60 ksi 5 20 ksi 5 F.S. 3.00 Since sall , sY, the tube remains in the elastic range and we can apply the results of Sec. 4.4. a. Bending Moment. s all 5 O Mc I With c 5 12 15 in.2 5 2.5 in., we write 12.97 in4 I M 5 s all 5 120 ksi2 c 2.5 in. M 5 103.8 kip ? in. ◀ b. Radius of Curvature. Recalling that E 5 10.6 3 106 psi, we substitute this value and the values obtained for I and M into Eq. (4.21) and find 1 M 103.8 3 103 lb ? in. 5 5 5 0.755 3 1023 in21 r EI 110.6 3 106 psi2 112.97 in4 2 r 5 1325 in. r 5 110.4 ft  ◀ Alternative Solution. Since we know that the maximum stress is sall 5 20 ksi, we can determine the maximum strain Pm and then use Eq. (4.9), M c c s all 20 ksi 5 5 1.887 3 1023 in./in. E 10.6 3 106 psi 2.5 in. c c r5 5 Pm 5 r Pm 1.887 3 1023 in./in. r 5 1325 in. r 5 110.4 ft Pm 5 ◀ 235 bee80288_ch04_220-313.indd Page 236 11/11/10 3:03:10 PM user-f499 /Users/user-f499/Desktop/Temp Work/Don't Delete Job/MHDQ251:Beer:201/ch04 SAMPLE PROBLEM 4.2 A cast-iron machine part is acted upon by the 3 kN ? m couple shown. Knowing that E 5 165 GPa and neglecting the effect of fillets, determine (a) the maximum tensile and compressive stresses in the casting, (b) the radius of curvature of the casting. 90 mm 20 mm 40 mm M  3 kN · m 30 mm SOLUTION 90 mm 1 20 mm x' C y1  50 mm 40 mm  2 y2  20 mm x 30 mm Centroid. We divide the T-shaped cross section into the two rectangles shown and write 1 2 Area, mm2 y, mm yA, mm3 12021902 5 1800 14021302 5 1200 ©A 5 3000 50 20 90 3 103 24 3 103 ©yA 5 114 3 103 Y ©A 5 ©yA Y 130002 5 114 3 106 Y 5 38 mm Centroidal Moment of Inertia. The parallel-axis theorem is used to determine the moment of inertia of each rectangle with respect to the axis x9 that passes through the centroid of the composite section. Adding the moments of inertia of the rectangles, we write 1 12 mm C 18 mm 2 22 mm x'   38 mm Ix¿ 5 © 1I 1 Ad 2 2 5 © 1 121 bh3 1 Ad 2 2 5 121 1902 1202 3 1 190 3 202 1122 2 1 5 868 3 103 mm4 I 5 868 3 1029 m4 A C  cB  0.038 m B 13 kN ? m2 10.022 m2 McA 5 I 868 3 1029 m4 Maximum Compressive Stress. x' sB 5 2 236 s A 5 176.0 MPa b This occurs at point B; we have 13 kN ? m2 10.038 m2 McB 52 I 868 3 1029 m4 b. Radius of Curvature. Center of curvature 1302 1402 3 1 130 3 402 1182 2 a. Maximum Tensile Stress. Since the applied couple bends the casting downward, the center of curvature is located below the cross section. The maximum tensile stress occurs at point A, which is farthest from the center of curvature. sA 5 cA  0.022 m 1 12 s B 5 2131.3 MPa b From Eq. (4.21), we have 1 M 3 kN ? m 5 5 r EI 1165 GPa2 1868 3 1029 m4 2 5 20.95 3 1023 m21 r 5 47.7 m b bee80288_ch04_220-313.indd Page 237 10/26/10 4:26:21 PM user-f499 /Volumes/201/MHDQ251/bee80288_disk1of1/0073380288/bee80288_pagefiles PROBLEMS 20 4.1 and 4.2 Knowing that the couple shown acts in a vertical plane, determine the stress at (a) point A, (b) point B. 20 20 2 in. 2 in. 2 in. M  15 kN · m A M  25 kip · in. A B 40 80 2 in. 1.5 in. 2 in. 20 B Fig. P4.1 Dimensions in mm 4.3 Using an allowable stress of 16 ksi, determine the largest couple that can be applied to each pipe. 4.4 A nylon spacing bar has the cross section shown. Knowing that the allowable stress for the grade of nylon used is 24 MPa, determine the largest couple Mz that can be applied to the bar. y Fig. P4.2 0.1 in. 0.5 in. M1 (a) z Mz C 0.2 in. 80 mm 0.5 in. r  25 mm M2 100 mm Fig. P4.4 (b) 4.5 A beam of the cross section shown is extruded from an aluminum alloy for which sY 5 250 MPa and sU 5 450 MPa. Using a factor of safety of 3.00, determine the largest couple that can be applied to the beam when it is bent about the z axis. Fig. P4.3 y 24 mm z Mz C 80 mm 24 mm 16 mm Fig. P4.5 4.6 Solve Prob. 4.5, assuming that the beam is bent about the y axis. 237 bee80288_ch04_220-313.indd Page 238 10/26/10 4:26:43 PM user-f499 238 /Volumes/201/MHDQ251/bee80288_disk1of1/0073380288/bee80288_pagefiles 4.7 and 4.8 Two W4 3 13 rolled sections are welded together as shown. Knowing that for the steel alloy used, sY 5 36 ksi and sU 5 58 ksi and using a factor of safety of 3.0, determine the largest couple that can be applied when the assembly is bent about the z axis. Pure Bending y y C z z Fig. P4.7 Fig. P4.8 C 25 mm 25 mm 4 kN A 4 kN B 300 mm 4.9 through 4.11 Two vertical forces are applied to a beam of the cross section shown. Determine the maximum tensile and compressive stresses in portion BC of the beam. C 300 mm 10 mm 10 mm Fig. P4.9 10 kN 50 mm B 10 kN C A D 10 mm 50 mm 8 in. 150 mm 250 mm 150 mm Fig. P4.10 1 in. 4.12 Knowing that a beam of the cross section shown is bent about a horizontal axis and that the bending moment is 6 kN ? m, determine the total force acting on the top flange. 6 in. 1 in. 1 in. 4 in. A 25 kips 25 kips B C 216 mm y D z 20 in. 60 in. 36 mm 54 mm C 20 in. 108 mm Fig. P4.11 72 mm Fig. P4.12 and P4.13 4.13 Knowing that a beam of the cross section shown is bent about a horizontal axis and that the bending moment is 6 kN ? m, determine the total force acting on the shaded portion of the web. bee80288_ch04_220-313.indd Page 239 10/26/10 4:26:57 PM user-f499 /Volumes/201/MHDQ251/bee80288_disk1of1/0073380288/bee80288_pagefiles Problems 4.14 Knowing that a beam of the cross section shown is bent about a horizontal axis and that the bending moment is 50 kip ? in., determine the total force acting (a) on the top flange (b) on the shaded portion of the web. 4.15 The beam shown is made of a nylon for which the allowable stress is 24 MPa in tension and 30 MPa in compression. Determine the largest couple M that can be applied to the beam. y 1.5 in. z 4 in. C 40 mm 1.5 in. 15 mm 2 in. 6 in. d  30 mm Fig. P4.14 20 mm M 0.5 in. 1.5 in. Fig. P4.15 4.17 Knowing that for the extruded beam shown the allowable stress is 12 ksi in tension and 16 ksi in compression, determine the largest couple M that can be applied. 4.18 Knowing that for the casting shown the allowable stress is 5 ksi in tension and 18 ksi in compression, determine the largest couple M that can be applied. 0.5 in. 0.5 in. 0.5 in. 1 in. M 0.5 in. Fig. P4.18 4.19 and 4.20 Knowing that for the extruded beam shown the allowable stress is 120 MPa in tension and 150 MPa in compression, determine the largest couple M that can be applied. 80 mm 125 mm 54 mm 50 mm 125 mm 40 mm 150 mm Fig. P4.19 M M Fig. P4.20 0.5 in. 1.5 in. 1.5 in. 4.16 Solve Prob. 4.15, assuming that d 5 40 mm. 0.5 in. 0.5 in. 0.5 in. M Fig. P4.17 239 bee80288_ch04_220-313.indd Page 240 11/11/10 3:03:11 PM user-f499 240 /Users/user-f499/Desktop/Temp Work/Don't Delete Job/MHDQ251:Beer:201/ch04 4.21 A steel band saw blade, that was originally straight, passes over 8-in.-diameter pulleys when mounted on a band saw. Determine the maximum stress in the blade, knowing that it is 0.018 in. thick and 0.625 in. wide. Use E 5 29 3 106 psi. Pure Bending 4.22 Straight rods of 0.30-in. diameter and 200-ft length are sometimes used to clear underground conduits of obstructions or to thread wires through a new conduit. The rods are made of high-strength steel and, for storage and transportation, are wrapped on spools of 5-ft diameter. Assuming that the yield strength is not exceeded, determine (a) the maximum stress in a rod, when the rod, which is initially straight, is wrapped on a spool, (b) the corresponding bending moment in the rod. Use E 5 29 3 106 psi. 0.018 in. Fig. P4.21 5 ft Fig. P4.22 M' M 8 mm t r 4.23 A 900-mm strip of steel is bent into a full circle by two couples applied as shown. Determine (a) the maximum thickness t of the strip if the allowable stress of the steel is 420 MPa, (b) the corresponding moment M of the couples. Use E 5 200 GPa. 4.24 A 60-N ? m couple is applied to the steel bar shown. (a) Assuming that the couple is applied about the z axis as shown, determine the maximum stress and the radius of curvature of the bar. (b) Solve part a, assuming that the couple is applied about the y axis. Use E 5 200 GPa. 900 mm Fig. P4.23 12 mm y 60 N · m 20 mm z Fig. P4.24 M M a (a) Fig. P4.25 (b) 4.25 A couple of magnitude M is applied to a square bar of side a. For each of the orientations shown, determine the maximum stress and the curvature of the bar. bee80288_ch04_220-313.indd Page 241 10/26/10 4:27:21 PM user-f499 /Volumes/201/MHDQ251/bee80288_disk1of1/0073380288/bee80288_pagefiles 4.26 A portion of a square bar is removed by milling, so that its cross section is as shown. The bar is then bent about its horizontal axis by a couple M. Considering the case where h 5 0.9h0, express the maximum stress in the bar in the form sm 5 ks0 where s0 is the maximum stress that would have occurred if the original square bar had been bent by the same couple M, and determine the value of k. 4.27 In Prob. 4.26, determine (a) the value of h for which the maximum stress sm is as small as possible, (b) the corresponding value of k. 4.28 A couple M will be applied to a beam of rectangular cross section that is to be sawed from a log of circular cross section. Determine the ratio dyb for which (a) the maximum stress sm will be as small as possible, (b) the radius of curvature of the beam will be maximum. Problems h0 C h h0 Fig. P4.26 M M' 4.29 For the aluminum bar and loading of Sample Prob. 4.1, determine (a) the radius of curvature r9 of a transverse cross section, (b) the angle between the sides of the bar that were originally vertical. Use E 5 10.6 3 106 psi and n 5 0.33. 4.30 For the bar and loading of Example 4.01, determine (a) the radius of curvature r, (b) the radius of curvature r9 of a transverse cross section, (c) the angle between the sides of the bar that were originally vertical. Use E 5 29 3 106 psi and n 5 0.29. h M d b Fig. P4.28 4.31 A W200 3 31.3 rolled-steel beam is subjected to a couple M of moment 45 kN ? m. Knowing that E 5 200 GPa and n 5 0.29, determine (a) the radius of curvature r, (b) the radius of curvature r9 of a transverse cross section. y A z M C x Fig. P4.31 y 4.32 It was assumed in Sec. 4.3 that the normal stresses sy in a member in pure bending are negligible. For an initially straight elastic member of rectangular cross section, (a) derive an approximate expression for s y as a function of y, (b) show that (sy)max 5 2(cy2r)(sx)max and, thus, that sy can be neglected in all practical situations. (Hint: Consider the free-body diagram of the portion of beam located below the surface of ordinate y and assume that the distribution of the stress sx is still linear.)  2 y  2 y  c x  2 Fig. P4.32 y x y  c  2 241 bee80288_ch04_220-313.indd Page 242 10/28/10 9:25:32 PM user-f499 242 4.6 Pure Bending 1 M 2 Fig. 4.20 Cross section with two materials. /Volumes/201/MHDQ251/bee80288_disk1of1/0073380288/bee80288_pagefiles BENDING OF MEMBERS MADE OF SEVERAL MATERIALS The derivations given in Sec. 4.4 were based on the assumption of a homogeneous material with a given modulus of elasticity E. If the member subjected to pure bending is made of two or more materials with different moduli of elasticity, our approach to the determination of the stresses in the member must be modified. Consider, for instance, a bar consisting of two portions of different materials bonded together as shown in cross section in Fig. 4.20. This composite bar will deform as described in Sec. 4.3, since its cross section remains the same throughout its entire length, and since no assumption was made in Sec. 4.3 regarding the stress-strain relationship of the material or materials involved. Thus, the normal strain Px still varies linearly with the distance y from the neutral axis of the section (Fig. 4.21a and b), and formula (4.8) holds: y Px 5 2 r (4.8) y 1 E1y ␴1 ⫽ – —– ␳ y ⑀x ⫽ – — ␳ ⑀x N. A. 2 (a) Fig. 4.21 y ␴x E2 y ␴2 ⫽ – —– ␳ (b) (c) Strain and stress distribution in bar made of two materials. However, we cannot assume that the neutral axis passes through the centroid of the composite section, and one of the goals of the present analysis will be to determine the location of this axis. Since the moduli of elasticity E1 and E2 of the two materials are different, the expressions obtained for the normal stress in each material will also be different. We write E 1y s 1 5 E 1P x 5 2 r E 2y s 2 5 E 2P x 5 2 r (4.24) and obtain a stress-distribution curve consisting of two segments of straight line (Fig. 4.21c). It follows from Eqs. (4.24) that the force dF1 exerted on an element of area dA of the upper portion of the cross section is E 1y dF1 5 s 1 dA 5 2 r dA (4.25) bee80288_ch04_220-313.indd Page 243 10/27/10 5:46:14 PM user-f499 /Users/user-f499/Desktop/Temp Work/Don't Delete Job/MHDQ251:Beer:201/ch04 4.6 Bending of Members Made of Several Materials while the force dF2 exerted on an element of the same area dA of the lower portion is E 2y (4.26) dF2 5 s 2 dA 5 2 r dA But, denoting by n the ratio E2yE1 of the two moduli of elasticity, we can express dF2 as 1nE1 2y E 1y (4.27) dF2 5 2 r dA 5 2 r 1n dA2 Comparing Eqs. (4.25) and (4.27), we note that the same force dF2 would be exerted on an element of area n dA of the first material. In other words, the resistance to bending of the bar would remain the same if both portions were made of the first material, provided that the width of each element of the lower portion were multiplied by the factor n. Note that this widening (if n . 1), or narrowing (if n , 1), must be effected in a direction parallel to the neutral axis of the section, since it is essential that the distance y of each element from the neutral axis remain the same. The new cross section obtained in this way is called the transformed section of the member (Fig. 4.22). Since the transformed section represents the cross section of a member made of a homogeneous material with a modulus of elasticity E1, the method described in Sec. 4.4 can be used to determine the neutral axis of the section and the normal stress at various points of the section. The neutral axis will be drawn through the centroid of the transformed section (Fig. 4.23), and the stress sx at any point of the corresponding fictitious homogeneous member will be obtained from Eq. (4.16) My (4.16) sx 5 2 I where y is the distance from the neutral surface, and I the moment of inertia of the transformed section with respect to its centroidal axis. To obtain the stress s1 at a point located in the upper portion of the cross section of the original composite bar, we simply compute the stress sx at the corresponding point of the transformed section. However, to obtain the stress s2 at a point in the lower portion of the cross section, we must multiply by n the stress sx computed at the corresponding point of the transformed section. Indeed, as we saw earlier, the same elementary force dF2 is applied to an element of area n dA of the transformed section and to an element of area dA of the original section. Thus, the stress s2 at a point of the original section must be n times larger than the stress at the corresponding point of the transformed section. The deformations of a composite member can also be determined by using the transformed section. We recall that the transformed section represents the cross section of a member, made of a homogeneous material of modulus E1, which deforms in the same manner as the composite member. Therefore, using Eq. (4.21), we write that the curvature of the composite member is 1 M 5 r E 1I where I is the moment of inertia of the transformed section with respect to its neutral axis. b b = dA ndA b nb Fig. 4.22 Transformed section for composite bar. y y My x  – —– I C N. A. Fig. 4.23 Distribution of stresses in transformed section. x 243 bee80288_ch04_220-313.indd Page 244 10/26/10 4:27:39 PM user-f499 EXAMPLE 4.03 /Volumes/201/MHDQ251/bee80288_disk1of1/0073380288/bee80288_pagefiles A bar obtained by bonding together pieces of steel (Es 5 29 3 106 psi) and brass (Eb 5 15 3 106 psi) has the cross section shown (Fig. 4.24). Determine the maximum stress in the steel and in the brass when the bar is in pure bending with a bending moment M 5 40 kip ? in. 0.75 in. 0.4 in. 0.4 in. 0.4 in. 1.45 in. 0.4 in. c  1.5 in. 3 in. 3 in. N. A. All brass Steel Brass 2.25 in. Brass Fig. 4.24 Fig. 4.25 The transformed section corresponding to an equivalent bar made entirely of brass is shown in Fig. 4.25. Since n5 29 3 106 psi Es 5 5 1.933 Eb 15 3 106 psi the width of the central portion of brass, which replaces the original steel portion, is obtained by multiplying the original width by 1.933, we have (0.75 in.)(1.933) 5 1.45 in. Note that this change in dimension occurs in a direction parallel to the neutral axis. The moment of inertia of the transformed section about its centroidal axis is I5 1 12 bh3 5 1 12 12.25 in.2 13 in.2 3 5 5.063 in4 and the maximum distance from the neutral axis is c 5 1.5 in. Using Eq. (4.15), we find the maximum stress in the transformed section: sm 5 Mc I 5 140 kip ? in.2 11.5 in.2 5.063 in4 5 11.85 ksi The value obtained also represents the maximum stress in the brass portion of the original composite bar. The maximum stress in the steel portion, however, will be larger than the value obtained for the transformed section, since the area of the central portion must be reduced by the factor n 5 1.933 when we return from the transformed section to the original one. We thus conclude that 1s brass 2 max 5 11.85 ksi 1s steel 2 max 5 11.9332 111.85 ksi2 5 22.9 ksi 244 bee80288_ch04_220-313.indd Page 245 10/27/10 5:46:14 PM user-f499 /Users/user-f499/Desktop/Temp Work/Don't Delete Job/MHDQ251:Beer:201/ch04 An important example of structural members made of two different materials is furnished by reinforced concrete beams (Photo 4.4). These beams, when subjected to positive bending moments, are reinforced by steel rods placed a short distance above their lower face (Fig. 4.26a). Since concrete is very weak in tension, it will crack below the neutral surface and the steel rods will carry the entire tensile load, while the upper part of the concrete beam will carry the compressive load. To obtain the transformed section of a reinforced concrete beam, we replace the total cross-sectional area As of the steel bars by an equivalent area nAs, where n is the ratio EsyEc of the moduli of elasticity of steel and concrete (Fig. 4.26b). On the other hand, since the concrete in the beam acts effectively only in compression, only the portion of the cross section located above the neutral axis should be used in the transformed section. b b x d 1 2 C x  N. A. d–x Photo 4.4 Reinforced concrete building. Fs nAs (a) Fig. 4.26 4.6 Bending of Members Made of Several Materials (b) (c) Reinforced concrete beam. The position of the neutral axis is obtained by determining the distance x from the upper face of the beam to the centroid C of the transformed section. Denoting by b the width of the beam, and by d the distance from the upper face to the center line of the steel rods, we write that the first moment of the transformed section with respect to the neutral axis must be zero. Since the first moment of each of the two portions of the transformed section is obtained by multiplying its area by the distance of its own centroid from the neutral axis, we have 1bx2 x 2 nAs 1d 2 x2 5 0 2 or 1 2 bx 1 nAs x 2 nAsd 5 0 2 (4.28) Solving this quadratic equation for x, we obtain both the position of the neutral axis in the beam, and the portion of the cross section of the concrete beam that is effectively used. The determination of the stresses in the transformed section is carried out as explained earlier in this section (see Sample Prob. 4.4). The distribution of the compressive stresses in the concrete and the resultant Fs of the tensile forces in the steel rods are shown in Fig. 4.26c. 245 bee80288_ch04_220-313.indd Page 246 10/26/10 4:27:50 PM user-f499 246 4.7 Pure Bending /Volumes/201/MHDQ251/bee80288_disk1of1/0073380288/bee80288_pagefiles STRESS CONCENTRATIONS The formula sm 5 McyI was derived in Sec. 4.4 for a member with a plane of symmetry and a uniform cross section, and we saw in Sec. 4.5 that it was accurate throughout the entire length of the member only if the couples M and M9 were applied through the use of rigid and smooth plates. Under other conditions of application of the loads, stress concentrations will exist near the points where the loads are applied. Higher stresses will also occur if the cross section of the member undergoes a sudden change. Two particular cases of interest have been studied,† the case of a flat bar with a sudden change in width, and the case of a flat bar with grooves. Since the distribution of stresses in the critical cross sections depends only upon the geometry of the members, stress-concentration factors can be determined for various ratios of the parameters involved and recorded as shown in Figs. 4.27 and 4.28. The value of the maximum stress in the critical cross section can then be expressed as sm 5 K Mc I (4.29) where K is the stress-concentration factor, and where c and I refer to the critical section, i.e., to the section of width d in both of the cases considered here. An examination of Figs. 4.27 and 4.28 clearly shows the importance of using fillets and grooves of radius r as large as practical. Finally, we should point out that, as was the case for axial loading and torsion, the values of the factors K have been computed under the assumption of a linear relation between stress and strain. In many applications, plastic deformations will occur and result in values of the maximum stress lower than those indicated by Eq. (4.29). 3.0 r M' 2.8 D M d 2.6 2.8 2.6 2.4 D  d 2.2 2 3 D  d 1.8 1.1 M' D r d M 2r 1.2 1.1 K 2.0 1.2 2 1.5 2.4 2.2 1.5 K 2.0 1.05 1.8 1.6 1.6 1.4 1.4 1.02 1.01 1.2 1.0 3.0 0 0.05 0.10 1.2 0.15 r/d 0.20 0.25 Fig. 4.27 Stress-concentration factors for flat bars with fillets under pure bending.† 0.3 1.0 0 0.05 0.10 0.15 r/d 0.20 0.25 0.30 Fig. 4.28 Stress-concentration factors for flat bars with grooves under pure bending.† †W. D. Pilkey, Peterson’s Stress Concentration Factors, 2d ed., John Wiley & Sons, New York, 1997. bee80288_ch04_220-313.indd Page 247 10/26/10 4:28:02 PM user-f499 /Volumes/201/MHDQ251/bee80288_disk1of1/0073380288/bee80288_pagefiles Grooves 10 mm deep are to be cut in a steel bar which is 60 mm wide and 9 mm thick (Fig. 4.29). Determine the smallest allowable width of the grooves if the stress in the bar is not to exceed 150 MPa when the bending moment is equal to 180 N ? m. EXAMPLE 4.04 r 10 mm c D  60 mm d 10 mm 2r b  9 mm (b) (a) Fig. 4.29 We note from Fig. 4.29a that d 5 60 mm 2 2110 mm2 5 40 mm b 5 9 mm c 5 12d 5 20 mm The moment of inertia of the critical cross section about its neutral axis is I 5 121 bd3 5 121 19 3 1023 m2 140 3 1023 m2 3 5 48 3 1029 m4 The value of the stress McyI is thus 1180 N ? m2 120 3 1023 m2 Mc 5 5 75 MPa I 48 3 1029 m4 Substituting this value for McyI into Eq. (4.29) and making sm 5 150 MPa, we write 150 MPa 5 K(75 MPa) K52 We have, on the other hand, D 60 mm 5 5 1.5 d 40 mm Using the curve of Fig. 4.32 corresponding to Dyd 5 1.5, we find that the value K 5 2 corresponds to a value of ryd equal to 0.13. We have, therefore, r 5 0.13 d r 5 0.13d 5 0.13(40 mm) 5 5.2 mm The smallest allowable width of the grooves is thus 2r 5 2(5.2 mm) 5 10.4 mm 247 bee80288_ch04_220-313.indd Page 248 11/11/10 3:03:12 PM user-f499 /Users/user-f499/Desktop/Temp Work/Don't Delete Job/MHDQ251:Beer:201/ch04 SAMPLE PROBLEM 4.3 200 mm 20 mm 300 mm 75 mm 20 mm Two steel plates have been welded together to form a beam in the shape of a T that has been strengthened by securely bolting to it the two oak timbers shown. The modulus of elasticity is 12.5 GPa for the wood and 200 GPa for the steel. Knowing that a bending moment M 5 50 kN ? m is applied to the composite beam, determine (a) the maximum stress in the wood, (b) the stress in the steel along the top edge. 75 mm SOLUTION Transformed Section. We first compute the ratio n5 0.020 m Multiplying the horizontal dimensions of the steel portion of the section by n 5 16, we obtain a transformed section made entirely of wood. y 16(0.200 m)  3.2 m 0.150 m C z Neutral Axis. The neutral axis passes through the centroid of the transformed section. Since the section consists of two rectangles, we have 0.160 m Y O 0.150 m 10.160 m2 13.2 m 3 0.020 m2 1 0 ©yA Y5 ©A 5 3.2 m 3 0.020 m 1 0.470 m 3 0.300 m Centroidal Moment of Inertia. 0.075 m 0.075 m 16(0.020 m)  0.32 m 1 12 sw 5 z 0.050 m O c1  0.120 m c2  0.200 m 150 3 103 N ? m2 10.200 m2 Mc2 5 I 2.19 3 1023 m4 s w 5 4.57 MPa b b. Stress in Steel. Along the top edge c1 5 0.120 m. From the transformed section we obtain an equivalent stress in wood, which must be multiplied by n to obtain the stress in steel. ss 5 n 248 Using the parallel-axis theorem: 3 a. Maximum Stress in Wood. The wood farthest from the neutral axis is located along the bottom edge, where c2 5 0.200 m. y C 5 0.050 m 10.4702 10.3002 1 10.470 3 0.3002 10.0502 2 1 121 13.22 10.0202 3 1 13.2 3 0.0202 10.160 2 0.0502 2 I 5 2.19 3 1023 m 4 I5 N. A. Es 200 GPa 5 16 5 12.5 GPa Ew 150 3 103 N ? m2 10.120 m2 Mc1 5 1162 I 2.19 3 1023 m4 s s 5 43.8 MPa b bee80288_ch04_220-313.indd Page 249 10/26/10 4:28:13 PM user-f499 /Volumes/201/MHDQ251/bee80288_disk1of1/0073380288/bee80288_pagefiles SAMPLE PROBLEM 4.4 A concrete floor slab is reinforced by 58 -in.-diameter steel rods placed 1.5 in. above the lower face of the slab and spaced 6 in. on centers. The modulus of elasticity is 3.6 3 106 psi for the concrete used and 29 3 106 psi for the steel. Knowing that a bending moment of 40 kip ? in. is applied to each 1-ft width of the slab, determine (a) the maximum stress in the concrete, (b) the stress in the steel. 4 in. 6 in. 6 in. 5.5 in. 6 in. 6 in. SOLUTION 12 in. x N. A. C 4 in. Transformed Section. We consider a portion of the slab 12 in. wide, in which there are two 58 -in.-diameter rods having a total cross-sectional area As 5 2 c 4x nAs  4.95 in2 2 p 5 a in.b d 5 0.614 in2 4 8 Since concrete acts only in compression, all the tensile forces are carried by the steel rods, and the transformed section consists of the two areas shown. One is the portion of concrete in compression (located above the neutral axis), and the other is the transformed steel area nAs. We have 29 3 106 psi Es 5 5 8.06 Ec 3.6 3 106 psi nAs 5 8.0610.614 in2 2 5 4.95 in2 n5 12 in. c1  x  1.450 in. 4 in. c2  4  x  2.55 in. Neutral Axis. The neutral axis of the slab passes through the centroid of the transformed section. Summing moments of the transformed area about the neutral axis, we write x 12x a b 2 4.9514 2 x2 5 0 2 4.95 in2 Moment of Inertia. formed area is x 5 1.450 in. The centroidal moment of inertia of the trans- I 5 13 1122 11.4502 3 1 4.9514 2 1.4502 2 5 44.4 in4 c  1.306 ksi a. Maximum Stress in Concrete. c1 5 1.450 in. and sc 5 s  18.52 ksi At the top of the slab, we have 140 kip ? in.2 11.450 in.2 Mc1 5 I 44.4 in4 s c 5 1.306 ksi b b. Stress in Steel. For the steel, we have c2 5 2.55 in., n 5 8.06 and ss 5 n 140 kip ? in.2 12.55 in.2 Mc2 5 8.06 I 44.4 in4 s s 5 18.52 ksi b 249 bee80288_ch04_220-313.indd Page 250 10/26/10 4:28:23 PM user-f499 /Volumes/201/MHDQ251/bee80288_disk1of1/0073380288/bee80288_pagefiles PROBLEMS 4.33 and 4.34 A bar having the cross section shown has been formed by securely bonding brass and aluminum stock. Using the data given below, determine the largest permissible bending moment when the composite bar is bent about a horizontal axis. Modulus of elasticity Allowable stress 8 mm 32 mm 8 mm 8 mm Aluminum Brass 70 GPa 100 MPa 105 GPa 160 MPa 8 mm 32 mm 8 mm 32 mm 32 mm 8 mm Aluminum Fig. P4.33 Brass Brass Aluminum Fig. P4.34 4.35 and 4.36 For the composite bar indicated, determine the largest permissible bending moment when the bar is bent about a vertical axis. 4.35 Bar of Prob. 4.33. 4.36 Bar of Prob. 4.34. 4.37 and 4.38 Wooden beams and steel plates are securely bolted together to form the composite member shown. Using the data given below, determine the largest permissible bending moment when the member is bent about a horizontal axis. Wood Modulus of elasticity Allowable stress Steel 6 29 3 106 psi 22 ksi 2 3 10 psi 2000 psi 5 1 2 in. 10 in. 3 in. 1 2 3 in. in. Fig. P4.37 250 10 in. 5 1 2 in. 6 in. Fig. P4.38 bee80288_ch04_220-313.indd Page 251 10/26/10 4:28:33 PM user-f499 /Volumes/201/MHDQ251/bee80288_disk1of1/0073380288/bee80288_pagefiles Problems 4.39 and 4.40 A steel bar and an aluminum bar are bonded together to form the composite beam shown. The modulus of elasticity for aluminum is 70 GPa and for steel is 200 GPa. Knowing that the beam is bent about a horizontal axis by a couple of moment M 5 1500 N ? m, determine the maximum stress in (a) the aluminum, (b) the steel. Steel 20 mm Aluminum 20 mm M M 40 mm Aluminum 40 mm Steel 30 mm 30 mm Fig. P4.39 Fig. P4.40 4.41 and 4.42 The 6 3 12-in. timber beam has been strengthened by bolting to it the steel reinforcement shown. The modulus of elasticity for wood is 1.8 3 106 psi and for steel is 29 3 106 psi. Knowing that the beam is bent about a horizontal axis by a couple of moment M 5 450 kip ? in., determine the maximum stress in (a) the wood, (b) the steel. 6 in. 6 in. M 12 in. 5 Fig. P4.41 1 2 M in. 12 in. C8 11.5 Fig. P4.42 4.43 and 4.44 For the composite beam indicated, determine the radius of curvature caused by the couple of moment 1500 N ? m. 4.43 Beam of Prob. 4.39. 4.44 Beam of Prob. 4.40. 4.45 and 4.46 For the composite beam indicated, determine the radius of curvature caused by the couple of moment 450 kip ? in. 4.45 Beam of Prob. 4.41. 4.46 Beam of Prob. 4.42. 4.47 The reinforced concrete beam shown is subjected to a positive bending moment of 175 kN ? m. Knowing that the modulus of elasticity is 25 GPa for the concrete and 200 GPa for the steel, determine (a) the stress in the steel, (b) the maximum stress in the concrete. 4.48 Solve Prob. 4.47, assuming that the 300-mm width is increased to 350 mm. 540 mm 25-mm diameter 60 mm 300 mm Fig. P4.47 251 bee80288_ch04_220-313.indd Page 252 11/11/10 3:03:13 PM user-f499 252 /Users/user-f499/Desktop/Temp Work/Don't Delete Job/MHDQ251:Beer:201/ch04 4.49 A concrete slab is reinforced by 16-mm-diameter steel rods placed on 180-mm centers as shown. The modulus of elasticity is 20 GPa for the concrete and 200 GPa for the steel. Using an allowable stress of 9 MPa for the concrete and 120 MPa for the steel, determine the largest bending moment in a portion of slab 1 m wide. Pure Bending 16-mm diameter 100 mm 4.50 Solve Prob. 4.49, assuming that the spacing of the 16-mm-diameter rods is increased to 225 mm on centers. 4.51 A concrete beam is reinforced by three steel rods placed as shown. The modulus of elasticity is 3 3 106 psi for the concrete and 29 3 106 psi for the steel. Using an allowable stress of 1350 psi for the concrete and 20 ksi for the steel, determine the largest allowable positive bending moment in the beam. 180 mm 140 mm Fig. P4.49 7 8 16 in. -in. diameter 2 in. 8 in. Fig. P4.51 4 in. 24 in. 20 in. 1-in. diameter 2.5 in. 12 in. Fig. P4.52 4.52 Knowing that the bending moment in the reinforced concrete beam is 1100 kip ? ft and that the modulus of elasticity is 3.625 3 106 psi for the concrete and 29 3 106 psi for the steel, determine (a) the stress in the steel, (b) the maximum stress in the concrete. 4.53 The design of a reinforced concrete beam is said to be balanced if the maximum stresses in the steel and concrete are equal, respectively, to the allowable stresses ss and sc. Show that to achieve a balanced design the distance x from the top of the beam to the neutral axis must be x5 d ssEc 11 scEs where Ec and Es are the moduli of elasticity of concrete and steel, respectively, and d is the distance from the top of the beam to the reinforcing steel. d b Fig. P4.53 and P4.54 4.54 For the concrete beam shown, the modulus of elasticity is 3.5 3 106 psi for the concrete and 29 3 106 psi for the steel. Knowing that b 5 8 in. and d 5 22 in., and using an allowable stress of 1800 psi for the concrete and 20 ksi for the steel, determine (a) the required area As of the steel reinforcement if the beam is to be balanced, (b) the largest allowable bending moment. (See Prob. 4.53 for definition of a balanced beam.) bee80288_ch04_220-313.indd Page 253 10/26/10 4:28:53 PM user-f499 /Volumes/201/MHDQ251/bee80288_disk1of1/0073380288/bee80288_pagefiles Problems 4.55 and 4.56 Five metal strips, each 40 mm wide, are bonded together to form the composite beam shown. The modulus of elasticity is 210 GPa for the steel, 105 GPa for the brass, and 70 GPa for the aluminum. Knowing that the beam is bent about a horizontal axis by a couple of moment 1800 N ? m, determine (a) the maximum stress in each of the three metals, (b) the radius of curvature of the composite beam. Aluminum 10 mm Brass 10 mm Brass 10 mm Steel 10 mm Steel 20 mm Aluminum 20 mm Brass 10 mm Steel 10 mm Aluminum 10 mm Brass 10 mm 253 40 mm 40 mm Fig. P4.55 Fig. P4.56 4.57 The composite beam shown is formed by bonding together a brass rod and an aluminum rod of semicircular cross sections. The modulus of elasticity is 15 3 106 psi for the brass and 10 3 106 psi for the aluminum. Knowing that the composite beam is bent about a horizontal axis by couples of moment 8 kip ? in., determine the maximum stress (a) in the brass, (b) in the aluminum. Brass Aluminum 0.8 in. 3 mm Steel Aluminum 6 mm Fig. P4.57 z 4.58 A steel pipe and an aluminum pipe are securely bonded together to form the composite beam shown. The modulus of elasticity is 200 GPa for the steel and 70 GPa for the aluminum. Knowing that the composite beam is bent by a couple of moment 500 N ? m, determine the maximum stress (a) in the aluminum, (b) in the steel. 4.59 The rectangular beam shown is made of a plastic for which the value of the modulus of elasticity in tension is one-half of its value in compression. For a bending moment M 5 600 N ? m, determine the maximum (a) tensile stress, (b) compressive stress. *4.60 A rectangular beam is made of material for which the modulus of elasticity is Et in tension and Ec in compression. Show that the curvature of the beam in pure bending is 1 M 5 r Er I where Er 5 y 4EtEc 1 1Et 1 1Ec 2 2 10 mm 38 mm Fig. P4.58  M Et  100 mm 1 2 Ec  50 mm Fig. P4.59 Ec bee80288_ch04_220-313.indd Page 254 10/26/10 4:29:04 PM user-f499 254 Pure Bending r 108 mm Fig. P4.61 and P4.62 18 mm M /Volumes/201/MHDQ251/bee80288_disk1of1/0073380288/bee80288_pagefiles 4.61 Semicircular grooves of radius r must be milled as shown in the sides of a steel member. Using an allowable stress of 60 MPa, determine the largest bending moment that can be applied to the member when (a) r 5 9 mm, (b) r 5 18 mm. 4.62 Semicircular grooves of radius r must be milled as shown in the sides of a steel member. Knowing that M 5 450 N ? m, determine the maximum stress in the member when the radius r of the semicircular grooves is (a) r 5 9 mm, (b) r 5 18 mm. 4.63 Knowing that the allowable stress for the beam shown is 90 MPa, determine the allowable bending moment M when the radius r of the fillets is (a) 8 mm, (b) 12 mm. 8 mm r M 80 mm 40 mm Fig. P4.63 and P4.64 4.64 Knowing that M 5 250 N ? m, determine the maximum stress in the beam shown when the radius r of the fillets is (a) 4 mm, (b) 8 mm. 4.65 The allowable stress used in the design of a steel bar is 12 ksi. Determine the largest couple M that can be applied to the bar (a) if the bar is designed with grooves having semicircular portions of radius r 5 34 in., as shown in Fig. a, (b) if the bar is redesigned by removing the material above and below the dashed lines as shown in Fig. b. 7 8 7 8 in. in. 7.5 in. 5 in. 7.5 in. 5 in. M M (a) (b) Fig. P4.65 and P4.66 4.66 A couple of moment M 5 20 kip ? in. is to be applied to the end of a steel bar. Determine the maximum stress in the bar (a) if the bar is designed with grooves having semicircular portions of radius r 5 12 in., as shown in Fig. a, (b) if the bar is redesigned by removing the material above and below the dashed lines as shown in Fig. b. bee80288_ch04_220-313.indd Page 255 11/11/10 3:03:13 PM user-f499 *4.8 /Users/user-f499/Desktop/Temp Work/Don't Delete Job/MHDQ251:Beer:201/ch04 4.8 Plastic Deformations PLASTIC DEFORMATIONS When we derived the fundamental relation sx 5 2MyyI in Sec. 4.4, we assumed that Hooke’s law applied throughout the member. If the yield strength is exceeded in some portion of the member, or if the material involved is a brittle material with a nonlinear stress-strain diagram, this relation ceases to be valid. The purpose of this section is to develop a more general method for the determination of the distribution of stresses in a member in pure bending, which can be used when Hooke’s law does not apply. We first recall that no specific stress-strain relationship was assumed in Sec. 4.3, when we proved that the normal strain Px varies linearly with the distance y from the neutral surface. Thus, we can still use this property in our present analysis and write y Px 5 2 Pm c (4.10) where y represents the distance of the point considered from the neutral surface, and c the maximum value of y. However, we cannot assume anymore that, in a given section, the neutral axis passes through the centroid of that section, since this property was derived in Sec. 4.4 under the assumption of elastic deformations. In general, the neutral axis must be located by trial and error, until a distribution of stresses has been found, that satisfies Eqs. (4.1) and (4.3) of Sec. 4.2. However, in the particular case of a member possessing both a vertical and a horizontal plane of symmetry, and made of a material characterized by the same stressstrain relation in tension and in compression, the neutral axis will coincide with the horizontal axis of symmetry of the section. Indeed, the properties of the material require that the stresses be symmetric with respect to the neutral axis, i.e., with respect to some horizontal axis, and it is clear that this condition will be met, and Eq. (4.1) satisfied at the same time, only if that axis is the horizontal axis of symmetry itself. Our analysis will first be limited to the special case we have just described. The distance y in Eq. (4.10) is thus measured from the horizontal axis of symmetry z of the cross section, and the distribution of strain Px is linear and symmetric with respect to that axis (Fig. 4.30). On the other hand, the stress-strain curve is symmetric with respect to the origin of coordinates (Fig. 4.31). The distribution of stresses in the cross section of the member, i.e., the plot of sx versus y, is obtained as follows. Assuming that smax has been specified, we first determine the corresponding value of Pm from the stress-strain diagram and carry this value into Eq. (4.10). Then, for each value of y, we determine the corresponding value of Px from Eq. (4.10) or Fig. 4.30, and obtain from the stress-strain diagram of Fig. 4.31 the stress sx corresponding to this value of Px. Plotting sx against y yields the desired distribution of stresses (Fig. 4.32). We now recall that, when we derived Eq. (4.3) in Sec. 4.2, we assumed no particular relation between stress and strain. We can therefore use Eq. (4.3) to determine the bending moment M corresponding to the stress distribution obtained in Fig. 4.32. Considering the particular y – m c M' M x z m –c Fig. 4.30 beam. Linear strain distribution in x max m 0 x Fig. 4.31 Nonlinear stressstrain material diagram. y c x –c Fig. 4.32 Nonlinear stress distribution in beam. max 255 bee80288_ch04_220-313.indd Page 256 10/27/10 5:46:17 PM user-f499 256 /Users/user-f499/Desktop/Temp Work/Don't Delete Job/MHDQ251:Beer:201/ch04 case of a member with a rectangular cross section of width b, we express the element of area in Eq. (4.3) as dA 5 b dy and write Pure Bending M 5 2b # c ysx dy (4.30) 2c where sx is the function of y plotted in Fig. 4.32. Since sx is an odd function of y, we can write Eq. (4.30) in the alternative form c M 5 22b # ys dy x (4.31) 0 y x U RB Fig. 4.33 Beam stress distribution at ultimate moment MU. If sx is a known analytical function of Px, Eq. (4.10) can be used to express sx as a function of y, and the integral in (4.31) can be determined analytically. Otherwise, the bending moment M can be obtained through a numerical integration. This computation becomes more meaningful if we note that the integral in Eq. (4.31) represents the first moment with respect to the horizontal axis of the area in Fig. 4.32 that is located above the horizontal axis and is bounded by the stress-distribution curve and the vertical axis. An important value of the bending moment is the ultimate bending moment MU that causes failure of the member. This value can be determined from the ultimate strength sU of the material by choosing smax 5 sU and carrying out the computations indicated earlier. However, it is found more convenient in practice to determine MU experimentally for a specimen of a given material. Assuming a fictitious linear distribution of stresses, Eq. (4.15) is then used to determine the corresponding maximum stress RB: RB 5 c N. A. c b Fig. 4.34 Beam with rectangular cross section. MUc I (4.32) The fictitious stress RB is called the modulus of rupture in bending of the given material. It can be used to determine the ultimate bending moment MU of a member made of the same material and having a cross section of the same shape, but of different dimensions, by solving Eq. (4.32) for MU. Since, in the case of a member with a rectangular cross section, the actual and the fictitious linear stress distributions shown in Fig. 4.33 must yield the same value MU for the ultimate bending moment, the areas they define must have the same first moment with respect to the horizontal axis. It is thus clear that the modulus of rupture RB will always be larger than the actual ultimate strength sU.  Y Y Y Fig. 4.35 Idealized steel stress-strain diagram. *4.9 MEMBERS MADE OF AN ELASTOPLASTIC MATERIAL  In order to gain a better insight into the plastic behavior of a member in bending, let us consider the case of a member made of an elastoplastic material and first assume the member to have a rectangular cross section of width b and depth 2c (Fig. 4.34). We recall from Sec. 2.17 that the stress-strain diagram for an idealized elastoplastic material is as shown in Fig. 4.35. bee80288_ch04_220-313.indd Page 257 11/11/10 3:03:14 PM user-f499 /Users/user-f499/Desktop/Temp Work/Don't Delete Job/MHDQ251:Beer:201/ch04 4.9 Members Made of an Elastoplastic Material As long as the normal stress sx does not exceed the yield strength sY, Hooke’s law applies, and the stress distribution across the section is linear (Fig. 4.36a). The maximum value of the stress is Mc I sm 5 (4.15) As the bending moment increases, sm eventually reaches the value sY (Fig. 4.36b). Substituting this value into Eq. (4.15), and solving for the corresponding value of M, we obtain the value MY of the bending moment at the onset of yield: MY 5 I s c Y y (4.33) The moment MY is referred to as the maximum elastic moment, since it is the largest moment for which the deformation remains fully elastic. Recalling that, for the rectangular cross section considered here, b12c2 3 I 2 5 5 bc2 c 3 12c c ELASTIC c (4.34) 2 MY 5 bc2sY 3 (4.35) As the bending moment further increases, plastic zones develop in the member, with the stress uniformly equal to 2sY in the upper zone, and to 1sY in the lower zone (Fig. 4.36c). Between the plastic zones, an elastic core subsists, in which the stress sx varies linearly with y, sY y yY M 5 22b # 0 sY ya2 yb dy 2 2b yY y  c ELASTIC x c PLASTIC  Y 2 5 by2YsY 1 bc2sY 2 by2YsY 3 2 1 yY M 5 bc2sYa1 2 b 3 c2 (4.37) c PLASTIC (c) M  max   M y c x PLASTIC c or, in view of Eq. (4.35), 2 3 1 yY b M 5 MY a1 2 2 3 c2 c x  yY y ELASTIC c # y12s 2 dy  max   m   (b) M  M (4.36) where yY represents half the thickness of the elastic core. As M increases, the plastic zones expand until, at the limit, the deformation is fully plastic (Fig. 4.36d). Equation (4.31) will be used to determine the value of the bending moment M corresponding to a given thickness 2yY of the elastic core. Recalling that sx is given by Eq. (4.36) for 0 # y # yY, and is equal to 2sY for yY # y # c, we write yY  max   m   (a) M  M we write sx 5 2 x  (d) M  Mp (4.38) Fig. 4.36 Bending stress distribution in beam for different moments. 257 bee80288_ch04_220-313.indd Page 258 10/27/10 5:46:17 PM user-f499 258 /Users/user-f499/Desktop/Temp Work/Don't Delete Job/MHDQ251:Beer:201/ch04 where MY is the maximum elastic moment. Note that as yY approaches zero, the bending moment approaches the limiting value Pure Bending Mp 5 3 MY 2 (4.39) This value of the bending moment, which corresponds to a fully plastic deformation (Fig. 4.36d), is called the plastic moment of the member considered. Note that Eq. (4.39) is valid only for a rectangular member made of an elastoplastic material. You should keep in mind that the distribution of strain across the section remains linear after the onset of yield. Therefore, Eq. (4.8) of Sec. 4.3 remains valid and can be used to determine the half-thickness yY of the elastic core. We have yY 5 PYr (4.40) where PY is the yield strain and r the radius of curvature corresponding to a bending moment M $ MY. When the bending moment is equal to MY, we have yY 5 c and Eq. (4.40) yields c 5 PYrY where rY is the radius of curvature corresponding to the maximum elastic moment MY. Dividing (4.40) by (4.41) member by member, we obtain the relation† yY r 5 (4.42) rY c y  Y b c RY c 2c/3 z x 2c/3 R'Y m  Y (a)  Y y b c Rp c z c/2 c/2 x R'p Y Fig. 4.37 Stress distributions in beam at maximum elastic moment and at plastic moment. (b) (4.41) Substituting for yYyc from (4.42) into Eq. (4.38), we express the bending moment M as a function of the radius of curvature r of the neutral surface: 3 1 r2 b M 5 MY a1 2 (4.43) 2 3 r2Y Note that Eq. (4.43) is valid only after the onset of yield, i.e., for values of M larger than MY. For M , MY, Eq. (4.21) of Sec. 4.4 should be used. We observe from Eq. (4.43) that the bending moment reaches the value Mp 5 32 MY only when r 5 0. Since we clearly cannot have a zero radius of curvature at every point of the neutral surface, we conclude that a fully plastic deformation cannot develop in pure bending. As you will see in Chap. 5, however, such a situation may occur at one point in the case of a beam under a transverse loading. The stress distributions in a rectangular member corresponding respectively to the maximum elastic moment MY and to the limiting case of the plastic moment Mp have been represented in three dimensions in Fig. 4.37. Since, in both cases, the resultants of the elementary tensile and compressive forces must pass through the centroids of the volumes representing the stress distributions and be equal in magnitude to these volumes, we check that RY 5 12 bcsY †Equation (4.42) applies to any member made of any ductile material with a well-defined yield point, since its derivation is independent of the shape of the cross section and of the shape of the stress-strain diagram beyond the yield point. bee80288_ch04_220-313.indd Page 259 10/27/10 5:46:18 PM user-f499 /Users/user-f499/Desktop/Temp Work/Don't Delete Job/MHDQ251:Beer:201/ch04 4.9 Members Made of an Elastoplastic Material and Rp 5 bcsY and that the moments of the corresponding couples are, respectively, MY 5 1 43 c2RY 5 23bc2sY (4.44) Mp 5 cRp 5 bc2sY (4.45) and We thus verify that, for a rectangular member, Mp 5 32 MY as required by Eq. (4.39). For beams of nonrectangular cross section, the computation of the maximum elastic moment MY and of the plastic moment Mp will usually be simplified if a graphical method of analysis is used, as shown in Sample Prob. 4.5. It will be found in this more general case that the ratio k 5 MpyMY is generally not equal to 32. For structural shapes such as wide-flange beams, for example, this ratio varies approximately from 1.08 to 1.14. Because it depends only upon the shape of the cross section, the ratio k 5 MpyMY is referred to as the shape factor of the cross section. We note that, if the shape factor k and the maximum elastic moment MY of a beam are known, the plastic moment Mp of the beam can be obtained by multiplying MY by k: Mp 5 kMY (4.46) The ratio MpysY obtained by dividing the plastic moment Mp of a member by the yield strength sY of its material is called the plastic section modulus of the member and is denoted by Z. When the plastic section modulus Z and the yield strength sY of a beam are known, the plastic moment Mp of the beam can be obtained by multiplying sY by Z: Mp 5 ZsY (4.47) Recalling from Eq. (4.18) that MY 5 SsY, and comparing this relation with Eq. (4.47), we note that the shape factor k 5 MpyMY of a given cross section can be expressed as the ratio of the plastic and elastic section moduli: Mp ZsY Z k5 5 5 (4.48) MY SsY S Considering the particular case of a rectangular beam of width b and depth h, we note from Eqs. (4.45) and (4.47) that the plastic section modulus of a rectangular beam is Mp bc2sY 5 5 bc2 5 14 bh2 Z5 sY sY On the other hand, we recall from Eq. (4.19) of Sec. 4.4 that the elastic section modulus of the same beam is S 5 16 bh2 Substituting into Eq. (4.48) the values obtained for Z and S, we verify that the shape factor of a rectangular beam is k5 1 2 Z 3 4 bh 51 25 S 2 6 bh 259 bee80288_ch04_220-313.indd Page 260 11/11/10 3:03:14 PM user-f499 EXAMPLE 4.05 b  50 mm c  60 mm c  60 mm yY /Users/user-f499/Desktop/Temp Work/Don't Delete Job/MHDQ251:Beer:201/ch04 A member of uniform rectangular cross section 50 3 120 mm (Fig. 4.38) is subjected to a bending moment M 5 36.8 kN ? m. Assuming that the member is made of an elastoplastic material with a yield strength of 240 MPa and a modulus of elasticity of 200 GPa, determine (a) the thickness of the elastic core, (b) the radius of curvature of the neutral surface. (a) Thickness of Elastic Core. We first determine the maximum elastic moment MY. Substituting the given data into Eq. (4.34), we have I 2 2 5 bc2 5 150 3 1023 m2 160 3 1023 m2 2 c 3 3 5 120 3 1026 m3 and carrying this value, as well as sY 5 240 MPa, into Eq. (4.33), I MY 5 sY 5 1120 3 1026 m3 2 1240 MPa2 5 28.8 kN ? m c Fig. 4.38 Substituting the values of M and MY into Eq. (4.38), we have 2 3 1 yY 128.8 kN ? m2a1 2 b 2 3 c2 yY 2 yY a b 5 0.444 5 0.666 c c 36.8 kN ? m 5 and, since c 5 60 mm, yY 5 0.666(60 mm) 5 40 mm The thickness 2yY of the elastic core is thus 80 mm. (b) Radius of Curvature. PY 5 We note that the yield strain is sY 240 3 106 Pa 5 5 1.2 3 1023 E 200 3 109 Pa Solving Eq. (4.40) for r and substituting the values obtained for yY and PY, we write yY 40 3 1023 m 5 5 33.3 m r5 PY 1.2 3 1023 *4.10 PLASTIC DEFORMATIONS OF MEMBERS WITH A SINGLE PLANE OF SYMMETRY In our discussion of plastic deformations, we have assumed so far that the member in bending had two planes of symmetry, one containing the couples M and M9, and the other perpendicular to that plane. Let us now consider the more general case when the member possesses only one plane of symmetry containing the couples M and M9. However, our analysis will be limited to the situation where the deformation is fully plastic, with the normal stress uniformly equal to 2sY above the neutral surface, and to 1sY below that surface (Fig. 4.39a). As indicated in Sec. 4.8, the neutral axis cannot be assumed to coincide with the centroidal axis of the cross section when the 260 bee80288_ch04_220-313.indd Page 261 10/26/10 4:29:34 PM user-f499 /Volumes/201/MHDQ251/bee80288_disk1of1/0073380288/bee80288_pagefiles cross section is not symmetric with respect to that axis. To locate the neutral axis, we consider the resultant R1 of the elementary compressive forces exerted on the portion A1 of the cross section located above the neutral axis, and the resultant R2 of the tensile forces exerted on the portion A2 located below the neutral axis (Fig. 4.39b). Since the forces R1 and R2 form a couple equivalent to the couple applied to the member, they must have the same magnitude. We have therefore R1 5 R2, or A1sY 5 A2sY, from which we conclude that A1 5 A2. In other words, the neutral axis divides the cross section into portions of equal areas. Note that the axis obtained in this fashion will not, in general, be a centroidal axis of the section. We also observe that the lines of action of the resultants R1 and R2 pass through the centroids C1 and C2 of the two portions we have just defined. Denoting by d the distance between C1 and C2, and by A the total area of the cross section, we express the plastic moment of the member as 4.11 Residual Stresses  Y Neutral surface  Y (a) Mp 5 1 12AsY 2 d C1 An example of the actual computation of the plastic moment of a member with only one plane of symmetry is given in Sample Prob. 4.6. *4.11 RESIDUAL STRESSES We saw in the preceding sections that plastic zones will develop in a member made of an elastoplastic material if the bending moment is large enough. When the bending moment is decreased back to zero, the corresponding reduction in stress and strain at any given point can be represented by a straight line on the stress-strain diagram, as shown in Fig. 4.40. As you will see presently, the final value of the stress at a point will not, in general, be zero. There will be a residual stress at most points, and that stress may or may not have the same sign as the maximum stress reached at the end of the loading phase. Since the linear relation between s x and Px applies at all points of the member during the unloading phase, Eq. (4.16) can be used to obtain the change in stress at any given point. In other words, the unloading phase can be handled by assuming the member to be fully elastic. The residual stresses are obtained by applying the principle of superposition in a manner similar to that described in Sec. 2.20 for an axial centric loading and used again in Sec. 3.11 for torsion. We consider, on one hand, the stresses due to the application of the given bending moment M and, on the other, the reverse stresses due to the equal and opposite bending moment 2M that is applied to unload the member. The first group of stresses reflect the elastoplastic behavior of the material during the loading phase, and the second group the linear behavior of the same material during the unloading phase. Adding the two groups of stresses, we obtain the distribution of residual stresses in the member. d A1 . N.A R1 C2 A2 R2 (b) Fig. 4.39 Nonsymmetrical beam subject to plastic moment. x Y Y  Y Fig. 4.40 Elastoplastic material stress-strain diagram. x 261 bee80288_ch04_220-313.indd Page 262 11/11/10 3:03:15 PM user-f499 EXAMPLE 4.06 /Users/user-f499/Desktop/Temp Work/Don't Delete Job/MHDQ251:Beer:201/ch04 For the member of Example 4.05, determine (a) the distribution of the residual stresses, (b) the radius of curvature, after the bending moment has been decreased from its maximum value of 36.8 kN ? m back to zero. (a) Distribution of Residual Stresses. We recall from Example 4.05 that the yield strength is sY 5 240 MPa and that the thickness of the elastic core is 2yY 5 80 mm. The distribution of the stresses in the loaded member is thus as shown in Fig. 4.41a. The distribution of the reverse stresses due to the opposite 36.8 kN ? m bending moment required to unload the member is linear and as shown in Fig. 4.41b. The maximum stress s9m in that distribution is obtained from Eq. (4.15). Recalling from Example 4.05 that Iyc 5 120 3 1026 m3, we write s¿m 5 36.8 kN ? m Mc 5 5 306.7 MPa I 120 3 1026 m3 Superposing the two distributions of stresses, we obtain the residual stresses shown in Fig. 4.41c. We check that, even though the reverse stresses exceed the yield strength sY, the assumption of a linear distribution of the reverse stresses is valid, since they do not exceed 2sY. (b) Radius of Curvature after Unloading. We can apply Hooke’s law at any point of the core |y| , 40 mm, since no plastic deformation has occurred in that portion of the member. Thus, the residual strain at the distance y 5 40 mm is Px 5 sx 235.5 3 106 Pa 5 5 2177.5 3 1026 E 200 3 109 Pa Solving Eq. (4.8) for r and substituting the appropriate values of y and Px, we write r52 y Px 5 40 3 1023 m 5 225 m 177.5 3 1026 The value obtained for r after the load has been removed represents a permanent deformation of the member. y(mm) y(mm) 60 60 40 40  'm 204.5 306.7 – 40 –60 (a) Fig. 4.41 262 60 40 240  x(MPa) –240 y(mm) x –35.5 66.7 –40 Y –60 (b) –60 (c)  x(MPa) bee80288_ch04_220-313.indd Page 263 10/26/10 4:29:42 PM user-f499 /Volumes/201/MHDQ251/bee80288_disk1of1/0073380288/bee80288_pagefiles SAMPLE PROBLEM 4.5 B Beam AB has been fabricated from a high-strength low-alloy steel that is assumed to be elastoplastic with E 5 29 3 106 psi and sY 5 50 ksi. Neglecting the effect of fillets, determine the bending moment M and the corresponding radius of curvature (a) when yield first occurs, (b) when the flanges have just become fully plastic. A 1 in. 3 4 16 in. in. M SOLUTION 1 in. 12 in. a. Onset of Yield. The centroidal moment of inertia of the section is I 5 121 112 in.2 116 in.2 3 2 121 112 in. 2 0.75 in.2 114 in.2 3 5 1524 in4 Bending Moment.  Y  50 ksi O MY 5 1 E  Y  0.001724 y 8 in. z C 8 in. Strain distribution 150 ksi2 11524 in4 2 sYI 5 c 8 in. M Y 5 9525 kip ? in. ◀ Radius of Curvature. Noting that, at c 5 8 in., the strain is PY 5 sYyE 5 (50 ksi)/(29 3 106 psi) 5 0.001724, we have from Eq. (4.41)   Y  0.001724 For smax 5 sY 5 50 ksi and c 5 8 in., we have c 5 PYrY Y 8 in. 5 0.001724rY rY 5 4640 in. ◀ b. Flanges Fully Plastic. When the flanges have just become fully plastic, the strains and stresses in the section are as shown in the figure below. We replace the elementary compressive forces exerted on the top flange and on the top half of the web by their resultants R1 and R2, and Stress similarly replace the tensile forces by R3 and R4. distribution R1 5 R4 5 (50 ksi)(12 in.)(1 in.) 5 600 kips R2 5 R3 5 12 150 ksi2 17 in.2 10.75 in.2 5 131.3 kips 3 4 in. 1 in.  Y  0.001724 7 in. 7 in. Y  50 ksi R1 R2 7.5 in. 4.67 in. C z 7 in. 7 in. 4.67 in. 7.5 in. R3 1 in. Y R4 Strain distribution Stress distribution Resultant force Bending Moment. Summing the moments of R1, R2, R3, and R4 about the z axis, we write M 5 2[R1(7.5 in.) 1 R 2(4.67 in.)] 5 2[(600)(7.5) 1 (131.3)(4.67)] Radius of Curvature. Eq. (4.40) yY 5 PYr M 5 10,230 kip ? in. ◀ Since yY 5 7 in. for this loading, we have from 7 in. 5 (0.001724)r r 5 4060 in. 5 338 ft ◀ 263 bee80288_ch04_220-313.indd Page 264 10/27/10 5:46:19 PM user-f499 SAMPLE PROBLEM 4.6 100 mm 20 mm 20 mm /Users/user-f499/Desktop/Temp Work/Don't Delete Job/MHDQ251:Beer:201/ch04 80 mm Determine the plastic moment Mp of a beam with the cross section shown when the beam is bent about a horizontal axis. Assume that the material is elastoplastic with a yield strength of 240 MPa. 20 mm SOLUTION 60 mm Neutral Axis. When the deformation is fully plastic, the neutral axis divides the cross section into two portions of equal areas. Since the total area is A 5 (100)(20) 1 (80)(20) 1 (60)(20) 5 4800 mm2 the area located above the neutral axis must be 2400 mm2. We write (20)(100) 1 20y 5 2400 100 mm y 5 20 mm Note that the neutral axis does not pass through the centroid of the cross section. 20 mm y Neutral axis 20 mm Plastic Moment. The resultant Ri of the elementary forces exerted on the partial area Ai is equal to Ri 5 AisY and passes through the centroid of that area. We have R1 R2 R3 R4 Y  240 MPa 100 mm 20 mm z 20 mm 3 10.100 m2 10.020 m2 4 240 MPa 3 10.020 m2 10.020 m2 4 240 MPa 3 10.020 m2 10.060 m2 4 240 MPa 3 10.060 m2 10.020 m2 4 240 MPa R1 R2 A2 20 mm A3 A4 5 480 kN 5 96 kN 5 288 kN 5 288 kN y A1 20 mm 60 mm 5 A1sY 5 5 A2sY 5 5 A3sY 5 5 A4sY 5 z R3 10 mm 30 mm 30 mm x 70 mm R4 60 mm The plastic moment Mp is obtained by summing the moments of the forces about the z axis. Mp 5 10.030 m2R1 1 10.010 m2R2 1 10.030 m2R3 1 10.070 m2R4 5 10.030 m2 1480 kN2 1 10.010 m2 196 kN2 110.030 m2 1288 kN2 1 10.070 m2 1288 kN2 5 44.16 kN ? m Mp 5 44.2 kN ? m ◀ Note: Since the cross section is not symmetric about the z axis, the sum of the moments of R1 and R2 is not equal to the sum of the moments of R3 and R4. 264 bee80288_ch04_220-313.indd Page 265 11/11/10 3:03:16 PM user-f499 /Users/user-f499/Desktop/Temp Work/Don't Delete Job/MHDQ251:Beer:201/ch04 SAMPLE PROBLEM 4.7 For the beam of Sample Prob. 4.5, determine the residual stresses and the permanent radius of curvature after the 10,230-kip ? in. couple M has been removed. SOLUTION Loading. In Sample Prob. 4.5 a couple of moment M 5 10,230 kip ? in. was applied and the stresses shown in Fig. 1 were obtained. Elastic Unloading. The beam is unloaded by the application of a couple of moment M 5 210,230 kip ? in. (which is equal and opposite to the couple originally applied). During this unloading, the action of the beam is fully elastic; recalling from Sample Prob. 4.5 that I 5 1524 in4, we compute the maximum stress s¿m 5 110,230 kip ? in.2 18 in.2 Mc 5 5 53.70 ksi I 1524 in4 The stresses caused by the unloading are shown in Fig. 2. Residual Stresses. We superpose the stresses due to the loading (Fig. 1) and to the unloading (Fig. 2) and obtain the residual stresses in the beam (Fig. 3). 10,230 kip · in. M  10,230 kip · in.  'm  53.70 ksi Y  50 ksi 8 in. 7 in. 8 in. 7 in. 3.01 ksi 3.70 ksi   46.99 ksi 3.01 ksi (1)   3.70 ksi (tension) (2) Permanent Radius of Curvature. At y 5 7 in. the residual stress is s 5 23.01 ksi. Since no plastic deformation occurred at this point, Hooke’s law can be used and we have Px 5 syE. Recalling Eq. (4.8), we write r52   3.70 ksi (compression) 3.70 ksi (3) y Px 52 yE s 52 17 in.2 129 3 106 psi2 23.01 ksi 5 167,400 in. r 5 5620 ft ◀ We note that the residual stress is tensile on the upper face of the beam and compressive on the lower face, even though the beam is concave upward. 265 bee80288_ch04_220-313.indd Page 266 10/26/10 4:30:06 PM user-f499 /Volumes/201/MHDQ251/bee80288_disk1of1/0073380288/bee80288_pagefiles PROBLEMS M 4.67 The prismatic bar shown is made of a steel that is assumed to be elastoplastic with sY 5 300 MPa and is subjected to a couple M parallel to the x axis. Determine the moment M of the couple for which (a) yield first occurs, (b) the elastic core of the bar is 4 mm thick. x z 12 mm 8 mm 4.68 Solve Prob. 4.67, assuming that the couple M is parallel to the z axis. 4.69 The prismatic bar shown, made of a steel that is assumed to be elastoplastic with E 5 29 3 106 psi and sY 5 36 ksi, is subjected to a couple of 1350 lb ? in. parallel to the z axis. Determine (a) the thickness of the elastic core, (b) the radius of curvature of the bar. Fig. P4.67 y 1 2 in. M 5 8 in. z Fig. P4.69 4.70 Solve Prob. 4.69, assuming that the 1350-lb ? in. couple is parallel to the y axis. 4.71 A bar of rectangular cross section shown is made of a steel that is assumed to be elastoplastic with E 5 200 GPa and sY 5 300 MPa. Determine the bending moment M for which (a) yield first occurs, (b) the plastic zones at the top and bottom of the bar are 12 mm thick. 30 mm M' M 40 mm Fig. P4.71 and P4.72 4.72 Bar AB is made of a steel that is assumed to be elastoplastic with E 5 200 GPa and sY 5 240 MPa. Determine the bending moment M for which the radius of curvature of the bar will be (a) 18 m, (b) 9 m. 266 bee80288_ch04_220-313.indd Page 267 10/27/10 5:46:20 PM user-f499 /Users/user-f499/Desktop/Temp Work/Don't Delete Job/MHDQ251:Beer:201/ch04 4.73 and 4.74 A beam of the cross section shown is made of a steel that is assumed to be elastoplastic with E 5 200 GPa and sY 5 240 MPa. For bending about the z axis, determine the bending moment at which (a) yield first occurs, (b) the plastic zones at the top and bottom of the bar are 30 mm thick. y y 30 mm z C 90 mm z 30 mm C 30 mm 60 mm 15 mm Fig. P4.73 15 mm 30 mm Fig. P4.74 4.75 and 4.76 A beam of the cross section shown is made of a steel that is assumed to be elastoplastic with E 5 29 3 106 psi and sY 5 42 ksi. For bending about the z axis, determine the bending moment at which (a) yield first occurs, (b) the plastic zones at the top and bottom of the bar are 3 in. thick. y y 3 in. C z 3 in. 3 in. z C 3 in. 3 in. 1.5 in. Fig. P4.75 3 in. 1.5 in. 3 in. 1.5 in. 3 in. 1.5 in. Fig. P4.76 4.77 through 4.80 For the beam indicated, determine (a) the plastic moment Mp, (b) the shape factor of the cross section. 4.77 Beam of Prob. 4.73. 4.78 Beam of Prob. 4.74. 4.79 Beam of Prob. 4.75. 4.80 Beam of Prob. 4.76. Problems 267 bee80288_ch04_220-313.indd Page 268 10/27/10 5:46:22 PM user-f499 268 /Users/user-f499/Desktop/Temp Work/Don't Delete Job/MHDQ251:Beer:201/ch04 4.81 through 4.84 Determine the plastic moment Mp of a steel beam of the cross section shown, assuming the steel to be elastoplastic with a yield strength of 240 MPa. Pure Bending 50 mm 36 mm 30 mm r  18 mm 10 mm 10 mm Fig. P4.81 30 mm Fig. P4.82 0.6 in. 0.6 in. 0.6 in. 1.2 in. Fig. P4.85 4 in. in. 1 2 in. 1 2 in. 3 in. 2 in. Fig. P4.86 y 1 in. z C 2 in. 1 in. 1 in. Fig. P4.92 1 in. 10 mm 30 mm Fig. P4.83 60 mm Fig. P4.84 4.85 and 4.86 Determine the plastic moment Mp of the cross section shown, assuming the steel to be elastoplastic with a yield strength of 36 ksi. 4.87 and 4.88 For the beam indicated, a couple of moment equal to the full plastic moment Mp is applied and then removed. Using a yield strength of 240 MPa, determine the residual stress at y 5 45 mm. 4.87 Beam of Prob. 4.73. 4.88 Beam of Prob. 4.74. 0.4 in. 1 2 40 mm 1 in. 4.89 and 4.90 A bending couple is applied to the bar indicated, causing plastic zones 3 in. thick to develop at the top and bottom of the bar. After the couple has been removed, determine (a) the residual stress at y 5 4.5 in., (b) the points where the residual stress is zero, (c) the radius of curvature corresponding to the permanent deformation of the bar. 4.89 Beam of Prob. 4.75. 4.90 Beam of Prob. 4.76. 4.91 A bending couple is applied to the beam of Prob. 4.73, causing plastic zones 30 mm thick to develop at the top and bottom of the beam. After the couple has been removed, determine (a) the residual stress at y 5 45 mm, (b) the points where the residual stress is zero, (c) the radius of curvature corresponding to the permanent deformation of the beam. 4.92 A beam of the cross section shown is made of a steel that is assumed to be elastoplastic with E 5 29 3 106 psi and sY 5 42 ksi. A bending couple is applied to the beam about the z axis, causing plastic zones 2 in. thick to develop at the top and bottom of the beam. After the couple has been removed, determine (a) the residual stress at y 5 2 in., (b) the points where the residual stress is zero, (c) the radius of curvature corresponding to the permanent deformation of the beam. 4.93 A rectangular bar that is straight and unstressed is bent into an arc of circle of radius r by two couples of moment M. After the couples are removed, it is observed that the radius of curvature of the bar is rR. Denoting by rY the radius of curvature of the bar at the onset of yield, show that the radii of curvature satisfy the following relation: 1 1 3 r 1 r 2 5 e1 2 c1 2 a b d f rR r 2 rY 3 rY bee80288_ch04_220-313.indd Page 269 10/26/10 4:30:32 PM user-f499 /Volumes/201/MHDQ251/bee80288_disk1of1/0073380288/bee80288_pagefiles Problems 4.94 A solid bar of rectangular cross section is made of a material that is assumed to be elastoplastic. Denoting by MY and rY, respectively, the bending moment and radius of curvature at the onset of yield, determine (a) the radius of curvature when a couple of moment M 5 1.25 MY is applied to the bar, (b) the radius of curvature after the couple is removed. Check the results obtained by using the relation derived in Prob. 4.93. 4.95 The prismatic bar AB is made of a steel that is assumed to be elastoplastic and for which E 5 200 GPa. Knowing that the radius of curvature of the bar is 2.4 m when a couple of moment M 5 350 N ? m is applied as shown, determine (a) the yield strength of the steel, (b) the thickness of the elastic core of the bar. 4.96 The prismatic bar AB is made of an aluminum alloy for which the tensile stress-strain diagram is as shown. Assuming that the s-P diagram is the same in compression as in tension, determine (a) the radius of curvature of the bar when the maximum stress is 250 MPa, (b) the corresponding value of the bending moment. (Hint: For part b, plot s versus y and use an approximate method of integration.) M B A 20 mm 16 mm Fig. P4.95  (MPa) 300 40 mm 200 B 0.8 in. M' M 60 mm  (ksi) A 100 B M 1.2 in. A 50 0 0.005 0.010  Fig. P4.96 4.97 The prismatic bar AB is made of a bronze alloy for which the tensile stress-strain diagram is as shown. Assuming that the s-P diagram is the same in compression as in tension, determine (a) the maximum stress in the bar when the radius of curvature of the bar is 100 in., (b) the corresponding value of the bending moment. (See hint given in Prob. 4.96.) 4.98 A prismatic bar of rectangular cross section is made of an alloy for which the stress-strain diagram can be represented by the relation P 5 ksn for s . 0 and P 5 2|ksn| for s , 0. If a couple M is applied to the bar, show that the maximum stress is sm 5   M Fig. P4.98 1 1 2n Mc 3n I 40 30 20 10 0 Fig. P4.97 0.004 0.008  269 bee80288_ch04_220-313.indd Page 270 10/26/10 4:30:40 PM user-f499 270 4.12 Pure Bending /Volumes/201/MHDQ251/bee80288_disk1of1/0073380288/bee80288_pagefiles ECCENTRIC AXIAL LOADING IN A PLANE OF SYMMETRY We saw in Sec. 1.5 that the distribution of stresses in the cross section of a member under axial loading can be assumed uniform only if the line of action of the loads P and P9 passes through the centroid of the cross section. Such a loading is said to be centric. Let us now analyze the distribution of stresses when the line of action of the loads does not pass through the centroid of the cross section, i.e., when the loading is eccentric. Two examples of an eccentric loading are shown in Photos 4.5 and 4.6. In the case of the walkway light, the weight of the lamp causes an eccentric loading on the post. Likewise, the vertical forces exerted on the press cause an eccentric loading on the back column of the press. D d E C P' A P M D F C P' A d (b) Fig. 4.42 loading. Member with eccentric M' D E C P' M' D P' Photo 4.5 Photo 4.6 B (a) M P (a) F5P M C In this section, our analysis will be limited to members that possess a plane of symmetry, and it will be assumed that the loads are applied in the plane of symmetry of the member (Fig. 4.42a). The internal forces acting on a given cross section may then be represented by a force F applied at the centroid C of the section and a couple M acting in the plane of symmetry of the member (Fig. 4.42b). The conditions of equilibrium of the free body AC require that the force F be equal and opposite to P9 and that the moment of the couple M be equal and opposite to the moment of P9 about C. Denoting by d the distance from the centroid C to the line of action AB of the forces P and P9, we have FP (b) Fig. 4.43 Internal forces in member with eccentric loading. and M 5 Pd (4.49) We now observe that the internal forces in the section would have been represented by the same force and couple if the straight portion DE of member AB had been detached from AB and subjected simultaneously to the centric loads P and P9 and to the bending couples M and M9 (Fig. 4.43). Thus, the stress distribution due bee80288_ch04_220-313.indd Page 271 10/26/10 4:30:46 PM user-f499 /Volumes/201/MHDQ251/bee80288_disk1of1/0073380288/bee80288_pagefiles 4.12 Eccentric Axial Loading in a Plane of Symmetry to the original eccentric loading can be obtained by superposing the uniform stress distribution corresponding to the centric loads P and P9 and the linear distribution corresponding to the bending couples M and M9 (Fig. 4.44). We write s x 5 1s x 2 centric 1 1s x 2 bending y C Fig. 4.44 y x + C y x = C x Stress distribution—eccentric loading. or, recalling Eqs. (1.5) and (4.16): sx 5 My P 2 A I (4.50) where A is the area of the cross section and I its centroidal moment of inertia, and where y is measured from the centroidal axis of the cross section. The relation obtained shows that the distribution of stresses across the section is linear but not uniform. Depending upon the geometry of the cross section and the eccentricity of the load, the combined stresses may all have the same sign, as shown in Fig. 4.44, or some may be positive and others negative, as shown in Fig. 4.45. In the latter case, there will be a line in the section, along which sx 5 0. This line represents the neutral axis of the section. We note that the neutral axis does not coincide with the centroidal axis of the section, since sx Z 0 for y 5 0. y C Fig. 4.45 y y x + C x = N.A. C Alternative stress distribution—eccentric loading. The results obtained are valid only to the extent that the conditions of applicability of the superposition principle (Sec. 2.12) and of Saint-Venant’s principle (Sec. 2.17) are met. This means that the stresses involved must not exceed the proportional limit of the material, that the deformations due to bending must not appreciably affect the distance d in Fig. 4.42a, and that the cross section where the stresses are computed must not be too close to points D or E in the same figure. The first of these requirements clearly shows that the superposition method cannot be applied to plastic deformations. x 271 bee80288_ch04_220-313.indd Page 272 11/11/10 3:03:16 PM user-f499 EXAMPLE 4.07 160 lb 0.5 in. /Users/user-f499/Desktop/Temp Work/Don't Delete Job/MHDQ251:Beer:201/ch04 An open-link chain is obtained by bending low-carbon steel rods of 0.5-in. diameter into the shape shown (Fig. 4.46). Knowing that the chain carries a load of 160 lb, determine (a) the largest tensile and compressive stresses in the straight portion of a link, (b) the distance between the centroidal and the neutral axis of a cross section. (a) Largest Tensile and Compressive Stresses. The internal forces in the cross section are equivalent to a centric force P and a bending couple M (Fig. 4.47) of magnitudes P 5 160 lb M 5 Pd 5 1160 lb2 10.65 in.2 5 104 lb ? in. 0.65 in. The corresponding stress distributions are shown in parts a and b of Fig. 4.48. The distribution due to the centric force P is uniform and equal to s0 5 PyA. We have A 5 pc2 5 p10.25 in.2 2 5 0.1963 in2 P 160 lb s0 5 5 5 815 psi A 0.1963 in2 160 lb Fig. 4.46 d  0.65 in. x P 8475 psi x 9290 psi x 815 psi M N.A. C y C + C y = – 7660 psi – 8475 psi 160 lb Fig. 4.47 (a) (b) y C (c) Fig. 4.48 The distribution due to the bending couple M is linear with a maximum stress sm 5 McyI. We write I 5 14 pc4 5 14 p10.25 in.2 4 5 3.068 3 1023 in4 1104 lb ? in.2 10.25 in.2 Mc 5 sm 5 5 8475 psi I 3.068 3 1023 in4 Superposing the two distributions, we obtain the stress distribution corresponding to the given eccentric loading (Fig. 4.48c). The largest tensile and compressive stresses in the section are found to be, respectively, s t 5 s 0 1 s m 5 815 1 8475 5 9290 psi s c 5 s 0 2 s m 5 815 2 8475 5 27660 psi (b) Distance Between Centroidal and Neutral Axes. The distance y0 from the centroidal to the neutral axis of the section is obtained by setting sx 5 0 in Eq. (4.50) and solving for y0: My0 P 2 A I P I 3.068 3 1023 in4 y0 5 a b a b 5 1815 psi2 104 lb ? in. A M y0 5 0.0240 in. 05 272 bee80288_ch04_220-313.indd Page 273 11/11/10 3:03:17 PM user-f499 SAMPLE PROBLEM 4.8 a A P P' D B 10 mm a 90 mm A C Properties of Cross Section. We now write: B d 5 (0.038 m) 2 (0.010 m) 5 0.028 m Force and Couple at C. system at the centroid C. 30 mm Section a– a P5P A cA  0.022 m C d cB  0.038 m D B 0.010 m A A C D B d M P B A 0 C P C A McA 1 I B (1) B McB 2 I (2) A A C C B From Sample Prob. 4.2, we have A 5 3000 mm2 5 3 3 1023 m2 Y 5 38 mm 5 0.038 m I 5 868 3 1029 m4 40 mm D 10 mm Knowing that for the cast iron link shown the allowable stresses are 30 MPa in tension and 120 MPa in compression, determine the largest force P which can be applied to the link. (Note: The T-shaped cross section of the link has previously been considered in Sample Prob. 4.2.) SOLUTION 20 mm  /Users/user-f499/Desktop/Temp Work/Don't Delete Job/MHDQ251:Beer:201/ch04 B We replace P by an equivalent force-couple M 5 P(d) 5 P(0.028 m) 5 0.028P The force P acting at the centroid causes a uniform stress distribution (Fig. 1). The bending couple M causes a linear stress distribution (Fig. 2). P P 5 5 333P 1Compression2 A 3 3 1023 10.028P2 10.0222 McA 5 710P 1Tension2 5 s1 5 I 868 3 1029 10.028P2 10.0382 McB 5 1226P 1Compression2 5 s2 5 I 868 3 1029 s0 5 Superposition. The total stress distribution (Fig. 3) is found by superposing the stress distributions caused by the centric force P and by the couple M. Since tension is positive, and compression negative, we have McA P 1 5 2333P 1 710P 5 1377P A I McB P 5 2333P 2 1226P 5 21559P sB 5 2 2 I A sA 5 2 1Compression2 Largest Allowable Force. The magnitude of P for which the tensile stress at point A is equal to the allowable tensile stress of 30 MPa is found by writing s A 5 377P 5 30 MPa P 5 79.6 kN ◀ We also determine the magnitude of P for which the stress at B is equal to the allowable compressive stress of 120 MPa. s B 5 21559P 5 2120 MPa (3) 1Tension2 P 5 77.0 kN ◀ The magnitude of the largest force P that can be applied without exceeding either of the allowable stresses is the smaller of the two values we have found. P 5 77.0 kN ◀ 273 bee80288_ch04_220-313.indd Page 274 10/26/10 4:31:12 PM user-f499 /Volumes/201/MHDQ251/bee80288_disk1of1/0073380288/bee80288_pagefiles PROBLEMS 4.99 A short wooden post supports a 6-kip axial load as shown. Determine the stress at point A when (a) b 5 0, (b) b 5 1.5 in., (c) b 5 3 in. y b 3 in. 6 kips 4.100 As many as three axial loads each of magnitude P 5 10 kips can be applied to the end of a W8 3 21 rolled-steel shape. Determine the stress at point A, (a) for the loading shown, (b) if loads are applied at points 1 and 2 only. C A x z A Fig. P4.99 P 30 mm 3.5 in. 3.5 in. B 24 mm D 45 mm P 15 mm 2 3 P 60 kN 150 mm 4.101 Knowing that the magnitude of the horizontal force P is 8 kN, determine the stress at (a) point A, (b) point B. 4.102 The vertical portion of the press shown consists of a rectangular tube of wall thickness t 5 10 mm. Knowing that the press has been tightened on wooden planks being glued together until P 5 20 kN, determine the stress at (a) point A, (b) point B. Fig. P4.101 1 C P Fig. P4.100 A 60 kN 1 60 kN 150 mm 3 2 t P P' a a t A 200 mm A 80 mm 60 mm 80 mm B Section a-a Fig. P4.102 B 4.103 Solve Prob. 4.102, assuming that t 5 8 mm. 120 mm Fig. P4.104 274 120 mm 90 mm 4.104 Determine the stress at points A and B, (a) for the loading shown, (b) if the 60-kN loads are applied at points 1 and 2 only. bee80288_ch04_220-313.indd Page 275 10/26/10 4:31:23 PM user-f499 /Volumes/201/MHDQ251/bee80288_disk1of1/0073380288/bee80288_pagefiles Problems 4.105 Knowing that the allowable stress in section ABD is 10 ksi, determine the largest force P that can be applied to the bracket shown. P 4.106 Portions of a 12 3 12-in. square bar have been bent to form the two machine components shown. Knowing that the allowable stress is 15 ksi, determine the maximum load that can be applied to each component. P P' P A D B P' 1 in. 0.9 in. 2 in. 0.6 in. 0.6 in. Fig. P4.105 (a) (b) Fig. P4.106 4.107 The four forces shown are applied to a rigid plate supported by a solid steel post of radius a. Knowing that P 5 100 kN and a 5 40 mm, determine the maximum stress in the post when (a) the force at D is removed, (b) the forces at C and D are removed. P P y P P B C D z a A x P' Fig. P4.107 4.108 A milling operation was used to remove a portion of a solid bar of square cross section. Knowing that a 5 30 mm, d 5 20 mm, and sall 5 60 MPa, determine the magnitude P of the largest forces that can be safely applied at the centers of the ends of the bar. a Fig. P4.108 and P4.109 12 kips 4.109 A milling operation was used to remove a portion of a solid bar of square cross section. Forces of magnitude P 5 18 kN are applied at the centers of the ends of the bar. Knowing that a 5 30 mm and sall 5 135 MPa, determine the smallest allowable depth d of the milled portion of the bar. 4.110 A short column is made by nailing two 1 3 4-in. planks to a 2 3 4-in. timber. Determine the largest compressive stress created in the column by a 12-kip load applied as shown at the center of the top section of the timber if (a) the column is as described, (b) plank 1 is removed, (c) both planks are removed. a d Fig. P4.110 P 275 bee80288_ch04_220-313.indd Page 276 10/26/10 4:31:35 PM user-f499 276 Pure Bending d P' P h P' P d /Volumes/201/MHDQ251/bee80288_disk1of1/0073380288/bee80288_pagefiles 4.111 An offset h must be introduced into a solid circular rod of diameter d. Knowing that the maximum stress after the offset is introduced must not exceed 5 times the stress in the rod when it is straight, determine the largest offset that can be used. 4.112 An offset h must be introduced into a metal tube of 0.75-in. outer diameter and 0.08-in. wall thickness. Knowing that the maximum stress after the offset is introduced must not exceed 4 times the stress in the tube when it is straight, determine the largest offset that can be used. 4.113 A steel rod is welded to a steel plate to form the machine element shown. Knowing that the allowable stress is 135 MPa, determine (a) the largest force P that can be applied to the element, (b) the corresponding location of the neutral axis. Given: The centroid of the cross section is at C and Iz 5 4195 mm4. Fig. P4.111 and P4.112 P' 6-mm diameter 18 mm 3 mm a C 13.12 mm a x Section a-a z P Fig. P4.113 4.114 A vertical rod is attached at point A to the cast iron hanger shown. Knowing that the allowable stresses in the hanger are sall 5 15 ksi and sall 5 212 ksi, determine the largest downward force and the largest upward force that can be exerted by the rod. 1 in. a 1.5 in. P A 3 in. 0.75 in. a 1.5 in. 3 in. 0.75 in. B Section a-a Fig. P4.114 50 mm 50 mm Fig. P4.116 4.115 Solve Prob. 4.114, assuming that the vertical rod is attached at point B instead of point A. 4.116 Three steel plates, each of 25 3 150-mm cross section, are welded together to form a short H-shaped column. Later, for architectural reasons, a 25-mm strip is removed from each side of one of the flanges. Knowing that the load remains centric with respect to the original cross section and that the allowable stress is 100 MPa, determine the largest force P (a) that could be applied to the original column, (b) that can be applied to the modified column. bee80288_ch04_220-313.indd Page 277 10/26/10 4:31:39 PM user-f499 /Volumes/201/MHDQ251/bee80288_disk1of1/0073380288/bee80288_pagefiles Problems 4.117 A vertical force P of magnitude 20 kips is applied at point C located on the axis of symmetry of the cross section of a short column. Knowing that y 5 5 in., determine (a) the stress at point A, (b) the stress at point B, (c) the location of the neutral axis. y P y B 3 in. y x 3 in. B 2 in. C A 4 in. A 2 in. x 2 in. 1 in. (a) (b) Fig. P4.117 and P4.118 4.118 A vertical force P is applied at point C located on the axis of symmetry of the cross section of a short column. Determine the range of values of y for which tensile stresses do not occur in the column. 4.119 Knowing that the clamp shown has been tightened until P 5 400 N, determine (a) the stress at point A, (b) the stress at point B, (c) the location of the neutral axis of section a-a. 2 mm radius A P P' 32 mm 20 mm a a P P P B P 4 mm Section a–a Fig. P4.119 4.120 The four bars shown have the same cross-sectional area. For the given loadings, show that (a) the maximum compressive stresses are in the ratio 4:5:7:9, (b) the maximum tensile stresses are in the ratio 2:3:5:3. (Note: the cross section of the triangular bar is an equilateral triangle.) Fig. P4.120 277 bee80288_ch04_220-313.indd Page 278 11/11/10 3:03:18 PM user-f499 278 Pure Bending P' /Users/user-f499/Desktop/Temp Work/Don't Delete Job/MHDQ251:Beer:201/ch04 4.121 The C-shaped steel bar is used as a dynamometer to determine the magnitude P of the forces shown. Knowing that the cross section of the bar is a square of side 40 mm and that the strain on the inner edge was measured and found to be 450 m, determine the magnitude P of the forces. Use E 5 200 GPa. 4.122 An eccentric force P is applied as shown to a steel bar of 25 3 90-mm cross section. The strains at A and B have been measured and found to be PA 5 1350 m 40 mm 80 mm PB 5 270 m Knowing that E 5 200 GPa, determine (a) the distance d, (b) the magnitude of the force P. 25 mm 30 mm P Fig. P4.121 A 90 mm P 45 mm B d 15 mm Fig. P4.122 4.123 Solve Prob. 4.122, assuming that the measured strains are PA 5 1600 m PB 5 1420 m 4.124 A short length of a W8 3 31 rolled-steel shape supports a rigid plate on which two loads P and Q are applied as shown. The strains at two points A and B on the centerline of the outer faces of the flanges have been measured and found to be PA 5 2550 3 1026 in./in. PB 5 2680 3 1026 in./in. Knowing that E 5 29 3 106 psi, determine the magnitude of each load. P 4.5 in. 4.5 in. A Q B b  40 mm A a  25 mm d B D P C 20 mm Fig. P4.126 Fig. P4.124 4.125 Solve Prob. 4.124, assuming that the measured strains are PA 5 135 3 1026 in./in. and PB 5 2450 3 1026 in./in. 4.126 The eccentric axial force P acts at point D, which must be located 25 mm below the top surface of the steel bar shown. For P 5 60 kN, determine (a) the depth d of the bar for which the tensile stress at point A is maximum, (b) the corresponding stress at point A. bee80288_ch04_220-313.indd Page 279 10/26/10 4:31:52 PM user-f499 4.13 /Volumes/201/MHDQ251/bee80288_disk1of1/0073380288/bee80288_pagefiles 4.13 Unsymmetric Bending UNSYMMETRIC BENDING Our analysis of pure bending has been limited so far to members possessing at least one plane of symmetry and subjected to couples acting in that plane. Because of the symmetry of such members and of their loadings, we concluded that the members would remain symmetric with respect to the plane of the couples and thus bend in that plane (Sec. 4.3). This is illustrated in Fig. 4.49; part a shows the cross section of a member possessing two planes of symmetry, one vertical and one horizontal, and part b the cross section of a member with a single, vertical plane of symmetry. In both cases the couple exerted on the section acts in the vertical plane of symmetry of the member and is represented by the horizontal couple vector M, and in both cases the neutral axis of the cross section is found to coincide with the axis of the couple. Let us now consider situations where the bending couples do not act in a plane of symmetry of the member, either because they act in a different plane, or because the member does not possess any plane of symmetry. In such situations, we cannot assume that the member will bend in the plane of the couples. This is illustrated in Fig. 4.50. In each part of the figure, the couple exerted on the section has again been assumed to act in a vertical plane and has been represented by a horizontal couple vector M. However, since the vertical plane is not a plane of symmetry, we cannot expect the member to bend in that plane, or the neutral axis of the section to coincide with the axis of the couple. y N.A. z M y N.A. z M (b) N.A. M M z C N.A. z C M (c) (b) (a) Fig. 4.50 Moment not in plane of symmetry. We propose to determine the precise conditions under which the neutral axis of a cross section of arbitrary shape coincides with the axis of the couple M representing the forces acting on that section. Such a section is shown in Fig. 4.51, and both the couple vector M and the y y = z C . N.A y z Fig. 4.51 C M x x  x dA Section with arbitrary shape. z C Fig. 4.49 Moment in plane of symmetry. y C C (a) y N.A. z y 279 bee80288_ch04_220-313.indd Page 280 10/26/10 4:32:00 PM user-f499 280 /Volumes/201/MHDQ251/bee80288_disk1of1/0073380288/bee80288_pagefiles neutral axis have been assumed to be directed along the z axis. We recall from Sec. 4.2 that, if we then express that the elementary internal forces sx dA form a system equivalent to the couple M, we obtain Pure Bending C (a) C M (b) Fig. 4.52 Moment on principal centroidal axis. y N.A. z C M (a) y N.A. z (4.1) moments about y axis: ezsxdA 5 0 (4.2) moments about z axis: e(2ysxdA) 5 M (4.3) # z a2 y N.A. z esxdA 5 0 As we saw earlier, when all the stresses are within the proportional limit, the first of these equations leads to the requirement that the neutral axis be a centroidal axis, and the last to the fundamental relation sx 5 2MyyI. Since we had assumed in Sec. 4.2 that the cross section was symmetric with respect to the y axis, Eq. (4.2) was dismissed as trivial at that time. Now that we are considering a cross section of arbitrary shape, Eq. (4.2) becomes highly significant. Assuming the stresses to remain within the proportional limit of the material, we can substitute sx 5 2sm yyc into Eq. (4.2) and write y N.A. z M x components: C M (b) Fig. 4.53 Moment not on principal centroidal axis. sm y b dA 5 0 c or e yz dA 5 0 (4.51) The integral eyzdA represents the product of inertia Iyz of the cross section with respect to the y and z axes, and will be zero if these axes are the principal centroidal axes of the cross section.† We thus conclude that the neutral axis of the cross section will coincide with the axis of the couple M representing the forces acting on that section if, and only if, the couple vector M is directed along one of the principal centroidal axes of the cross section. We note that the cross sections shown in Fig. 4.49 are symmetric with respect to at least one of the coordinate axes. It follows that, in each case, the y and z axes are the principal centroidal axes of the section. Since the couple vector M is directed along one of the principal centroidal axes, we verify that the neutral axis will coincide with the axis of the couple. We also note that, if the cross sections are rotated through 908 (Fig. 4.52), the couple vector M will still be directed along a principal centroidal axis, and the neutral axis will again coincide with the axis of the couple, even though in case b the couple does not act in a plane of symmetry of the member. In Fig. 4.50, on the other hand, neither of the coordinate axes is an axis of symmetry for the sections shown, and the coordinate axes are not principal axes. Thus, the couple vector M is not directed along a principal centroidal axis, and the neutral axis does not coincide with the axis of the couple. However, any given section possesses principal centroidal axes, even if it is unsymmetric, as the section shown in Fig. 4.50c, and these axes may be determined analytically or by using Mohr’s circle.† If the couple vector M is directed along one of the principal centroidal axes of the section, the neutral axis will coincide with the axis of the couple (Fig. 4.53) and the equations †See Ferdinand P. Beer and E. Russell Johnston, Jr., Mechanics for Engineers, 5th ed., McGraw-Hill, New York, 2008, or Vector Mechanics for Engineers, 9th ed., McGraw-Hill, New York, 2010, Secs. 9.8–9.10. bee80288_ch04_220-313.indd Page 281 10/27/10 5:46:23 PM user-f499 /Users/user-f499/Desktop/Temp Work/Don't Delete Job/MHDQ251:Beer:201/ch04 4.13 Unsymmetric Bending derived in Secs. 4.3 and 4.4 for symmetric members can be used to determine the stresses in this case as well. As you will see presently, the principle of superposition can be used to determine stresses in the most general case of unsymmetric bending. Consider first a member with a vertical plane of symmetry, which is subjected to bending couples M and M9 acting in a plane forming an angle u with the vertical plane (Fig. 4.54). The couple y M'  M y M My x z Fig. 4.54 Unsymmetric bending.  z vector M representing the forces acting on a given cross section will form the same angle u with the horizontal z axis (Fig. 4.55). Resolving the vector M into component vectors Mz and My along the z and y axes, respectively, we write M z 5 M cos u M y 5 M sin u sx 5 2 Iz where Iy is the moment of inertia of the section about the principal centroidal y axis, and where the positive sign is due to the fact that we have tension to the left of the vertical xy plane (z . 0) and compression to its right (z , 0). The distribution of the stresses caused by the original couple M is obtained by superposing the stress distributions defined by Eqs. (4.53) and (4.54), respectively. We have Mz y Iz 1 My z Iy (4.55) y M'z Mz y x z (4.53) where Iz is the moment of inertia of the section about the principal centroidal z axis. The negative sign is due to the fact that we have compression above the xz plane (y . 0) and tension below (y , 0). On the other hand, the couple My acts in a horizontal plane and bends the member in that plane (Fig. 4.57). The resulting stresses are found to be My z sx 5 1 (4.54) Iy sx 5 2 Fig. 4.55 (4.52) Since the y and z axes are the principal centroidal axes of the cross section, we can use Eq. (4.16) to determine the stresses resulting from the application of either of the couples represented by Mz and My. The couple Mz acts in a vertical plane and bends the member in that plane (Fig. 4.56). The resulting stresses are Mz y C Mz Fig. 4.56 y z M'y My x z Fig. 4.57 281 bee80288_ch04_220-313.indd Page 282 10/26/10 4:32:18 PM user-f499 282 Pure Bending z y C Fig. 4.58 Unsymmetric cross section. /Volumes/201/MHDQ251/bee80288_disk1of1/0073380288/bee80288_pagefiles We note that the expression obtained can also be used to compute the stresses in an unsymmetric section, such as the one shown in Fig. 4.58, once the principal centroidal y and z axes have been determined. On the other hand, Eq. (4.55) is valid only if the conditions of applicability of the principle of superposition are met. In other words, it should not be used if the combined stresses exceed the proportional limit of the material, or if the deformations caused by one of the component couples appreciably affect the distribution of the stresses due to the other. Equation (4.55) shows that the distribution of stresses caused by unsymmetric bending is linear. However, as we have indicated earlier in this section, the neutral axis of the cross section will not, in general, coincide with the axis of the bending couple. Since the normal stress is zero at any point of the neutral axis, the equation defining that axis can be obtained by setting sx 5 0 in Eq. (4.55). We write 2 Mzy Iz 1 M yz Iy 50 or, solving for y and substituting for Mz and My from Eqs. (4.52), y5a Iz tan ub z Iy (4.56) The equation obtained is that of a straight line of slope m 5 (IzyIy) tan u. Thus, the angle f that the neutral axis forms with the z axis (Fig. 4.59) is defined by the relation tan f 5 Iz tan u Iy (4.57) where u is the angle that the couple vector M forms with the same axis. Since Iz and Iy are both positive, f and u have the same sign. Furthermore, we note that f . u when Iz . Iy, and f , u when Iz , Iy. Thus, the neutral axis is always located between the couple vector M and the principal axis corresponding to the minimum moment of inertia. y . A N. M  z Fig. 4.59 C bee80288_ch04_220-313.indd Page 283 10/26/10 4:32:24 PM user-f499 /Volumes/201/MHDQ251/bee80288_disk1of1/0073380288/bee80288_pagefiles A 1600-lb ? in. couple is applied to a wooden beam, of rectangular cross section 1.5 by 3.5 in., in a plane forming an angle of 308 with the vertical (Fig. 4.60). Determine (a) the maximum stress in the beam, (b) the angle that the neutral surface forms with the horizontal plane. EXAMPLE 4.08 y 1600 lb · in. D 30 E 1600 lb · in. 3.5 in. C z Mz C   30 1.75 in. A 1.5 in. B 0.75 in. Fig. 4.60 Fig. 4.61 (a) Maximum Stress. The components Mz and My of the couple vector are first determined (Fig. 4.61): M z 5 11600 lb ? in.2 cos 30° 5 1386 lb ? in. M y 5 11600 lb ? in.2 sin 30° 5 800 lb ? in. y Iz 5 Iy 5 1 12 1 12 11.5 in.2 13.5 in.2 3 5 5.359 in4 13.5 in.2 11.5 in.2 3 5 0.9844 in4 The largest tensile stress due to Mz occurs along AB and is s1 5 M zy Iz 5 11386 lb ? in.2 11.75 in.2 5.359 in4 D . N. A We also compute the moments of inertia of the cross section with respect to the z and y axes: E C z 5 452.6 psi A B The largest tensile stress due to My occurs along AD and is s2 5 1800 lb ? in.2 10.75 in.2 M yz 5 5 609.5 psi Iy 0.9844 in4 Fig. 4.62 The largest tensile stress due to the combined loading, therefore, occurs at A and is 1062 psi D s max 5 s 1 1 s 2 5 452.6 1 609.5 5 1062 psi E (b) Angle of Neutral Surface with Horizontal Plane. The angle f that the neutral surface forms with the horizontal plane (Fig. 4.62) is obtained from Eq. (4.57): is Iz 5.359 in 4 tan u 5 tan 30° 5 3.143 Iy 0.9844 in4 f 5 72.4° C A tan f 5 The distribution of the stresses across the section is shown in Fig. 4.63. ral ax Neut The largest compressive stress has the same magnitude and occurs at E. 1062 psi B Fig. 4.63 283 bee80288_ch04_220-313.indd Page 284 10/27/10 5:46:24 PM user-f499 284 4.14 Pure Bending A S y B P' C x z b P a (a) M'y y A P' S My B M'z Mz z (b) Fig. 4.64 Eccentric axial loading. C P x /Users/user-f499/Desktop/Temp Work/Don't Delete Job/MHDQ251:Beer:201/ch04 GENERAL CASE OF ECCENTRIC AXIAL LOADING In Sec. 4.12 you analyzed the stresses produced in a member by an eccentric axial load applied in a plane of symmetry of the member. You will now study the more general case when the axial load is not applied in a plane of symmetry. Consider a straight member AB subjected to equal and opposite eccentric axial forces P and P9 (Fig. 4.64a), and let a and b denote the distances from the line of action of the forces to the principal centroidal axes of the cross section of the member. The eccentric force P is statically equivalent to the system consisting of a centric force P and of the two couples My and Mz of moments My 5 Pa and Mz 5 Pb represented in Fig. 4.64b. Similarly, the eccentric force P9 is equivalent to the centric force P9 and the couples M9y and M9z. By virtue of Saint-Venant’s principle (Sec. 2.17), we can replace the original loading of Fig. 4.64a by the statically equivalent loading of Fig. 4.64b in order to determine the distribution of stresses in a section S of the member, as long as that section is not too close to either end of the member. Furthermore, the stresses due to the loading of Fig. 4.64b can be obtained by superposing the stresses corresponding to the centric axial load P and to the bending couples My and Mz, as long as the conditions of applicability of the principle of superposition are satisfied (Sec. 2.12). The stresses due to the centric load P are given by Eq. (1.5), and the stresses due to the bending couples by Eq. (4.55), since the corresponding couple vectors are directed along the principal centroidal axes of the section. We write, therefore, sx 5 My z Mz y P 2 1 Iz A Iy (4.58) where y and z are measured from the principal centroidal axes of the section. The relation obtained shows that the distribution of stresses across the section is linear. In computing the combined stress sx from Eq. (4.58), care should be taken to correctly determine the sign of each of the three terms in the right-hand member, since each of these terms can be positive or negative, depending upon the sense of the loads P and P9 and the location of their line of action with respect to the principal centroidal axes of the cross section. Depending upon the geometry of the cross section and the location of the line of action of P and P9, the combined stresses sx obtained from Eq. (4.58) at various points of the section may all have the same sign, or some may be positive and others negative. In the latter case, there will be a line in the section, along which the stresses are zero. Setting sx 5 0 in Eq. (4.58), we obtain the equation of a straight line, which represents the neutral axis of the section: My Mz P y2 z5 A Iz Iy bee80288_ch04_220-313.indd Page 285 10/26/10 4:32:39 PM user-f499 /Volumes/201/MHDQ251/bee80288_disk1of1/0073380288/bee80288_pagefiles A vertical 4.80-kN load is applied as shown on a wooden post of rectangular cross section, 80 by 120 mm (Fig. 4.65). (a) Determine the stress at points A, B, C, and D. (b) Locate the neutral axis of the cross section. EXAMPLE 4.09 4.80 kN 35 mm y 120 mm 80 mm D C A z B x Fig. 4.65 (a) Stresses. The given eccentric load is replaced by an equivalent system consisting of a centric load P and two couples Mx and Mz represented by vectors directed along the principal centroidal axes of the section (Fig. 4.66). We have y P  4.80 kN Mz  12 M x 5 14.80 kN2 140 mm2 5 192 N ? m M z 5 14.80 kN2 160 mm 2 35 mm2 5 120 N ? m 192 N · m We also compute the area and the centroidal moments of inertia of the cross section: A 5 10.080 m2 10.120 m2 5 9.60 3 1023 m 2 Ix 5 121 10.120 m2 10.080 m2 3 5 5.12 3 1026 m 4 Iz 5 121 10.080 m2 10.120 m2 3 5 11.52 3 1026 m 4 The stress s0 due to the centric load P is negative and uniform across the section. We have s0 5 x Fig. 4.66 P 24.80 kN 5 5 20.5 MPa A 9.60 3 1023 m 2 The stresses due to the bending couples Mx and Mz are linearly distributed across the section, with maximum values equal, respectively, to 1192 N ? m2 140 mm2 M x zmax 5 5 1.5 MPa Ix 5.12 3 1026 m 4 1120 N ? m2 160 mm2 M z xmax 5 5 0.625 MPa s2 5 Iz 11.52 3 1026 m 4 s1 5 The stresses at the corners of the section are sy 5 s0 6 s1 6 s2 where the signs must be determined from Fig. 4.66. Noting that the stresses due to Mx are positive at C and D, and negative at A and B, and 285 bee80288_ch04_220-313.indd Page 286 10/26/10 4:32:47 PM user-f499 /Volumes/201/MHDQ251/bee80288_disk1of1/0073380288/bee80288_pagefiles that the stresses due to Mz are positive at B and C, and negative at A and D, we obtain sA sB sC sD 5 5 5 5 20.5 20.5 20.5 20.5 2 2 1 1 1.5 1.5 1.5 1.5 2 1 1 2 0.625 0.625 0.625 0.625 1.625 MPa B 5 5 5 5 22.625 MPa 21.375 MPa 11.625 MPa 10.375 MPa 80 mm 0.375 MPa H C D G A 1.375 MPa 80 mm 2.625 MPa (a) (b) Fig. 4.67 (b) Neutral Axis. We note that the stress will be zero at a point G between B and C, and at a point H between D and A (Fig. 4.67). Since the stress distribution is linear, we write 1.375 BG 5 80 mm 1.625 1 1.375 HA 2.625 5 80 mm 2.625 1 0.375 BG 5 36.7 mm HA 5 70 mm The neutral axis can be drawn through points G and H (Fig. 4.68). D C Neu tral H O axis x G B A z Fig. 4.68 The distribution of the stresses across the section is shown in Fig. 4.69. 0.375 MPa H A 2.625 MPa Fig. 4.69 286 1.625 MPa Ne u axi tral s B G 1.375 MPa C bee80288_ch04_220-313.indd Page 287 10/27/10 5:46:24 PM user-f499 /Users/user-f499/Desktop/Temp Work/Don't Delete Job/MHDQ251:Beer:201/ch04 SAMPLE PROBLEM 4.9 A horizontal load P is applied as shown to a short section of an S10 3 25.4 rolled-steel member. Knowing that the compressive stress in the member is not to exceed 12 ksi, determine the largest permissible load P. 4.75 in. C S10  25.4 P 1.5 in. SOLUTION y C 10 in. Properties of Cross Section. The following data are taken from Appendix C. Area: A 5 7.46 in2 Section moduli: Sx 5 24.7 in3 Sy 5 2.91 in3 x Force and Couple at C. We replace P by an equivalent force-couple system at the centroid C of the cross section. 4.66 in. M x 5 14.75 in.2P M y 5 11.5 in.2P Note that the couple vectors Mx and My are directed along the principal axes of the cross section. y B A x My Mx C s3 5 P E D Normal Stresses. The absolute values of the stresses at points A, B, D, and E due, respectively, to the centric load P and to the couples Mx and My are P P s1 5 5 5 0.1340P A 7.46 in2 Mx 4.75P 5 5 0.1923P s2 5 Sx 24.7 in3 My Sy 5 1.5P 5 0.5155P 2.91 in3 Superposition. The total stress at each point is found by superposing the stresses due to P, Mx, and My. We determine the sign of each stress by carefully examining the sketch of the force-couple system. sA 5 2s1 sB 5 2s1 sD 5 2s1 sE 5 2s1 1 s2 1 s2 2 s2 2 s2 1 s3 2 s3 1 s3 2 s3 5 20.1340P 1 0.1923P 1 0.5155P 5 10.574P 5 20.1340P 1 0.1923P 2 0.5155P 5 20.457P 5 20.1340P 2 0.1923P 1 0.5155P 5 10.189P 5 20.1340P 2 0.1923P 2 0.5155P 5 20.842P Largest Permissible Load. The maximum compressive stress occurs at point E. Recalling that sall 5 212 ksi, we write sall 5 sE 212 ksi 5 20.842P P 5 14.3 kips b 287 bee80288_ch04_220-313.indd Page 288 10/28/10 9:25:52 PM user-f499 *SAMPLE PROBLEM 4.10 y M0 z /Volumes/201/MHDQ251/bee80288_disk1of1/0073380288/bee80288_pagefiles A couple of magnitude M0 5 1.5 kN ? m acting in a vertical plane is applied to a beam having the Z-shaped cross section shown. Determine (a) the stress at point A, (b) the angle that the neutral axis forms with the horizontal plane. The moments and product of inertia of the section with respect to the y and z axes have been computed and are as follows: y x 80 mm A 12 mm 12 mm C z M0 ⫽ 1.5 kN · m 12 mm Iy 5 3.25 3 1026 m4 Iz 5 4.18 3 1026 m4 Iyz 5 2.87 3 1026 m4 100 mm Iyz(10–6 m4) Y(3.25, 2.87) SOLUTION R O U D E F Iy, Iz (10–6 m4) V 2␪ m R y u ␪ m ⫽ 40.4° A Loading. The applied couple M0 is resolved into components parallel to the principal axes. Mu M0 ⫽ 1.5 kN · m C z Mu 5 M0 sin um 5 1500 sin 40.4° 5 972 N ? m Mv 5 M0 cos um 5 1500 cos 40.4° 5 1142 N ? m ␪m Mv v a. Stress at A. The perpendicular distances from each principal axis to point A are zA ⫽ 74 mm y u vA zA sin ␪ m yA cos ␪ m A yA ⫽ 50 mm ␪m uA z C u 1972 N ? m2 10.0239 m2 11142 N ? m2 10.0860 m2 MuvA MvuA 2 51 2 26 4 Iu Iv 0.810 3 10 m 6.63 3 1026 m4 5 1(28.68 MPa) 2 (14.81 MPa) sA 5 113.87 MPa ◀ tan f 5 Iv 6.63 tan um 5 tan 40.4° Iu 0.810 f 5 81.8° The angle b formed by the neutral axis and the horizontal is ␤ M0 ␪m v Considering separately the bending about each principal axis, we note that Mu produces a tensile stress at point A while Mv produces a compressive stress at the same point. b. Neutral Axis. Using Eq. (4.57), we find the angle f that the neutral axis forms with the v axis. N.A. ␾ uA 5 yA cos um 1 zA sin um 5 50 cos 40.4° 1 74 sin 40.4° 5 86.0 mm vA 5 2yA sin um 1 zA cos um 5 250 sin 40.4° 1 74 cos 40.4° 5 23.9 mm sA 5 1 v 288 2.87 FZ 5 2um 5 80.8° um 5 40.4° EF 0.465 R2 5 1EF2 2 1 1FZ2 2 5 10.4652 2 1 12.872 2 R 5 2.91 3 1026 m4 Iu 5 Imin 5 OU 5 Iave 2 R 5 3.72 2 2.91 5 0.810 3 1026 m4 Iv 5 Imax 5 OV 5 Iave 1 R 5 3.72 1 2.91 5 6.63 3 1026 m4 tan 2um 5 Z(4.18, –2.87) Iave ⫽ 3.72 Principal Axes. We draw Mohr’s circle and determine the orientation of the principal axes and the corresponding principal moments of inertia.† C b 5 f 2 um 5 81.88 2 40.48 5 41.48 b 5 41.48 ◀ †See Ferdinand F. Beer and E. Russell Johnston, Jr., Mechanics for Engineers, 5th ed., McGraw-Hill, New York, 2008, or Vector Mechanics for Engineers–9th ed., McGraw-Hill, New York, 2010, Secs. 9.8–9.10. bee80288_ch04_220-313.indd Page 289 10/26/10 4:33:18 PM user-f499 /Volumes/201/MHDQ251/bee80288_disk1of1/0073380288/bee80288_pagefiles PROBLEMS 4.127 through 4.134 The couple M is applied to a beam of the cross section shown in a plane forming an angle b with the vertical. Determine the stress at (a) point A, (b) point B, (c) point D. y y   30 A y M  300 N · m z A 50 mm B z D 16 mm D M  25 kN · m A z 2 in. C B A y   30 B A B 80 mm z 20 mm z M  250 kip · in. 80 mm 10 in. C 0.3 in. D D 2 in. 8 in. 30 mm 4 in. Fig. P4.130 0.5 in. C 3 in. D 1 in. 2.5 in. 2.5 in. 5 in. 5 in.   15 M  10 kip · in. 3 in. 1 in. Fig. P4.129 y   20 C 3 in. 40 mm Fig. P4.128 y z D C 40 mm B 3 in. B 16 mm 40 mm 40 mm Fig. P4.127 A   60 50 mm C M  250 N · m M  60 kip · in.   50 Fig. P4.131 0.5 in. Fig. P4.132 y y M  75 kip · in.   75 A 2.4 in. 1.6 in. z B B C D 4 in. z C D A 4.8 in. Fig. P4.133   30 M  100 N · m r  20 mm Fig. P4.134 289 bee80288_ch04_220-313.indd Page 290 10/26/10 4:33:38 PM user-f499 290 /Volumes/201/MHDQ251/bee80288_disk1of1/0073380288/bee80288_pagefiles 4.135 through 4.140 The couple M acts in a vertical plane and is applied to a beam oriented as shown. Determine (a) the angle that the neutral axis forms with the horizontal, (b) the maximum tensile stress in the beam. Pure Bending W310 10 C200 38.7 15 B A 17.1 C A M  2.8 kN · m E M  16 kN · m 57 mm C B D E 203 mm D 14.4 mm 165 mm Fig. P4.135 Fig. P4.136 y' 45 y' B 50 mm 5 mm 5 mm 0.859 in. M  15 kip · in. 30 A B z' C M  400 N · m A 310 mm D C 1 2 in. 4 in. D z' 4 in. 18.57 mm 4 in. E 5 mm 50 mm Iy'  281 103 mm4 Iz'  176.9 103 mm4 Iy'  6.74 in4 Iz'  21.4 in4 Fig. P4.137 Fig. P4.138 y' 20 B 10 mm A 15 6 mm M  120 N · m A B C M  35 kip · in. E 1 in. Fig. P4.139 2 in. 1 in. C 0.4 in. z' 1.6 in. D 0.4 in. Iy'  14.77 103 mm4 Iz'  53.6 103 mm4 Fig. P4.140 10 mm D 6 mm E 10 mm 10 mm bee80288_ch04_220-313.indd Page 291 10/26/10 4:33:51 PM user-f499 /Volumes/201/MHDQ251/bee80288_disk1of1/0073380288/bee80288_pagefiles Problems *4.141 through *4.143 The couple M acts in a vertical plane and is applied to a beam oriented as shown. Determine the stress at point A. 291 y 1.08 in. 0.75 in. y 2.08 in. A 2.4 in. z 0.75 in. z C 2.4 in. z A 10 mm Iy  8.7 in4 Iz  24.5 in4 Iyz  8.3 in4 2.4 in. 2.4 in. Fig. P4.141 4.145 Solve Prob. 4.144, assuming that the 28-kN force at point E is removed. 4.146 A rigid circular plate of 125-mm radius is attached to a solid 150 3 200-mm rectangular post, with the center of the plate directly above the center of the post. If a 4-kN force P is applied at E with u 5 308, determine (a) the stress at point A, (b) the stress at point B, (c) the point where the neutral axis intersects line ABD. C z P  4 kN E  x A 70 mm 10 mm Fig. P4.143 4.144 The tube shown has a uniform wall thickness of 12 mm. For the loading given, determine (a) the stress at points A and B, (b) the point where the neutral axis intersects line ABD. R  125 mm 40 mm Iy  1.894 106 mm4 Iz  0.614 106 mm4 Iyz  0.800 106 mm4 Fig. P4.142 y 10 mm C M  1.2 kN · m 4 in. 2.4 in. 40 mm y 6 in. C M  60 kip · in. 2.4 in. M  125 kip · in. A D B 200 mm 150 mm Fig. P4.146 4.147 In Prob. 4.146, determine (a) the value of u for which the stress at D reaches its largest value, (b) the corresponding values of the stress at A, B, C, and D. D B G H 14 kN 28 kN E A F Fig. P4.144 125 mm 28 kN 75 mm bee80288_ch04_220-313.indd Page 292 10/26/10 4:34:02 PM user-f499 292 4.148 Knowing that P 5 90 kips, determine the largest distance a for which the maximum compressive stress does not exceed 18 ksi. Pure Bending 1 in. 4 in. 1 in. /Volumes/201/MHDQ251/bee80288_disk1of1/0073380288/bee80288_pagefiles 1 in. 4.149 Knowing that a 5 1.25 in., determine the largest value of P that can be applied without exceeding either of the following allowable stresses: 5 in. P sten 5 10 ksi a 2.5 in. scomp 5 18 ksi 4.150 The Z section shown is subjected to a couple M0 acting in a vertical plane. Determine the largest permissible value of the moment M0 of the couple if the maximum stress is not to exceed 80 MPa. Given: Imax 5 2.28 3 1026 m4, Imin 5 0.23 3 1026 m4, principal axes 25.78 c and 64.38 a. y Fig. P4.148 and P4.149 z 0.5 in. 40 mm 70 mm 10 mm Fig. P4.150 1.43 in. C M0 10 mm C 10 mm y z M0 40 mm 5 in. 4.151 Solve Prob. 4.150, assuming that the couple M0 acts in a horizontal plane. 4.152 A beam having the cross section shown is subjected to a couple M0 that acts in a vertical plane. Determine the largest permissible value of the moment M0 of the couple if the maximum stress in the beam is not to exceed 12 ksi. Given: Iy 5 Iz 5 11.3 in4, A 5 4.75 in2, kmin 5 0.983 in. (Hint: By reason of symmetry, the principal axes form an angle of 458 with the coordinate axes. Use the relations Imin 5 Ak2min and Imin 1 Imax 5 Iy 1 Iz.) 0.5 in. 1.43 in. 5 in. Fig. P4.152 4.153 Solve Prob. 4.152, assuming that the couple M0 acts in a horizontal plane. y 0.3 in. M0 C z 0.3 in. 0.6 in. 1.5 in. 0.6 in. Fig. P4.154 1.5 in. 4.154 An extruded aluminum member having the cross section shown is subjected to a couple acting in a vertical plane. Determine the largest permissible value of the moment M0 of the couple if the maximum stress is not to exceed 12 ksi. Given: Imax 5 0.957 in4, Imin 5 0.427 in4, principal axes 29.48 a and 60.68 c. 4.155 A couple M0 acting in a vertical plane is applied to a W12 3 16 rolled-steel beam, whose web forms an angle u with the vertical. Denoting by s0 the maximum stress in the beam when u 5 0, determine the angle of inclination u of the beam for which the maximum stress is 2s0. M0  Fig. P4.155  bee80288_ch04_220-313.indd Page 293 10/26/10 4:34:11 PM user-f499 /Volumes/201/MHDQ251/bee80288_disk1of1/0073380288/bee80288_pagefiles Problems 4.156 Show that, if a solid rectangular beam is bent by a couple applied in a plane containing one diagonal of a rectangular cross section, the neutral axis will lie along the other diagonal. 4.157 A beam of unsymmetric cross section is subjected to a couple M0 acting in the horizontal plane xz. Show that the stress at point A, of coordinates y and z, is zIz 2 yIyz My sA 5 IyIz 2 I2yz where Iy, Iz, and Iyz denote the moments and product of inertia of the cross section with respect to the coordinate axes, and My the moment of the couple. b A h M C D E Fig. P4.156 4.158 A beam of unsymmetric cross section is subjected to a couple M0 acting in the vertical plane xy. Show that the stress at point A, of coordinates y and z, is yIy 2 zIyz Mz sA 5 2 IyIz 2 I2yz where Iy, Iz, and Iyz denote the moments and product of inertia of the cross section with respect to the coordinate axes, and Mz the moment of the couple. 4.159 (a) Show that, if a vertical force P is applied at point A of the section shown, the equation of the neutral axis BD is a xA r2z bx 1 a zA r2x B y z A y C Fig. P4.157 and P4.158 b z 5 21 y where rz and rx denote the radius of gyration of the cross section with respect to the z axis and the x axis, respectively. (b) Further show that, if a vertical force Q is applied at any point located on line BD, the stress at point A will be zero. 4.160 (a) Show that the stress at corner A of the prismatic member shown in Fig. P4.160a will be zero if the vertical force P is applied at a point located on the line D B (b) Further show that, if no tensile stress is to occur in the member, the force P must be applied at a point located within the area bounded by the line found in part a and three similar lines corresponding to the condition of zero stress at B, C, and D, respectively. This area, shown in Fig. P4.160b, is known as the kern of the cross section. A D B C A D P B z x z C h 6 h b (a) Fig. P4.160 x (b) b 6 P C A z xA x z 1 51 by6 hy6 y x z Fig. P4.159 zA x 293 bee80288_ch04_220-313.indd Page 294 10/26/10 4:34:21 PM user-f499 294 /Volumes/201/MHDQ251/bee80288_disk1of1/0073380288/bee80288_pagefiles *4.15 Pure Bending BENDING OF CURVED MEMBERS Our analysis of stresses due to bending has been restricted so far to straight members. In this section we will consider the stresses caused by the application of equal and opposite couples to members that are initially curved. Our discussion will be limited to curved members of uniform cross section possessing a plane of symmetry in which the bending couples are applied, and it will be assumed that all stresses remain below the proportional limit. If the initial curvature of the member is small, i.e., if its radius of curvature is large compared to the depth of its cross section, a good approximation can be obtained for the distribution of stresses by assuming the member to be straight and using the formulas derived in Secs. 4.3 and 4.4.† However, when the radius of curvature and the dimensions of the cross section of the member are of the same order of magnitude, we must use a different method of analysis, which was first introduced by the German engineer E. Winkler (1835–1888). Consider the curved member of uniform cross section shown in Fig. 4.70. Its transverse section is symmetric with respect to the y axis (Fig. 4.70b) and, in its unstressed state, its upper and lower surfaces intersect the vertical xy plane along arcs of circle AB and FG centered at C (Fig. 4.70a). We now apply two equal and opposite couples M y y C C C'  R r r' r R A y y G x z (a) y B' K' E' G' F' x N. A. (b) M y D' E F  '    J' K D Fig. 4.70 M' A' B J R' (c) Curved member in pure bending. and M9 in the plane of symmetry of the member (Fig. 4.70c). A reasoning similar to that of Sec. 4.3 would show that any transverse plane section containing C will remain plane, and that the various arcs of circle indicated in Fig. 4.70a will be transformed into circular and concentric arcs with a center C9 different from C. More specifically, if the couples M and M9 are directed as shown, the curvature of the various arcs of circle will increase; that is A9C9 , AC. We also note that the couples M and M9 will cause the length of the upper surface †See Prob. 4.166. bee80288_ch04_220-313.indd Page 295 10/26/10 4:34:21 PM user-f499 /Volumes/201/MHDQ251/bee80288_disk1of1/0073380288/bee80288_pagefiles 4.15 Bending of Curved Members of the member to decrease (A9B9 , AB) and the length of the lower surface to increase (F9G9 . FG). We conclude that a neutral surface must exist in the member, the length of which remains constant. The intersection of the neutral surface with the xy plane has been represented in Fig. 4.70a by the arc DE of radius R, and in Fig. 4.70c by the arc D9E9 of radius R9. Denoting by u and u9 the central angles corresponding respectively to DE and D9E9, we express the fact that the length of the neutral surface remains constant by writing Ru 5 R9u9 295 (4.59) Considering now the arc of circle JK located at a distance y above the neutral surface, and denoting respectively by r and r9 the radius of this arc before and after the bending couples have been applied, we express the deformation of JK as d 5 r9u9 2 ru (4.60) Observing from Fig. 4.70 that r5R2y r9 5 R9 2 y (4.61) and substituting these expressions into Eq. (4.60), we write d 5 (R9 2 y)u9 2 (R 2 y)u or, recalling Eq. (4.59) and setting u9 2 u 5 Du, d 5 2y Du (4.62) The normal strain Px in the elements of JK is obtained by dividing the deformation d by the original length ru of arc JK. We write Px 5 y ¢u d 52 ru ru or, recalling the first of the relations (4.61), Px 5 2 ¢u y u R2y (4.63) The relation obtained shows that, while each transverse section remains plane, the normal strain Px does not vary linearly with the distance y from the neutral surface. The normal stress sx can now be obtained from Hooke’s law, sx 5 EPx, by substituting for Px from Eq. (4.63). We have sx 5 2 E ¢u y u R2y (4.64) y or, alternatively, recalling the first of Eqs. (4.61), sx 5 2 E ¢u R 2 r r u y (4.65) Equation (4.64) shows that, like Px, the normal stress sx does not vary linearly with the distance y from the neutral surface. Plotting sx versus y, we obtain an arc of hyperbola (Fig. 4.71). In order to determine the location of the neutral surface in the member and the value of the coefficient E Duyu used in Eqs. (4.64) z Fig. 4.71 N. A. x bee80288_ch04_220-313.indd Page 296 10/26/10 4:34:22 PM user-f499 296 /Volumes/201/MHDQ251/bee80288_disk1of1/0073380288/bee80288_pagefiles and (4.65), we now recall that the elementary forces acting on any transverse section must be statically equivalent to the bending couple M. Expressing, as we did in Sec. 4.2 for a straight member, that the sum of the elementary forces acting on the section must be zero, and that the sum of their moments about the transverse z axis must be equal to the bending moment M, we write the equations Pure Bending # s dA 5 0 (4.1) # 12ys dA2 5 M (4.3) x and x Substituting for sx from (4.65) into Eq. (4.1), we write 2 # E ¢u R 2 r dA 5 0 r u R2r dA 5 0 r # R dA #r from which it follows that the distance R from the center of curvature C to the neutral surface is defined by the relation y C R5 R z r r5 e Centroid # A dA r (4.66) We note that the value obtained for R is not equal to the distance r from C to the centroid of the cross section, since r is defined by a different relation, namely, N. A. Fig. 4.72 # dA 5 0 2 1 A # r dA (4.67) We thus conclude that, in a curved member, the neutral axis of a transverse section does not pass through the centroid of that section (Fig. 4.72).† Expressions for the radius R of the neutral surface will be derived for some specific cross-sectional shapes in Example 4.10 and in Probs. 4.188 through 4.190. For convenience, these expressions are shown in Fig. 4.73. Substituting now for sx from (4.65) into Eq. (4.3), we write # E ¢u R 2 r y dA 5 M r u †However, an interesting property of the neutral surface can be noted if we write Eq. (4.66) in the alternative form 1 1 5 R A 1 # r dA (4.669) Equation (4.669) shows that, if the member is divided into a large number of fibers of cross-sectional area dA, the curvature 1yR of the neutral surface will be equal to the average value of the curvature 1yr of the various fibers. bee80288_ch04_220-313.indd Page 297 11/11/10 3:03:19 PM user-f499 C C C r1 r1 c h b Rectangle Fig. 4.73 1 2 R (r r1 r2 h h b2 Trapezoid Triangle r 2  c 2) b1 r2 Circle h r2 ln r 1 C b r r2 R /Users/user-f499/Desktop/Temp Work/Don't Delete Job/MHDQ251:Beer:201/ch04 R 1 2h r2 r ln 2  1 r1 h R 1 2 h2(b1 b2) r2 (b1r2  b2r1) ln r  h(b1  b2) 1 Radius of neutral surface for various cross-sectional shapes. or, since y 5 R 2 r, 2 E ¢u 1R 2 r2 dA 5 M r u # Expanding the square in the integrand, we obtain after reductions E ¢u 2 cR u dA #r # 2 2RA 1 r dA d 5 M Recalling Eqs. (4.66) and (4.67), we note that the first term in the brackets is equal to RA, while the last term is equal to rA. We have, therefore, E ¢u 1RA 2 2RA 1 rA2 5 M u and, solving for E Duyu, M E ¢u 5 u A1r 2 R2 (4.68) Referring to Fig. 4.70, we note that Du . 0 for M . 0. It follows that r 2 R . 0, or R , r, regardless of the shape of the section. Thus, the neutral axis of a transverse section is always located between the centroid of the section and the center of curvature of the member (Fig. 4.72). Setting r 2 R 5 e, we write Eq. (4.68) in the form E ¢u M 5 u Ae (4.69) Substituting now for E Duyu from (4.69) into Eqs. (4.64) and (4.65), we obtain the following alternative expressions for the normal stress sx in a curved beam: sx 5 2 My Ae1R 2 y2 (4.70) and sx 5 M1r 2 R2 Aer (4.71) 297 bee80288_ch04_220-313.indd Page 298 10/26/10 4:34:31 PM user-f499 298 /Volumes/201/MHDQ251/bee80288_disk1of1/0073380288/bee80288_pagefiles We should note that the parameter e in the previous equations is a small quantity obtained by subtracting two lengths of comparable size, R and r. In order to determine sx with a reasonable degree of accuracy, it is therefore necessary to compute R and r very accurately, particularly when both of these quantities are large, i.e., when the curvature of the member is small. However, as we indicated earlier, it is possible in such a case to obtain a good approximation for sx by using the formula sx 5 2MyyI developed for straight members. Let us now determine the change in curvature of the neutral surface caused by the bending moment M. Solving Eq. (4.59) for the curvature 1yR9 of the neutral surface in the deformed member, we write Pure Bending 1 1 u¿ 5 R¿ R u or, setting u9 5 u 1 Du and recalling Eq. (4.69), 1 1 ¢u 1 M b 5 a1 1 b 5 a1 1 R¿ R u R EAe from which it follows that the change in curvature of the neutral surface is 1 1 M 2 5 (4.72) R¿ R EAeR EXAMPLE 4.10 A curved rectangular bar has a mean radius r 5 6 in. and a cross section of width b 5 2.5 in. and depth h 5 1.5 in. (Fig. 4.74). Determine the distance e between the centroid and the neutral axis of the cross section. C C r r h/2 h b C r1 r C r1 Fig. 4.74 We first derive the expression for the radius R of the neutral surface. Denoting by r1 and r2, respectively, the inner and outer radius of the bar (Fig. 4.75), we use Eq. (4.66) and write r r2 r2 R5 A 5 dA r r # r2 1 dr # dr b Fig. 4.75 bh 5 b dr r r R5 r2 1 h r2 ln r1 h dr r r # r2 1 (4.73) bee80288_ch04_220-313.indd Page 299 10/27/10 5:46:26 PM user-f499 /Users/user-f499/Desktop/Temp Work/Don't Delete Job/MHDQ251:Beer:201/ch04 For the given data, we have r1 5 r 2 12 h 5 6 2 0.75 5 5.25 in. C r2 5 r 1 12 h 5 6 1 0.75 5 6.75 in. Substituting for h, r1, and r2 into Eq. (4.73), we have R5 h 1.5 in. 5 5 5.9686 in. r2 6.75 ln ln r1 5.25 The distance between the centroid and the neutral axis of the cross section (Fig. 4.76) is thus e 5 r 2 R 5 6 2 5.9686 5 0.0314 in. r  6 in. R  5.9686 in. Neutral axis e  0.0314 in. Centroid We note that it was necessary to calculate R with five significant figures in order to obtain e with the usual degree of accuracy. Fig. 4.76 For the bar of Example 4.10, determine the largest tensile and compressive stresses, knowing that the bending moment in the bar is M 5 8 kip ? in. EXAMPLE 4.11 We use Eq. (4.71) with the given data, A 5 bh 5 (2.5 in.)(1.5 in.) 5 3.75 in2 M 5 8 kip ? in. and the values obtained in Example 4.10 for R and e, R 5 5.969 e 5 0.0314 in. Making first r 5 r2 5 6.75 in. in Eq. (4.71), we write smax 5 5 smax M1r2 2 R2 Aer2 18 kip ? in.2 16.75 in. 2 5.969 in.2 13.75 in2 2 10.0314 in.2 16.75 in.2 5 7.86 ksi Making now r 5 r1 5 5.25 in. in Eq. (4.71), we have smin 5 5 smin M1r1 2 R2 Aer1 18 kip ? in.2 15.25 in. 2 5.969 in.2 13.75 in2 2 10.0314 in.2 15.25 in.2 5 29.30 ksi Remark. Let us compare the values obtained for smax and smin with the result we would get for a straight bar. Using Eq. (4.15) of Sec. 4.4, we write smax, min 5 6 56 Mc I 18 kip ? in.2 10.75 in.2 1 12 12.5 in.2 11.5 in.2 3 5 68.53 ksi 299 bee80288_ch04_220-313.indd Page 300 11/11/10 3:03:20 PM user-f499 /Users/user-f499/Desktop/Temp Work/Don't Delete Job/MHDQ251:Beer:201/ch04 SAMPLE PROBLEM 4.11 20 mm a 40 mm A machine component has a T-shaped cross section and is loaded as shown. Knowing that the allowable compressive stress is 50 MPa, determine the largest force P that can be applied to the component. 20 mm 80 mm a Section a-a 60 mm P' P SOLUTION 20 mm Centroid of the Cross Section. We locate the centroid D of the cross section 2 40 mm 20 mm r2  70 mm 1 B M D P 60 mm P – A  B D D A A C C M (r – R) Aer 64 3 10 r 124002 5 120 3 103 56 3 103 r 5 50 mm 5 0.050 m © ri Ai 5 120 3 103 R 2400 mm2 r3 180 mm2 dr 120 mm2 dr 1 r r r1 r2 2400 2400 5 5 5 45.61 mm 50 90 40.866 1 11.756 80 ln 1 20 ln 30 50 5 0.04561 m # A 5 dA r # r2 # We also compute: e 5 r 2 R 5 0.05000 m 2 0.04561 m 5 0.00439 m Allowable Load. We observe that the largest compressive stress will occur at point A where r 5 0.030 m. Recalling that sall 5 50 MPa and using Eq. (1), we write B D dr A 80 mm C Superposition. The centric force P causes a uniform compressive stress on section a-a. The bending couple M causes a varying stress distribution [Eq. (4.71)]. We note that the couple M tends to increase the curvature of the member and is therefore positive (cf. Fig. 4.70). The total stress at a point of section a-a located at distance r from the center of curvature C is M1r 2 R2 P (1) s52 1 Aer A R5 20 mm r1  30 mm 40 70 r ©Ai 5 ©ri Ai 3 Radius of Neutral Surface. We now determine the radius R of the neutral surface by using Eq. (4.66). r r2  50 mm 12021802 5 1600 14021202 5 800 © Ai 5 2400 riAi, mm3 M 5 P(50 mm 1 60 mm) 5 (0.110 m)P 50 mm P' r3  90 mm ri, mm Force and Couple at D. The internal forces in section a-a are equivalent to a force P acting at D and a couple M of moment A C B Ai, mm2 1 2 r1  40 mm 80 mm 30 mm 300 30 mm r 10.110 P2 10.030 m 2 0.04561 m2 P 1 23 2 2.4 3 10 m 12.4 3 1023 m2 2 10.00439 m2 10.030 m2 6 250 3 10 5 2417P 2 5432P P 5 8.55 kN ◀ 250 3 106 Pa 5 2 bee80288_ch04_220-313.indd Page 301 10/26/10 4:34:54 PM user-f499 /Volumes/201/MHDQ251/bee80288_disk1of1/0073380288/bee80288_pagefiles PROBLEMS 4 kip · in. 4.161 For the machine component and loading shown, determine the stress at point A when (a) h 5 2 in., (b) h 5 2.6 in. 4.162 For the machine component and loading shown, determine the stress at points A and B when h 5 2.5 in. 4.163 The curved portion of the bar shown has an inner radius of 20 mm. Knowing that the allowable stress in the bar is 150 MPa, determine the largest permissible distance a from the line of action of the 3-kN force to the vertical plane containing the center of curvature of the bar. 4.164 The curved portion of the bar shown has an inner radius of 20 mm. Knowing that the line of action of the 3-kN force is located at a distance a 5 60 mm from the vertical plane containing the center of curvature of the bar, determine the largest compressive stress in the bar. 3 in. C 4 kip · in. h A 0.75 in. B Fig. P4.161 and P4.162 a r  20 mm P  3 kN 25 mm 25 mm 4.165 The curved bar shown has a cross section of 40 3 60 mm and an inner radius r1 5 15 mm. For the loading shown determine the largest tensile and compressive stresses. Fig. P4.163 and P4.164 r1 40 mm 60 mm 120 N · m Fig. P4.165 and P4.166 5 kN 4.166 For the curved bar and loading shown, determine the percent error introduced in the computation of the maximum stress by assuming that the bar is straight. Consider the case when (a) r1 5 20 mm, (b) r1 5 200 mm, (c) r1 5 2 m. 4.167 The curved bar shown has a cross section of 30 3 30 mm. Knowing that a 5 60 mm, determine the stress at (a) point A, (b) point B. 4.168 The curved bar shown has a cross section of 30 3 30 mm. Knowing that the allowable compressive stress is 175 MPa, determine the largest allowable distance a. a 30 mm B A 20 mm 20 mm C 30 mm 5 kN Fig. P4.167 and P4.168 301 bee80288_ch04_220-313.indd Page 302 11/11/10 3:03:21 PM user-f499 302 /Users/user-f499/Desktop/Temp Work/Don't Delete Job/MHDQ251:Beer:201/ch04 4.169 Steel links having the cross section shown are available with different central angles b. Knowing that the allowable stress is 12 ksi, determine the largest force P that can be applied to a link for which b 5 908. Pure Bending 0.3 in. B B 0.4 in. P' A 100 0.8 in. P 0.4 in. A 0.8 in. 1.2 in. M 2.5 kN · m C Fig. P4.169 D A C B 4.170 Solve Prob. 4.169, assuming that b 5 608. 20 A B 60 40 50 20 Dimensions in mm Fig. P4.171 and P4.172 4.171 A machine component has a T-shaped cross section that is orientated as shown. Knowing that M 5 2.5 kN ? m, determine the stress at (a) point A, (b) point B. 4.172 Assuming that the couple shown is replaced by a vertical 10-kN force attached at point D and acting downward, determine the stress at (a) point A, (b) point B. 4.173 Three plates are welded together to form the curved beam shown. For the given loading, determine the distance e between the neutral axis and the centroid of the cross section. 2 in. B 0.5 in. 0.5 in. A M' 2.5 kN d r1 B A Fig. P4.175 and P4.176 2 in. 0.5 in. M 3 in. 3 in. C Fig. P4.173 and P4.174 4.174 Three plates are welded together to form the curved beam shown. For M 5 8 kip ? in., determine the stress at (a) point A, (b) point B, (c) the centroid of the cross section. 4.175 The split ring shown has an inner radius r1 5 20 mm and a circular cross section of diameter d 5 32 mm. For the loading shown, determine the stress at (a) point A, (b) point B. 4.176 The split ring shown has an inner radius r1 5 16 mm and a circular cross section of diameter d 5 32 mm. For the loading shown, determine the stress at (a) point A, (b) point B. bee80288_ch04_220-313.indd Page 303 11/11/10 3:03:21 PM user-f499 /Users/user-f499/Desktop/Temp Work/Don't Delete Job/MHDQ251:Beer:201/ch04 Problems 4.177 The curved bar shown has a circular cross section of 32-mm diameter. Determine the largest couple M that can be applied to the bar about a horizontal axis if the maximum stress is not to exceed 60 MPa. 303 50 lb M B C A C 16 mm 12 mm 50 lb a 0.6 in. 0.5 in. Fig. P4.178 and P4.179 Fig. P4.177 4.178 The bar shown has a circular cross section of 0.6 in.-diameter. Knowing that a 5 1.2 in., determine the stress at (a) point A, (b) point B. 4.179 The bar shown has a circular cross section of 0.6-in. diameter. Knowing that the allowable stress is 8 ksi, determine the largest permissible distance a from the line of action of the 50-lb forces to the plane containing the center of curvature of the bar. 4.180 Knowing that P 5 10 kN, determine the stress at (a) point A, (b) point B. P 90 mm B 100 mm Fig. P4.180 4.181 and 4.182 Knowing that M 5 5 kip ? in., determine the stress at (a) point A, (b) point B. M 2.5 in. M A 2 in. 2 in. 2 in. M C 3 in. C 3 in. B 2.5 in. B M A 80 mm A 2 in. 3 in. 3 in. Fig. P4.181 Fig. P4.182 4.183 For the curved beam and loading shown, determine the stress at (a) point A, (b) point B. B a 20 mm B A 30 mm a 250 N · m 250 N · m A 40 mm 35 mm 25 mm 60 mm 35 mm Section a-a Fig. P4.183 4.184 For the crane hook shown, determine the largest tensile stress in section a-a. a 40 mm a 60 mm Section a-a Fig. P4.184 15 kN bee80288_ch04_220-313.indd Page 304 10/26/10 4:35:20 PM user-f499 304 Pure Bending 80 kip · in. b B A B A /Volumes/201/MHDQ251/bee80288_disk1of1/0073380288/bee80288_pagefiles 4.185 Knowing that the machine component shown has a trapezoidal cross section with a 5 3.5 in. and b 5 2.5 in., determine the stress at (a) point A, (b) point B. 4.186 Knowing that the machine component shown has a trapezoidal cross section with a 5 2.5 in. and b 5 3.5 in., determine the stress at (a) point A, (b) point B. C a 4.187 Show that if the cross section of a curved beam consists of two or more rectangles, the radius R of the neutral surface can be expressed as 6 in. 4 in. Fig. P4.185 and P4.186 R5 b2 A r3 b2 r4 b3 r2 ln c a b a b a b d r1 r2 r3 b1 where A is the total area of the cross section. b3 b1 r1 r2 4.188 through 4.190 Using Eq. (4.66), derive the expression for R given in Fig. 4.73 for *4.188 A circular cross section. 4.189 A trapezoidal cross section. 4.190 A triangular cross section. r3 r4 Fig. P4.187 *4.191 For a curved bar of rectangular cross section subjected to a bending couple M, show that the radial stress at the neutral surface is sr 5 r1 M R a1 2 2 ln b r1 Ae R and compute the value of sr for the curved bar of Examples 4.10 and 4.11. (Hint: consider the free-body diagram of the portion of the beam located above the neutral surface.) C  2  2 r1 x x b r r Fig. P4.191 R bee80288_ch04_220-313.indd Page 305 10/26/10 4:35:28 PM user-f499 /Volumes/201/MHDQ251/bee80288_disk1of1/0073380288/bee80288_pagefiles REVIEW AND SUMMARY This chapter was devoted to the analysis of members in pure bending. That is, we considered the stresses and deformation in members subjected to equal and opposite couples M and M9 acting in the same longitudinal plane (Fig. 4.77). We first studied members possessing a plane of symmetry and subjected to couples acting in that plane. Considering possible deformations of the member, we proved that transverse sections remain plane as a member is deformed [Sec. 4.3]. We then noted that a member in pure bending has a neutral surface along which normal strains and stresses are zero and that the longitudinal normal strain Px varies linearly with the distance y from the neutral surface: Px 5 2 y r M' M A B Fig. 4.77 Normal strain in bending C (4.8)  where r is the radius of curvature of the neutral surface (Fig. 4.78). The intersection of the neutral surface with a transverse section is known as the neutral axis of the section.  –y y B K A For members made of a material that follows Hooke’s law [Sec. 4.4], we found that the normal stress sx varies linearly with the distance from the neutral axis (Fig. 4.79). Denoting by sm the maximum stress we wrote y sx 5 2 sm c (4.12) J D A⬘ O Mc sm 5 I (4.15) E B⬘ Fig. 4.78 Normal stress in elastic range where c is the largest distance from the neutral axis to a point in the section. By setting the sum of the elementary forces, sx dA, equal to zero, we proved that the neutral axis passes through the centroid of the cross section of a member in pure bending. Then by setting the sum of the moments of the elementary forces equal to the bending moment, we derived the elastic flexure formula for the maximum normal stress x y m y c Neutral surface x Fig. 4.79 Elastic flexure formula where I is the moment of inertia of the cross section with respect to the neutral axis. We also obtained the normal stress at any distance y from the neutral axis: sx 5 2 My I (4.16) 305 bee80288_ch04_220-313.indd Page 306 11/11/10 3:03:22 PM user-f499 306 /Users/user-f499/Desktop/Temp Work/Don't Delete Job/MHDQ251:Beer:201/ch04 Noting that I and c depend only on the geometry of the cross section, we introduced the elastic section modulus Pure Bending S5 Elastic section modulus I c (4.17) and then used the section modulus to write an alternative expression for the maximum normal stress: sm 5 Curvature of member M S (4.18) Recalling that the curvature of a member is the reciprocal of its radius of curvature, we expressed the curvature of the member as 1 M 5 r EI (4.21) Anticlastic curvature In Sec. 4.5, we completed our study of the bending of homogeneous members possessing a plane of symmetry by noting that deformations occur in the plane of a transverse cross section and result in anticlastic curvature of the members. Members made of several materials Next we considered the bending of members made of several materials with different moduli of elasticity [Sec. 4.6]. While transverse sections remain plane, we found that, in general, the neutral axis does not pass through the centroid of the composite cross section (Fig. 4.80). Using the ratio of the moduli of elasticity of the materials, y 1 y E1y 1  – —–  y x  – —  x N. A. 2 (a) y (b) Fig. 4.80 y My x  – —– I x E2 y 2  – —–  C N. A. x (c) Fig. 4.81 we obtained a transformed section corresponding to an equivalent member made entirely of one material. We then used the methods previously developed to determine the stresses in this equivalent homogeneous member (Fig. 4.81) and then again used the ratio of the moduli of elasticity to determine the stresses in the composite beam [Sample Probs. 4.3 and 4.4]. Stress concentrations In Sec. 4.7, stress concentrations that occur in members in pure bending were discussed and charts giving stress-concentration factors for flat bars with fillets and grooves were presented in Figs. 4.27 and 4.28. bee80288_ch04_220-313.indd Page 307 10/26/10 4:35:47 PM user-f499 /Volumes/201/MHDQ251/bee80288_disk1of1/0073380288/bee80288_pagefiles Review and Summary We next investigated members made of materials that do not follow Hooke’s law [Sec. 4.8]. A rectangular beam made of an elastoplastic material (Fig. 4.82) was analyzed as the magnitude of the bending moment was increased. The maximum elastic moment MY occurred when yielding was initiated in the beam (Fig. 4.83). As the bending moment was further increased, plastic zones developed and the size of the elastic core of the member decreased [Sec. 4.9]. Finally the beam became fully plastic and we obtained the maximum or plastic moment Mp. In Sec. 4.11, we found that permanent deformations and residual stresses remain in a member after the loads that caused yielding have been removed. y ELASTIC c (a) M PLASTIC   max   m c  max   m   y  c x (c) M Plastic deformations (b) M  M y c  Fig. 4.82 x  M ELASTIC PLASTIC Y c ELASTIC x Y Y y  c  c x PLASTIC c  max    (d) M  Mp M Fig. 4.83 Eccentric axial loading In Sec. 4.12, we studied the stresses in members loaded eccentrically in a plane of symmetry. Our analysis made use of methods developed earlier. We replaced the eccentric load by a force-couple system located at the centroid of the cross section (Fig. 4.84) and then superposed stresses due to the centric load and the bending couple (Fig. 4.85): sx 5 y C Fig. 4.85 My P 2 I A x + C A Fig. 4.84 y x = N.A. C C P' (4.50) y M D x F d 307 bee80288_ch04_220-313.indd Page 308 11/11/10 3:03:22 PM user-f499 308 Pure Bending Unsymmetric bending /Users/user-f499/Desktop/Temp Work/Don't Delete Job/MHDQ251:Beer:201/ch04 The bending of members of unsymmetric cross section was considered next [Sec. 4.13]. We found that the flexure formula may be used, provided that the couple vector M is directed along one of the principal centroidal axes of the cross section. When necessary we y M y M' My   M z Mz C x z Fig. 4.86 N. M resolved M into components along the principal axes and superposed the stresses due to the component couples (Figs. 4.86 and 4.87). Myz Mzy sx 5 2 1 (4.55) Iy Iz y A.   Fig. 4.87 C z For the couple M shown in Fig. 4.88, we determined the orientation of the neutral axis by writing tan f 5 Fig. 4.88 General eccentric axial loading Curved members C  A J D F Fig. 4.89 (4.57) The general case of eccentric axial loading was considered in Sec. 4.14, where we again replaced the load by a force-couple system located at the centroid. We then superposed the stresses due to the centric load and two component couples directed along the principal axes: Myz Mzy P sx 5 2 1 (4.58) Iy Iz A The chapter concluded with the analysis of stresses in curved members (Fig. 4.89). While transverse sections remain plane when the member is subjected to bending, we found that the stresses do not vary linearly and the neutral surface does not pass through the centroid of the section. The distance R from the center of curvature of the member to the neutral surface was found to be y R Iz tan u Iy R5 r B y K (4.66) where A is the area of the cross section. The normal stress at a distance y from the neutral surface was expressed as E G x # A dA r sx 5 2 My Ae1R 2 y2 (4.70) where M is the bending moment and e the distance from the centroid of the section to the neutral surface. bee80288_ch04_220-313.indd Page 309 10/26/10 4:36:50 PM user-f499 /Volumes/201/MHDQ251/bee80288_disk1of1/0073380288/bee80288_pagefiles REVIEW PROBLEMS 4.192 Two vertical forces are applied to a beam of the cross section 3 in. 3 in. 3 in. shown. Determine the maximum tensile and compressive stresses in portion BC of the beam. 6 in. 4.193 Straight rods of 6-mm diameter and 30-m length are stored by 2 in. coiling the rods inside a drum of 1.25-m inside diameter. Assuming that the yield strength is not exceeded, determine (a) the maximum stress in a coiled rod, (b) the corresponding bending moment in the rod. Use E 5 200 GPa. A 15 kips 15 kips B C 60 in. 40 in. D 40 in. Fig. P4.192 2.4 in. Fig. P4.193 4.194 Knowing that for the beam shown the allowable stress is 12 ksi in 4.195 In order to increase corrosion resistance, a 2-mm-thick cladding of aluminum has been added to a steel bar as shown. The modulus of elasticity is 200 GPa for steel and 70 GPa for aluminum. For a bending moment of 300 N ? m, determine (a) the maximum stress in the steel, (b) the maximum stress in the aluminum, (c) the radius of curvature of the bar. M  300 N · m 1.2 in. 0.75 in. tension and 16 ksi in compression, determine the largest couple M that can be applied. M Fig. P4.194 26 mm 30 mm y 46 mm 50 mm P Fig. P4.195 z x 4.196 A single vertical force P is applied to a short steel post as shown. Gages located at A, B, and C indicate the following strains: PA 5 2500 m PB 5 21000 m 6 B PC 5 2200 m Knowing that E 5 29 3 10 psi, determine (a) the magnitude of P, (b) the line of action of P, (c) the corresponding strain at the hidden edge of the post, where x 5 22.5 in. and z 5 21.5 in. C A 5 in. 3 in. Fig. P4.196 309 bee80288_ch04_220-313.indd Page 310 11/11/10 8:01:54 PM user-f499 310 /Users/user-f499/Desktop/Temp Work/Don't Delete Job/MHDQ251:Beer:201/ch04 4.197 For the split ring shown, determine the stress at (a) point A, Pure Bending (b) point B. 2500 N 90 mm 40 mm B A 14 mm 5 y' W200  19.3 A B Fig. P4.197 z' C M  8 kN · m 4.198 A couple M of moment 8 kN ? m acting in a vertical plane is E applied to a W200 3 19.3 rolled-steel beam as shown. Determine (a) the angle that the neutral axis forms with the horizontal plane, (b) the maximum stress in the beam. D Fig. P4.198 4.199 Determine the maximum stress in each of the two machine ele- ments shown. 400 lb 400 lb 2.5 400 lb 400 lb 2.5 3 r ⫽ 0.3 r ⫽ 0.3 P 1.5 a a 0.5 3 0.5 1.5 90⬚ 0.5 0.5 (a) t B (b) Fig. P4.199 All dimensions given in inches. C A P' Fig. P4.200 4.200 The shape shown was formed by bending a thin steel plate. Assum- ing that the thickness t is small compared to the length a of a side of the shape, determine the stress (a) at A, (b) at B, (c) at C. bee80288_ch04_220-313.indd Page 311 10/26/10 4:37:11 PM user-f499 /Volumes/201/MHDQ251/bee80288_disk1of1/0073380288/bee80288_pagefiles 4.201 Three 120 3 10-mm steel plates have been welded together to form the beam shown. Assuming that the steel is elastoplastic with E 5 200 GPa and sY 5 300 MPa, determine (a) the bending moment for which the plastic zones at the top and bottom of the beam are 40 mm thick, (b) the corresponding radius of curvature of the beam. 120 mm 10 mm M 120 mm 10 mm 10 mm Fig. P4.201 4.202 A short column is made by nailing four 1 3 4-in. planks to a 4 3 4-in. timber. Determine the largest compressive stress created in the column by a 16-kip load applied as shown in the center of the top section of the timber if (a) the column is as described, (b) plank 1 is removed, (c) planks 1 and 2 are removed, (d) planks 1, 2, and 3 are removed, (e) all planks are removed. 16 kips 2 4 3 1 Fig. P4.202 4.203 Two thin strips of the same material and same cross section are bent by couples of the same magnitude and glued together. After the two surfaces of contact have been securely bonded, the couples are removed. Denoting by s1 the maximum stress and by r1 the radius of curvature of each strip while the couples were applied, determine (a) the final stresses at points A, B, C, and D, (b) the final radius of curvature. M1 M1 1 A M'1 M'1 1 B C D Fig. P4.203 1 1 Review Problems 311 bee80288_ch04_220-313.indd Page 312 10/26/10 4:37:17 PM user-f499 /Volumes/201/MHDQ251/bee80288_disk1of1/0073380288/bee80288_pagefiles COMPUTER PROBLEMS The following problems are designed to be solved with a computer. Aluminum a Steel h  40 mm a b  60 mm Fig. P4.C1 4.C1 Two aluminum strips and a steel strip are to be bonded together to form a composite member of width b 5 60 mm and depth h 5 40 mm. The modulus of elasticity is 200 GPa for the steel and 75 GPa for the aluminum. Knowing that M 5 1500 N ? m, write a computer program to calculate the maximum stress in the aluminum and in the steel for values of a from 0 to 20 mm using 2-mm increments. Using appropriate smaller increments, determine (a) the largest stress that can occur in the steel, (b) the corresponding value of a. 4.C2 A beam of the cross section shown, made of a steel that is assumed to be elastoplastic with a yield strength sY and a modulus of elasticity E, is bent about the x axis. (a) Denoting by yY the half thickness of the elastic core, write a computer program to calculate the bending moment M and the radius of curvature r for values of yY from 12 d to 16 d using decrements equal to 12 tf. Neglect the effect of fillets. (b) Use this program to solve Prob. 4.201. tf y x tw d bf Fig. P4.C2 4.C3 An 8-kip ? in. couple M is applied to a beam of the cross section shown in a plane forming an angle b with the vertical. Noting that the centroid of the cross section is located at C and that the y and z axes are principal axes, write a computer program to calculate the stress at A, B, C, and D for values of b from 0 to 1808 using 108 increments. (Given: Iy 5 6.23 in4 and Iz 5 1.481 in4.)  y 0.4 0.4 A B z M 0.4 C  1.2 D E 0.8 0.4 1.6 0.4 0.8 Dimensions in inches Fig. P4.C3 312 1.2 bee80288_ch04_220-313.indd Page 313 10/26/10 4:37:28 PM user-f499 /Volumes/201/MHDQ251/bee80288_disk1of1/0073380288/bee80288_pagefiles 4.C4 Couples of moment M 5 2 kN ? m are applied as shown to a curved bar having a rectangular cross section with h 5 100 mm and b 5 25 mm. Write a computer program and use it to calculate the stresses at points A and B for values of the ratio r1/h from 10 to 1 using decrements of 1, and from 1 to 0.1 using decrements of 0.1. Using appropriate smaller increments, determine the ratio r1/h for which the maximum stress in the curved bar is 50% larger than the maximum stress in a straight bar of the same cross section. 4.C5 The couple M is applied to a beam of the cross section shown. (a) Write a computer program that, for loads expressed in either SI or U.S. customary units, can be used to calculate the maximum tensile and compressive stresses in the beam. (b) Use this program to solve Probs. 4.10, 4.11, and 4.192. Computer Problems b B B A A h M' M r1 C Fig. P4.C4 bn hn M h2 b2 h1 b1 Fig. P4.C5 y y y 4.C6 A solid rod of radius c 5 1.2 in. is made of a steel that is assumed to be elastoplastic with E 5 29,000 ksi and sY 5 42 ksi. The rod is subjected to a couple of moment M that increases from zero to the maximum elastic moment MY and then to the plastic moment Mp. Denoting by yY the half thickness of the elastic core, write a computer program and use it to calculate the bending moment M and the radius of curvature r for values of yY from 1.2 in. to 0 using 0.2-in. decrements. (Hint: Divide the cross section into 80 horizontal elements of 0.03-in. height.) 4.C7 The machine element of Prob. 4.182 is to be redesigned by removing part of the triangular cross section. It is believed that the removal of a small triangular area of width a will lower the maximum stress in the element. In order to verify this design concept, write a computer program to calculate the maximum stress in the element for values of a from 0 to 1 in. using 0.1-in. increments. Using appropriate smaller increments, determine the distance a for which the maximum stress is as small as possible and the corresponding value of the maximum stress. 2 in. C 3 in. A 2.5 in. a Fig. P4.C7 B c M z Fig. P4.C6 313 bee80288_ch05_314-379.indd Page 314 10/27/10 9:48:37 PM user-f499 The beams supporting the multiple overhead cranes system shown in this picture are subjected to transverse loads causing the beams to bend. The normal stresses resulting from such loadings will be determined in this chapter. 314 /Volumes/201/MHDQ251/bee80288_disk1of1/0073380288/bee80288_pagefiles bee80288_ch05_314-379.indd Page 315 10/27/10 9:50:57 PM user-f499 /Volumes/201/MHDQ251/bee80288_disk1of1/0073380288/bee80288_pagefiles C H A P T E R Analysis and Design of Beams for Bending 315 bee80288_ch05_314-379.indd Page 316 10/29/10 7:07:38 PM user-f499 5.1 Chapter 5 Analysis and Design of Beams for Bending 5.1 5.2 5.3 5.4 *5.5 *5.6 P2 B A C w A C B (b) Distributed load Transversely loaded Statically Determinate Beams Photo 5.1 Timber beams used in residential dwelling. D (a) Concentrated loads Fig. 5.1 beams. The transverse loading of a beam may consist of concentrated loads P1, P2, . . . , expressed in newtons, pounds, or their multiples, kilonewtons and kips (Fig. 5.1a), of a distributed load w, expressed in N/m, kN/m, lb/ft, or kips/ft (Fig. 5.1b), or of a combination of both. When the load w per unit length has a constant value over part of the beam (as between A and B in Fig. 5.1b), the load is said to be uniformly distributed over that part of the beam. Beams are classified according to the way in which they are supported. Several types of beams frequently used are shown in Fig. 5.2. The distance L shown in the various parts of the figure is L L (a) Simply supported beam Statically Indeterminate Beams L1 L2 (d) Continuous beam Fig. 5.2 Common beam support configurations. 316 INTRODUCTION This chapter and most of the next one will be devoted to the analysis and the design of beams, i.e., structural members supporting loads applied at various points along the member. Beams are usually long, straight prismatic members, as shown in the photo on the previous page. Steel and aluminum beams play an important part in both structural and mechanical engineering. Timber beams are widely used in home construction (Photo 5.1). In most cases, the loads are perpendicular to the axis of the beam. Such a transverse loading causes only bending and shear in the beam. When the loads are not at a right angle to the beam, they also produce axial forces in the beam. Introduction Shear and Bending-Moment Diagrams Relations Among Load, Shear, and Bending Moment Design of Prismatic Beams for Bending Using Singularity Functions to Determine Shear and Bending Moment in a Beam Nonprismatic Beams P1 /Volumes/201/MHDQ251/bee80288_disk1of1/0073380288/bee80288_pagefiles L (b) Overhanging beam L (e) Beam fixed at one end and simply supported at the other end (c) Cantilever beam L ( f ) Fixed beam bee80288_ch05_314-379.indd Page 317 11/12/10 7:30:37 PM user-f499 /Users/user-f499/Desktop/Temp Work/Don't Delete Job/MHDQ251:Beer:201/ch05 called the span. Note that the reactions at the supports of the beams in parts a, b, and c of the figure involve a total of only three unknowns and, therefore, can be determined by the methods of statics. Such beams are said to be statically determinate and will be discussed in this chapter and the next. On the other hand, the reactions at the supports of the beams in parts d, e, and f of Fig. 5.2 involve more than three unknowns and cannot be determined by the methods of statics alone. The properties of the beams with regard to their resistance to deformations must be taken into consideration. Such beams are said to be statically indeterminate and their analysis will be postponed until Chap. 9, where deformations of beams will be discussed. Sometimes two or more beams are connected by hinges to form a single continuous structure. Two examples of beams hinged at a point H are shown in Fig. 5.3. It will be noted that the reactions at the supports involve four unknowns and cannot be determined from the free-body diagram of the two-beam system. They can be determined, however, by recognizing that the internal moment at the hinge is zero. Then, after considering the free-body diagram of each beam separately, six unknowns are involved (including two force components at the hinge), and six equations are available. When a beam is subjected to transverse loads, the internal forces in any section of the beam will generally consist of a shear force V and a bending couple M. Consider, for example, a simply supported beam AB carrying two concentrated loads and a uniformly distributed load (Fig. 5.4a). To determine the internal forces in a section through point C we first draw the free-body diagram of the entire beam to obtain the reactions at the supports (Fig. 5.4b). Passing a section through C, we then draw the free-body diagram of AC (Fig. 5.4c), from which we determine the shear force V and the bending couple M. The bending couple M creates normal stresses in the cross section, while the shear force V creates shearing stresses in that section. In most cases the dominant criterion in the design of a beam for strength is the maximum value of the normal stress in the beam. The determination of the normal stresses in a beam will be the subject of this chapter, while shearing stresses will be discussed in Chap. 6. Since the distribution of the normal stresses in a given section depends only upon the value of the bending moment M in that section and the geometry of the section,† the elastic flexure formulas derived in Sec. 4.4 can be used to determine the maximum stress, as well as the stress at any given point, in the section. We write‡ sm 5 ZMZc I sx 5 2 My I (5.1, 5.2) †It is assumed that the distribution of the normal stresses in a given cross section is not affected by the deformations caused by the shearing stresses. This assumption will be verified in Sec. 6.5. ‡We recall from Sec. 4.2 that M can be positive or negative, depending upon whether the concavity of the beam at the point considered faces upward or downward. Thus, in the case considered here of a transverse loading, the sign of M can vary along the beam. On the other hand, since sm is a positive quantity, the absolute value of M is used in Eq. (5.1). 5.1 Introduction H B A (a) H A C B (b) Fig. 5.3 Beams connected by hinges. w P2 P1 C B A a (a) Transversely-loaded beam w P2 P1 C A B RA RB (b) Free-body diagram to find support reactions wa P1 C M A V RA (c) Free-body diagram to find internal forces at C Fig. 5.4 Analysis of a simply supported beam. 317 bee80288_ch05_314-379.indd Page 318 10/27/10 9:51:23 PM user-f499 318 Analysis and Design of Beams for Bending /Volumes/201/MHDQ251/bee80288_disk1of1/0073380288/bee80288_pagefiles where I is the moment of inertia of the cross section with respect to a centroidal axis perpendicular to the plane of the couple, y is the distance from the neutral surface, and c is the maximum value of that distance (Fig. 4.11). We also recall from Sec. 4.4 that, introducing the elastic section modulus S 5 Iyc of the beam, the maximum value s m of the normal stress in the section can be expressed as sm 5 ZMZ S (5.3) The fact that sm is inversely proportional to S underlines the importance of selecting beams with a large section modulus. Section moduli of various rolled-steel shapes are given in Appendix C, while the section modulus of a rectangular shape can be expressed, as shown in Sec. 4.4, as S 5 16 bh2 (5.4) where b and h are, respectively, the width and the depth of the cross section. Equation (5.3) also shows that, for a beam of uniform cross section, sm is proportional to |M|: Thus, the maximum value of the normal stress in the beam occurs in the section where |M| is largest. It follows that one of the most important parts of the design of a beam for a given loading condition is the determination of the location and magnitude of the largest bending moment. This task is made easier if a bending-moment diagram is drawn, i.e., if the value of the bending moment M is determined at various points of the beam and plotted against the distance x measured from one end of the beam. It is further facilitated if a shear diagram is drawn at the same time by plotting the shear V against x. The sign convention to be used to record the values of the shear and bending moment will be discussed in Sec. 5.2. The values of V and M will then be obtained at various points of the beam by drawing free-body diagrams of successive portions of the beam. In Sec. 5.3 relations among load, shear, and bending moment will be derived and used to obtain the shear and bending-moment diagrams. This approach facilitates the determination of the largest absolute value of the bending moment and, thus, the determination of the maximum normal stress in the beam. In Sec. 5.4 you will learn to design a beam for bending, i.e., so that the maximum normal stress in the beam will not exceed its allowable value. As indicated earlier, this is the dominant criterion in the design of a beam. Another method for the determination of the maximum values of the shear and bending moment, based on expressing V and M in terms of singularity functions, will be discussed in Sec. 5.5. This approach lends itself well to the use of computers and will be expanded in Chap. 9 to facilitate the determination of the slope and deflection of beams. Finally, the design of nonprismatic beams, i.e., beams with a variable cross section, will be discussed in Sec. 5.6. By selecting bee80288_ch05_314-379.indd Page 319 10/27/10 9:51:23 PM user-f499 /Volumes/201/MHDQ251/bee80288_disk1of1/0073380288/bee80288_pagefiles 5.2 Shear and Bending-Moment Diagrams the shape and size of the variable cross section so that its elastic section modulus S 5 Iyc varies along the length of the beam in the same way as |M|, it is possible to design beams for which the maximum normal stress in each section is equal to the allowable stress of the material. Such beams are said to be of constant strength. 5.2 SHEAR AND BENDING-MOMENT DIAGRAMS As indicated in Sec. 5.1, the determination of the maximum absolute values of the shear and of the bending moment in a beam are greatly facilitated if V and M are plotted against the distance x measured from one end of the beam. Besides, as you will see in Chap. 9, the knowledge of M as a function of x is essential to the determination of the deflection of a beam. In the examples and sample problems of this section, the shear and bending-moment diagrams will be obtained by determining the values of V and M at selected points of the beam. These values will be found in the usual way, i.e., by passing a section through the point where they are to be determined (Fig. 5.5a) and considering the equilibrium of the portion of beam located on either side of the section (Fig. 5.5b). Since the shear forces V and V9 have opposite senses, recording the shear at point C with an up or down arrow would be meaningless, unless we indicated at the same time which of the free bodies AC and CB we are considering. For this reason, the shear V will be recorded with a sign: a plus sign if the shearing forces are directed as shown in Fig. 5.5b, and a minus sign otherwise. A similar convention will apply for the bending moment M. It will be considered as positive if the bending couples are directed as shown in that figure, and negative otherwise.† Summarizing the sign conventions we have presented, we state: The shear V and the bending moment M at a given point of a beam are said to be positive when the internal forces and couples acting on each portion of the beam are directed as shown in Fig. 5.6a. These conventions can be more easily remembered if we note that P1 P2 w C A B x (a) P1 w A C M V (b) RA P2 V' B M' C RB Fig. 5.5 Determination of V and M. 1. The shear at any given point of a beam is positive when the external forces (loads and reactions) acting on the beam tend to shear off the beam at that point as indicated in Fig. 5.6b. M V' M' V (a) Internal forces (positive shear and positive bending moment) (b) Effect of external forces (positive shear) Fig. 5.6 Sign convention for shear and bending moment. †Note that this convention is the same that we used earlier in Sec. 4.2 (c) Effect of external forces (positive bending moment) 319 bee80288_ch05_314-379.indd Page 320 10/27/10 9:51:28 PM user-f499 320 /Volumes/201/MHDQ251/bee80288_disk1of1/0073380288/bee80288_pagefiles 2. The bending moment at any given point of a beam is positive Analysis and Design of Beams for Bending when the external forces acting on the beam tend to bend the beam at that point as indicated in Fig. 5.6c. It is also of help to note that the situation described in Fig. 5.6, in which the values of the shear and of the bending moment are positive, is precisely the situation that occurs in the left half of a simply supported beam carrying a single concentrated load at its midpoint. This particular case is fully discussed in the next example. EXAMPLE 5.01 P 1 2L D A RA P x A 1 2 E D V 1 2L RB P A C V' B (b) C 1 RB 2 P E 1 RA 2 P M' V' x (c) V V M E B Lx 1 RB 2 P P L 1 2 L (d) x  12 P M PL 1 2 L (e) Fig. 5.8 C B P D A 1 4 1 2L Fig. 5.7 M P 1 2 P B 1 2 (a) M' RA P 1 2L C 1 2 Draw the shear and bending-moment diagrams for a simply supported beam AB of span L subjected to a single concentrated load P at its midpoint C (Fig. 5.7). L x We first determine the reactions at the supports from the free-body diagram of the entire beam (Fig. 5.8a); we find that the magnitude of each reaction is equal to Py2. Next we cut the beam at a point D between A and C and draw the free-body diagrams of AD and DB (Fig. 5.8b). Assuming that shear and bending moment are positive, we direct the internal forces V and V9 and the internal couples M and M9 as indicated in Fig. 5.6a. Considering the free body AD and writing that the sum of the vertical components and the sum of the moments about D of the forces acting on the free body are zero, we find V 5 1Py2 and M 5 1Pxy2. Both the shear and the bending moment are therefore positive; this may be checked by observing that the reaction at A tends to shear off and to bend the beam at D as indicated in Figs. 5.6b and c. We now plot V and M between A and C (Figs. 5.8d and e); the shear has a constant value V 5 Py2, while the bending moment increases linearly from M 5 0 at x 5 0 to M 5 PLy4 at x 5 Ly2. Cutting, now, the beam at a point E between C and B and considering the free body EB (Fig. 5.8c), we write that the sum of the vertical components and the sum of the moments about E of the forces acting on the free body are zero. We obtain V 5 2Py2 and M 5 P(L 2 x)y2. The shear is therefore negative and the bending moment positive; this can be checked by observing that the reaction at B bends the beam at E as indicated in Fig. 5.6c but tends to shear it off in a manner opposite to that shown in Fig. 5.6b. We can complete, now, the shear and bendingmoment diagrams of Figs. 5.8d and e; the shear has a constant value V 5 2Py2 between C and B, while the bending moment decreases linearly from M 5 PLy4 at x 5 Ly2 to M 5 0 at x 5 L. bee80288_ch05_314-379.indd Page 321 11/12/10 7:30:42 PM user-f499 /Users/user-f499/Desktop/Temp Work/Don't Delete Job/MHDQ251:Beer:201/ch05 Draw the shear and bending-moment diagrams for a cantilever beam AB of span L supporting a uniformly distributed load w (Fig. 5.9). 321 5.2 Shear and Bending-Moment Diagrams We note from the foregoing example that, when a beam is subjected only to concentrated loads, the shear is constant between loads and the bending moment varies linearly between loads. In such situations, therefore, the shear and bending-moment diagrams can easily be drawn, once the values of V and M have been obtained at sections selected just to the left and just to the right of the points where the loads and reactions are applied (see Sample Prob. 5.1). EXAMPLE 5.02 w A B L Fig. 5.9 We cut the beam at a point C between A and B and draw the free-body diagram of AC (Fig. 5.10a), directing V and M as indicated in Fig. 5.6a. Denoting by x the distance from A to C and replacing the distributed load over AC by its resultant wx applied at the midpoint of AC, we write wx 1 2 x w M A x 1x©Fy 5 0: 2wx 2 V 5 0 V 5 2wx V C V (a) L B A 1 l©MC 5 0: x wx a b 1 M 5 0 2 x 1 M 5 2 wx2 2 (b) VB  wL M We note that the shear diagram is represented by an oblique straight line (Fig. 5.10b) and the bending-moment diagram by a parabola (Fig. 5.10c). The maximum values of V and M both occur at B, where we have VB 5 2wL L B A MB 5 212wL2 (c) Fig. 5.10 1 x MB  2 wL2 bee80288_ch05_314-379.indd Page 322 11/16/10 6:41:54 PM user-f499 20 kN SAMPLE PROBLEM 5.1 40 kN B A 250 mm For the timber beam and loading shown, draw the shear and bending-moment D C 2.5 m 3m diagrams and determine the maximum normal stress due to bending. 2m 80 mm SOLUTION 40 kN 20 kN D B A C 1 2 3 4 46 kN 2.5 m 3m 20 kN 5 6 Reactions. Considering the entire beam as a free body, we find 14 kN RB 5 40 kNx 2m V1 20 kN V2 V3 20 kN M4 V3 V4 V5 V6 M5 V5 40 kN 20 kN M6 40 kN M'4 V ⫹26 kN x ⫺14 kN ⫺20 kN 2.5 m 3m M M3 M4 M5 M6 5 250 kN ? m 5 128 kN ? m 5 128 kN ? m 50 V4 2 40 kN 1 14 kN 5 0 2M4 1 114 kN2 12 m2 5 0 1x©Fy 5 0 : 1l©M4 5 0 : 14 kN V'4 5 126 kN 5 126 kN 5 214 kN 5 214 kN For several of the latter sections, the results may be more easily obtained by considering as a free body the portion of the beam to the right of the section. For example, for the portion of the beam to the right of section 4, we have V6 46 kN V 2 5 220 kN M2 5 250 kN ? m The shear and bending moment at sections 3, 4, 5, and 6 are determined in a similar way from the free-body diagrams shown. We obtain 40 kN 46 kN 220 kN 2 V 2 5 0 120 kN2 12.5 m2 1 M2 5 0 1x©F y 5 0 : 1l©M 2 5 0 : V4 46 kN V 1 5 220 kN M1 5 0 We next consider as a free body the portion of beam to the left of section 2 and write M3 20 kN 220 kN 2 V 1 5 0 120 kN2 10 m2 1 M 1 5 0 1x©F y 5 0 : 1l©M 1 5 0 : M2 46 kN RD 5 14 kNx Shear and Bending-Moment Diagrams. We first determine the internal forces just to the right of the 20-kN load at A. Considering the stub of beam to the left of section 1 as a free body and assuming V and M to be positive (according to the standard convention), we write M1 20 kN /Volumes/201/MHDQ251/bee80288_disk1of1/0073380288/bee80288_pagefiles 2m V4 5 126 kN M4 5 128 kN ? m We can now plot the six points shown on the shear and bendingmoment diagrams. As indicated earlier in this section, the shear is of constant value between concentrated loads, and the bending moment varies linearly; we obtain therefore the shear and bending-moment diagrams shown. Maximum Normal Stress. It occurs at B, where |M| is largest. We use Eq. (5.4) to determine the section modulus of the beam: ⫹28 kN ? m x S 5 16bh2 5 16 10.080 m2 10.250 m2 2 5 833.33 3 1026 m3 Substituting this value and |M| 5 |MB| 5 50 3 103 N ? m into Eq. (5.3) gives ⫺50 kN ? m sm 5 ZMBZ S 5 150 3 103 N ? m2 833.33 3 1026 5 60.00 3 106 Pa Maximum normal stress in the beam 5 60.0 MPa 322 ◀ bee80288_ch05_314-379.indd Page 323 11/16/10 6:47:15 PM user-f499 8 ft SAMPLE PROBLEM 5.2 10 kips 2 ft 3 ft 3 ft 3 kips/ft The structure shown consists of a W10 3 112 rolled-steel beam AB and of two short members welded together and to the beam. (a) Draw the shear and bending-moment diagrams for the beam and the given loading. (b) Determine the maximum normal stress in sections just to the left and just to the right of point D. E B A C 3 kips/ft 1 A D 20 kip ? ft C 2 D 10 kips 3x /Users/user-f499/Desktop/Temp Work/Don't Delete Job/MHDQ251:Beer:201/ch05 318 kip ? ft SOLUTION Equivalent Loading of Beam. The 10-kip load is replaced by an equivalent force-couple system at D. The reaction at B is determined by considering the beam as a free body. 3 B 34 kips x 2 a. Shear and Bending-Moment Diagrams From A to C. We determine the internal forces at a distance x from point A by considering the portion of beam to the left of section 1. That part of the distributed load acting on the free body is replaced by its resultant, and we write M x V 24 kips x⫺4 M x x⫺4 20 kip ? ft 10 kips From C to D. Considering the portion of beam to the left of section 2 and again replacing the distributed load by its resultant, we obtain M V x ⫺ 11 x 224 2 V 5 0 241x 2 42 1 M 5 0 1x©Fy 5 0 : 1l©M2 5 0 : V 5 224 kips M 5 96 2 24 x kip ? ft These expressions are valid in the region 8 ft , x , 11 ft. V 8 ft V 5 23 x kips M 5 21.5 x2 kip ? ft Since the free-body diagram shown can be used for all values of x smaller than 8 ft, the expressions obtained for V and M are valid in the region 0 , x , 8 ft. V 24 kips 23 x 2 V 5 0 3 x1 12 x2 1 M 5 0 1x©Fy 5 0 : 1l©M1 5 0 : 11 ft 16 ft x From D to B. Using the position of beam to the left of section 3, we obtain for the region 11 ft , x , 16 ft V 5 234 kips ⫺ 24 kips M ⫺148 kip ? ft ⫺ 168 kip ? ft ⫺ 318 kip ? ft kip ? ft The shear and bending-moment diagrams for the entire beam can now be plotted. We note that the couple of moment 20 kip ? ft applied at point D introduces a discontinuity into the bending-moment diagram. ⫺ 34 kips ⫺ 96 kip ? ft M 5 226 2 34 x x b. Maximum Normal Stress to the Left and Right of Point D. From Appendix C we find that for the W10 3 112 rolled-steel shape, S 5 126 in3 about the X-X axis. To the left of D: We have |M| 5 168 kip ? ft 5 2016 kip ? in. Substituting for |M| and S into Eq. (5.3), we write 0M 0 2016 kip ? in. sm 5 16.00 ksi ◀ 5 16.00 ksi 126 in3 To the right of D: We have |M| 5 148 kip ? ft 5 1776 kip ? in. Substituting for |M| and S into Eq. (5.3), we write sm 5 sm 5 S 0M 0 S 5 5 1776 kip ? in. 126 in3 5 14.10 ksi sm 5 14.10 ksi ◀ 323 bee80288_ch05_314-379.indd Page 324 10/27/10 9:51:52 PM user-f499 /Volumes/201/MHDQ251/bee80288_disk1of1/0073380288/bee80288_pagefiles PROBLEMS 5.1 through 5.6 For the beam and loading shown, (a) draw the shear and bending-moment diagrams, (b) determine the equations of the shear and bending-moment curves. P A w B C B A a b L L Fig. P5.1 Fig. P5.2 w0 w B A A C B L a L Fig. P5.3 Fig. P5.4 w B A w w C a D B A C a a D a L L Fig. P5.5 Fig. P5.6 5.7 and 5.8 Draw the shear and bending-moment diagrams for the beam and loading shown, and determine the maximum absolute value (a) of the shear, (b) of the bending moment. 300 lb 240 lb 360 lb D E C A 4 in. 3 in. 4 in. B 5 in. A 200 N 200 N 500 N 200 N C E 300 D 225 300 Dimensions in mm Fig. P5.7 324 Fig. P5.8 B 225 bee80288_ch05_314-379.indd Page 325 10/27/10 9:52:07 PM user-f499 /Volumes/201/MHDQ251/bee80288_disk1of1/0073380288/bee80288_pagefiles Problems 5.9 and 5.10 Draw the shear and bending-moment diagrams for the beam and loading shown, and determine the maximum absolute value (a) of the shear, (b) of the bending moment. 12 kN/m C A B C A 4 ft 1m 2m 15 kips 2 kips/ft 40 kN Fig. P5.9 325 D B 4 ft 4 ft Fig. P5.10 5.11 and 5.12 Draw the shear and bending-moment diagrams for the beam and loading shown, and determine the maximum absolute value (a) of the shear, (b) of the bending moment. E 250 mm F 3 in. C A 60 kips 8 in. C 60 kips 8 in. 8 in. 250 mm A B D 250 mm 50 mm 50 mm 75 N Fig. P5.11 B D 75 N Fig. P5.12 5.13 and 5.14 Assuming that the reaction of the ground is uniformly distributed, draw the shear and bending-moment diagrams for the beam AB and determine the maximum absolute value (a) of the shear, (b) of the bending moment. 24 kips 2 kips/ft C A 3 ft D 3 ft 2 kips/ft E 3 ft B 1.5 kN 1.5 kN C D A 0.9 m 0.3 m 3 ft Fig. P5.13 B 0.3 m Fig. P5.14 5.15 and 5.16 For the beam and loading shown, determine the maximum normal stress due to bending on a transverse section at C. 3 kN 3 kN 1.8 kN/m A C 1.5 m Fig. P5.15 B D 1.5 m 1.5 m 2000 lb 80 mm 300 mm 4 in. 200 lb/ft C A B 4 ft Fig. P5.16 4 ft 6 ft 8 in. bee80288_ch05_314-379.indd Page 326 10/27/10 9:52:17 PM user-f499 326 Analysis and Design of Beams for Bending /Volumes/201/MHDQ251/bee80288_disk1of1/0073380288/bee80288_pagefiles 5.17 For the beam and loading shown, determine the maximum normal stress due to bending on a transverse section at C. 8 kN 3 kN/m C A B W310  60 1.5 m 2.1 m Fig. P5.17 5.18 For the beam and loading shown, determine the maximum normal stress due to bending on section a-a. 30 kN 50 kN 50 kN 30 kN W310  52 a B A a 2m 5 @ 0.8 m  4 m Fig. P5.18 5.19 and 5.20 For the beam and loading shown, determine the maximum normal stress due to bending on a transverse section at C. 5 5 2 2 2 kips kips kips kips kips C D E F C G D E A B A 150 kN 150 kN 90 kN/m B S8  18.4 W460  113 2.4 m 6 @ 15 in.  90 in. 0.8 m 0.8 m 0.8 m Fig. P5.19 Fig. P5.20 5.21 Draw the shear and bending-moment diagrams for the beam and loading shown and determine the maximum normal stress due to bending. 25 kips 25 kips 25 kips C D E A B S12  35 1 ft 2 ft Fig. P5.21 6 ft 2 ft bee80288_ch05_314-364.indd Page 327 11/29/10 6:44:18 PM user-f499 /Users/user-f499/Desktop/Temp Work/Don't Delete Job/MHDQ251:Beer:201 Problems 5.22 and 5.23 Draw the shear and bending-moment diagrams for the beam and loading shown and determine the maximum normal stress due to bending. 160 kN 80 kN/m 24 kN/m 64 kN ? m C B A W310 ⫻ 60 Hinge 2.4 m S250 ⫻ 52 2m D E B 2m C A D 327 1.5 m 2m 1.5 m 0.6 m Fig. P5.22 Fig. P5.23 5.24 and 5.25 Draw the shear and bending-moment diagrams for the beam and loading shown and determine the maximum normal stress due to bending. 5 kips 25 kN/m 40 kN ? m 10 kips C C A B D A B W14 ⫻ 22 W200 ⫻ 31.3 1.6 m 3.2 m 5 ft Fig. P5.24 8 ft 5 ft Fig. P5.25 5.26 Knowing that W 5 12 kN, draw the shear and bending-moment diagrams for beam AB and determine the maximum normal stress due to bending. 5.27 Determine (a) the magnitude of the counterweight W for which the maximum absolute value of the bending moment in the beam is as small as possible, (b) the corresponding maximum normal stress due to bending. (Hint: Draw the bending-moment diagram and equate the absolute values of the largest positive and negative bending moments obtained.) 5.28 Determine (a) the distance a for which the absolute value of the bending moment in the beam is as small as possible, (b) the corresponding maximum normal stress due to bending. (See hint of Prob. 5.27.) 4 kips/ft B A C a Hinge 18 ft Fig. P5.28 W14 ⫻ 68 W 8 kN C 8 kN D W310 ⫻ 23.8 E B A 1m 1m 1m Figs. P5.26 and P5.27 1m bee80288_ch05_314-379.indd Page 328 10/27/10 9:52:34 PM user-f499 328 Analysis and Design of Beams for Bending /Volumes/201/MHDQ251/bee80288_disk1of1/0073380288/bee80288_pagefiles 5.29 Determine (a) the distance a for which the absolute value of the bending moment in the beam is as small as possible, (b) the corresponding maximum normal stress due to bending. (See hint of Prob. 5.27.) 1.2 kips 0.8 kips C 1.2 kips D E B A S3  5.7 1.5 ft a 1.2 ft 0.9 ft Fig. P5.29 5.30 Knowing that P 5 Q 5 480 N, determine (a) the distance a for which the absolute value of the bending moment in the beam is as small as possible, (b) the corresponding maximum normal stress due to bending. (See hint of Prob. 5.27.) P Q 500 mm 500 mm C A D 12 mm 18 mm B a Fig. P5.30 5.31 Solve Prob. 5.30, assuming that P 5 480 N and Q 5 320 N. 5.32 A solid steel bar has a square cross section of side b and is supported as shown. Knowing that for steel r 5 7860 kg/m3, determine the dimension b for which the maximum normal stress due to bending is (a) 10 MPa, (b) 50 MPa. b A C 1.2 m D 1.2 m B b 1.2 m Fig. P5.32 5.33 A solid steel rod of diameter d is supported as shown. Knowing that for steel g 5 490 lb/ft3, determine the smallest diameter d that can be used if the normal stress due to bending is not to exceed 4 ksi. d A B L  10 ft Fig. P5.33 bee80288_ch05_314-379.indd Page 329 10/27/10 9:52:43 PM user-f499 5.3 /Volumes/201/MHDQ251/bee80288_disk1of1/0073380288/bee80288_pagefiles 5.3 Relations Among Load, Shear, and Bending Moment RELATIONS AMONG LOAD, SHEAR, AND BENDING MOMENT When a beam carries more than two or three concentrated loads, or when it carries distributed loads, the method outlined in Sec. 5.2 for plotting shear and bending moment can prove quite cumbersome. The construction of the shear diagram and, especially, of the bending-moment diagram will be greatly facilitated if certain relations existing among load, shear, and bending moment are taken into consideration. Let us consider a simply supported beam AB carrying a distributed load w per unit length (Fig. 5.11a), and let C and C9 be two points of the beam at a distance Dx from each other. The shear and bending moment at C will be denoted by V and M, respectively, and will be assumed positive; the shear and bending moment at C9 will be denoted by V 1 DV and M 1 DM. We now detach the portion of beam CC9 and draw its free-body diagram (Fig. 5.11b). The forces exerted on the free body include a load of magnitude w Dx and internal forces and couples at C and C9. Since shear and bending moment have been assumed positive, the forces and couples will be directed as shown in the figure. Relations between Load and Shear. Writing that the sum of the vertical components of the forces acting on the free body CC9 is zero, we have V 2 1V 1 ¢V2 2 w ¢x 5 0 ¢V 5 2w ¢x 1x©Fy 5 0: Dividing both members of the equation by Dx and then letting Dx approach zero, we obtain dV 5 2w dx (5.5) Equation (5.5) indicates that, for a beam loaded as shown in Fig. 5.11a, the slope dVydx of the shear curve is negative; the numerical value w x 1 2 x w w V A C x C' D B M  M M x (a) Fig. 5.11 Simply supported beam subjected to a distributed load. C C' V  V x (b) 329 bee80288_ch05_314-379.indd Page 330 10/27/10 9:52:49 PM user-f499 330 Analysis and Design of Beams for Bending /Volumes/201/MHDQ251/bee80288_disk1of1/0073380288/bee80288_pagefiles of the slope at any point is equal to the load per unit length at that point. Integrating (5.5) between points C and D, we write VD 2 VC 5 2 # xD w dx (5.6) xC VD 2 VC 5 21area under load curve between C and D2 (5.69) Note that this result could also have been obtained by considering the equilibrium of the portion of beam CD, since the area under the load curve represents the total load applied between C and D. It should be observed that Eq. (5.5) is not valid at a point where a concentrated load is applied; the shear curve is discontinuous at such a point, as seen in Sec. 5.2. Similarly, Eqs. (5.6) and (5.69) cease to be valid when concentrated loads are applied between C and D, since they do not take into account the sudden change in shear caused by a concentrated load. Equations (5.6) and (5.69), therefore, should be applied only between successive concentrated loads. w x 1 2 x Relations between Shear and Bending Moment. Returning to the free-body diagram of Fig. 5.11b, and writing now that the sum of the moments about C9 is zero, we have w 1loMC¿ 5 0 : V M  M M C C' V  V x 1M 1 ¢M2 2 M 2 V ¢x 1 w ¢x ¢M 5 V ¢x 2 ¢x 50 2 1 w 1 ¢x2 2 2 Dividing both members of the equation by Dx and then letting Dx approach zero, we obtain (b) Fig. 5.11 (repeated) dM 5V dx (5.7) Equation (5.7) indicates that the slope dMydx of the bending-moment curve is equal to the value of the shear. This is true at any point where the shear has a well-defined value, i.e., at any point where no concentrated load is applied. Equation (5.7) also shows that V 5 0 at points where M is maximum. This property facilitates the determination of the points where the beam is likely to fail under bending. Integrating (5.7) between points C and D, we write MD 2 MC 5 # xD V dx (5.8) xC MD 2 MC 5 area under shear curve between C and D (5.89) bee80288_ch05_314-379.indd Page 331 10/27/10 9:53:04 PM user-f499 /Volumes/201/MHDQ251/bee80288_disk1of1/0073380288/bee80288_pagefiles 5.3 Relations Among Load, Shear, and Bending Moment Note that the area under the shear curve should be considered positive where the shear is positive and negative where the shear is negative. Equations (5.8) and (5.89) are valid even when concentrated loads are applied between C and D, as long as the shear curve has been correctly drawn. The equations cease to be valid, however, if a couple is applied at a point between C and D, since they do not take into account the sudden change in bending moment caused by a couple (see Sample Prob. 5.6). Draw the shear and bending-moment diagrams for the simply supported beam shown in Fig. 5.12 and determine the maximum value of the bending moment. 331 EXAMPLE 5.03 w From the free-body diagram of the entire beam, we determine the magnitude of the reactions at the supports. B A L w RA 5 RB 5 12wL A Next, we draw the shear diagram. Close to the end A of the beam, the shear is equal to RA, that is, to 12wL, as we can check by considering as a free body a very small portion of the beam. Using Eq. (5.6), we then determine the shear V at any distance x from A; we write B 1 1 RA 2 wL RB 2 wL Fig. 5.12 x V 2 VA 5 2 # w dx 5 2wx 1 2 0 wL V V 5 VA 2 wx 5 12 wL 2 wx 5 w1 12L 2 x2 L 1 2 The shear curve is thus an oblique straight line which crosses the x axis at x 5 Ly2 (Fig. 5.13a). Considering, now, the bending moment, we first observe that MA 5 0. The value M of the bending moment at any distance x from A may then be obtained from Eq. (5.8); we have x L 1 ⫺ 2 wL (a) x M 2 MA 5 # V dx 0 M5 # x w1 12L 2 x2dx 5 1 2 w1Lx 2 2x2 1 8 wL2 M 0 The bending-moment curve is a parabola. The maximum value of the bending moment occurs when x 5 Ly2, since V (and thus dMydx) is zero for that value of x. Substituting x 5 Ly2 in the last equation, we obtain Mmax 5 wL2y8 (Fig. 5.13b). 1 2 Fig. 5.13 L L (b) x bee80288_ch05_314-379.indd Page 332 10/29/10 7:07:46 PM user-f499 332 Analysis and Design of Beams for Bending /Volumes/201/MHDQ251/bee80288_disk1of1/0073380288/bee80288_pagefiles In most engineering applications, one needs to know the value of the bending moment only at a few specific points. Once the shear diagram has been drawn, and after M has been determined at one of the ends of the beam, the value of the bending moment can then be obtained at any given point by computing the area under the shear curve and using Eq. (5.89). For instance, since MA 5 0 for the beam of Example 5.03, the maximum value of the bending moment for that beam can be obtained simply by measuring the area of the shaded triangle in the shear diagram of Fig. 5.13a. We have Mmax 5 1 2 wL 1 L wL wL2 5 2 2 2 8 V L 1 2 x L 1 ⫺ 2 wL (a) 1 8 M wL2 1 2 L L (b) x Fig. 5.13 We note that, in this example, the load curve is a horizontal straight line, the shear curve an oblique straight line, and the bendingmoment curve a parabola. If the load curve had been an oblique straight line (first degree), the shear curve would have been a parabola (second degree) and the bending-moment curve a cubic (third degree). The shear and bending-moment curves will always be, respectively, one and two degrees higher than the load curve. With this in mind, we should be able to sketch the shear and bendingmoment diagrams without actually determining the functions V(x) and M(x), once a few values of the shear and bending moment have been computed. The sketches obtained will be more accurate if we make use of the fact that, at any point where the curves are continuous, the slope of the shear curve is equal to 2w and the slope of the bending-moment curve is equal to V. bee80288_ch05_314-379.indd Page 333 11/12/10 7:31:15 PM user-f499 20 kips A 12 kips B 6 ft 1.5 kips/ft C 8 ft 10 ft 12 kips 12 kips B Ay D 8 ft 20 kips A E D C 6 ft B 10 ft 8 ft 12 kips 1 15 kips/ft C E D 18 kips 26 kips 20 kips M We also note that at both A and E the bending moment is zero; thus, two points (indicated by dots) are obtained on the bending-moment diagram. Shear Diagram. Since dVydx 5 2w, we find that between concentrated loads and reactions the slope of the shear diagram is zero (i.e., the shear is constant). The shear at any point is determined by dividing the beam into two parts and considering either part as a free body. For example, using the portion of beam to the left of section 1, we obtain the shear between B